Vous êtes sur la page 1sur 500

H2

Mathematics
Textbook
CHOO YAN MIN

& Answers.
Covers both 9740 & 9758 syllabuses.
Includes TYS

This version: 24th July 2016.


The latest version will always be at this link.

This book is optimised for viewing in PDF format (click the above link).
Other existing formats are crude conversions and may be sub-optimal.

Recent changes: First complete draft done. Spent a few days checking for errors.
Upcoming changes: None planned.

Page 2, Table of Contents

www.EconsPhDTutor.com

, Errors? Feedback? Email me! ,


With your help, I plan to keep improving this textbook.

Page 3, Table of Contents

www.EconsPhDTutor.com

Please do not be intimidated by the length of this book (~1,400 pages).


The actual main content takes up only about 700+ pages.
The other 700 pages are for things like front matter, TYS questions, appendices,
reproductions of formula lists and syllabuses, and answers to exercises.

Page 4, Table of Contents

www.EconsPhDTutor.com

This book is licensed under the Creative Commons license CC-BY-NC-SA 4.0.

You are free to:


Share copy and redistribute the material in any medium or format
Adapt remix, transform, and build upon the material
Under the following terms:
Attribution You must give appropriate credit, provide a link to the license, and
indicate if changes were made. You may do so in any reasonable manner, but not in any
way that suggests the licensor endorses you or your use.
NonCommercial You may not use the material for commercial purposes.
ShareAlike If you remix, transform, or build upon the material, you must distribute
your contributions under the same license as the original.

Author: Choo, Yan Min.


Title: H2 Mathematics Textbook.
ISBN: 978-981-11-0383-4 (e-book).

Page 5, Table of Contents

www.EconsPhDTutor.com

The first thing to understand is that mathematics is an art.


Paul Lockhart (2009, A Mathematicians Lament, p. 22).

A mathematician, like a painter or a poet, is a maker of patterns. If his patterns are


more permanent than theirs, it is because they are made with ideas. ... Beauty is the
first test: there is no permanent place in the world for ugly mathematics.
- G.H. Hardy (1940 [1967], A Mathematicians Apology, pp. 84-85).

The scientist does not study nature because it is useful to do so. He studies it because
he takes pleasure in it, and he takes pleasure in it because it is beautiful.
- Henri Poincar (1908 [1914], Science and Method, English trans., p. 22).

Page 6, Table of Contents

www.EconsPhDTutor.com

About This Book


This textbook is for Singaporean H2 Maths students (hence the occasional Singlish and
TLAs1 ). Of course, I hope that anyone else in the world will also find this useful!
I needed a definitive reference for my own teaching needs, but could find nothing satisfying.
So I decided to just write my own textbook.
This textbook is based exactly on the old (9740) and revised (9758) syllabuses (also
reproduced in Part VIII). Do check to make sure which exam youre taking. The revised
syllabus (9758) is the same as the old syllabus (9740), but with noticeable chunks excised
and is thus easier.2
9740 (old) examined? 9758 (revised) examined?
2016
Yes.
No.
2017
Yes, for the last time.
Yes, for the first time.
2018
No.
Yes.

SYLLABUS ALERT
Where there are any differences between the old and revised syllabuses, Ill let you know
with a yellow box like this.

FREE! This book is free. But if you paid any money for it, I certainly hope your money
is going to me! This book is free because:
1. It is a shameless advertising vehicle for my awesome tutoring services.
2. The marginal cost of reproducing this book is zero.
DONATE! This book may be free, but donations are more than welcome! Donation
methods in footnote.3
Its irrational for Homo economicus to donate. But please consider donating because:
1. Youre a nice human being , [*emotional_manipulation*].
2. Your donations will encourage me and others to continue producing awesome free content
for the world.
Three Letter Abbreviations.
Indeed, some chunks of the old syllabus (9740) have simply been moved into the syllabus of Further Maths (9649), which
the authorities have kindly resurrected for the 2017 exam season.
3
Singapore. POSB Savings Account 174052271 or OCBC Savings Account 5523016383 (Name: Choo Yan Min). International. Bitcoin wallet: 1GDGNAdGZhEq9pz2SaoAdLb1uu34LFwViz. Paypal ychoo@umich.edu (Name: Yan Min Choo,
USD preferred because this account was set up in the US). USA. Venmo link (Name: Yanmin Choo).

Page 7, Table of Contents

www.EconsPhDTutor.com

HELP ME IMPROVE THIS BOOK! Feel free to email me if:


1. There are any errors in this book. Please let me know even if its something as trivial
as a spelling mistake or a grammatical error.
2. You have absolutely any suggestions for improvement.
3. Any part of this book is less than crystal clear.
Heres an anecdote about Richard Feynman, the great teacher and physicist:
Feynman was once asked by a Caltech faculty member to explain why spin
1/2 particles obey Fermi-Dirac statistics. He gauged his audience perfectly
and said, Ill prepare a freshman lecture on it. But a few days later he
returned and said, You know, I couldnt do it. I couldnt reduce it to
the freshman level. That means we really dont understand it.
I agree: If you cant explain something simply, you dont understand it well enough.4 And
as a corollary, the best way to gauge whether you understand something is to see if you
can explain it simply to someone else.
If at any point in this textbook, you have read the same passage a few times, tried to reason
it through, and still find things confusing, then it is a failure on MY part. Please let me
know and I will try to rewrite it so that its clearer. (There is also the possibility that I
simply messed up! So please let me know if theres anything confusing!)
I deeply value any feedback, because Id like to keep improving this textbook
for the benefit of everyone! I am very grateful to all the kind folks whove already
written in, allowing me to rid this book of more than a few embarrassing errors.
LyX rocks!
This book was written using LYX.5
Is the font size big enough?
Youre probably reading this on some device. So Ive tried to set the font sizes and stuff so
that one can comfortably read this on a device as small as a seven-inch tablet. It should
also be possible to read this on a phone, though somewhat less comfortably. (Please let me
know if you have any feedback about this!)
(Ill probably be contacting some publishers to see if they want to do a print version of
this, for anyone who prefers it in print.)
This quote or some similar variant is often (mis)attributed to Einstein. But as Einstein himself once said, 73% of Einstein
quotes are misattributed.
5 A
L TEX is the typesetting programme used by most economists and scientists. But LATEX can be difficult to use. LYX is a
user-friendly GUI version of LATEX. LYX has boosted my productivity by countless hours over the years and you should use
LYX too!

Page 8, Table of Contents

www.EconsPhDTutor.com

Tips for the Student


Read maths slowly.
Reading maths is not like reading Harry Potter. Most of Harry Potter is fluff. There is
little fluff in maths.
So go slowly. Dwell upon and carefully consider every sentence in this textbook. Make sure
you completely understand what each statement says and why it is true. Reading maths
is very different from reading any other subject matter.
If you dont quite understand some material, you might be tempted to move forward anyway.
Dont. In maths, later material usually builds on earlier material. So if you simply move
forward, this will usually cost you more time and frustration in the long run.
Better then to stop right there. Keep working on it until you get it. Ask a friend or
a teacher for help. Feel free to even email me! (Im always interested to know what the
common points of confusion are and how I can better clear them up.)
Examples and exercises are your best friends. So work through them.
A good stock of examples, as large as possible, is indispensable for a
thorough understanding of any concept, and when I want to learn something
new, I make it my first job to build one.

- Paul Halmos (1983, Google Books).


Work through all the examples and exercises. Merely moving your eyeballs is not the same
as working. Working means having pencil and paper by your side and going through each
example/exercise word-by-word, line-by-line.
For example, I might say something like x2 y 2 = 0. Thus, (x y)(x + y) = 0. If its not
obvious to you why the first sentence implies the second, stop right there and work on it
until you understand why. Dont just let your eyeballs fly over these sentences and pretend
that your brain is getting it.
I will often not bother to explain some steps, especially if they simply involve some simple
algebra.
You get a List of Formulae during the A-level exam.
So theres no need to memorise all the formulae that are already on the list youre getting.
Note that you get a different list depending on which exam youre taking List of Formulae
(MF15) for the old 9740 exam and List of Formulae (MF26) for the revised 9758 exam.
(Both lists are reproduced in Part VIII of this book.) I cannot guarantee though that your
JC will give you the List during your JC common tests and exams.
Page 9, Table of Contents

www.EconsPhDTutor.com

Remember your O-Level Additional Mathematics?


Youve probably forgotten some (or most?) of it, but unfortunately, you are still assumed
to know EVERYTHING from O-Level A Maths. See the lists near the end of either the
9740 (old) or the 9758 (revised) syllabus. Skim through and see if anything looks totally
alien to you!
Some chapters (e.g. Chapters 5 and 26) in this textbook will give a quick review of some of
the O-Level Maths material that you may have forgotten but which well use quite often.
Online Calculators
Google is probably the quickest for simple calculations. Type in anything into your
browsers Google search bar and the answer will instantly show up:

Wolfram Alpha is somewhat more advanced (but also slower). Enter sin x for example
and youll get graphs, the derivative, the indefinite integral, the Maclaurin series, and a
bunch of other stuff you neither know nor care about.
The Derivative Calculator and the Integral Calculator are probably unbeatable for the
specific purposes of differentiation and integration. Both give step-by-step solutions for
anything you want to differentiate or integrate.
Here is a collection of spreadsheets I made. These spreadsheets are for doing tedious and
repetitive calculations youll often encounter in H2 maths (e.g. with vectors, complex
numbers, etc.). As with anything I do, I welcome any feedback you may have about
these spreadsheets. Perhaps in the future I will make a more attractive version of it.
(Instructions: Click Make a copy to open up your own independent copy of
this spreadsheet. Enter your input in the yellow cells. Output is produced in
the blue cells. If you mess up anything, simply click the same link and Make
a copy again.)

Page 10, Table of Contents

www.EconsPhDTutor.com

Other Online Resources


There are way too many websites out there catering to primary, secondary, and lower-level
undergraduate maths. Unfortunately, some of them can be awful and can get things wrong.
Three resources I like (though are probably a bit advanced for JC students) are:
1. Math StackExchange
A great resource where you can ask maths questions and often get them answered fairly
promptly. Note though that this site is mostly frequented by fairly advanced students of
maths (not to mention also mathematicians), so they can be pretty impatient and quick
to downvote questions they perceive to be stupid. Nonetheless, if you make an effort to
write down a carefully-crafted question and show also that youve made some effort to look
for an answer (either on your own or online), they can be very helpful.6
2. ProofWiki gives succinct and rigorous definitions and proofs. Unfortunately it is very
incomplete.
3. Mathworld.Wolfram is also great, but at times excessively encyclopaedic, at the cost of
clarity and brevity.

And of course, you can find countless free maths textbooks online (some less legal than
others). Two totally illegal7 resources are: LibGen for books and SciHub for articles.8
An old reliable is Bittorrent.

There is an entire StackExchange family of websites. The flagship site is StackOverflow where you can ask any programming
question and get it answered amazingly quickly.
7
Well, depending on which jurisdiction you live in. Of course, in Singapore, unless told otherwise, you should assume that
everything is illegal.
8
Note though that these sites are constantly playing whac-a-mole with the fascist authorities so the URLs often change
if so, simply google to look up the current URLs.

Page 11, Table of Contents

www.EconsPhDTutor.com

Preface
Divide students into two extremes:
1. Type #1 is happy to get an A, even if this means learning absolutely nothing.
2. Type #2 would rather learn a lot, even if this means getting a C.
The good Singaporean is trained to view pragmatism is the highest virtue (and obedience
second). She is thus also trained to be a Type #1 student (and indeed a Type #1 human
being).
If youre a Type #1 student, then this textbook may not be the best use of your time
(though you may still find the TYS and answers useful). Please use instead these three
resources, which are provided with the efficient Type #1 student in mind:
The H2 Mathematics CheatSheet, which contains all the formulae youll ever need
on two sides of a single A4 sheet of paper.9
The H2 Maths Exercise Book (coming soon), which teaches you how to mindlessly
apply formulae and give the correct answer to every exam question.
My totally awesome tuition classes!
Of course, it is fully intended that this textbook (complemented by a capable teacher) will
help any student get her A. But that for me is quite beside the point.
My broader goal in writing this textbook is to impart genuine understanding or at least
as much as is possible, within the stultifying confines of the A-level syllabus.
Maths education in Singapore is at least every bit as stupid and boring and formulaic
and mindless as in the US.10 But at least the average US student has the consolation
that only a very small portion of her life will have been squandered on mindless pseudomaths.
The same cannot be said for the average Singaporean student. By the time she turns 18,
she will have just for the subject of maths alone clocked many thousands of hours
attending classes; doing homework; doing practice exam questions; doing assessment books,
Ten Year Series; going to tuition classes; taking common tests, promos, prelims, one big
exam after another; etc.
This textbook is for the Singaporean A-level student. So a good deal of mindless formulae
is unavoidable. But at the same time, I try in this textbook to give the student a tiny
glimpse of what maths really is the art of explanation.11 So for example, this textbook
explains
Two things: (1) This CheatSheet does not include many of the formulae already printed in List MF26. (2) It is written for
9758 (revised) students (so 9740 students may find a few things missing).
10
At least as described by Paul Lockhart, in A Mathematicians Lament.
11
Lockhart, p. 29.

Page 12, Table of Contents

www.EconsPhDTutor.com

A bit of intuition behind differentiation, integration, and the Fundamental Theorems of


Calculus. (To get an A, no understanding of these is necessary. Instead, one need merely
know how to do differentiation and integration problems.)
Why the Central Limit Theorem is so amazing. (To get an A, one need merely treat the
CLT as yet another mysterious mathematical trick that helps solve exam questions. No
appreciation of why it is wonderful is necessary.)
A bit of intuition behind the Maclaurin series. (To get an A, it suffices to know how to
mindlessly apply this strange formula that falls out from the sky.)
Why it is terribly wrong to believe that a high correlation coefficient means a good
model. (Yet this is exactly what the writers of the A-level exams seem to believe. See
Section 73.9.)
Once upon a time, I had the misfortune of being a Singaporean JC student myself. I
remember being deeply mystified by why the scalar (or dot) product, despite having a
simple algebraic definition, could at the same time also tell us about the cosine of the
angle between the two vectors. I never figured it out,12 but it didnt matter, because this
was simply yet another formula that we were required to know, for the sole purpose of
answering exam questions.
I remember being confused about the difference between the sample mean, the mean of the
sample mean, the variance of the sample mean, and the sample variance. But this confusion
didnt matter, because once again, all we needed to do to get an A was to mindlessly apply
formulae and algorithms. Monkey see, monkey do.
This textbook is thus partly in response to my unhappy and unsatisfactory experience as
a maths student in Singapore. Almost all results are proven. I often try to supply the
intuition for each result in the simplest possible terms. Many proofs are relegated to the
appendices, but where a proof is especially simple and beautiful, I encourage the student
to savour it by leaving it in the main text. In the rare instances where proofs are entirely
omitted from this book usually because they are too advanced I make sure to clearly
state so, lest the student wonder whether the result is supposed to be obvious.
Finally, I also hope that this textbook will serve as an authoritative resource to which
teachers and students alike can refer.

12

The internet was, at that time, not so well-developed, so one could not easily find answers online.

Page 13, Table of Contents

www.EconsPhDTutor.com

Tuition Ad
I give tuition for the following at any level:

Economics.

Mathematics.

Writing, English, General Paper.


I have a PhD in economics (University of Michigan)
and have been teaching and tutoring since 2010.

For more information, please visit:

www.EconsPhDTutor.com
Or email:

DrChooYanMin@gmail.com

Contents
About This Book

Tips for the Student

Preface

12

35

Functions and Graphs

1 Sets

36

1.1

In and Not In . . . . . . . . . . . . . . . . . . . . . . . . . . . . . . . . . . . .

37

1.2

Greater than >, Less Than <, Positive > 0, and Negative < 0 . . . . . . . . . .

38

1.3

Types of Numbers . . . . . . . . . . . . . . . . . . . . . . . . . . . . . . . . . . .

39

1.4

The Order of the Elements Doesnt Matter . . . . . . . . . . . . . . . . . . . .

40

1.5

Repeated Elements Dont Count . . . . . . . . . . . . . . . . . . . . . . . . . .

41

1.6

Ellipsis . . . Means Continue in the Obvious Fashion . . . . . . . . . . . . . . .

42

1.7

Sets can be Finite or Infinite . . . . . . . . . . . . . . . . . . . . . . . . . . . . .

43

1.8

Special Names of Sets: Z, Q, R, and C . . . . . . . . . . . . . . . . . . . . . . .

44

1.9

Special Names of Sets: Intervals

. . . . . . . . . . . . . . . . . . . . . . . . . .

45

1.10 Special Names of Sets: The Empty Set . . . . . . . . . . . . . . . . . . . . .

47

1.11 Subset Of . . . . . . . . . . . . . . . . . . . . . . . . . . . . . . . . . . . . . . .

48

1.12 Proper Subset Of . . . . . . . . . . . . . . . . . . . . . . . . . . . . . . . . . .

49

1.13 Union . . . . . . . . . . . . . . . . . . . . . . . . . . . . . . . . . . . . . . . . .

50

1.14 Intersection . . . . . . . . . . . . . . . . . . . . . . . . . . . . . . . . . . . . . .

51

1.15 Set Minus / . . . . . . . . . . . . . . . . . . . . . . . . . . . . . . . . . . . . . . .

52

1.16 Set Complement A . . . . . . . . . . . . . . . . . . . . . . . . . . . . . . . . . .

53

1.17 Set-Builder Notation . . . . . . . . . . . . . . . . . . . . . . . . . . . . . . . . . .

54

2 Dividing By Zero

Page 15, Table of Contents

55

www.EconsPhDTutor.com

3 Functions

56

3.1

Formal Mathematical Notation for Functions

. . . . . . . . . . . . . . . . . .

58

3.2

EVERY x D Must be Mapped to EXACTLY ONE y C . . . . . . . . . . .

62

3.3

Real-Valued Functions of a Real Variable . . . . . . . . . . . . . . . . . . . . .

64

3.4

The Range of a Function . . . . . . . . . . . . . . . . . . . . . . . . . . . . . . .

65

3.5

Creating New Functions . . . . . . . . . . . . . . . . . . . . . . . . . . . . . . . .

66

3.6

One-to-One Functions . . . . . . . . . . . . . . . . . . . . . . . . . . . . . . . . .

67

3.7

Inverse Functions . . . . . . . . . . . . . . . . . . . . . . . . . . . . . . . . . . . .

68

3.8

Domain Restriction to Create an Invertible Function . . . . . . . . . . . . . .

73

3.9

Composite Functions . . . . . . . . . . . . . . . . . . . . . . . . . . . . . . . . .

74

4 Graphs
4.1

77

Graphing with Your TI84 Graphing Calculator . . . . . . . . . . . . . . . . . .

5 Quick Revision: Exponents, Surds, Absolute Value

83
86

5.1

Laws of Exponents . . . . . . . . . . . . . . . . . . . . . . . . . . . . . . . . . . .

86

5.2

Rationalising the Denominator of a Surd . . . . . . . . . . . . . . . . . . . . .

87

5.3

Absolute Value . . . . . . . . . . . . . . . . . . . . . . . . . . . . . . . . . . . . .

88

6 Intercepts

90

7 Symmetry

93

7.1

Reflection of a Point in a Line . . . . . . . . . . . . . . . . . . . . . . . . . . . .

93

7.2

Reflection of a Graph in a Line . . . . . . . . . . . . . . . . . . . . . . . . . . .

94

7.3

Lines of Symmetry . . . . . . . . . . . . . . . . . . . . . . . . . . . . . . . . . . .

97

8 Limits, Continuity, and Asymptotes

99

8.1

Limits: Introduction and Examples . . . . . . . . . . . . . . . . . . . . . . . . .

8.2

Continuity . . . . . . . . . . . . . . . . . . . . . . . . . . . . . . . . . . . . . . . . 103

8.3

Limits: More Examples

8.4

Infinite Limits and Vertical Asymptotes . . . . . . . . . . . . . . . . . . . . . . 109

8.5

Limits at Infinity, Horizontal and Oblique Asymptotes . . . . . . . . . . . . . 111

Page 16, Table of Contents

99

. . . . . . . . . . . . . . . . . . . . . . . . . . . . . . . 106

www.EconsPhDTutor.com

9 Differentiation

114

9.1

Motivation: The Derivative as Slope of the Tangent . . . . . . . . . . . . . . . 114

9.2

Lagranges, Leibnizs, and Newtons Notation . . . . . . . . . . . . . . . . . . . 117

9.3

The Derivative is a Function . . . . . . . . . . . . . . . . . . . . . . . . . . . . . 120

9.4

Second and Higher-Order Derivatives

. . . . . . . . . . . . . . . . . . . . . . . 121

9.5

More About Leibnizs Notation: The

d
Operator . . . . . . . . . . . . . . . . 123
dx

9.6

Standard Rules of Differentiation . . . . . . . . . . . . . . . . . . . . . . . . . . 124

9.7

Differentiable and Twice-Differentiable Functions . . . . . . . . . . . . . . . . 127

9.8

Differentiability Implies (i.e. is Stronger Than) Continuity . . . . . . . . . . . 130

9.9

Implicit Differentiation . . . . . . . . . . . . . . . . . . . . . . . . . . . . . . . . 131

10 Increasing, Decreasing, and f

133

10.1 When a Function is Increasing or Decreasing . . . . . . . . . . . . . . . . . . . 133


10.2 The First Derivative Increasing/Decreasing Test . . . . . . . . . . . . . . . . . 134
11 Extreme, Stationary, and Turning Points

135

11.1 Maximum and Minimum Points . . . . . . . . . . . . . . . . . . . . . . . . . . . 135


11.2 Global Maximum and Minimum Points . . . . . . . . . . . . . . . . . . . . . . 139
11.3 Stationary and Turning Points . . . . . . . . . . . . . . . . . . . . . . . . . . . . 143
11.4 The Interior Extremum Theorem . . . . . . . . . . . . . . . . . . . . . . . . . . 145
11.5 How to Find Maximum and Minimum Points . . . . . . . . . . . . . . . . . . . 147
12 Concavity, Inflexion Points, and the 2DT

151

12.1 The Second Derivative Test (2DT) . . . . . . . . . . . . . . . . . . . . . . . . . 155


12.2 Summary of Points and Venn Diagram

. . . . . . . . . . . . . . . . . . . . . . 157

13 Relating the Graph of f to that of f

159

14 Quick Revision: Quadratic Equations y = ax2 + bx + c

162

Page 17, Table of Contents

www.EconsPhDTutor.com

15 Transformations

166

15.1 y = f (x) + a . . . . . . . . . . . . . . . . . . . . . . . . . . . . . . . . . . . . . . . 166


15.2 y = f (x + a) . . . . . . . . . . . . . . . . . . . . . . . . . . . . . . . . . . . . . . . 167
15.3 y = af (x) . . . . . . . . . . . . . . . . . . . . . . . . . . . . . . . . . . . . . . . . 168
15.4 y = f (ax) . . . . . . . . . . . . . . . . . . . . . . . . . . . . . . . . . . . . . . . . 169
15.5 Combinations of the Above . . . . . . . . . . . . . . . . . . . . . . . . . . . . . . 170
15.6 y = f (x) . . . . . . . . . . . . . . . . . . . . . . . . . . . . . . . . . . . . . . . . 171
15.7 y = f (x) . . . . . . . . . . . . . . . . . . . . . . . . . . . . . . . . . . . . . . . . . 172
15.8 y =

1
. . . . . . . . . . . . . . . . . . . . . . . . . . . . . . . . . . . . . . . . . 173
f (x)

15.9 y 2 = f (x) . . . . . . . . . . . . . . . . . . . . . . . . . . . . . . . . . . . . . . . . . 174


16 Conic Sections

176

16.1 The Ellipse x2 + y 2 = 1 (The Unit Circle) . . . . . . . . . . . . . . . . . . . . . . 179


x2 y 2
16.2 The Ellipse 2 + 2 = 1 . . . . . . . . . . . . . . . . . . . . . . . . . . . . . . . . 181
a
b
16.3 The Hyperbola: y = 1/x . . . . . . . . . . . . . . . . . . . . . . . . . . . . . . . . 183
16.4 The Hyperbola x2 y 2 = 1 . . . . . . . . . . . . . . . . . . . . . . . . . . . . . . . 185
x2 y 2
16.5 The Hyperbola 2 2 = 1 . . . . . . . . . . . . . . . . . . . . . . . . . . . . . . 186
a
b
16.6 The Hyperbola

y 2 x2

= 1 . . . . . . . . . . . . . . . . . . . . . . . . . . . . . . 188
b2 a2

16.7 Long Division of Polynomials . . . . . . . . . . . . . . . . . . . . . . . . . . . . 190


16.8 The Hyperbola y =

bx + c
. . . . . . . . . . . . . . . . . . . . . . . . . . . . . . . 193
dx + e

ax2 + bx + c
. . . . . . . . . . . . . . . . . . . . . . . . . . . 198
16.9 The Hyperbola y =
dx + e
17 Simple Parametric Equations

203

17.1 Eliminating the Parameter t . . . . . . . . . . . . . . . . . . . . . . . . . . . . . 208

Page 18, Table of Contents

www.EconsPhDTutor.com

18 Equations and Inequalities

211

18.1

ax + b
> 0 . . . . . . . . . . . . . . . . . . . . . . . . . . . . . . . . . . . . . . . . 212
cx + d

18.2

ax2 + bx + c
> 0 . . . . . . . . . . . . . . . . . . . . . . . . . . . . . . . . . . . . . 214
dx2 + ex + f

18.3 Solving Inequalities by Graphical Methods . . . . . . . . . . . . . . . . . . . . 218


18.4 Systems of Equations . . . . . . . . . . . . . . . . . . . . . . . . . . . . . . . . . 221

II

Sequences and Series

19 Finite Sequences

224
225

19.1 A Corresponding Function for a Sequence . . . . . . . . . . . . . . . . . . . . . 226


19.2 Recurrence Relations . . . . . . . . . . . . . . . . . . . . . . . . . . . . . . . . . 227
19.3 Creating New Sequences . . . . . . . . . . . . . . . . . . . . . . . . . . . . . . . 229
20 Infinite Sequences

231

20.1 Creating New Sequences . . . . . . . . . . . . . . . . . . . . . . . . . . . . . . . 232


21 Series

233

21.1 Convergent and Divergent Sequences and Series . . . . . . . . . . . . . . . . . 234


22 Summation Notation

236

23 Arithmetic Sequences and Series

240

23.1 Finite Arithmetic Sequences and Series . . . . . . . . . . . . . . . . . . . . . . 241


24 Geometric Sequences and Series

242

24.1 Finite Geometric Sequences and Series . . . . . . . . . . . . . . . . . . . . . . . 243


24.2 Infinite Geometric Sequences and Series . . . . . . . . . . . . . . . . . . . . . . 244
25 Proof by the Method of Mathematical Induction

245

III

251

Vectors

Page 19, Table of Contents

www.EconsPhDTutor.com

26 Quick Revision of Some O-Level Maths

252

26.1 Lines vs. Line Segments vs. Rays . . . . . . . . . . . . . . . . . . . . . . . . . . 252


26.2 Angles - Acute, Right, Obtuse, Straight, Reflex . . . . . . . . . . . . . . . . . . 253
26.3 Triangles - Acute, Right, Obtuse . . . . . . . . . . . . . . . . . . . . . . . . . . 254
26.4 Sine, Cosine, Tangent - Definitions . . . . . . . . . . . . . . . . . . . . . . . . . 255
26.5 Sine, Cosine, Tangent - Values and Graphs . . . . . . . . . . . . . . . . . . . . 256
26.6 Formulae for Sine, Cosine, and Tangent . . . . . . . . . . . . . . . . . . . . . . 257
26.7 Arcsine, Arccosine, Arctangent . . . . . . . . . . . . . . . . . . . . . . . . . . . 258
26.8 The Law of Sines and the Law of Cosines . . . . . . . . . . . . . . . . . . . . . 262
27 Vectors in Two Dimensions (2D)

264

27.1 Sum and Difference of Points and Vectors . . . . . . . . . . . . . . . . . . . . . 269


27.2 Sum, Additive Inverse, and Difference of Vectors . . . . . . . . . . . . . . . . . 272
27.3 Displacement Vectors . . . . . . . . . . . . . . . . . . . . . . . . . . . . . . . . . 275
27.4 Length (or Magnitude) of a Vector . . . . . . . . . . . . . . . . . . . . . . . . . 276
27.5 Scalar Multiplication of a Vector . . . . . . . . . . . . . . . . . . . . . . . . . . 278
27.6 Unit Vectors . . . . . . . . . . . . . . . . . . . . . . . . . . . . . . . . . . . . . . . 279
27.7 The Ratio Theorem . . . . . . . . . . . . . . . . . . . . . . . . . . . . . . . . . . 282
28 Scalar Product

284

28.1 The Angle between Two Vectors . . . . . . . . . . . . . . . . . . . . . . . . . . . 285


28.2 Projection of One Vector on Another . . . . . . . . . . . . . . . . . . . . . . . . 289
28.3 Direction Cosines . . . . . . . . . . . . . . . . . . . . . . . . . . . . . . . . . . . . 291
29 Vectors in 3D

293

30 Vector Product

297

30.1 Vector Product in 2D . . . . . . . . . . . . . . . . . . . . . . . . . . . . . . . . . 297


30.2 Areas of Triangles and Parallelograms . . . . . . . . . . . . . . . . . . . . . . . 299
30.3 Vector Product in 3D . . . . . . . . . . . . . . . . . . . . . . . . . . . . . . . . . 301
Page 20, Table of Contents

www.EconsPhDTutor.com

31 Lines

305

31.1 Lines on a 2D Plane: Cartesian to Vector Equations . . . . . . . . . . . . . . 305


31.2 Lines on a 2D Plane: Vector to Cartesian Equations . . . . . . . . . . . . . . 310
31.3 Lines in 3D Space: Vector Equations . . . . . . . . . . . . . . . . . . . . . . . . 312
31.4 Lines in 3D Space: Vector to and from Cartesian Equations . . . . . . . . . . 314
31.5 Collinearity . . . . . . . . . . . . . . . . . . . . . . . . . . . . . . . . . . . . . . . 321
32 Planes

323

32.1 Planes: Vector to Cartesian Equations . . . . . . . . . . . . . . . . . . . . . . . 330


32.2 Planes: Hessian Normal Form . . . . . . . . . . . . . . . . . . . . . . . . . . . . 332
33 Distances

333

33.1 Distance of a Point from a Line . . . . . . . . . . . . . . . . . . . . . . . . . . . 334


33.2 Distance of a Point from a Plane . . . . . . . . . . . . . . . . . . . . . . . . . . 341
34 Angles

345

34.1 Angle between Two Lines (2D) . . . . . . . . . . . . . . . . . . . . . . . . . . . 345


34.2 Angle between Two Lines (3D) . . . . . . . . . . . . . . . . . . . . . . . . . . . 350
34.3 Angle between A Line and a Plane . . . . . . . . . . . . . . . . . . . . . . . . . 353
34.4 Angle between Two Planes . . . . . . . . . . . . . . . . . . . . . . . . . . . . . . 355
35 Relationships between Lines and Planes

357

35.1 Relationship between Two Lines . . . . . . . . . . . . . . . . . . . . . . . . . . . 357


35.2 Relationship between a Line and a Plane . . . . . . . . . . . . . . . . . . . . . 361
35.3 Relationship between Two Planes . . . . . . . . . . . . . . . . . . . . . . . . . . 363
35.4 Relationship between Three Planes . . . . . . . . . . . . . . . . . . . . . . . . . 368

IV

Complex Numbers

36 Complex Numbers: Introduction

374
375

36.1 The Real and Imaginary Parts of Complex Numbers . . . . . . . . . . . . . . 378


Page 21, Table of Contents

www.EconsPhDTutor.com

37 Basic Arithmetic of Complex Numbers

379

37.1 Addition and Subtraction . . . . . . . . . . . . . . . . . . . . . . . . . . . . . . . 379


37.2 Multiplication . . . . . . . . . . . . . . . . . . . . . . . . . . . . . . . . . . . . . . 380
37.3 Division . . . . . . . . . . . . . . . . . . . . . . . . . . . . . . . . . . . . . . . . . 382
38 Solving Polynomial Equations

384

38.1 Complex Roots to Quadratic Equations . . . . . . . . . . . . . . . . . . . . . . 384


38.2 The Fundamental Theorem of Algebra . . . . . . . . . . . . . . . . . . . . . . . 386
38.3 The Complex Conjugate Roots Theorem . . . . . . . . . . . . . . . . . . . . . . 389
39 The Argand Diagram

390

39.1 Complex Numbers in Polar Form . . . . . . . . . . . . . . . . . . . . . . . . . . 392


39.2 Complex Numbers in Exponential Form . . . . . . . . . . . . . . . . . . . . . . 397
40 More Arithmetic of Complex Numbers

398

40.1 The Product of Two Complex Numbers . . . . . . . . . . . . . . . . . . . . . . 398


40.2 The Ratio of Two Complex Numbers . . . . . . . . . . . . . . . . . . . . . . . . 401
40.3 Sine and Cosine as Weighted Sums of the Exponential . . . . . . . . . . . . . 403
41 Geometry of Complex Numbers

406

41.1 The Sum and Difference of Two Complex Numbers . . . . . . . . . . . . . . . 406


41.2 The Product and Ratio of Two Complex Numbers . . . . . . . . . . . . . . . . 408
41.3 Conjugating a Complex Number . . . . . . . . . . . . . . . . . . . . . . . . . . . 410
42 Loci Involving Cartesian Equations

411

42.1 Circles . . . . . . . . . . . . . . . . . . . . . . . . . . . . . . . . . . . . . . . . . . 411


42.2 Lines . . . . . . . . . . . . . . . . . . . . . . . . . . . . . . . . . . . . . . . . . . . 416
42.3 Intersection of Lines and Circles . . . . . . . . . . . . . . . . . . . . . . . . . . . 420

Page 22, Table of Contents

www.EconsPhDTutor.com

43 Loci Involving Complex Equations

423

43.1 Circles . . . . . . . . . . . . . . . . . . . . . . . . . . . . . . . . . . . . . . . . . . 423


43.2 Lines . . . . . . . . . . . . . . . . . . . . . . . . . . . . . . . . . . . . . . . . . . . 425
43.3 Rays . . . . . . . . . . . . . . . . . . . . . . . . . . . . . . . . . . . . . . . . . . . 426
43.4 Quick O-Level Revision: Properties of The Circle . . . . . . . . . . . . . . . . 429
44 De Moivres Theorem

432

44.1 Powers of a Complex Number . . . . . . . . . . . . . . . . . . . . . . . . . . . . 433


44.2 Roots of a Complex Number . . . . . . . . . . . . . . . . . . . . . . . . . . . . . 436

Calculus

45 Solving Problems Involving Differentiation

441
442

45.1 Inverse Function Theorem (IFT) . . . . . . . . . . . . . . . . . . . . . . . . . . 442


45.2 Differentiation of Simple Parametric Functions

. . . . . . . . . . . . . . . . . 443

45.3 Equations of Tangents and Normals . . . . . . . . . . . . . . . . . . . . . . . . 444


45.4 Connected Rates of Change Problems . . . . . . . . . . . . . . . . . . . . . . . 445
45.5 Finding Max/Min Points on the TI84 . . . . . . . . . . . . . . . . . . . . . . . 447
45.6 Finding the Derivative at a Point on the TI84 . . . . . . . . . . . . . . . . . . 449
46 The Maclaurin Series

451

46.1 Power Series . . . . . . . . . . . . . . . . . . . . . . . . . . . . . . . . . . . . . . . 451


46.2 Maclaurin Series . . . . . . . . . . . . . . . . . . . . . . . . . . . . . . . . . . . . 452
46.3 The Amazing Maclaurin Series . . . . . . . . . . . . . . . . . . . . . . . . . . . . 454
46.4 Finite-Order Maclaurin Series as Approximations . . . . . . . . . . . . . . . . 456
46.5 Product of Two Power Series . . . . . . . . . . . . . . . . . . . . . . . . . . . . . 460
46.6 Composition of Two Functions . . . . . . . . . . . . . . . . . . . . . . . . . . . . 462
46.7 How the Maclaurin Series Works (Optional) . . . . . . . . . . . . . . . . . . . . 465

Page 23, Table of Contents

www.EconsPhDTutor.com

47 The Indefinite Integral

466

47.1 The Constant of Integration . . . . . . . . . . . . . . . . . . . . . . . . . . . . . 468


47.2 The Indefinite Integral is Unique Up to the C.O.I. . . . . . . . . . . . . . . . . 469
48 Integration Techniques

470

48.1 Basic Rules of Integration . . . . . . . . . . . . . . . . . . . . . . . . . . . . . . . 471


48.2 More Basic Rules of Integration . . . . . . . . . . . . . . . . . . . . . . . . . . . 472
48.3 Trigonometric Functions . . . . . . . . . . . . . . . . . . . . . . . . . . . . . . . 474
48.4 Integration by Substitution (IBS) . . . . . . . . . . . . . . . . . . . . . . . . . . 475
48.5 Integration by Parts (IBP) . . . . . . . . . . . . . . . . . . . . . . . . . . . . . . 480
49 The Fundamental Theorems of Calculus (FTCs)

481

49.1 The Area Function . . . . . . . . . . . . . . . . . . . . . . . . . . . . . . . . . . . 481


49.2 The First Fundamental Theorem of Calculus (FTC1) . . . . . . . . . . . . . . 486
49.3 The Definite (or Riemann) Integral

. . . . . . . . . . . . . . . . . . . . . . . . 490

49.4 The Second Fundamental Theorem of Calculus (FTC2) . . . . . . . . . . . . . 491


50 Definite Integrals

492

50.1 Area between a Curve and Lines Parallel to Axes . . . . . . . . . . . . . . . . 493


50.2 Area between a Curve and a Line . . . . . . . . . . . . . . . . . . . . . . . . . . 494
50.3 Area between Two Curves . . . . . . . . . . . . . . . . . . . . . . . . . . . . . . 495
50.4 Area below the x-Axis . . . . . . . . . . . . . . . . . . . . . . . . . . . . . . . . . 496
50.5 Area under a Parametrically-Defined Curve

. . . . . . . . . . . . . . . . . . . 497

50.6 Volume of Rotation about the y- or x-Axis . . . . . . . . . . . . . . . . . . . . 498


50.7 Finding Definite Integrals on your TI84 . . . . . . . . . . . . . . . . . . . . . . 501
51 Differential Equations
dy
= f (x) . . . . . . .
dx
dy
51.2
= f (y) . . . . . . .
dx
d2 y
51.3
= f (x) . . . . . . .
dx2
51.4 Word Problems . . .
51.1

502
. . . . . . . . . . . . . . . . . . . . . . . . . . . . . . . . . 502
. . . . . . . . . . . . . . . . . . . . . . . . . . . . . . . . . 503
. . . . . . . . . . . . . . . . . . . . . . . . . . . . . . . . . 505
. . . . . . . . . . . . . . . . . . . . . . . . . . . . . . . . . 507

51.5 Family of Solution Curves to Represent the General Solution . . . . . . . . . 510


Page 24, Table of Contents

www.EconsPhDTutor.com

VI

Probability and Statistics

52 How to Count: Four Principles

511
512

52.1 How to Count: The Addition Principle . . . . . . . . . . . . . . . . . . . . . . . 513


52.2 How to Count: The Multiplication Principle . . . . . . . . . . . . . . . . . . . 516
52.3 How to Count: The Inclusion-Exclusion Principle . . . . . . . . . . . . . . . . 520
52.4 How to Count: The Complements Principle . . . . . . . . . . . . . . . . . . . . 522
53 How to Count: Permutations

523

53.1 Permutations with Repeated Elements . . . . . . . . . . . . . . . . . . . . . . . 527


53.2 Circular Permutations . . . . . . . . . . . . . . . . . . . . . . . . . . . . . . . . . 533
53.3 Partial Permutations . . . . . . . . . . . . . . . . . . . . . . . . . . . . . . . . . . 536
53.4 Permutations with Restrictions . . . . . . . . . . . . . . . . . . . . . . . . . . . 537
54 How to Count: Combinations

539

54.1 Pascals Triangle . . . . . . . . . . . . . . . . . . . . . . . . . . . . . . . . . . . . 542


54.2 The Combination as Binomial Coefficient . . . . . . . . . . . . . . . . . . . . . 543
54.3 The Number of Subsets of a Set is 2n . . . . . . . . . . . . . . . . . . . . . . . . 545
55 Probability: Introduction

547

55.1 Mathematical Modelling . . . . . . . . . . . . . . . . . . . . . . . . . . . . . . . 547


55.2 The Experiment as a Model of Scenarios Involving Chance . . . . . . . . . . . 549
55.3 The Kolmogorov Axioms . . . . . . . . . . . . . . . . . . . . . . . . . . . . . . . 554
55.4 Implications of the Kolmogorov Axioms . . . . . . . . . . . . . . . . . . . . . . 555
56 Probability: Conditional Probability

557

56.1 The Conditional Probability Fallacy (CPF) . . . . . . . . . . . . . . . . . . . . 559


56.2 Two-Boys Problem (Fun, Optional) . . . . . . . . . . . . . . . . . . . . . . . . . 564
57 Probability: Independence

566

57.1 Warning: Not Everything is Independent . . . . . . . . . . . . . . . . . . . . . 571


57.2 Probability: Independence of Multiple Events . . . . . . . . . . . . . . . . . . . 573
Page 25, Table of Contents

www.EconsPhDTutor.com

58 Fun Probability Puzzles

574

58.1 The Monty Hall Problem . . . . . . . . . . . . . . . . . . . . . . . . . . . . . . . 574


58.2 The Birthday Problem . . . . . . . . . . . . . . . . . . . . . . . . . . . . . . . . 577
59 Random Variables: Introduction

578

59.1 A Random Variable vs. Its Observed Values . . . . . . . . . . . . . . . . . . . 579


59.2 X = k Denotes the Event {s S X(s) = k} . . . . . . . . . . . . . . . . . . . . 580
59.3 The Probability Distribution of a Random Variable . . . . . . . . . . . . . . . 581
59.4 Random Variables Are Simply Functions . . . . . . . . . . . . . . . . . . . . . 584
60 Random Variables: Independence

586

61 Random Variables: Expectation

589

61.1 The Expected Value of a Constant R.V. is Constant . . . . . . . . . . . . . . . 592


61.2 The Expectation Operator is Linear . . . . . . . . . . . . . . . . . . . . . . . . 594
62 Random Variables: Variance

596

62.1 The Variance of a Constant R.V. is 0 . . . . . . . . . . . . . . . . . . . . . . . . 603


62.2 Standard Deviation . . . . . . . . . . . . . . . . . . . . . . . . . . . . . . . . . . 604
62.3 The Variance Operator is Not Linear . . . . . . . . . . . . . . . . . . . . . . . . 605
62.4 The Definition of the Variance (Optional) . . . . . . . . . . . . . . . . . . . . . 607
63 The Coin-Flips Problem (Fun, Optional)

608

64 The Bernoulli Trial and the Bernoulli Distribution

609

64.1 Mean and Variance of the Bernoulli Random Variable . . . . . . . . . . . . . 611


65 The Binomial Distribution

612

65.1 Probability Distribution of the Binomial R.V. . . . . . . . . . . . . . . . . . . 614


65.2 The Mean and Variance of the Binomial Random Variable

Page 26, Table of Contents

. . . . . . . . . . 615

www.EconsPhDTutor.com

66 The Poisson Distribution

617

66.1 Formal Definition of the Poisson Random Variable . . . . . . . . . . . . . . . 619


66.2 When is the Poisson Random Variable an Appropriate Model? . . . . . . . . 620
66.3 The Mean and Variance of the Poisson Random Variable

. . . . . . . . . . . 623

66.4 The Poisson Distribution as an Approximation of the Binomial Distribution 624


66.5 The Sum of Two Independent Poisson R.V.s is a Poisson R.V. . . . . . . . . 627
67 The Continuous Uniform Distribution

630

67.1 The Continuous Uniform Distribution . . . . . . . . . . . . . . . . . . . . . . . 631


67.2 The Cumulative Distribution Function (CDF) . . . . . . . . . . . . . . . . . . 633
67.3 Important Digression: P (X k) = P (X < k) . . . . . . . . . . . . . . . . . . . 634
67.4 The Probability Density Function (PDF) . . . . . . . . . . . . . . . . . . . . . 635
68 The Normal Distribution

636

68.1 The Normal Distribution, in General . . . . . . . . . . . . . . . . . . . . . . . . 642


68.2 Sum of Independent Normal Random Variables . . . . . . . . . . . . . . . . . 651
68.3 The Central Limit Theorem and The Normal Approximation . . . . . . . . . 655
68.3.1 Normal Approximation to the Binomial Distribution
68.3.2 Normal Approximation to the Poisson Distribution
69 The CLT is Amazing (Optional)

. . . . . . . . . 658
. . . . . . . . . . 659
660

69.1 The Normal Distribution in Nature . . . . . . . . . . . . . . . . . . . . . . . . . 660


69.2 Illustrating the Central Limit Theorem (CLT) . . . . . . . . . . . . . . . . . . 664
69.3 Why Are So Many Things Normally Distributed? . . . . . . . . . . . . . . . . 668
69.4 Dont Assume That Everything is Normal . . . . . . . . . . . . . . . . . . . . . 669
70 Statistics: Introduction (Optional)

675

70.1 Probability vs. Statistics . . . . . . . . . . . . . . . . . . . . . . . . . . . . . . . 675


70.2 Objectivists vs Subjectivists . . . . . . . . . . . . . . . . . . . . . . . . . . . . . 676

Page 27, Table of Contents

www.EconsPhDTutor.com

71 Sampling

678

71.1 Population . . . . . . . . . . . . . . . . . . . . . . . . . . . . . . . . . . . . . . . . 678


71.2 Population Mean and Population Variance . . . . . . . . . . . . . . . . . . . . 679
71.3 Parameter . . . . . . . . . . . . . . . . . . . . . . . . . . . . . . . . . . . . . . . . 680
71.4 Distribution of a Population . . . . . . . . . . . . . . . . . . . . . . . . . . . . . 681
71.5 A Random Sample . . . . . . . . . . . . . . . . . . . . . . . . . . . . . . . . . . . 682
71.6 Sample Mean and Sample Variance . . . . . . . . . . . . . . . . . . . . . . . . . 684
71.7 Sample Mean and Sample Variance are Unbiased Estimators . . . . . . . . . 690
71.8 The Sample Mean is a Random Variable

. . . . . . . . . . . . . . . . . . . . . 693

71.9 The Distribution of the Sample Mean . . . . . . . . . . . . . . . . . . . . . . . 694


71.10Non-Random Samples . . . . . . . . . . . . . . . . . . . . . . . . . . . . . . . . . 695
71.11Stratified, Quota, and Systematic Sampling . . . . . . . . . . . . . . . . . . . . 696
72 Null Hypothesis Significance Testing (NHST)

701

72.1 One-Tailed vs Two-Tailed Tests . . . . . . . . . . . . . . . . . . . . . . . . . . . 705


72.2 The Abuse of NHST (Optional) . . . . . . . . . . . . . . . . . . . . . . . . . . . 708
72.3 Common Misinterpretations of the Margin of Error (Optional) . . . . . . . . 709
72.4 Critical Region and Critical Value . . . . . . . . . . . . . . . . . . . . . . . . . . 712
72.5 Testing
of
a
Population
Mean
2
(Small Sample, Normal Distribution, Known) . . . . . . . . . . . . . . . . . 714
72.6 Testing
of
a
Population
Mean
(Large Sample, Any Distribution, 2 Known) . . . . . . . . . . . . . . . . . . . 716
72.7 Testing
of
a
Population
Mean
2
(Large Sample, Any Distribution, Unknown) . . . . . . . . . . . . . . . . . 718
72.8 Testing
of
a
Population
Mean
2
(Small Sample, Normal Distribution, Unknown) . . . . . . . . . . . . . . . 720
72.9 Formulation of Hypotheses . . . . . . . . . . . . . . . . . . . . . . . . . . . . . . 724

Page 28, Table of Contents

www.EconsPhDTutor.com

73 Correlation and Linear Regression

725

73.1 Bivariate Data and Scatter Diagrams . . . . . . . . . . . . . . . . . . . . . . . . 725


73.2 Product Moment Correlation Coefficient (PMCC) . . . . . . . . . . . . . . . . 727
73.3 Correlation Does Not Imply Causation (Optional) . . . . . . . . . . . . . . . . 733
73.4 Linear Regression . . . . . . . . . . . . . . . . . . . . . . . . . . . . . . . . . . . 734
73.5 Ordinary Least Squares (OLS) . . . . . . . . . . . . . . . . . . . . . . . . . . . . 736
73.6 TI84 to Calculate the PMCC and the OLS Estimates . . . . . . . . . . . . . . 741
73.7 Interpolation and Extrapolation . . . . . . . . . . . . . . . . . . . . . . . . . . . 743
73.8 Transformations to Achieve Linearity . . . . . . . . . . . . . . . . . . . . . . . . 751
73.9 The Higher the PMCC, the Better the Model? . . . . . . . . . . . . . . . . . . 755

VII

Ten-Year Series

757

74 Past-Year Questions for Part I: Functions and Graphs

758

75 Past-Year Questions for Part II: Sequences and Series

769

76 Past-Year Questions for Part III: Vectors

779

77 Past-Year Questions for Part IV: Complex Numbers

788

78 Past-Year Questions for Part V: Calculus

794

79 Past-Year Questions for Part VI: Prob. and Stats.

823

VIII

854

Syllabuses and Lists of Formulae

80 Revised (9758) Syllabus

855

81 New List of Formulae (MF26)

876

82 Old (9740) Syllabus

889

Page 29, Table of Contents

www.EconsPhDTutor.com

83 Old List of Formulae (MF15)

908

IX

921

Appendices (Optional)

84 Appendices for Part I: Functions and Graphs

922

84.1 Sets . . . . . . . . . . . . . . . . . . . . . . . . . . . . . . . . . . . . . . . . . . . . 922


84.2 Functions . . . . . . . . . . . . . . . . . . . . . . . . . . . . . . . . . . . . . . . . 923
84.3 Reflection in a Line . . . . . . . . . . . . . . . . . . . . . . . . . . . . . . . . . . 924
84.4 The Hyperbola y =

bx + c
. . . . . . . . . . . . . . . . . . . . . . . . . . . . . . . 926
dx + e

84.5 The Hyperbola y =

ax2 + bx + c
. . . . . . . . . . . . . . . . . . . . . . . . . . . 927
dx + e

85 Appendices for Part II: Sequences and Series

929

86 Appendices for Part III: Vectors

930

86.1 Vectors in 2D . . . . . . . . . . . . . . . . . . . . . . . . . . . . . . . . . . . . . . 930


86.2 Scalar Product . . . . . . . . . . . . . . . . . . . . . . . . . . . . . . . . . . . . . 931
86.3 The Ratio Theorem . . . . . . . . . . . . . . . . . . . . . . . . . . . . . . . . . . 932
86.4 Vector Product . . . . . . . . . . . . . . . . . . . . . . . . . . . . . . . . . . . . . 933
86.5 2D Geometry . . . . . . . . . . . . . . . . . . . . . . . . . . . . . . . . . . . . . . 936
86.6 3D Geometry . . . . . . . . . . . . . . . . . . . . . . . . . . . . . . . . . . . . . . 937
87 Appendices for Part IV: Complex Numbers

943

88 Appendices for Part V: Calculus

946

88.1 Limits Formally Defined . . . . . . . . . . . . . . . . . . . . . . . . . . . . . . . 946


88.2 Left- and Right-Sided Limits . . . . . . . . . . . . . . . . . . . . . . . . . . . . . 948
88.3 Infinite Limits and Vertical Asymptotes . . . . . . . . . . . . . . . . . . . . . . 949
88.4 Limits at Infinity, Horizontal, and Oblique Asymptotes

. . . . . . . . . . . . 950

88.5 Limit Laws . . . . . . . . . . . . . . . . . . . . . . . . . . . . . . . . . . . . . . . 951


88.6 Continuity . . . . . . . . . . . . . . . . . . . . . . . . . . . . . . . . . . . . . . . . 954
Page 30, Table of Contents

www.EconsPhDTutor.com

88.7 Differentiation . . . . . . . . . . . . . . . . . . . . . . . . . . . . . . . . . . . . . 955


88.8 Differentiability Implies Continuity . . . . . . . . . . . . . . . . . . . . . . . . . 961
88.9 Maximum, Minimum, and Turning Points . . . . . . . . . . . . . . . . . . . . . 962
88.10Concavity and Inflexion Points . . . . . . . . . . . . . . . . . . . . . . . . . . . . 964
88.11Concavity and Inflexion Points with Differentiability . . . . . . . . . . . . . . 966
88.12Inverse Function Theorem . . . . . . . . . . . . . . . . . . . . . . . . . . . . . . 969
88.13Parametric Differentiation . . . . . . . . . . . . . . . . . . . . . . . . . . . . . . 970
88.14Maclaurin Series . . . . . . . . . . . . . . . . . . . . . . . . . . . . . . . . . . . . 971
88.15Product of Two Power Series . . . . . . . . . . . . . . . . . . . . . . . . . . . . . 975
88.16Composition of Two Functions . . . . . . . . . . . . . . . . . . . . . . . . . . . . 977
88.17The Fundamental Theorems of Calculus . . . . . . . . . . . . . . . . . . . . . . 978
88.18The Natural Logarithm and Eulers Number e . . . . . . . . . . . . . . . . . . 982
88.19Eulers Number . . . . . . . . . . . . . . . . . . . . . . . . . . . . . . . . . . . . . 984
89 Appendices for Part VI: Probability and Statistics

986

89.1 How to Count . . . . . . . . . . . . . . . . . . . . . . . . . . . . . . . . . . . . . . 986


89.2 Circular Permutations . . . . . . . . . . . . . . . . . . . . . . . . . . . . . . . . . 988
89.3 Probability . . . . . . . . . . . . . . . . . . . . . . . . . . . . . . . . . . . . . . . 989
89.4 Random Variables . . . . . . . . . . . . . . . . . . . . . . . . . . . . . . . . . . . 990
89.5 The Poisson Distribution . . . . . . . . . . . . . . . . . . . . . . . . . . . . . . . 994
89.6 The Normal Distribution . . . . . . . . . . . . . . . . . . . . . . . . . . . . . . . 996
89.7 Sampling . . . . . . . . . . . . . . . . . . . . . . . . . . . . . . . . . . . . . . . . . 999
89.8 Null Hypothesis Significance Testing . . . . . . . . . . . . . . . . . . . . . . . . 1001
89.9 Calculating the Margin of Error . . . . . . . . . . . . . . . . . . . . . . . . . . . 1002
89.10Correlation and Linear Regression . . . . . . . . . . . . . . . . . . . . . . . . . 1004
89.10.1 Deriving a Linear Model from the Barometric Formula . . . . . . . . . 1006

Answers to Exercises

Page 31, Table of Contents

1007

www.EconsPhDTutor.com

90 Answers to Exercises in Part I: Functions and Graphs

1008

90.1 Answers to Exercises in Ch. 1: Sets . . . . . . . . . . . . . . . . . . . . . . . . . 1008


90.2 Answers to Exercises in Ch. 2: Dividing by Zero . . . . . . . . . . . . . . . . . 1010
90.3 Answers to Exercises in Ch. 3: Functions . . . . . . . . . . . . . . . . . . . . . 1011
90.4 Answers to Exercises in Ch. 4. Graphs . . . . . . . . . . . . . . . . . . . . . . . 1017
90.5 Answers to Exercises in Ch. 5. Quick Revision . . . . . . . . . . . . . . . . . . 1020
90.6 Answers to Exercises in Ch. 6. Intercepts . . . . . . . . . . . . . . . . . . . . . 1022
90.7 Answers to Exercises in Ch. 7. Symmetry . . . . . . . . . . . . . . . . . . . . . 1023
90.8 Answers to Exercises in Ch. 8. Limits, Continuity, and Asymptotes . . . . . 1024
90.9 Answers to Exercises in Ch. 9. Differentiation . . . . . . . . . . . . . . . . . . 1025
90.10Answers to Exercises in Ch. 11. Stationary, Maximum, Minimum, and
Inflexion Points . . . . . . . . . . . . . . . . . . . . . . . . . . . . . . . . . . . . . 1027
90.11Answers to Exercises in Ch. 14. Quadratic Equations . . . . . . . . . . . . . . 1035
90.12Answers to Exercises in Ch. 15. Transformations . . . . . . . . . . . . . . . . 1036
90.13Answers to Exercises in Ch. 16: Conic Sections . . . . . . . . . . . . . . . . . 1038
90.14Answers to Exercises in Ch. 17. Simple Parametric Equations . . . . . . . . . 1050
90.15Answers to Exercises in Ch. 18: Equations and Inequalities . . . . . . . . . . 1055
91 Answers to Exercises in Part II: Sequences and Series

1068

91.1 Answers for Ch. 19: Finite Sequences . . . . . . . . . . . . . . . . . . . . . . . 1068


91.2 Answers for Ch. 20: Infinite Sequences . . . . . . . . . . . . . . . . . . . . . . . 1070
91.3 Answers for Ch. 22: Summation . . . . . . . . . . . . . . . . . . . . . . . . . . . 1071
91.4 Answers for Ch. 23: Arithmetic Sequences and Series . . . . . . . . . . . . . . 1072
91.5 Answers for Ch. 24: Geometric Sequences and Series . . . . . . . . . . . . . . 1073
91.6 Answers for Ch. 25: Proof by Induction . . . . . . . . . . . . . . . . . . . . . . 1074

Page 32, Table of Contents

www.EconsPhDTutor.com

92 Answers to Exercises in Part III: Vectors

1077

92.1 Answers for Ch. 27: Vectors in 2D . . . . . . . . . . . . . . . . . . . . . . . . . 1077


92.2 Answers for Ch. 29: Vectors in 3D . . . . . . . . . . . . . . . . . . . . . . . . . 1082
92.3 Answers for Ch. 30: Vector Product . . . . . . . . . . . . . . . . . . . . . . . . 1085
92.4 Answers for Ch. 31: Lines . . . . . . . . . . . . . . . . . . . . . . . . . . . . . . 1086
92.5 Answers for Ch. 32: Planes . . . . . . . . . . . . . . . . . . . . . . . . . . . . . . 1089
92.6 Answers for Ch. 33: Distances . . . . . . . . . . . . . . . . . . . . . . . . . . . . 1091
92.7 Answers for Ch. 34: Angles . . . . . . . . . . . . . . . . . . . . . . . . . . . . . 1098
92.8 Answers for Ch. 35: Relationships between Lines and Planes . . . . . . . . . 1101
93 Answers to Exercises in Part IV: Complex Numbers

1105

93.1 Answers for Ch. 36: Introduction to Complex Numbers . . . . . . . . . . . . . 1105


93.2 Answers for Ch. 37: Basic Arithmetic of Complex Numbers . . . . . . . . . . 1106
93.3 Answers for Ch. 38: Solving Polynomial Equations . . . . . . . . . . . . . . . 1108
93.4 Answers for Ch. 39: The Argand Diagram . . . . . . . . . . . . . . . . . . . . 1111
93.5 Answers for Ch. 40: More Arithmetic of Complex Numbers . . . . . . . . . . 1114
93.6 Answers for Ch. 41: Geometry of Complex Numbers . . . . . . . . . . . . . . 1117
93.7 Answers for Ch. 42: Loci Involving Cartesian Equations . . . . . . . . . . . . 1118
93.8 Answers for Ch. 43: Loci Involving Complex Equations . . . . . . . . . . . . . 1122
93.9 Answers for Ch. 44: De Moivres Theorem . . . . . . . . . . . . . . . . . . . . 1125
94 Answers to Exercises in Part V: Calculus

1128

94.1 Answers for Ch. 45: Solving Problems Involving Differentiation . . . . . . . . 1128
94.2 Answers for Ch. 46: Maclaurin Series . . . . . . . . . . . . . . . . . . . . . . . 1130
94.3 Answers for Ch. 47: The Indefinite Integral . . . . . . . . . . . . . . . . . . . . 1133
94.4 Answers for Ch. 48: Integration Techniques . . . . . . . . . . . . . . . . . . . . 1134
94.5 Answers for Ch. 49: The Fundamental Theorems of Calculus . . . . . . . . . 1143
94.6 Answers for Ch. 50: Definite Integrals . . . . . . . . . . . . . . . . . . . . . . . 1144
94.7 Answers for Ch. 51: Differential Equations . . . . . . . . . . . . . . . . . . . . 1147
Page 33, Table of Contents

www.EconsPhDTutor.com

95 Answers to Exercises in Part VI: Probability and Statistics

1152

95.1 Answers for Ch. 52: How to Count: Four Principles . . . . . . . . . . . . . . . 1152
95.2 Answers for Ch. 53: How to Count: Permutations . . . . . . . . . . . . . . . . 1155
95.3 Answers for Ch. 54: How to Count: Combinations . . . . . . . . . . . . . . . . 1157
95.4 Answers for Ch. 55: Probability: Introduction . . . . . . . . . . . . . . . . . . 1160
95.5 Answers for Ch. 56: Conditional Probability . . . . . . . . . . . . . . . . . . . 1164
95.6 Answers for Ch. 57: Probability: Independence . . . . . . . . . . . . . . . . . 1165
95.7 Answers for Ch. 59: Random Variables: Introduction . . . . . . . . . . . . . . 1166
95.8 Answers for Ch. 60: Random Variables: Independence . . . . . . . . . . . . . 1170
95.9 Answers for Ch. 61: Random Variables: Expectation . . . . . . . . . . . . . . 1171
95.10Answers for Ch. 62: Random Variables: Variance . . . . . . . . . . . . . . . . 1173
95.11Answers for Ch. 64: Bernoulli Trial and Distribution . . . . . . . . . . . . . . 1174
95.12Answers for Ch. 65: Binomial Distribution . . . . . . . . . . . . . . . . . . . . 1175
95.13Answers for Ch. 66: Poisson Distribution . . . . . . . . . . . . . . . . . . . . . 1176
95.14Answers for Ch. 67: Continuous Uniform Distribution . . . . . . . . . . . . . 1178
95.15Answers for Ch. 68: Normal Distribution . . . . . . . . . . . . . . . . . . . . . 1179
95.16Answers for Ch. 71: Sampling . . . . . . . . . . . . . . . . . . . . . . . . . . . . 1186
95.17Answers for Ch. 72: Null Hypothesis Significance Testing . . . . . . . . . . . 1189
95.18Answers for Ch. 73: Correlation and Linear Regression . . . . . . . . . . . . . 1194
96 Answers to Exercises in Part VII (2006-2015 A-Level Exams) 1198
96.1 Answers for Ch. 74: Functions and Graphs . . . . . . . . . . . . . . . . . . . . 1198
96.2 Answers for Ch. 75: Sequences and Series . . . . . . . . . . . . . . . . . . . . . 1225
96.3 Answers for Ch. 76: Vectors . . . . . . . . . . . . . . . . . . . . . . . . . . . . . 1255
96.4 Answers for Ch. 77: Complex Numbers . . . . . . . . . . . . . . . . . . . . . . 1269
96.5 Answers for Ch. 78: Calculus . . . . . . . . . . . . . . . . . . . . . . . . . . . . 1292
96.6 Answers for Ch. 79: Probability and Statistics . . . . . . . . . . . . . . . . . . 1345

Page 34, Table of Contents

www.EconsPhDTutor.com

Part I

Functions and Graphs

Page 35, Table of Contents

www.EconsPhDTutor.com

Sets

The glory of [maths] is its complete irrelevance to our lives. Thats why its so fun!
Paul Lockhart (2009, A Mathematicians Lament, p. 38).

I have never done anything useful. No discovery of mine has made, or is likely to make,
directly or indirectly, for good or ill, the least difference to the amenity of the world.
- G.H. Hardy (1940 [1967], A Mathematicians Apology, p. 150).

The set is the basic building block of mathematics. Informally, a set is a container that
usually has some objects in it, but can sometimes also be empty.
Each object in a set is called an element (of that set).
Example 1. Let A = {3, 2 , Clementi Mall, Love, the colour green}.
Observations:
The name of a set is often an upper-case letter; in this case, it is A.
Mathematical punctuation marks called braces {} are used to denote a set. Listed within
these braces are the elements of the set.
Elements of the set are separated by commas (,). This mathematical punctuation mark
means and.
Thus, {3, 2 , Clementi Mall, Love, the colour green} is the set consisting of five elements, namely 3 and 2 and Clementi Mall and Love and the colour green.
Elements in a set can be almost anything whatsoever! In this example, the
elements included a building (Clementi Mall), an abstract notion (Love), and even a
colour (green). The elements of a set can even be another set! But dont worry, in the
context of A-level maths, the elements of a set will almost always be numbers.
When we talk about a set, we refer to both the container itself and all the objects in
it.

Exercise 1. B is the set of the first 7 positive integers. Write down B in set notation.
(Answer on p. 1008.)
Exercise 2. C is the set of even prime numbers. Write down C in set notation. (Answer
on p. 1008.)

Page 36, Table of Contents

www.EconsPhDTutor.com

1.1

In and Not In

The mathematical punctuation mark means is in, while means is not in.
Example 2. Let B = {1, 2, 3, 4, 5, 6, 7}. Then 1 B, 2 B, 3 B, etc. You can read these
statements aloud as 1 is in B, 2 is in B, 3 is in B, etc.
We can also write 1, 2, 3 B (1, 2, and 3 are in B).
Also, 8 B, 9 B, 10 B, etc. (8 is not in B, 9 is not in B, 10 is not in B, etc.).
We can also write 8, 9, 10 B (8, 9, and 10 are not in B).
Example 3. Cow {Cow, Chicken} reads aloud as Cow is in the set consisting of Cow
and Chicken.
Cow, Chicken {Cow, Chicken} reads aloud as Cow and Chicken are in the set consisting
of Cow and Chicken.

Page 37, Table of Contents

www.EconsPhDTutor.com

1.2

Greater than >, Less Than <, Positive > 0, and Negative < 0

In this textbook:
Greater than means strictly greater than (>). So I wont bother saying strictly,
unless its something I want to emphasise.
Less than means strictly less than (<).
If I want to say greater than or equal to () or smaller than or equal to (), Ill
say exactly that.
Positive means greater than zero (> 0).
Negative means less than zero (< 0).
Non-negative means greater than or equal to zero ( 0).
Non-positive means less than or equal to zero ( 0).
0 is neither positive nor negative. Instead, 0 is both non-negative and non-positive.

Page 38, Table of Contents

www.EconsPhDTutor.com

1.3

Types of Numbers

The following taxonomy lists the several types of numbers youll encounter in this textbook.

Complex
Numbers

Real
Numbers

Rational
Numbers

Imaginary
Numbers

Irrational
Numbers

Integers
NonIntegers

Well study imaginary numbers only later on in Part IV of this textbook. For now, all
numbers well consider are real numbers (or reals). We wont define what real numbers
are. Instead, well simply assume (like in secondary school) that everyone knows what
real numbers are.
Infinity () and negative infinity () are NOT numbers. Informally, is the thing
that is greater than any real number. Similarly, is the thing that is smaller than any
real number. I repeat: INFINITY IS NOT A NUMBER.13
So what exactly are real numbers, infinity, and negative infinity? This is actually a fascinating question that mathematicians were able to answer satisfactorily only from the 19th
century, but is beyond the scope of the A-levels.
Definition 1. An integer is any one of these real numbers: . . . , 3, 2, 1, 0, 1, 2, 3, . . .

Definition 2. A rational number (or simply rationals) is any real number that can be
expressed as the ratio of two integers. An irrational number (or simply irrationals) is any
other real number.

Example 4. The number 16 is an integer, a rational, and a real.

Example 5. The number 1.87 is a rational and a real, but it is not an integer.

Example 6. The number 3.14159 is an irrational and a real, but it is neither an integer
nor a rational.
13

Actually, the truth is somewhat more complicated. Under certain special contexts in more advanced mathematics, infinity
is treated as a number. But in this textbook, Ill simply keep it simple and insist that infinity is not a number.

Page 39, Table of Contents

www.EconsPhDTutor.com

1.4

The Order of the Elements Doesnt Matter

The order in which we write out the elements of the set does not matter:
Definition 3. Two sets are equal (or identical) if both sets contain exactly the same elements.

Example 7. There are at least six equivalent ways to write the set of the 3 smallest positive
even numbers: {2, 4, 6} = {2, 6, 4} = {4, 2, 6} = {4, 6, 2} = {6, 2, 4} = {6, 4, 2}.
Example 8. {Cow, Chicken} = {Chicken, Cow}.

Page 40, Table of Contents

www.EconsPhDTutor.com

1.5

Repeated Elements Dont Count

Repeated elements are simply ignored.


Example 9. The set of the 3 smallest positive even numbers can be written as {2, 4, 6}. It
can also be written as: {2, 2, 4, 6} or {2, 6, 6, 6, 4, 4}. Repeated elements are simply ignored.
The notation n({2, 4, 6}) denotes the number of elements in the set of the first 3 even numbers. Hence, n({2, 4, 6}) = 3. And we also have n({2, 2, 4, 6}) = 3 and n({2, 6, 6, 6, 4, 4}) = 3.
Example 10. {Cow, Chicken} = {Cow, Cow, Chicken} = {Chicken, Cow, Chicken}. And
n({Cow, Chicken}) = n({Cow, Cow, Chicken}) = n({Chicken, Cow, Chicken}) = 2.

Note that more commonly, the number of elements in the set A is written as A. But for
some reason, the A-level syllabus instead uses the notation n(A), so thats what well use.

Exercise 3. W = {Apple, Apple, Apple, Banana, Banana, Apple}. What is n(W )? (Answer on p. 1008.)
Exercise 4. C is the set of even prime numbers. What is n(C)? (Answer on p. 1008.)

Page 41, Table of Contents

www.EconsPhDTutor.com

1.6

Ellipsis . . . Means Continue in the Obvious Fashion

The mathematical punctuation mark . . . is called the ellipsis and means continue in the
obvious fashion.
Example 11. D is the set of all odd positive integers smaller than 100. So in set notation,
we can write D = {1, 3, 5, 9, 11, . . . , 99}.

Example 12. T is the set of all negative integers greater than 100. So in set notation, we
can write T = {99, 98, 97, . . . , 2, 1}.

What is obvious to one person might not be obvious to another. So only use the ellipsis
when youre confident it will be obvious to your reader! And never be shy to write a few
more of the sets elements (as I did with the sets above)!

Exercise 5. Let D and T be as in the above two examples. What are n(D) and n(T )?
(Answer on p. 1008.)

Page 42, Table of Contents

www.EconsPhDTutor.com

1.7

Sets can be Finite or Infinite

Example 13. Z+ is the set of all positive integers. So, Z+ = {1, 2, 3, . . . }. And since Z+ is
infinite, we write n(Z+ ) = .
Example 14. Z is the set of all integers. So, Z = {. . . , 3, 2, 1, 0, 1, 2, 3, . . . }. And since
Z is infinite, we write n(Z) = .

Obviously, for an infinite set, we cannot explicitly list out all of its elements. So well often
use ellipses to help out, as we did in the above examples. Alternatively, we can use interval
notation or set-builder notation, which well learn about shortly.

Exercise 6. H is the set of all prime numbers. Write down H in set notation. (Answer on
p. 1008.)

Page 43, Table of Contents

www.EconsPhDTutor.com

1.8

Special Names of Sets: Z, Q, R, and C

The following sets are so common that they have special symbols:
1. Z = {. . . , 3, 2, 1, 0, 1, 2, 3, . . . } is the set of all integers. (Z is for Zahl, German for
number.)
2. Q is the set of all rational numbers. (Q is for quoziente, Italian for quotient.)
3. R is the set of all real numbers.
4. C is the set of all complex numbers. (To be studied only in Part IV of this textbook.)
To create a new set that contains only the positive (or negative) elements of the old set,
append a superscript plus (+ ) or minus ( ) to the name of a set:
1. Z+ = {1, 2, 3, . . . } is the set of all positive integers. Z = {. . . , 3, 2, 1} is the set of all
negative integers.
2. Q+ is the set of all positive rational numbers. Q is the set of all negative rational
numbers.
3. R+ is the set of all positive real numbers. R is the set of all negative real numbers.
As well learn later, there is no such thing as a positive or negative complex number. Hence,
there is no such set named C+ or C .
To add the number 0 to a set, append a subscript zero (0 ) to its name:
Example 15. The set A = {3, 2 , Clementi Mall, Love, the colour green}. And so the set
A0 = {3, 2 , Clementi Mall, Love, the colour green, 0}.
Example 16. The set B = {1, 2, 3, 4, 5, 6, 7}. And so the set B0 = {0, 1, 2, 3, 4, 5, 6, 7}.

Adding both a superscript + and a subscript 0 to the name of a set creates a new set that
contains all positive elements of the old set and in addition the number 0.
Similarly, adding both a superscript and a subscript 0 to the name of a set creates a new
set that contains all negative elements of the old set and in addition the number 0.
Example 17. If V = {2, 1, 3, 4}, then V + = {3, 4}, V = {2, 1}, V0+ = {0, 3, 4}, and
V0 = {2, 1, 0}.

Exercise 7. If U = {1, 0, 2}, then what are U + , U , U0 , U0+ , and U0 ? (Answer on p. 1008.)

Page 44, Table of Contents

www.EconsPhDTutor.com

1.9

Special Names of Sets: Intervals

Here are four new mathematical punctuation marks:


The left-parenthesis: (
The right-parenthesis: )

The left-bracket: [
The right-bracket: ]

Together, () are called parentheses and [] are called brackets.


An interval is a (usually infinite) set of real numbers. It is written using parentheses
and/or brackets. Let a and b be real numbers where b a. Then:
1. (a, b) is the set of all real numbers that are greater than a and smaller than b. Such a
set is also called an open interval.
Example 18. The set I = (0, 3) denotes
the set of all real numbers that are greater than
0 and smaller than 3. So for example, 2 1.41 I, but 0, 3 I.

2. [a, b] is the set of all real numbers that are greater than or equal to a and smaller than
or equal to b. Such a set is also called an closed interval.
Example 19. The set J = [0, 3] denotes the set of all real numbers that are
greater than
or equal to 0 and smaller than or equal to 3. So for example, the numbers 0, 2, 3 J.

3. (a, b] is the set of all real numbers that are greater than a and smaller than or equal to
b. Such a set is also called a half-open interval or a half-closed interval.
Example 20. The set K = (0, 3] denotes the set of all real numbers
that are greater than

0 and smaller than or equal to 3. So for example, the numbers 2, 3 K, but 0 K.

4. [a, b) is the set of all real numbers that are greater than or equal to a and smaller than
b. Such a set is also called a half-open interval or a half-closed interval.
Example 21. The set L = [0, 3) denotes the set of all real numbers that are greater
than

or equal to 0 and smaller than or equal to 3. So for example, the numbers 0, 2 L, but
3 L.

Page 45, Table of Contents

www.EconsPhDTutor.com

Exercise 8. How many elements does the set Z = [1, 1] contain? (Answer on p. 1008.)
Exercise 9. How many elements does the set Y = (1, 1) contain? (Answer on p. 1008.)
Exercise 10. How many elements does the set X = (1, 1.01) contain? (Answer on p. 1008.)
Exercise 11. Write down R, R+ , R+0 , R , and R0 in interval notation. (Answer on p.
1008.)

Page 46, Table of Contents

www.EconsPhDTutor.com

1.10

Special Names of Sets: The Empty Set

The empty set is literally the set that contains no elements. Hence the name!
Definition 4. The empty set is the set {}. It can also be denoted .
Example 22. In 2016, the set of all Singapore Ministers who are younger than 30 is {} or
. This means that there is no Singapore Minister who is younger than 30.
Example 23. The set of all even prime numbers greater than 2 is {} or . This means
that there is no even prime number that is greater than 2.

Example 24. The set of numbers that are greater than 4 and smaller than 4 is {} or .
This means that there is no number that is simultaneously greater than 4 and smaller than
4.
As already mentioned, in this textbook (and also for the A-levels), the elements in a set will
almost always be numbers. But in general, the elements of a set can be (nearly) anything
whatsoever. In other words, a set really and simply is a container that can contain
(nearly) anything whatsoever.
Indeed, the elements of a set can be other sets, including even the empty set! Here are two
examples to illustrate:
Example 25. The set {} is not the same as the set . The former is a set containing a
single element, namely the empty set. The latter is the empty set. It is perhaps clearer if
we rewrite them as
{} = {{}} and = {} .
Now we clearly see that {{}} {}.
Note that the set {} = {{}} is certainly not empty, because it contains a single element
(namely the empty set).
Example 26. The set {, 1, {}} is the set containing three elements, namely the empty
set, the number 1, and a set containing the empty set.

Page 47, Table of Contents

www.EconsPhDTutor.com

1.11

Subset Of

Definition 5. A is a subset of B (written as A B) if every element in A is also an element


in B.
Not surprisingly, A / B denotes that A is not a subset of B.
Example 27. Let M = {1, 2}, N = {1, 2, 3}, and O = {1, 2, 4, 5}. Then M N , but N M .
Also, M O, but O M . Further, N O and O N .

Exercise 12. State whether Z, Q, and R are subsets of each other. (Answer on p. 1008.)
Exercise 13. True or false: The set of currently-serving Singapore Prime Ministers is a
subset of the set of currently-serving Singapore Ministers. (Answer on p. 1008.)

The next fact is useful for showing that two sets are equal.
Fact 1. Two sets are subsets of each other They are identical.

Proof. Optional, see p. 922 in the Appendices.


The symbol stands for is equivalent to or if and only if.
The above claim may be decomposed into two separate claims:
1. Two sets are subsets of each other they are identical. (The symbol stands
for implies or only if.)
2. Two sets are subsets of each other they are identical. (The symbol stands
for is implied by or if.)
Note importantly that A B is different from its converse B A. For example,
x > 10 x > 3, but it is certainly not the case that x > 3 x > 10.

Page 48, Table of Contents

www.EconsPhDTutor.com

1.12

Proper Subset Of

Definition 6. A is a proper subset of B (written as A B) if A B but A B.


Not surprisingly, A / B denotes that A is not a proper subset of B.
Example 28. Let M = {1, 2}, N = {1, 2, 3}, O = {1, 2, 4, 5}, and P = {1, 2, 3}. Then
M N, O, P and M N, O, P . In contrast, N P , but N / P ; this is because N = P .

Exercise 14. Is the set of all squares (call it S) a proper subset of the set of all rectangles
(call it R)? (Answer on p. 1008.)
Exercise 15. Does A B imply that A B? (Answer on p. 1009.)
Exercise 16. Does A B imply that A B? (Answer on p. 1009.)
Exercise 17. True or false statement: If A is a subset of B, then A is either a proper
subset of or is equal to B. (Answer on p. 1009.)

Remark 1. The official A-level syllabus uses the symbol to mean subset of and to
mean proper subset of. So this is what well use in this textbook.
However, confusingly enough, many writers use the symbol to mean subset of and to
mean proper subset of. We will not follow such practice in this textbook. Just to let you
know, in case you get confused while reading other mathematical texts!

Page 49, Table of Contents

www.EconsPhDTutor.com

1.13

Union

Definition 7. The union of the sets A and B (denoted A B) is the set of elements that
are either in A OR B.
Tip: U for Union.
Example 29. Let T = {1, 2}, U = {3, 4}, and V = {1, 2, 3}. Then T U = {1, 2, 3, 4},
T V = {1, 2, 3}, and U V = {1, 2, 3, 4}. And T U V = {1, 2, 3, 4}.

Exercise 18. Rewrite each of the following sets more simply: (a) [1, 2] [2, 3]. (b)
(, 3) [16, 7). (c) {0} Z+ ? (Answer on p. 1009.)
Exercise 19. What is the union of the set of squares (S) and the set of rectangles (R)?
(Answer on p. 1009.)
Exercise 20. What is the union of the set of rationals (Q) and the set of irrationals?
(Answer on p. 1009.)

Page 50, Table of Contents

www.EconsPhDTutor.com

1.14

Intersection

Definition 8. The intersection of the sets A and B (denoted A B) is the set of elements
that are in A AND B.

Definition 9. Two sets intersect if their intersection contains at least one element (i.e.
A B ).

Definition 10. Two sets are mutually exclusive or disjoint if their intersection is empty
(i.e. A B = ).
Example 30. Let T = {1, 2}, U = {3, 4}, and V = {1, 2, 3}. Then T U = , T V = {1, 2},
and U V = {3}. And T U V = .

Exercise 21. Rewrite each of the following sets more simply: (a) (4, 7] (6, 9). (b) [1, 2]
[5, 6]. (c) (, 3) (16, 7). (Answer on p. 1009.)
Exercise 22. What is the intersection of the set of squares (S) and the set of rectangles
(R)? (Answer on p. 1009.)
Exercise 23. What is the intersection of the set of rationals (Q) and the set of irrationals?
(Answer on p. 1009.)

Page 51, Table of Contents

www.EconsPhDTutor.com

1.15

Set Minus /

The set minus (sometimes also called set difference) operator is very convenient. Sadly,
it is not in the A-level syllabus and so Ill avoid using it in this textbook. Nonetheless, its
worth a quick mention.
Definition 11. A set minus B (denoted A/B or A B) is the set that contains every
element in A that is not also in B.
Example 31. Let T = {1, 2}, U = {3, 4}, and V = {1, 2, 3}. Then T /U = T , T /V = , and
U /V = {4}.

Page 52, Table of Contents

www.EconsPhDTutor.com

1.16

Set Complement A

Definition 12. The set complement of A (denoted A or Ac ) is the set of all elements that
are not in A.
Example 32. Consider the set of positive integers. Let A = {2, 4, 6, 8, 10, . . . }. Then in
this context, A = {1, 3, 5, 7, 9, 11, . . . }.
Example 33. Consider the set of all reals. Let A = R+ . Then in this context, A = R0 .
Example 34. Consider the roll of a die. The set of desired outcomes was A = {1, 6}.
Unfortunately, no desired outcome occurred. In other words, the actual outcome was an
element in the set A = {2, 3, 4, 5}.

Page 53, Table of Contents

www.EconsPhDTutor.com

1.17

Set-Builder Notation

Set-builder notation is an alternative method of writing down sets. In the current context,
the mathematical punctuation mark colon will mean such that.
Example 35. The set {x R x > 0} contains all x R such that x > 0. In words, this set
contains all real numbers that are positive.

What comes after the colon are the conditions or criteria that x must satisfy, in order to
qualify as a member of the set. Our sets will usually contain only numbers, but heres an
example to show you how we can write down one particular set of musical artists.
Example 36. The set {x x is an artist that has had a US Billboard Hot 100 #1 Single}
contains all the artists who have ever had a US Billboard Hot 100 #1 Single.

It will however be more typical for our sets to be sets such as these:
Example 37. {x R x > 0} = R+ , Q+ = {x Q x > 0}, Z+ = {x Z x > 0},
R+0 = {x R x 0}, Q+0 = {x Q x 0}, and Z+0 = {x Z x 0}.

Remark 2. We use the colon but some writers use instead the pipe .
Exercise 24. Write down R , Q , Z , R0 , Q0 , and Z0 in set-builder notation. (Answer
on p. 1009.)
Exercise 25. Write down (a, b), [a, b], (a, b], and [a, b) in set-builder notation. (Answer
on p. 1009.)
Exercise 26. Let X = {x x is a living current or former Prime Minister of Singapore}.
Write down the set X so that all its elements are explicitly stated. (Answer on p. 1009.)
Exercise 27. Rewrite
each of the following sets in set-builder notation: (a) (, 3)

(5, ). (b) (, 2] (e, ) (, ). (c) (, 3) (0, 7). (Answer on p. 1009.)

Page 54, Table of Contents

www.EconsPhDTutor.com

Dividing By Zero

This very brief chapter is to warn you against making a common mistake dividing by
0. Students have little trouble avoiding this mistake if the divisor is obviously a big fat 0.
Instead, students usually make this mistake when the divisor is an unknown constant or
variable that might be 0.
Example 38. Find the values of x for which x(x 1) = (2x 2)(x 1).
Heres the wrong solution: Divide both sides by x 1 to get x = 2x 2. So x = 2.
Heres the correct solution: Case #1. Suppose x 1 = 0. Then the given equation is
satisfied. So x = 1 is one possible value for which x(x 1) = (2x 1)(x 1). Case #2.
Now suppose x 1 0. So we can divide both sides by x 1 to get x = 2x 2. So x = 2.
Conclusion. The two possible values of x for which x(x 1) = (2x 1)(x 1) are x = 1 and
x = 2.
Moral of the story. Whenever you divide by a certain quantity, make sure its non-zero.
If youre not sure whether it equals 0, then break up your analysis into two cases, as was
done in the above example: Case #1 the quantity equals 0 (and see what happens
in this case); Case #2 the quantity is non-zero (in which case you can go ahead and
divide).

By the way, lets take this opportunity to clear up another popular misconception You
1
1
may have heard that = . This is wrong. . Instead, any non-zero number divided
0
0
by 0 is undefined.14 Undefined is the mathematicians way of saying, You havent told
me what you are talking about. So what you are saying is meaningless.15
Exercise 28. Whats wrong with this proof that 1 = 0? (Answer on p. 1010.)
1. Let x, y be positive numbers such that x = y.
2. Square both sides: x2 = y 2 .
3. Rearrange: x2 y 2 = 0
4. Factorise: (x y)(x + y) = 0.

5. Divide both sides by x y to get x + y = 0.


6. Since x = y, sub y = x into the above equation to get 2x = 0.
7. Divide both sides by 2x to get 1 = 0.
Once again, the truth is actually somewhat more complicated. Under certain special contexts in more advanced mathe1
1
matics, is well-defined. But in this textbook, Ill simply keep it simple and insist that is undefined.
0
0
0
15
On the other hand,
is indeterminate, which means that its typically undefined, but can sometimes be defined under
0
certain circumstances.

14

Page 55, Table of Contents

www.EconsPhDTutor.com

Functions

Undoubtedly the most important concept in all of mathematics is that of a function in


almost every branch of modern mathematics functions turn out to be the central objects of
investigation.
- Michael Spivak (1994 [2006], Calculus, p. 39).
You are probably familiar from secondary school with such statements as: Let f (x) = x + 8
be a function. Strictly speaking, this is not the correct way of describing a function.
Here is a more precise definition of a function.16 A function consists of three pieces:
1. A set called the domain;
2. A set called the codomain; and
3. A mapping rule (or simply mapping or simply rule) which specifies how each and every
element in the domain is mapped (or assigned) to one (and exactly one) element in the
codomain.
Remark 3. The codomain is not the same thing as the range. Well learn about the range
only in the next section.
Altogether then, a function simply maps (or assigns) each element in the domain to one
(and exactly one) element in the codomain.
Example 39. Let f be the function whose ...
Domain is the set {Cow, Chicken};
Codomain is the set {Produces eggs, Produces milk, Guards the home}; and
Mapping rule is, informally, match the animal to its role.
According to the mapping rule, Cow (in the domain) is mapped to Produces milk
(in the codomain) and Chicken (in the domain) is mapped to Produces eggs (in the
codomain). Every element in the domain is mapped to exactly one element in the codomain.
This is indeed a function, because it has a domain, codomain, and a correctly-specified
mapping rule.

16

This definition is still informal. See Definition 135 in the Appendices for the exact, formal definition (optional).

Page 56, Table of Contents

www.EconsPhDTutor.com

Example 40. Let f be the function whose ...


Domain is the set {1, 2};
Codomain is the set {1, 2, 3, 4, 5}; and
Mapping rule is, informally, multiply by 2.
According to the mapping rule, 1 (in the domain) is mapped to 2 (in the codomain)
and 2 (in the domain) is mapped to 4 (in the codomain). Every element in the domain
is mapped to exactly one element in the codomain.
This is indeed a function, because it has a domain, codomain, and a correctly-specified
mapping rule.

Example 41. Let f be the function whose ...


Domain is the set R;
Codomain is the set R; and
Mapping rule is, informally, round off to the nearest integer, where half-integers are
rounded up
According to the mapping rule, 3 (in the domain) is mapped to 3 (in the codomain),
3.14159 (in the domain) is mapped to 3 (in the codomain), 3.5 (in the domain) is
mapped to 4 (in the codomain), and 3.88 (in the domain) is mapped to 4 (in the
codomain). Every element in the domain is mapped to exactly one element in the codomain.
This is indeed a function, because it has a domain, codomain, and a correctly-specified
mapping rule.

Page 57, Table of Contents

www.EconsPhDTutor.com

3.1

Formal Mathematical Notation for Functions

In general, the correct way to describe a function is:


The function f D C is defined by f x f (x) for all x D.
Or alternatively:
The function f D C is defined by f (x) = ... for all x D.
This says that the functions name is f , its domain is D, and its codomain is C. The last
bit f x f (x) is the mapping rule and this mapping rule applies to all x D (all
elements in the domain).
To save ourselves a bit of writing, if its clear from the context that were talking about the
function f , then well omit f from the front of the mapping rule. Also, if the mapping
rule applies universally to all elements of the domain, then we also omit the for all x D
at the end.
Altogether then, we will often simply write:
The function f D C is defined by x f (x).
We will sometimes also denote the domain and codomain of f by Dom(f ) and Cod(f ).

Page 58, Table of Contents

www.EconsPhDTutor.com

Example 40 (revisited). In formal mathematical notation, we write: the function


f {1, 2} {1, 2, 3, 4, 5} is defined by x 2x. Or alternatively, the function f {1, 2}
{1, 2, 3, 4, 5} is defined by f (x) = 2x.
This says that the function has:
Name f ;
Domain {1, 2};
Codomain {1, 2, 3, 4, 5}; and
Mapping rule: Map every element x in the domain to the element 2x in the codomain.

Lets examine this formal notation a little more closely.


In the context of set-builder notation (Section 1.17), the mathematical punctuation mark
colon stood for such that. However, in the context of functions, the colon stands
instead for from. Unfortunately there are only so many symbols and punctuation marks,
so invariably some symbols will have to play more than one role!
The mathematical punctuation mark (right arrow) simply stands for to.
Altogether then, f D C reads as f is the function from domain D to domain C.
The mathematical punctuation mark stands for maps to. Hence, x f (x) reads
as x is mapped to f (x).

Page 59, Table of Contents

www.EconsPhDTutor.com

Example 39 (revisited). In formal mathematical notation, we write: the function f


{Cow, Chicken} {Produces eggs, Produces milk, Guards the home} is defined by Cow
Produces milk and Chicken Produces eggs. Or alternatively, the function f {Cow,
Chicken} {Produces eggs, Produces milk, Guards the home} is defined by f (Cow) =
Produces milk and f (Chicken) = Produces eggs.
This says that the function has
Name f ;
Domain {Cow, Chicken};
Codomain {Produces eggs, Produces milk, Guards the home}; and
Mapping rule: Map the element Cow in the domain to the element Produces milk in
the codomain and the element Chicken in the domain to the element Produces eggs
in the codomain.

Example 41 (revisited). In formal mathematical notation, we write: the function


f R R is defined by x Integer closest to x.
This says that the function has
Name f ;
Domain R;
Codomain R;
Mapping rule: Map every element x in the domain to the closest integer in the codomain.

Students frequently believe that f (x) denotes a function. This is wrong.


f and f (x) refer to two different things.
f denotes a function. f (x) denotes the value of f at x.
This may seem like an excessively pedantic distinction. But maths is precise and pedantic.
In maths, what we mean is precisely what we say and what we say is precisely what we
mean. There is never any room for ambiguity or alternative interpretations.
More examples:

Page 60, Table of Contents

www.EconsPhDTutor.com

Example 42. The function f [0, 1] R is defined by x 3x + 4. Or alternatively, The


function f [0, 1] R is defined by f (x) = 3x + 4.
This says that the functions name is f , its domain is [0, 1] (the set of all reals between 0
and 1, including 0 and 1), its codomain is R (the set of all reals), and its mapping rule is
that we map each element x in the domain to the element 3x + 4 in the codomain. The
value of f at 0.5 is f (0.5) = 3(0.5) + 4 = 5.5.
What is f (3)? It is not 3(3) + 4 = 13. This is because 3 is not in the domain of f . Hence,
f (3) is simply undefined.

Example 43. The function f R+ R is defined by x ln x. Or alternatively, The


function f R+ R is defined by f (x) = ln x.
This says that the functions name is f , its domain is R+ (the set of all positive reals), its
codomain is R (the set of all reals), and its mapping rule is that we map each element x in
the domain to the element ln x in the codomain. The value of f at 2 is f (2) = ln 2 0.693.
f (0) is simply undefined, because 0 is not in the domain of f . Likewise, f (a) is undefined,
for any a < 0.

Exercise 29. For each of the following functions, write down the value of the function at
1. (a) The function f R R is defined by x x + 1. (b) The function g [1, 1] R is
defined by x 17x. (c) The function h Z+ R is defined by x 3x . (d) The function
i Z R is defined by x 3x . (Answer on p. 1011.)

Page 61, Table of Contents

www.EconsPhDTutor.com

3.2

EVERY x D Must be Mapped to EXACTLY ONE y C

This section simply repeats and emphasises what was already said above.
Example 44. Say we have ...
Domain {Cow, Chicken};
Codomain {Produces eggs, Produces milk, Guards the home}; and
Mapping rule: Chicken is mapped to Produces eggs.
Can we define a function using the above domain, codomain, and mapping rule?
No. The reason is that the mapping rule fails to specify what Cow (an element of the
domain) should be mapped to. It thus fails the requirement that every element in the
domain be mapped to an element in the codomain.
Example 45. Say we have ...
Domain {Cow, Chicken};
Codomain {Produces eggs, Produces milk, Guards the home}; and
Mapping rule: Cow is mapped to both Produces milk and Guards the home; and
Chicken is mapped to Produces eggs.
Can we define a function using the above domain, codomain, and mapping rule?
No. The reason is that the mapping rule maps Cow (an element of the domain) to more
than one element in the codomain. It thus fails the requirement that every element in the
domain be mapped to exactly one element in the codomain.
Example 46. Say we have ...
Domain R;
Codomain [0, 1]; and
Mapping rule: x x + 1.
Can we define a function using the above domain, codomain, and mapping rule?
No. The reason is that the mapping rule fails to map some elements in the domain (e.g.
14) to any element in the codomain. It thus fails the requirement that every element in the
domain be mapped to an element in the codomain.

Page 62, Table of Contents

www.EconsPhDTutor.com

Example 47. Say we have ...


Domain R;
Codomain R; and
Mapping rule: x x.
Can we define a function using the above domain, codomain, and mapping rule?
No. The reason is that the mapping rule maps each element in the codomain (e.g. 14) to
more than one element in the codomain (+14 and -14). It thus fails the requirement that
every element in the domain be mapped to exactly one element in the codomain.

For Exercises 30-37: (i) State (yes/no) whether we can define a function using the given
domain, codomain, and rule. (ii) Explain why or why not. (iii) If we can, then write down
the function in formal notation.
Exercise 30. Let the domain be {5, 6, 7}, the codomain be Z+ , and the mapping rule be
x 2x (Answer on p. 1011.)
Exercise 31. Let the domain be {0, 3}, the codomain be {3, 4}, and the mapping rule be
(informally) any larger number will work. (Answer on p. 1011.)
Exercise 32. Let the domain be {2, 4}, the codomain be {3, 4}, and the mapping rule be
(informally) any smaller number will work. (Answer on p. 1011.)
Exercise 33. Let the domain be {1}, the codomain be {1}, and the mapping rule be
(informally) stay exactly the same. (Answer on p. 1011.)
Exercise 34. Let the domain be {1}, the codomain be {1, 2}, and the mapping rule be
(informally) stay exactly the same. (Answer on p. 1011.)
Exercise 35. Let the domain be {1, 2}, the codomain be {1}, and the mapping rule be
(informally) stay exactly the same. (Answer on p. 1011.)

Exercise 36. Let the domain be R, the codomain be R, and the mapping rule be x x.
(Answer on p. 1011.)
1
Exercise 37. Let the domain be R, the codomain be R, and the mapping rule be x .
x
(Answer on p. 1011.)
Exercise 38. How might you change the domain in Exercise 36 so that a function can be
defined? (Answer on p. 1012.)
Exercise 39. How might you change the domain in Exercise 37 so that a function can be
defined? (Answer on p. 1012.)

Page 63, Table of Contents

www.EconsPhDTutor.com

3.3

Real-Valued Functions of a Real Variable

Definition 13. A function of a real variable is any function whose domain is a subset of
R.

Definition 14. A real-valued function is any function whose codomain is a subset of R.

Altogether then, a real-valued function of a real variable is any function both of whose
domain and codomain are subsets of R.
Example 48. Consider the functions f R R, g R R, and h R R defined by
x x2 . All three are real-valued functions, functions of a real variable, and thus also
real-valued functions of a real variable.
Consider the function i {Cow, Chicken} Z defined by Cow 5 and Chicken 32. This
is a real-valued function, but not a function of a real variable. Thus, it is not a real-valued
function of a real variable.
Consider the function j Z {Cow, Chicken} defined by x Cow if x is odd and x
Chicken if x is even. This is a function of a real variable, but not a real-valued function.

Almost all functions considered in H2 Maths are real-valued functions of a real variable. So
well see plenty of functions like f , g, and h from the above example, but rarely (if ever)
will we see functions like i or j.
In this textbook, unless otherwise clearly-stated, it may be assumed that all functions are
real-valued functions of a real variable.

Page 64, Table of Contents

www.EconsPhDTutor.com

3.4

The Range of a Function

Informally, the range of a function f D C denoted f (D) is the set of elements


in the codomain C that are hit by the function. Formally:
Definition 15. The range of a function f D C is f (D) = {y C There is x D such
that f (x) = y}.
The range of f may be denoted Range(f ) or f (D) (if D is the domain of f ).
The range is not the same thing as the codomain. Because this is such a common
misconception, let me repeat:

The range is not the same thing as the codomain.


Indeed, the range is usually a proper subset of the codomain, as was the case in each of the
following examples.
Example 49. Define f [0, 1] R by x x + 1. Then Range(f ) = f ([0, 1]) = [1, 2].
Example 50. Define f {2, 3} R by x x + 1. Then Range(f ) = f ({2, 3}) = {3, 4}.
Example 51. Define f R R by x ex . Then Range(f ) = f (R) = R+ .
The range is often is a proper subset of the codomain, but sometimes they can be equal:
Example 52. Define f R+ R by x ln x. Then Range(f ) = f (R+ ) = R = Cod(f ).

Exercise 40. Let the function f R+0 R be defined by x


(Answer on p. 1012.)

x. What is the range of f ?

Exercise 41. Let the function f Z Z be defined by x x2 . What is the range of f ?


(Answer on p. 1012.)
Exercise 42. Which of the following statements is/are true? (a) The range of any function
is a subset of its domain. (b) The range of any function is a subset of its codomain. (c)
The range of any function is a proper subset of its codomain. (Answer on p. 1012.)

Page 65, Table of Contents

www.EconsPhDTutor.com

3.5

Creating New Functions

Let f A R and g B R be functions with mapping rules f x f (x) and g x g(x).


Let k R be a constant. Then we can create the function f + g in the obvious fashion.
f
We can also create the functions f g, f g, , and kf in the obvious fashions.17
g
The symbol is an alternative symbol for multiplication. We will often prefer using rather
than because there is the slight risk of confusing with the letter x.
As we shall see, f g shall refer to a function that is entirely different from f g, so we must
really be careful to write f g when that is what we mean.
Example 53. Let f R R be defined by x 7x + 5 and g R R be defined by x x3 .
Let k = 2. Then
f + g is the function with domain R, codomain R, and mapping rule x 7x + 5 + x3 ;
f g is the function with domain R, codomain R, and mapping rule x 7x + 5 x3 ;

f g is the function with domain R, codomain R, and mapping rule x (7x + 5) x3 ; and
f
is the function with domain R R+ , codomain R, and mapping rule x (7x + 5) /x3 .
g
kf is the function with domain R, codomain R, and mapping rule x 2 (7x + 5).
We can of course give these four new functions new names (perhaps a single-letter name
for each), but this is not necessary. We can simplywrite:
(f + g) (1) = 7(1) + 5 + 13 = 13,
(f g) (1) = 7(1) + 5 13 = 11,

(f g) (1) = [7 (1) + 5] (1)3 = 12,


f
( ) (1) = [7 (1) + 5] /13 = 12,
g

(kf )(1) = 2 [7 (1) + 5] = 24,


where the pairs of parentheses around each of the five new functions are just to be clear
that we are talking about a single, fully-fledged function.

17

Formally, f + g is the function with domain A B, codomain R, and mapping rule x f (x) + g(x). Similarly, f g is the
function with domain A B, codomain R, and mapping rule x f (x) g(x).
f g is the function with domain A B, codomain R, and mapping rule x f (x)g(x).
f
is the function with domain {x x A B, g(x) 0}, codomain R, and mapping rule x f (x)/g(x). The set A
g
B/ {x g(x) = 0} is the set of all elements x that are in both A and B, excluding those for which g(x) = 0. This exclusion
is necessary, otherwise f (x)/g(x) may sometimes not be well-defined.
Finally, kf is simply the function with domain A, codomain R, and mapping rule x kf (x).

Page 66, Table of Contents

www.EconsPhDTutor.com

3.6

One-to-One Functions

Informally, a function is one-to-one (or invertible) if every element in its range is hit
exactly once (by exactly one element in the domain). Put another way: every element y in
the range corresponds to exactly one element in the domain. Formally:
Definition 16. A function f D C is one-to-one (or invertible) if for every y f (D),
there is only one x D such that f (x) = y.
Example 54. Consider the function f whose domain is the set {Cow, Chicken}, codomain
is the set {Produces eggs, Produces milk, Guards the home}, and mapping rule is Cow
Produces milk and Chicken Produces eggs.
The range is {Produces eggs, Produces milk}.
This function is one-to-one because each element in the range is hit exactly once, as we
can easily verify: Produces eggs is hit once by Chicken and Produces milk is hit once
by Cow.

Example 55. Let f [0, 1] R be defined by x x + 1. The range of f is [1, 2].


To check whether this function is one-to-one, we need to show that every element y in the
range corresponds to exactly one element x in the codomain. To this end, lets pick any
element y in the range and write: y = x + 1 y 1 = x.
Thus, indeed, this function is one-to-one every element y in the range corresponds to
exactly one element y 1 in the domain.
To show that a function is not one-to-one, simply give a counter-example:
Example 56. Let f R R be defined by x x2 . The range of f is R+ .
This function is not one-to-one for example, the element 9 in the range is hit twice,
once by 3 and again by 3.
Remark 4. One-to-one or invertible functions are also known as injective functions (or
simply injections), but we wont use this term in this textbook.
Exercise 43. State and explain
of the following functions is one-to-one. (a)
whether each
+
+
f R0 R is defined by x x. (b) g R0 R is defined by x x2 . (c) h R R is
defined by x x. (d) i R+0 R is defined by x x. (e) j R R is defined by x sin x.
(Answer on p. 1013.)

Page 67, Table of Contents

www.EconsPhDTutor.com

3.7

Inverse Functions

Definition 17. If f D C is invertible, then its inverse function f 1 f (D) D is


defined by the mapping rule f 1 (y) = x y = f (x).
Only invertible functions have inverse functions. If a function is not invertible, then
its inverse function simply does not exist.

Given a one-to-one (or invertible) function f , to find its inverse function f 1 , follow these
steps:
1. Dom (f 1 ) = Range (f ).
2. Cod (f 1 ) = Dom (f ).
3. Write down an expression f 1 (y) that involves only y and show that f 1 (y) = x
y = f (x) .

Page 68, Table of Contents

www.EconsPhDTutor.com

Example 57. As we showed above, the function f [0, 1] R defined by x x + 1 is


one-to-one. So its inverse function f 1 exists. Lets find it.
1. f has range [1, 2]. So f 1 has domain [1, 2].
2. f has domain [0, 1]. So f 1 has codomain [0, 1].
3. Pick any element y in the range of f and write:
y =f (x) y = x + 1 y 1 = x.

1
f

(y)

So f 1 has mapping rule y y 1.


Well actually only formally talk about graphs in the next few chapters. But for now, as a
visual aid, Ill provide the graphs of f 1 (blue) and f (red) anyway.
Observe that f 1 is simply the reflection of f in the line y = x (dotted). Section 7.2 (in
particular Fact 7) will explain why exactly this is so.

f(x), f -1(x)

4
3
2
1
x
0
-5

-4

-3

-2

-1

-1
-2
-3
-4
-5

Page 69, Table of Contents

www.EconsPhDTutor.com

Example 58. You can verify for yourself that the function f R R defined by x 2x is
one-to-one. Lets find its inverse function f 1 .
1. f has range R. So f 1 has domain R.
2. f has domain R. So f 1 has codomain R.
3. As usual, lets pick any element y in the range of f and write:
y =f (x) y = 2x 0.5y = x.

1
f

(y)

So f 1 has mapping rule y 0.5y.

f(x), f -1(x)

4
3
2
1
x
0
-5

-4

-3

-2

-1

-1
-2
-3
-4
-5

Page 70, Table of Contents

www.EconsPhDTutor.com

Example 59. You can verify for yourself that the function f R+ R R defined by
1
x is one-to-one. Lets find its inverse function f 1 .
x
1. f has range R+ R . So f 1 has domain R+ R .

2. f has domain R+ R . So f 1 has codomain R+ R .


3. As usual, lets pick any element y in the range of f and write:
1
1

y =f (x) y =
= x ( y 0).
x
y

1
f

(y)

1
So f 1 has mapping rule y .
y
(Note that is the shorthand symbol for because. Similarly, is the shorthand symbol
for therefore.)
The condition here that y 0 is important and goes back to our warning that was Chapter
2 (Dividing by Zero). We know for sure that the range of f does not contain 0. This is
why in the last line above, we can safely divide both sides of the equation by y.
5

f(x), f -1(x)

4
3
2
1
x
0
-5

-4

-3

-2

-1

-1
-2
-3
-4
-5

Page 71, Table of Contents

www.EconsPhDTutor.com

Example 60. You can verify for yourself that the function f R+0 R defined by x x2
is one-to-one. Lets find its inverse function f 1 .
1. f has range R+0 . So f 1 has domain R+0 .

2. f has domain R+0 . So f 1 has codomain R+0 .


3. As usual, lets pick any element y in the range of f and write:

y =f (x) y = x2 y = x.

1
f

(y)

Here there are two possibilities for the mapping rule of f 1 , namely y y and y y.
We must pick one. We know that the domain of f and
hence the codomain of f 1 is

R+0 . So we should pick as the mapping rule of f 1 y y.

f(x), f -1(x)
4
3
2
1
x
0
0.0

0.5

1.0

1.5

2.0

+
Exercise 44. Find
the inverse function for each of the following functions. (a) f R0 R
defined by x x. (b) g [0.5, 0.5] R defined by x sin x. (c) h R R defined by
x x3 .(Answers on p. 1014.)

Page 72, Table of Contents

www.EconsPhDTutor.com

3.8

Domain Restriction to Create an Invertible Function

We saw that some functions were not one-to-one (or non-invertible). And so for these
functions, an inverse function simply does not exist.
Nonetheless, we can often transform a non-invertible function into an invertible function.
One way to do this is by restricting the domain. The new invertible function will then have
an inverse function.
Example 61. We saw in Exercise 43 that the function j R R defined by x sin x was
not one-to-one. However, we can restrict the domain to [0.5, 0.5] to get a brand new
function g [0.5, 0.5] R defined by x sin x. This brand new function g is identical
to the original function j except for its domain. g is one-to-one, as you should verify for
yourself.
We can thus go ahead and construct the inverse function g 1 . Actually, we already did this
in Exercise 44.

Example 62. We saw in Example 56 that the function f R R defined by x x2 was


not one-to-one. However, we can restrict the domain to R+0 to get a brand new function
g R+0 R defined by x x2 . This brand new function g is identical to the original function
f except for its domain. g is one-to-one, as we verified in Exercise 43.
We can thus go ahead and construct the inverse function g 1 . I leave this as an exercise for
you.

There is almost always more than one way to restrict the domain of a non-invertible function
to obtain an invertible function. Indeed, a trivial case would be where we restrict its domain
to be the empty set! In which case the function thus formed would certainly be invertible,
though not very interesting (it would have an empty domain and an empty range so too
would its inverse function).

1
(x 1)2
is not one-to-one. (b) Show that by restricting its domain to (1, ), we can create a new
invertible function g (you must prove that this new function is invertible). (c) Then find
the inverse function g 1 . (Answer on p. 1015.)
Exercise 45. (a) Show that the function f (, 1) (1, ) R defined by x

Exercise 46. For the function f in Example 62, lets instead restrict the domain to [20, 30].
Show that the new function thus obtained is one-to-one and find its inverse. (Answer on
p. 1015.)

Page 73, Table of Contents

www.EconsPhDTutor.com

3.9

Composite Functions

Definition 18. Let f and g be functions such that the range of g is a subset of the domain
of f . Then the composite function f g is the function with the same domain as g, the same
codomain as f , and mapping rule x f (g(x)).
The composite function f g can be read aloud as f circle g and is sometimes denoted
f g, especially when we want to make clear that we are not talking about f g. But well
rarely use the f g notation, unless there is some risk of confusion with f g.
The underlined condition is important: The range of g must be a subset of the domain of
f in order for the composite function f g to exist. This condition ensures that given any x
from the domain of g, the value g(x) is itself also in the domain of f , so that f (g(x)) is
well-defined.
If this condition fails, then the composite function f g simply does not exist.
Example 63. The functions g, f R R are defined by g x x + 1 and f x 2x. The
range of g is R this is indeed a subset of the domain of f (which is R). So the composite
function f g R R exists and is defined by x f (g(x)) = 2 (g(x)) = 2(x + 1).
Lets try computing f g(2). We can use the definition of a composite function: f g(2) =
f (g(2)) = f (2 + 1) = f (3) = 6.
Alternatively, we can directly use f g(x) = 2(x + 1) to compute f g(2) = 2(2 + 1) = 6.

Notice that for the composite function f g, we apply the function g first before applying the
function f . So for example, to compute, say f g(7), we compute g(7) first, then compute
f (g(7)). (A common mistake by students is to instinctiely read from left to right, and so
apply f first before g.)

Example 64. The functions g, f R R are defined by g x x2 and f x x + 1. The


range of g is R+0 this is indeed a subset of the domain of f (which is R). So the composite
function f g R R exists and is defined by x f (g(x)) = g(x) + 1 = x2 + 1.
Lets try computing f g(3). We can use either the definition of a composite function:
f g(3) = f (g(3)) = f (32 ) = f (9) = 10.
Alternatively, we can directly compute, using f g(x) = x2 + 1: f g(3) = 32 + 1 = 10.

Page 74, Table of Contents

www.EconsPhDTutor.com

Example 65. The function g R R is defined by x x + 1. The function f R+ R is


defined by x ln x. The range of g is R, which is not a subset of the domain of f (which
is R+ ). Hence, the composite function f g simply does not exist.
We saw that if f is non-invertible, then its inverse function f 1 simply does not exist.
Nonetheless, we could restrict its domain to create a new invertible function g, whose
inverse function g 1 we could then write down.
By analogy, suppose we have functions f and g where gs range is not a subset of f s
domain. Thus, the composite function f g simply does not exist. But we can play a similar
trick: We can restrict the domain of g to create a new function g, so that the range of g is
a subset of f s domain. We can then write down the composite function f g.
Fortunately, this is not in the syllabus, so you dont need to know how to do this. Yay!

Exercise 47. For each of the following pairs of functions f and g, verify that the composite
function f g exists and write it out in full. Also, compute f g(1) and f g(2). (a) The functions
g, f R R defined by g x x2 + 1 and f x ex . (b) The functions g, f R R defined
by g x ex and f x x2 + 1. (c) The functions g, f R R+ R defined by g x 1/2x
and f x 1/x. (d) The functions g, f R R+ R defined by g x 1/x and f x 1/2x.
(Answer on p. 1016.)
We can of course also build a composite function out of a single function.
Example 66. The function f R R is defined by x 2x. The range of f is R and this
is indeed a subset of the domain of f (which is R). So the composite function f f R R
exists and is defined by x f (f (x)) = 2f (x) = 2(2x) = 4x. And so for example f f (3) =
2(2 3) = 12.
The composite function f f can instead be written as f 2 . So in the above example, wed
write f 2 (3) = 12.
We can, analogously, define the composite function f f 2 and denote it f 3 . Using the above
example, f 3 (x) = 8x and f 3 (3) = 24.
Of course, there are also f 4 , f 5 , etc.

Page 75, Table of Contents

www.EconsPhDTutor.com

Remark 5. The official A-level syllabus uses f 2 to mean the composite function f f and
nothing else. So this is what well do in this textbook.
But confusingly enough, some writers use the symbol f 2 to mean the second derivative of
f , f 3 to mean the third derivative of f , etc.. We wont follow such practice. Just to let
you know, in case you read other mathematical texts and get confused.
However, we will use f (3) to mean the third derivative of, f (4) to mean the fourth
derivative of, etc. This will show up occasionally in Part V (Calculus).
Exercise 48. For each of the following functions f , verify that the composite function f 2
exists and write it out in full. Also, compute f 2 (1) and f 2 (2). (a) The function f R R
defined by x ex . (b) The function f R R defined by x 3x + 2. (c) The function
f R R defined by x 2x2 + 1. (Answer on p. 1016.)

Page 76, Table of Contents

www.EconsPhDTutor.com

Graphs

An ordered pair is a mathematical object. Like set of two objects, an ordered pair is,
informally, a container with two objects, where the objects are listed out with a comma
separating them.
The only difference between a set of two objects and an ordered pair is that order
matters for the latter.
To distinguish an ordered pair from a set of two objects, we use parentheses (instead of
braces).
Example 67. (Cow, Chicken) is an ordered pair. (5, 4) is an ordered pair.
We also refer to (a, b) as ordered set notation. So (Cow, Chicken) and (5, 4) are both
examples of ordered pairs, written out in ordered set notation.

Example 68. Let (Cow, Chicken) and (Chicken, Cow) be ordered pairs. Let {Cow,
Chicken} and {Chicken, Cow} be sets.
Recall that for sets, order did not matter. Hence, {Cow, Chicken} = {Chicken, Cow}.
In contrast, for ordered pairs, order does matter. And so (Cow, Chicken) (Chicken, Cow).
Definition 19. An ordered pair of real numbers is any (x, y) where both x, y are real.
Example 69. (5, 4), (1, 1), and (2, 3) are all ordered pairs of real numbers.
Confusingly, above in Section 1.9 (Intervals), we said that (5, 4) was a set, namely {x R
5 < x < 4}. Here we say instead that (5, 4) is an ordered pair, consisting of two objects
(5 and 4), the order of which matters.
Unfortunately this is yet another bit of confusing notation youll have to live with. Youll
have to learn to tell, from the context, whether (5, 4) is a set of infinitely-many real
numbers or an ordered pair. But dont worry, this is usually pretty obvious.

Page 77, Table of Contents

www.EconsPhDTutor.com

Definition 20. In any ordered pair of real numbers, the first real is called the x-coordinate
and the second is the y-coordinate.

Definition 21. The cartesian plane is the set of all ordered pairs of real numbers.

In set-builder notation, the cartesian plane can be written as {(x, y) x R, y R}. This
reads aloud as the cartesian plane is the set of ordered pairs of real number (x, y).
In this textbook, well usually only ever look at ordered pairs of real numbers.
Hence, rather than say ordered pair of real numbers, well simply say ordered pair. And
so whenever you see the notation (x, y), it should be understood that this is an ordered
pair of real numbers (and not cows or chickens).
And so instead of writing the cartesian plane as {(x, y) x R, y R}, well simply write it
as {(x, y)}, with the understanding that x, y are reals.
In the present context, well also simply call any ordered pair of real numbers a point.
(Later on, in the context of three-dimensional geometry, points will also refer to ordered
triples of real numbers.)
Definition 22. In the context of the cartesian plane, the origin is the point (0, 0).
Points are usually given lower-case letters as names.

Page 78, Table of Contents

www.EconsPhDTutor.com

We can illustrate the cartesian plane graphically. The horizontal axis corresponds to the
x-coordinate of the points and is thus also called the x-axis. The vertical axis corresponds
to the y-coordinate of the points and is thus also called the y-axis.
Example 70. The points (or ordered pairs of real numbers) a = (5, 4), b = (1, 1), and
c = (2, 3) are illustrated graphically on the cartesian plane:

a
3

b
x

-5

-3

-1

-1

-3

-5

Definition 23. A graph (or curve) is any set of points.

Example 71. The set of three points {a, b, c} = {(5, 4), (1, 1), (2, 3)} is a graph.

Page 79, Table of Contents

www.EconsPhDTutor.com

The graph of a function f is simply the set of points (x, y) that satisfy x D, y C, and
f (x) = y. Formally:
Definition 24. The graph of a function f D C is the set {(x, y) x D, y C, y = f (x)}.
Given a function that is named with a lower-case letter, we will often use the upper-case
version of that same letter to denote that functions graph. So for example, given the
function f , we often give its graph the name F .
Example 72. Consider the function f R R defined by x x2 . Its graph may be written
as F = {(x, y) y = x2 }.

f(x), y

x
0
-2

-1

Weve defined graph as a noun. But at the slight risk of confusion, well also use it as a
verb that means draw in the cartesian plane a given set of points. So we can say either
we draw the graph of f (graph as a noun), or we graph f (graph as a verb).

Page 80, Table of Contents

www.EconsPhDTutor.com

Definition 25. Given a function f D C, a point of the function is any element of its
graph. That is, it is any ordered pair (x, f (x)), where x D.
To use the above example, we say that (2, 4) and (5, 25) are both points of f .
But since x determines f (x), it is nice but not necessary to specify the complete ordered
pair (x, f (x)). Instead, we can refer to the point simply as x. So in the above example, we
can simply say that 2 and 5 are both points of f , with the understanding that what we
really mean is (2, 4) and (5, 25) are both points of f . This is a bit sloppy and at the risk
of some confusion, but will save us a lot of messy notation.
So in the context of functions, x does double duty. It can either refer to an element in the
functions domain OR it can refer to a point of the function.
On exams though, it is probably safer to simply list out the full co-ordinates, whenever
youre referring to a point. Just in case your marker is damn niao.

Page 81, Table of Contents

www.EconsPhDTutor.com

We just learnt about the graph of a function. A graph of an equation is very similarly
defined:
Definition 26. Given an equation involving x and y as the only two variables, the graph
of the equation is the set of points (x, y) that satisfy the equation.
Example 73. The graph below is of the equation x2 + y 2 = 1. It is simply the set {(x, y)
x2 + y 2 = 1}.

1
f(x), g(x), y
p = (x, y)

x 2 + y2 = 1

y
x

-1.0

-0.5

0.0

0.5

1.0

(-0, 0)
Centre

-1

Exercise 49. (a) Can the equation x2 + y 2 = 1 be rewritten into the form of a single
function? (b) Can it be rewritten into the form of two functions? (Answer on p. 1017.)
Exercise 50. Draw the graphs of each of the following equations. (a) y = ex . (b) y = 3x + 2.
(c) y = 2x2 + 1. (Answers on pp. 1017, 1018, and 1019.)

Page 82, Table of Contents

www.EconsPhDTutor.com

4.1

Graphing with Your TI84 Graphing Calculator

You are required to know how to use a graphing calculator to graph a given function.
Pretty bizarre that in this age of the smartphone, they want you to learn how to use these
clunky and now-useless devices from the 80s and 90s. It is the equivalent of having to
learn how to program a VCR.
This textbook will give only a very few examples involving graphing calculators.
There is no better way of learning to use it than to play around with it yourself. By the
time you sit down for your A-level exams, you will have had plenty of practice with it.
You can also use any of the seven calculators in the list below (last updated by SEAB on
March 1st, 2016, PDF). But this textbook will stick with the TI-84 PLUS Silver Edition
(which Ill simply call the TI84).18

Ill always start each example with the calculator freshly reset.
Example 74. Graph the function f R R defined by x x2 .
1. Press ON to turn on your calculator.
2. Press Y= to bring up the Y= editor.
3. Press X,T,,n to enter X; then x2 to enter the squared 2 symbol.
4. Now press GRAPH and the calculator will graph y = x2 .
After Step 1.

18

After Step 2.

After Step 3.

After Step 4.

My understanding is that most students use a TI calculator and that the five approved TI calculators are pretty similar.

Page 83, Table of Contents

www.EconsPhDTutor.com

Example 75. Graph the equation x2 + y 2 = 1.


The TI84 requires that we enter equations in a form where y is directly expressed in terms
of y. But there is no way to do this here. As explained in Exercise 49, there is no way to
rewrite this equation into a single function.
However, we can rewrite it into two functions:
Namely, f [1, 1] R defined by x

2
1 x2 and g [1, 1] R defined
by x 1 x. We can thus tell our calculator to
graph two separate equations: y = 1 x2 and y = 1 x2 .
1. Press ON to turn on your calculator.
2. Press Y= to bring up the Y= editor.
Most buttons on the TI84 have three different roles. If you simply press a button, then the
TI84 executes the role that is printed on the button itself. If you press the blue 2ND and
then a button, then the TI84 executes the role printed in blue above the button. And if
you press the green ALPHA and then a button, then the TI84 exexcutes the role printed
in green above the button.

(which corresponds to the x2 button) to enter


3. Press the blue 2ND button and then

(. Next press 1 X,T,,n x2 ) to enter 1 X 2 ). Altogether you will have

entered 1 x2 .
4. Now press ENTER and the blinking cursor will move down, to the right of Y2 =.
After Step 1.

After Step 2.

After Step 3.

After Step 4.

(... Example continued on the next page ...)

Page 84, Table of Contents

www.EconsPhDTutor.com

(... Example continued from the previous page ...)


After Step 5.

After Step 6.

After Step 8.

After Step 9.

After Step 7.

Well now enter the second equation.


4. Press the (-) button. (Warning: This is different from the - button. If you use the button, you will get an error message when you try to generate your graphs later.) Now

repeat what we did in step 3 above: Press the blue 2ND button and then
(which

corresponds to the x2 button) to enter (. Next press 1 X,T,,n x2 ) to enter

2
1 X ). Altogether you will have entered 1 x2 .

5. Now press GRAPH and the calculator will graph both y = 1 x2 and y = 1 x2 .
Notice the graphs are very small. To zoom in:
6. Press the ZOOM button to bring up a menu of ZOOM options.
7. Press 2 to select the Zoom In option. Nothing seems to happen. But now press ENTER
and the TI84 will zoom in a little for you.
We expected to see a perfect circle. Instead we get an elongated oval whats going on?
The reason is that by default, the x- and y- axes are scaled differently. To set them to the
same scale:
8. Press the ZOOM button again to bring up the ZOOM menu of options. Press 5 to
select the ZSquare option. Nothing seems to happen. But now press ENTER and the
TI84 will adjust the x- and y- axes to have the same scale. And now we have a perfect
circle.

Page 85, Table of Contents

www.EconsPhDTutor.com

Quick Revision: Exponents, Surds, Absolute Value

Heres a super quick revision of some O-Level Maths well be using. If you have severe
difficulty with these exercises, you should go back and review your O-Level Maths material!

5.1

Laws of Exponents

For all real numbers x, we have x1 = x and x0 = 1.19


For all real numbers x, y, a, and b (provided any denominators are non-zero):
x x
a

=x

a+b

x a
xa
( ) = a,
y
y

xa
= xab ,
b
x
(xa )

= xab ,

(xy)a = xa y a ,

xa =

1
,
xa

a1/b =

b
a,

ac/b =

c
b
ac = ( b a) .

Exercise 51. Simplify the two expressions below. (Answer on p. 1020.)


(53x 251x )
52x+1 + 3(25x ) + 17(52x )

(8x+2 34(23x ))
.
2x+1
( 8)

Exercise 52. Is it true that x(a ) = xab ? (Answer on p. 1021.)


b

19

By convention, 00 is usually defined to be equal to 1 this textbook will follow this practice.

Page 86, Table of Contents

www.EconsPhDTutor.com

5.2

Rationalising the Denominator of a Surd

Example 76. Heres a case where theres just a surd in the denominator:

1
2
2
= =
.
2
2
2 2
For more complicated cases, the trick is to use the fact that (a + b)(a b) = a2 b2 .
Given a + b, we call a b the conjugate of a + b. We refer to a + b and a b as a conjugate
pair.
Example 77.

1 2
1 2 1 2
1
1 2
=

=
=
=
= 2 1.

12
1
1 + 2 (1 + 2) (1 2) 12 ( 2)2

Exercise 53. Prove the following equality: (Answer on p. 1021.)

x
y

Page 87, Table of Contents

x2
y2

+1

x
x2
+1 .
2
y
y

www.EconsPhDTutor.com

5.3

Absolute Value

The notation z returns the absolute value of z.

z,
z =

z,

if z 0,
if z < 0.

(See footnote for a more formal definition.20 )

Example 78. 4 = 4 and 4 = 4. 2 = 2 and 2 = 2.

Fact 2. (a) x < b b < x < b.


(b) x b b x b.
Proof. (a) x < b 0 x < b OR b < x < 0 b < x < b.
(b) Very similar.

Fact 3. (a) x a < b a b < x < a + b.


(b) x a b a b x a b.
Proof. (a) By Fact 2, x a < b if and only if b < x a < b. Rearranging the latter set of
inequalities yields a b < x < a + b.
(b) Very similar.

20

The absolute value operator is the function with domain R, codomain R+0 , and mapping rule x x if x 0 and x x
if x < 0.

Page 88, Table of Contents

www.EconsPhDTutor.com

Fact 4.

a a
= (provided b 0).
b
b

Proof. If a = 0, then clearly this is true.


a
a
a
0 so that = . Moreover,
b
b
b
a a
a a a
a a
either
= or
=
= . Altogether then, indeed
= .
b b
b b b
b
b
If a and b have the same signs (and are non-zero), then

a
a
a
< 0 so that = . Moreover,
b
b
b
a a
a a
a a
either
=
or
= . Altogether then, indeed
= .
b
b
b b
b
b
If a and b have opposite signs (and are non-zero), then

Page 89, Table of Contents

www.EconsPhDTutor.com

Intercepts

Example 79. The graph below is of the equation y = x + 3. It has horizontal intercept 3
and vertical intercept 3.

5
y
y=x+3
3

1
x
-5

-3

-1

-1

-3

-5

Horizontal intercepts are the x-coordinates of the points at which the graph intersects
the horizontal or x-axis. Similarly, vertical intercepts are the y-coordinates of the points
at which the graph intersects the vertical or y-axis.
Definition 27. a is a horizontal intercept (or x-intercept) of a graph G if (a, 0) G.
Definition 28. b is a vertical intercept (or y-intercept) of a graph G if (0, b) G.

Page 90, Table of Contents

www.EconsPhDTutor.com

Where the graph G is of an equation (or function), we sometimes also call the horizontal
intercepts zeros or roots of the equation (or function). (Well use the terms zeros and
roots interchangeably in this textbook.)
Example 80. Consider the equation y = x2 1. Its zeros or roots are 1 and 1, because
these are the values of x for which y = 0.
Of course, 1 and 1 are also the horizontal intercepts (or x-intercepts) of the graph of the
equation.
Example 81. Consider the the function f with domain R, codomain R, and mapping rule
x x2 1. Its zeros or roots are 1 and 1, because these are the values of x for which
f (x) = 0.
Of course, 1 and 1 are also the horizontal intercepts (or x-intercepts) of the graph of f .

Page 91, Table of Contents

www.EconsPhDTutor.com

Example 82. The graph below is of the function f R R defined by x x2 1. It has


vertical intercept 1 and two horizontal intercepts, 1 and 1.
1 and 1 are also the zeros or roots of f , because f (1) = 0 and f (1) = 0.

f(x)

f(x) = x2 - 1

x
0
-2

-1

-1

The A-level exams will often ask you to write down the full co-ordinates of the points at
which a graph (or curve) crosses the axes this means writing down both the x- and
y-coordinates, and not just the horizontal intercept or the vertical intercept. Heres an
exercise to help you make this a habit.
Exercise 54. Write down in full the point(s) at which the graphs of each the following
equations crosses the axes: (a) x2 +y 2 = 1. (b) y = x2 4. (c) y = x2 +2x+1. (d) y = x2 +2x+2.
(Answer on p. 1022.)

Page 92, Table of Contents

www.EconsPhDTutor.com

7
7.1

Symmetry

Reflection of a Point in a Line

A reflection of a point in a line is its mirror image point on that line. Formally:
Definition 29. Let a be a point and l1 be a line. Let l2 be the line that is perpendicular
to l1 and runs through a. Let x be the point where l1 and l2 intersect. Then the reflection
of a in l1 is the point a on l2 such that the distances ax and a x are equal.

l1
l2

a'

Fact 5. Let (a, b) be a point. Its reflection in the line y = x is the point (b, a).

Proof. Optional, see p. 924 in the Appendices.

Fact 6. Let (a, b) be a point. Its reflection in the line y = x is the point (b, a).

Proof. Optional, see p. 924 in the Appendices.

Example 83. (a) Given the point (3, 17), its reflection in the line y = x is (17, 3) and its
reflection in the line y = x is (17, 3).
(b) Given the point (1, 5), its reflection in the line y = x is (5, 1) and its reflection in the
line y = x is (5, 1).
(c) Given the point (0, 0), its reflection in the line y = x is (0, 0) and its reflection in the
line y = x is (0, 0).
Exercise 55. For each of the following points, write down their reflections in the lines (i)
y = x; and (ii) y = x. (a) (3, 17). (b) (1, 5). (c) (0, 0). (Answer on p. 1023.)

Page 93, Table of Contents

www.EconsPhDTutor.com

7.2

Reflection of a Graph in a Line

Definition 30. The reflection of a graph G in a line is the graph G where each point in
G is a reflection of a point in G.

Example 84. The reflection of the graph G = {(x, y) y = x2 + 4} in the line y = 2 is the
graph G = {(x, y) y = x2 }.

G : y = x2 + 4

y=2
line of reflection

G ' : y = -x2

Page 94, Table of Contents

www.EconsPhDTutor.com

Example 85. The reflection of the graph G = {(x, y) y = ln x} in the line x = 0 is the
graph G = {(x, y) y = ln(x)}.

y
x=0
line of reflection

G ' : y = ln (-x)

G : y = ln x
x

Fact 7 formalises our earlier observation in section 3.7 (Inverse Functions) that the graphs
of f and its inverse f 1 are reflections in the line y = x.
Fact 7. Let f be an invertible function. Then the reflection of the graph of f in the line
y = x is the graph of its inverse function f 1 .

Proof. Optional, see p. 925 in Appendices.

Page 95, Table of Contents

www.EconsPhDTutor.com

The next Fact simply makes the obvious observation that the reflection in the line y = x of
any point along the line y = x is itself.
Fact 8. Let (a, a) be a point. Its reflection in the line y = x is (a, a).

The above two Facts together imply that


Fact 9. Let f be invertible. Suppose f passes through (a, a). Then so too does its inverse
f 1 . And hence, f and f 1 intersect at those points where x = f (x).

The above Fact is useful for finding where a function and its inverse intersect.
Example 86. Let f R R be the invertible function defined by x 2x. The graph of f
intersects the graph of f 1 at the point(s) where x = f (x) x = 2x x = 0. Notice
the intersection point (0, 0) is also on the line y = x. See figure on p. 70.
Example 87. Let f R+0 R be the invertible function defined by x x2 . The graph of
f intersects the graph of f 1 at the point(s) where x = f (x) x = x2 x(x 1) = 0
x = 0, 1. Notice the intersection points (0, 0) and (1, 1) are also on the line y = x. See
figure on p. 72.

Be careful not to make the mistake of believing that f and f 1 can only intersect at points
where x = f (x). A function and its inverse can certainly intersect at points that
are not on the y = x line.
Example 88. Let f R+ R R be the invertible function defined by x 1/x. The
graph of f intersects the graph of f 1 at the point(s) where x = f (x) x = 1/x
x = 1, 1.
We have merely found two points at which f and f 1 intersect. There may very well be
other intersection points. Indeed, in this example, f and f 1 also intersect at every other
x 0! See figure on p. 71.

Page 96, Table of Contents

www.EconsPhDTutor.com

7.3

Lines of Symmetry

Definition 31. A graph is symmetric in a line if it is unchanged after being reflected in


that line.

Example 89. The graph of y = x2 is symmetric in the line x = 0 (which also happens to
be the vertical axis).

4
y
x=0
Reflection
line
3
y = x2

x
0
-2

Page 97, Table of Contents

-1

www.EconsPhDTutor.com

Example 90. The graph of y =

1
is symmetric in the lines y = x and y = x.
x

5
y
4
y = -x
line

y=x
line

3
2
1

y=1/x
0
-5

-4

-3

-2

-1

0
-1

5
x

-2
-3
-4
-5

Page 98, Table of Contents

www.EconsPhDTutor.com

Limits, Continuity, and Asymptotes

The syllabus makes nearly no mention of limits and none of continuity. Yet differentiation
and integration are built entirely on the concept of limits. Continuity is also almost always
assumed. It is thus well-worth spending an hour or two on these concepts, especially since
theyre not difficult and everything will become that much clearer.

8.1

Limits: Introduction and Examples

Here is a very simple example to illustrate the idea of limits.


Example 91. Graphed below is the function f R R defined by x 5x + 2. Observe
that as x approaches 3, f (x) approaches 17. We write this as:
Statement #1. As x 3, f (x) 17.
(The right arrow symbol means to in the context of functions, but now means approaches in the context of limits.)
Equivalently, we may say The limit of f (x) as x approaches 3 is equal to 17. We write:
Statement #2. lim f (x) = 17.
x3

Statements #1 and #2 are entirely equivalent. Either may be (informally) interpreted thus:

For all values of x that are close to but not equal to 3,


f (x) is close to (or possibly even equal to) 17.
y

x
-5

Page 99, Table of Contents

-3

-1

www.EconsPhDTutor.com

In general, lim f (x) = L is informally interpreted as:


xa

For all values of x that are close to but not equal to a,


f (x) is close to (or possibly even equal to) L.
This interpretation is informal because the words close to are vague. For the formal
definitions of limits (optional), see Section 88.1 in the Appendices.
The subtle condition but not equal to requires emphasis. When considering the limit
of f at 3, we do NOT care about the value f (3). Indeed, we do NOT care even if f (3) is
undefined!
Heres an example where lim g(x) is well-defined, even though g(3) is not.
x3

Example 92. Graphed below is the function g (, 3)(3, ) R defined by x 5x+2.


It looks almost exactly like that of f (from the previous example), except there is now a
hole (or more formally, a discontinuity) at x = 3.
Nonetheless, it is still true that

For all values of x that are close to but not equal to 3,


g(x) is close to (or possibly even equal to) 17.
In formal notation, we write as x 3, g(x) 17 or lim g(x) = 17.
x3

x
-5

Page 100, Table of Contents

-3

-1

www.EconsPhDTutor.com

In the next example, both h(3) and lim h(x) are well-defined, but lim h(x) h(3).
x3

x3

Example 93. Graphed below is the function h R R defined by x 5x + 2 for x 3


and h(3) = 0. The graph of h looks almost exactly like those of f and g (from the previous
examples), except that now the value of h at x = 3 is, strangely enough, 0.
Nonetheless, it is still true that

For all values of x that are close to but not equal to 3,


h(x) is close to (or possibly even equal to) 17.
In formal notation, we write as x 3, h(x) 17 or lim h(x) = 17.
x3

x
-5

-3

-1

The next example is similar.

Page 101, Table of Contents

www.EconsPhDTutor.com

Example 94. Graphed below is the function i R R defined by x 0 for x 3 and


i(3) = 17. This graph looks very different from those of f , g, and h (from the previous
examples). But like f , we again have i(3) = 17.
We are tempted to conclude that therefore lim i(x) = 17. This though is wrong, because
x3

we cannot make i(x) as close to 17 as we like by restricting x to values that are close to
but not equal to 3. Hence,
As x 3, i(x)
/ 17,

or equivalently, lim i(x) 17.


x3

Instead, lim i(x) = 0, because:


x3

For all values of x that are close to but not equal to 3,


i(x) is close to (or possibly even equal to) 0.

x
-5

-3

-1

Section 8.3 gives more examples of limits. But first, lets learn about continuity.

Page 102, Table of Contents

www.EconsPhDTutor.com

8.2

Continuity

Informally, a function is continuous at a point a if there is no hole or jump at a.


And so a function is continuous on an interval (of points) if you can smoothly draw its
graph for that entire interval without once lifting your pencil. Formally:
Definition 32. f D R is continuous at a D if lim f (x) = f (a).
xa

Section 88.6 in the Appendices contains additional definitions and results concerning continuity (optional).
Example 91 (revisited). Graphed below is the function f R R defined by x 5x + 2.
It is continuous at 3, because lim f (x) = 17 and f (3) = 17.
x3

It is also continuous at 1, because lim f (x) = 7 and f (1) = 7.


x1

Indeed, it is continuous on R, because for any a R, we have lim f (x) = f (a).


xa

20

10
f is continuous
everywhere
-5

-3

0
-1

x
1

-10

-20

Page 103, Table of Contents

www.EconsPhDTutor.com

Example 92 (revisited). Graphed below is the function g (, 3) (3, ) R defined


by x 5x + 2. It is continuous at 1, because lim g(1) = 7 and g(1) = 7.
x5

However, it is not continuous at 3, because lim g(x) = 17, but g(3) is undefined, and so
x3

lim g(x) g(3).


x3

Altogether, g is continuous at any a (, 3)(3, ), because for any a (, 3)(3, ),


we have lim g(x) = g(a). But g fails to be continuous at 3.
xa

g is continuous
everywhere
except at x = 3.
x
-5

Page 104, Table of Contents

-3

-1

www.EconsPhDTutor.com

Example 93 (revisited). Graphed below is the function h R R defined by x 5x + 2


for x 3 and h(3) = 0. It is continuous at 1, because lim h(x) = 7 and h(1) = 7.
x1

However, it is not continuous at 3, because lim h(x) = 17, but h(3) = 0 and so lim h(x)
x3

x3

h(3).
Altogether, h is continuous at any a (, 3)(3, ), because for any a (, 3)(3, ),
we have lim h(x) = h(a). But h fails to be continuous at 3.
xa

y
h is continuous
everywhere
except at x = 3.
x
-5

Page 105, Table of Contents

-3

-1

www.EconsPhDTutor.com

8.3

Limits: More Examples

We now turn to examples where limits do not exist. We start with a trivial example.
Example 95. Graphed below is the function f R+0 R defined by x 5x + 2.
There is no number L such that for all values of x that are close to but not equal to 3,
f (x) is also close to L. And so we simply say that lim f (x) does not exist.
x3

This is a trivial example because 3 is far from the domain of f . So obviously, for all
values of x that are close to but not equal to 3, f (x) is undefined and so of course there
is no number L that f (x) is always close to!

x
-5

-3

-1

is
undefined.

The next example is less trivial.

Page 106, Table of Contents

www.EconsPhDTutor.com

1
x
for all x 0. This is a very strange function. As x gets ever closer to 0, g(x) fluctuates
ever more rapidly between 1 and 1.

Example 96. Graphed below is the function g R R defined by g(0) = 0 and g(x) = sin

Its difficult or even impossible to draw an accurate graph of g near the origin.
In this example, lim g(x) does not exist. The reason is that for all values of x that are
x0

close to but not equal to 0, there is no number L that g(x) is close to. When x is close
to 0, g(x) takes on every value in [1, 1] infinitely often! And so g(x) can never be said
to be close to any one single number L.
Altogether then, g is not continuous at 0. (With a little work, we can actually prove that
g is continuous on R and also on R+ , but this is beyond the scope of A-levels.)

In the next example, h is nowhere-continuous!

Page 107, Table of Contents

www.EconsPhDTutor.com

Example 97. Graphed below is the function h R R defined by x 1 if the decimal


form representation of x contains the digit 7 and x 2 otherwise.
This function is arguably even stranger than the previous one. We have for example,
h(7) = h(70) = h(1.27) = h(0.0007) = 1 and h(15) = h(16) = h(16.335) = 2.
There are infinitely many points along the line y = 1. And there are also infinitely many
points along the line y = 2! It is quite impossible to sketch its graph accurately.
Nonetheless, h is a perfectly well-defined function. Indeed, h(3) is well-defined and h(x) is
well-defined for any x R.
However, lim h(x) does not exist. However we try to restrict x to values that are close to
x3

(but not equal to) 3, h(x) is never close to any one single value; instead, h(x) switches
infinitely often between 1 and 2.
Indeed, lim h(x) does not exist for any a R! However we try to restrict x to values that
xa
are close to (but not equal to) a, h(x) is never close to any one single value; instead, h(x)
switches infinitely often between 1 and 2.
h is nowhere-continuous: For every a R, h(a) is perfectly well-defined, lim h(x) is not.
xa
And so for every a R, lim h(x) h(a).
xa

y
2

1
is nowherecontinuous.
x

1,
Exercise 56. Consider the function f R R defined by f (x) =

2,

lim f (x), lim f (x), and lim f (x)? (Answer on p. 1024.)


x5

x0

Page 108, Table of Contents

if x 0,
if x > 0.

What are

x5

www.EconsPhDTutor.com

8.4

Infinite Limits and Vertical Asymptotes

This section considers infinite limits, i.e. where as x approaches some number, f (x)
increases (or decreases) grows without bound .
Example 98. Graphed below are the functions f and g, both with domain (, 3)(3, )
1
1
and codomain R, defined by f x 2 +
and g x 2
.
2
(x 3)
(x 3)2

vertical asymptote
-2

-1

x
3

Observe that for all values of x that are close to but not equal to 3, there is no number
L that f (x) is close to. Hence, we say that lim f (x) simply does not exist. Similarly,
x3

lim g(x) does not exist either.


x3

Nonetheless, we observe that as x 3, f (x) increases without bound, while g(x)


decreases without bound. By a very special convention, we are allowed to write these
observations as:
lim f (x) = and
x3

lim g(x) = .
x3

We say that x = 3 is a vertical asymptote of both the graph of f and that of g.


lim f (x) = must NOT be interpreted to mean that there exists something called
x3

lim f (x) (no such thing exists); or that this thing is equal to some other thing called
x3

(recall that is not a number!). Instead, lim f (x) = is interpreted informally as:
x3

f (x) can be made as large as we like,


for all values of x that are sufficiently close to but not equal to 3.
Again, see Section 88.3 in the Appendices (optional) for the formal definitions.

Page 109, Table of Contents

www.EconsPhDTutor.com

Here is another example of vertical asymptotes.


Example 99. Graphed below is the equation y = tan x. It has two vertical asymptotes
x = /2, because lim = and lim = .
x/2

x/2

15

Vertical
asymptote
x = - /2

10

y = tan x
x

0
/2

/2
-5

Vertical
asymptote
x = /2
-10

-15

Page 110, Table of Contents

www.EconsPhDTutor.com

8.5

Limits at Infinity, Horizontal and Oblique Asymptotes

This section considers limits at infinity (not to be confused with the infinite limits discussed in the previous section). That is, the behaviour of f (x) as x increases (or decreases) grows without bound.
Example 98 (revisited). Reproduced below are the graphs of the functions f and g,
1
and
both with domain (, 3) (3, ) and codomain R, defined by f x 2 +
(x 3)2
1
g x 2
.
(x 3)2
We already saw that f and g both have vertical asymptote x = 3, because as x , f (x)
increases without bound and g(x) decreases without bound. We now consider instead what
happens as x increases or decreases without bound.

horizontal asymptotes

As x increases without bound, f (x) 2 and g(x) 2. And as x decreases without


bound, f (x) 2 and g(x) 2. We can write these observations as lim f (x) = 2,
x
lim g(x) = 2, lim f (x) = 2, and lim g(x) = 2.

We also say that y = 2 is a horizontal asymptote of the graph of f . Similarly, y = 2 is a


horizontal asymptote of the graph of g. [See Section 88.4 in the Appendices (optional) for
the formal definition of a horizontal asymptote.]
Pedantic point: Infinite limits do not exist. In contrast, limits at infinity DO exist. Here
in this example, lim f (x) does not exist. In contrast, lim f (x) and lim f (x) both exist
x
x
x3
(and are both equal to 2).

Page 111, Table of Contents

www.EconsPhDTutor.com

Example 100. Graphed below is the equation y = ex .


As x , y 0. We can also write this as lim y = 0. And we can also say that this
x
graph has horizontal asymptote y = 0.

20

y = ex

15

10

Horizontal asymptote
x
y=0
0
-4

Page 112, Table of Contents

-2

www.EconsPhDTutor.com

The previous two examples were of horizontal asymptotes. Next is an example of an


oblique (or slant) asymptote.
1
Example 101. Consider the function f R R+ R defined by x x + .
x
As x increases without bound or decreases without bound, f (x) approaches the line
y = x. We can also write these observations as lim f (x) = x and lim f (x) = x.
x

We can moreover say that y = x is an oblique asymptote of the graph of g.


Again, see Section 88.4 in the Appendices (optional) for the formal definition of an oblique
asymptote.

1
x
-5

-3

-1

-1
Oblique
asymptote
y=x
-3

-5

Page 113, Table of Contents

www.EconsPhDTutor.com

9
9.1

Differentiation

Motivation: The Derivative as Slope of the Tangent

The problem of finding the derivative is the problem of finding the slope of the tangent to
a graph at a given point.
Graphed below is some function f R R. Pick some point A = (a, f (a)). Draw the line l
which is tangent to the graph at the point A.
How do we find the slope of l? Unsure of how to proceed, we try a crude approximation.
Pick some point X1 = (x1 , f (x1 )) that is also on the graph. Consider the line AX1 . Whats
f (x1 ) f (a)
its slope? Slope = Rise Run and so AX1 has slope
.
x1 a
This number serves as our first crude approximation of the slope of l.
How can we improve on this approximation? Simple just pick some point X2 = (x2 , f (x2 ))
f (x2 ) f (a)
that is closer to A. The line AX2 has slope
.
x2 a
This number serves as our second, improved approximation of the slope of l.

y
f (x1)

X1

l
f (x2)
X2
A
f (a)

y = f (x)

x
a

x2

x1

At least in theory, we can keep repeating this procedure, by picking points that are ever
closer to A. Our estimates of the slope of l will get ever better. Altogether then, we are
motivated to make the following formal definition of the derivative:
Page 114, Table of Contents

www.EconsPhDTutor.com

Definition 33. Let f D R be a function. Consider


f (x) f (a)
.
xa
xa
lim

If this limit exists, then we say that f is differentiable at the point a D and we call this
limit the value of f s derivative at the point a D.
But if this limit does not exist, then we say that f is not differentiable at the point a D
and the value of f s derivative at the point a D is undefined or does not exist.

Heres a simple example to illustrate.

Page 115, Table of Contents

www.EconsPhDTutor.com

Example 102. Graphed below is the function f R R defined by x x.

y
Derivative = -1 for a < 0

Derivative = 1 for a > 0


Derivative does not exist at a = 0.

The value of f s derivative at the point x = 5 is


f (x) f (5)
x 5
x 5
= lim
= lim
= lim 1 = 1.
x5
x5 x + 5
x5 x + 5
x5
x (5)
lim

Similarly, the value of f s derivative at the point x = 3 is also


x 3
x 3
f (x) f (3)
= lim
= lim
= lim 1 = 1.
x5 x + 3
x3 x + 3
x3
x3
x (3)
lim

Indeed, the value of f s derivative at any point a < 0 is 1, because for any a < 0,
f (x) f (a)
x a
x + a
= lim
= lim
= lim 1 = 1.
xa
xa x a
xa x a
xa
xa
lim

In contrast, for any a > 0, we have


f (x) f (a)
x a
xa
= lim
= lim
= lim 1 = 1.
xa x a
xa x a
xa
xa
xa
lim

At x = 0, f is not differentiable, as we now prove:

lim
= lim 1 = 1,

x0 x
x0

f (x) f (0)
x 0
x
lim
= lim
= lim
=
x0 x 0
x0 x
x0

x0

lim
= lim (1) = 1,

x0 x x0
So as x 0, there is no one single value towards which the expression
proaches. So the limit does not exist.
Page 116, Table of Contents

for x > 0,
for x < 0.
f (x) f (0)
apx0

www.EconsPhDTutor.com

9.2

Lagranges, Leibnizs, and Newtons Notation


The Value of the Derivative of f at a

Lagranges notation:

Leibnizs notation:

Newtons notation:

f (x) f (a)
.
xa
xa

f (a) = lim

R
f (x) f (a)
df (x) RRRR
RRR = lim
xa
dx RR
xa
Rx=a

or

R
f (x) f (a)
df RRRR
RRR = lim
.
xa
dx RR
xa
Rx=a

f (x) f (a)
.
xa
xa

f (a) = lim

Some remarks.
Lagranges and Leibnizs notation are widely-used. Newtons notation is not.
But Newtons notation is sometimes used in physics (especially when the independent
variable is time). You certainly need to know about Newtons notation because it is on the
A-level syllabus. Nonetheless, this textbook will avoid using Newtons notation.

d
Leibnizs notation is convenient in that it allows us to interpret
as the differentiate
dx
with respect to operator.
Section 9.5 will give some examples of how this operator works.

Here is the motivation behind Leibnizs notation.


Define x to be equal to x a, and f (x) to be equal to f (x) f (a), so that we can write:
f (x) f (x) f (a)
=
.
x
xa
The limit of this expression as x a is precisely the value of the derivative of f at a:
R
f (x) f (a)
f (x) df RRRR
= RRR = lim
.
lim
xa x
xa
dx RR
xa
Rx=a

Page 117, Table of Contents

www.EconsPhDTutor.com

Example 102 (revisited). Consider again the function f R R defined by x x.


a = 5

a=2

a=0

Lagranges notation: f (5) = 1,

f (2) = 1,

f (0) is undefined,

Leibnizs notation:

R
df RRRR
= 1,
R
dx RRRR
Rx=5

R
df RRRR
R = 1,
dx RRRR
Rx=2

R
df RRRR
is undefined,
R
dx RRRR
Rx=0

Newtons notation:

f (5) = 1,

f (2) = 1,

f (0) is undefined.

Historical Note (optional).


Here is a very oversimplified history of Leibnizs notation, to give you a better sense of why
we use it.
Leibniz (1646-1716) thought of dx as an infinitesimal change in x. And dy was the
corresponding infinitesimal change in y. Leibniz then defined the derivative to be literally
the quotient dy/dx. Unfortunately, the idea of infinitesimals was rather vague, imprecise,
and non-rigorous. So in the 19th century, mathematicians embarked on a project to put
calculus on a firmer footing. In particular, they wished to rid mathematics of all references
to infinitesimals. Eventually, they settled on the modern notion of limits, in which no
reference to infinitesimals was necessary. This modern notion of limits is also what youve
just learnt.
So simply put, Leibniz was wrong to think of the derivative as a fraction. And
you should be very careful not to think of the derivative as a fraction, even though it looks
very much like one.
You are now being taught things in the correct order. First you are taught about limits.
Next we define the derivative in terms of limits. We are careful to note that the derivative
is not a fraction.
But if Leibniz was wrong to think of the derivative as a fraction, then why are we still
using his notation? The main reason is that it is highly intuitive. In particular, it reminds
us of what calculus is really about how a small change in one variable affects another
variable. It also allows us to quickly grasp the intuition behind such results as the Chain
Rule, which may informally be stated as:
dz dz dy
=
.
dx dy dx
It is tempting to navely interpret the expressions in the above equation as fractions, navely
apply simple algebra, navely cancel out the dys, so that the equation is indeed true. But
Page 118, Table of Contents

www.EconsPhDTutor.com

the correct informal interpretation (easily seen when written in Leibnizs notation) is this:
The change in z caused by a small unit change in x is equal to The change in z caused
by a small unit change in y The change in y caused by a small unit change in x.
Another result is the Inverse Function Theorem, which may informally be stated as:
dy 1
= .
dx dx
dy
dy
dx
and
as fractions, so that indeed by nave
dx
dy
algebra, the above equation is true. But again, the correct informal interpretation (easily
seen when written in Leibnizs notation) is this: The change in y caused by a small unit
change in x is equal to The reciprocal of the change in x caused by a small unit change
in y.
Again, the nave interpretation would be of

For a more detailed discussion, see the leading answer to this question on Math StackExchange.

Page 119, Table of Contents

www.EconsPhDTutor.com

9.3

The Derivative is a Function

Above we defined the value of the derivative at a given point to be a number.


In contrast, we now define the derivative to be a function:
Definition 34. Let f D R be a function and A be the set of points at which f is
differentiable. Then the derivative of f is the function with domain A, the same codomain
as f (namely R), and mapping rule x f (x).

The Derivative of f
Lagranges notation:

f .

Leibnizs notation:

df (x)
dx

Newtons notation:

f.

or

df
.
dx

For the next example, I assume you already know that

d 2
d
cx = 2cx and
cx = c.
dx
dx

Example 103. Let f R R be defined by f (x) = 7x2 . Its derivative is the function
f R R defined by f (x) = 14x. This derivative may be denoted
f or

df (x)
df
or
or f .
dx
dx

df (x)
df

=
= f (0.5) = 1.75.
dx x=0.5 dx x=0.5

df (x)
df
The value of the derivative of f at 1 is f (1) =
= = f (1) = 7.
dx x=1 dx x=1

df (x)
df
The value of the derivative of f at 2 is f (2) =
= = f (2) = 28.
dx x=2 dx x=2

The value of the derivative of f at 0.5 is f (0.5) =

Page 120, Table of Contents

www.EconsPhDTutor.com

9.4

Second and Higher-Order Derivatives

The derivative is also known as the first derivative. The second derivative is, similarly, also
a function:
Definition 35. Let f D R be a function. The second derivative of f is simply the
derivative of the derivative of f .

The Derivative of f
Lagranges notation:

f .

Leibnizs notation:

d2 f (x)
dx2

Newtons notation:

f.

Under Leibnizs notation, since


derivative of f by

d2
d2 f
f
or
.
dx2
dx2

or

d2 f
.
dx2

d
is the operator, it makes sense to denote the second
dx

Example 103 (revisited). Let f R R be defined by x 7x2 . Its second derivative


is the function with domain and codomain both R, and mapping rule x 14. This second
derivative may be denoted
f or

d2 f (x)
d2 f
or
or
f
.
dx2
dx2

df (x)
df

=
= f (0.5) = 14.
dx x=0.5 dx x=0.5

df (x)
df
The value of the second derivative of f at 1 is f (1) =
= = f (1) = 14.
dx x=1 dx x=1

df (x)
df
The value of the second derivative of f at 2 is f (2) =
= = f (2) = 14.
dx x=2 dx x=2

The value of the second derivative of f at 0.5 is f (0.5) =

Page 121, Table of Contents

www.EconsPhDTutor.com

We similarly define the third, fourth, fifth, etc. derivatives in the obvious fashion.
Definition 36. Let f D R be a function. For n 3, the nth derivative of f is simply
the derivative of the (n 1)th derivative of f .

The 3rd
The 4th
Derivative of f Derivative of f
Lagranges notation:

f (3) .

f (4) .

Leibnizs notation:

d3 f
dx3 .

d4 f
dx4

Newtons notation:

f.

Etc.

f.

Example 103 (revisited). Let f R R be defined by x 7x2 . Its first derivative is the
function f R R defined by x 14x. Its second derivative is the function f R R
defined by x 14. We have f (2) = 28 and f (2) = 14.
Its third derivative is the function f (3) R R defined by x 0. Its fourth derivative is
the function f (4) R R defined by x 0. Observe that f (3) = f (4) . Indeed, the third and
all higher-order derivatives are identical functions: f (3) = f (4) = f (5) = . . .
We have f (3) (2) = f (4) (2) = f (5) (2) = = 0. Indeed, for any x R, we have f (3) (x) =
f (4) (x) = f (5) (x) = = 0.
Exercise 57. Given f D R and f A R, what is f ? (Answer on p. 1025.)
Exercise 58. (Tedious but easy.) Let g R R be defined by x x4 x3 + x2 x + 1.
Write down all of its derivatives. Evaluate all of these derivatives at 1. Write your answers
in Lagranges, Leibnizs, and Newtons notation. (Answer on p. 1025.)

Page 122, Table of Contents

www.EconsPhDTutor.com

9.5

More About Leibnizs Notation: The

Example 104.

d
Operator
dx

d 2
x = 2x is simply shorthand for this statement:
dx

The derivative of the function with mapping rule x x2 is the function with mapping rule
x 2x.

Example 105.

d
f = g is simply shorthand for this statement:
dx
The derivative of the function f is the function g.

Example 106.

d
f g = g f + f g is simply shorthand for this statement:
dx

The derivative of the function f g is the function with mapping rule


x g(x) f (x) + f (x) g (x).

Page 123, Table of Contents

www.EconsPhDTutor.com

9.6

Standard Rules of Differentiation

Proposition 1. Let f A R and g B R be differentiable functions with derivatives f


and g . Suppose also that the composite function f g A R is well-defined. Let k R be a
constant. Then:
d
dx

d
sin x =
dx

cos x,

d
f g = f g,
dx

d
cos x =
dx

sin x,

d
dx

kf

d
dx

f g

g f + f g,

d
dx

xk

= kxk1 ,

d
dx

f
g

g f f g
,
gg

d
dx

ex

ex ,

d
d (f g) dg
f g =
.
dx
dg
dx

d
dx

ln x

1
,
x

0,

kf ,

(My mnemonic for the Quotient Rule is: Lo-D-Hi minus Hi-D-Lo; cross over and square
the low.)
Proof. Optional, see p. 957 in the Appendices.

Of the above rules, the Chain Rule is the most powerful. We can also write it more elegantly
(if a little imprecisely) as
dz dz dy
=
.
dx dy dx
As discussed above in the historical note (p. 118), thus written, the Chain Rule has a
beautiful informal interpretation: The change in z caused by a small unit change in x is
equal to The change in z caused by a small unit change in y The change in y caused
by a small unit change in x. This makes perfect sense:

Page 124, Table of Contents

www.EconsPhDTutor.com

Example 107. When I add 1 g of Milo (the x-variable) to a cup of water, the volume of
dy
the water increases by 2 cm3 (the y-variable). That is,
= 2 cm3 g-1
dx
When the volume of the water increases by 1 cm3 (the y-variable), the water level (in the
dz
cup) rises by 0.3 cm (the z-variable). That is
= 0.3 cm cm-3 = 0.3 cm-2 .
dy
Altogether then, when I add 1 g of Milo (the x-variable) to a cup of water, I should expect
dz
the water level to rise by 0.6 cm. That is,
= 0.6 cm g-1 . This is indeed consistent with
dx
dz dz dy
=
= 2 0.3 = 0.6 cm g-1 .
dx dy dx
In case youve forgotten how it works, here are a few examples to illustrate:
Example 108. Let h R R be defined by x esin x .
desin x desin x d sin x
h (x) =
=
= esin x cos x.
dx
d sin x dx

Another simple example:


Example 109. Let g R R be defined by x

4x 1.

d 4x 1 d 4x 1 d (4x 1)
0.5
0.5
g (x) =
=
= 0.5 (4x 1)
4 = 2 (4x 1)
.
dx
d (4x 1)
dx

Heres a more complicated example, where the Chain Rule is applied twice.
3

Example 110. Let f R R be defined by x [sin(2x 3) + cos(5 2x)] . Then


3

d [sin(2x 3) + cos(5 2x)]


f (x) =
dx
3
d [sin(2x 3) + cos(5 2x)] d [sin(2x 3) + cos(5 2x)]
=
d [sin(2x 3) + cos(5 2x)]
dx
d cos(5 2x) d(5 2x)
2 d sin(2x 3) d(2x 3)
= 3 [sin(2x 3) + cos(5 2x)] [
+
]
d(2x 3)
dx
d(5 2x)
dx

= 3 [sin(2x 3) + cos(5 2x)] [cos(2x 3) 2 sin(5 2x) (2)]


2

= 6 [sin(2x 3) + cos(5 2x)] [cos(2x 3) + sin(5 2x)] .

Page 125, Table of Contents

www.EconsPhDTutor.com

Exercise 59. For each of the following functions (assume they have a suitably defined
domain and codomain), evaluate the first derivative at 0. (a) f (x) = x2 . (b) g(x) =
x
2
1 + [x ln (x + 1)] . (c) h(x) = sin
2 . (Answer on p. 1026.)
1 + [x ln (x + 1)]

Corollary 1.

d
d
d
tan x = sec2 x,
cot x = csc2 x, and
csc x = csc x cot x.
dx
dx
dx

Proof. Using the quotient rule,


d
d sin x cos x cos x sin x( sin x) cos2 x + sin2 x
1
tan x =
=
=
=
= sec2 x.
2
2
2
dx
dx cos x
cos x
cos x
cos x
For the derivatives of cot x and csc x, see Exercise 60.

SYLLABUS ALERT
d
csc x = csc x cot x is in the List of Formulae for 9758 (revised), but not for 9740 (old).
dx

Exercise 60. Prove the following: (Answer on p. 1026.)


d
cot x = csc2 x and
dx

d
csc x = csc x cot x.
dx

Exercise 61. (Answer on p. 1026.)


(a) Newtons Second Law of Motion is that force is equal to the rate of change of momentum,
where momentum is the product of mass and velocity. Write down this law in mathematical
notation, with F , m, v, and t denoting force, mass, velocity, and time.
(b) Assume that mass is constant. Explain why Newtons Second Law then simplifies into
the more-familiar F = ma, where a is acceleration (i.e. the rate of change of velocity).

Page 126, Table of Contents

www.EconsPhDTutor.com

9.7

Differentiable and Twice-Differentiable Functions

Definition 37. A function is differentiable on a set of points if it is differentiable at every


point in that set.

Definition 38. A function is differentiable if it is differentiable on its domain.

Definition 39. A function is twice-differentiable on a set of points if it is twice-differentiable


at every point in that set.

Definition 40. A function is twice-differentiable if it is twice-differentiable on its domain.

In other words, f is differentiable if and only if f has the same domain as f . Similarly, f
is twice-differentiable if and only if f has the same domain as f .
And of course, if a function is twice-differentiable, then it is also differentiable.
(The definitions for a function to be thrice-differentiable, four-times-differentiable,
etc. are very much analogous, but this textbook will have no reason to use these terms.)
The condition that the first derivative (or second derivative) exists at every point in the
domain is important. Failing which, we do not consider the function to be differentiable
(or twice-differentiable). The three functions in the next example illustrate:

Page 127, Table of Contents

www.EconsPhDTutor.com

Example 111. Consider f R R defined by f (x) = x2 . We have f (x) = 2x and f (x) = 2


for all x R. And so f is both differentiable and twice-differentiable.
Now consider g R R defined by g(x) = x x (graphed below). We have g (x) = 2 x for
all x R and

2,

g (x) =

2,

for x < 0,
for x > 0.

But g (0) does not exist. And so g is differentiable but NOT twice-differentiable.

, for all .
x
- 2, for x < 0,
2, for x > 0.
is undefined.

Consider h R R defined by x x. We have

2,

h (x) =

2,

for x < 0,
for x > 0.

But h (0) does not exist. So h is not even once-differentiable. (And thus it is certainly not
twice-differentiable either.)

Page 128, Table of Contents

www.EconsPhDTutor.com

We can of course also consider thrice-differentiable, four-times-differentiable, etc. functions. We can even consider infinitely-differentiable functions. Indeed, in the A-levels, most
functions are usually infinitely differentiable. For example, all polynomials are infinitelydifferentiable, as illustrated in the next example.
Example 112. Consider i R R defined by x x5 x4 + x3 x2 + x 1. We have, for all
x R,
i (x) = 5x4 4x3 + 3x2 2x + 1,

i(4) (x) = 120x 24,

i (x) = 20x3 12x2 + 6x 2,

i(5) (x) = 120,

i(3) (x) = 60x2 24x + 6,

i(6) (x) = i(7) (x) = i(8) (x) = 0.

The function i is infinitely-differentiable, with the 6th and higher-order derivatives all having the mapping rule x 0.

Simple exponential functions are also infinitely differentiable:


Example 113. Consider j R R defined by x ex . We have, for all x R,
j (x) = j (x) = j (3) (x) = j (4) (x) = = ex .
The function j is infinitely-differentiable, with every derivative simply being the same
function as j.

Page 129, Table of Contents

www.EconsPhDTutor.com

9.8

Differentiability Implies (i.e. is Stronger Than) Continuity

Informally, continuity is a smoothness condition if a function is continuous, then its


graph has no holes or jumps anywhere and can be drawn smoothly without lifting
your pencil.
Differentiability is a stronger smoothness condition. If a function is differentiable,
then its graph is continuous (i.e. has no holes or jumps) and moreover has no kinks
or other abrupt turns.
Example 114. Graphed below are the functions f , g, and h.
f is both continuous and differentiable.
g is continuous you can draw its entire graph without lifting your pencil. However, it is
not differentiable because of the kink.
h is neither continuous nor differentiable, because of the hole.

y
h is neither continuous
nor differentiable.
f is both continuous
and differentiable.

g is continuous, but
not differentiable.

Theorem 1. If f D R is differentiable at a D, then f is continuous at a D.


Proof. Optional, see p. 961 in the Appendices.

Page 130, Table of Contents

www.EconsPhDTutor.com

9.9

Implicit Differentiation

Example 115. Consider the equation x2 + y 2 = 1. What is

dy
?
dx

Method #1. First write y in terms of x: y = 1 x2 . Then differentiate:


dy
2x
x
x
=
=
=
.
dx
2 1 x2
1 x2
1 x2
Method #2 (implicit differentiation). Directly apply

d
to the given equation:
dx

d
d
dy
dy
x
(x2 + y 2 ) = (1) 2x + 2y
= 0
= .
dx
dx
dx
dx
y

If desired, we can plug in y = 1 x2 to get the same answer as before:


x
dy
x
=
.
=
dx
1 x2
1 x2
In the above example, the second method (implicit differentiation) is not obviously superior
to the first. However, it is sometimes difficult (or impossible) to express y in terms of x.
Nonetheless we might still want to compute dy/dx. In such cases, the method of implicit
differentiation is wonderful. The next example illustrates:

Example 116. Consider the equation x2 y +


when evaluated at x = 0)?

y
dy
dy
= 1. What is

(i.e. what is
cos x
dx x=0
dx

In this example, its difficult to express y in terms of x. But this doesnt matter, because
we can use implicit differentiation:

d
y
d
1 dy y( sin x) cos x dx
(x2 y +
) = (1) 2x y + x2
+
= 0.
dx
cos x
dx
2 y dx
cos2 x
dy

Now plug in x = 0:

1 dy y( sin 0) cos 0 dx
dy
2 0 y + 02
+
=
0

= 0.
2 y dx
cos2 0
dx
dy

Page 131, Table of Contents

www.EconsPhDTutor.com

The four rules of differentiation in the next corollary are in the List of Formulae you get
during A-level exams (both 9740 and 9758), so you need not know these by heart.
d
1
d
1
d
sec x = sec x tan x,
sin1 x =
,
cos1 x =
, and
dx
dx
1 x2 dx
1 x2
d
1
tan1 x =
.
dx
1 + x2

Corollary 2.

d
1
, first rewrite y = sin1 x as x = sin y. Next
sin1 x =
2
dx
1x
d
dy
then apply
(implicit differentiation) to get 1 = cos y . But sin2 y + cos2 y = 1, so
dx
dx
2
cos y = 1 x . And so,
Proof. To prove that

dy
d
1
1
.
=
sin1 x =
=
dx dx
cos y
1 x2
Exercise 62 asks you the prove the derivatives of sec x, cos1 x and tan1 x are as claimed.

d
d
1
d
sec x = sec x tan x,
cos1 x =
, and
tan1 x =
2
dx
dx
dx
1x
1
. (Answer on p. 1026.)
1 + x2

Exercise 62. Prove that

Page 132, Table of Contents

www.EconsPhDTutor.com

10
10.1

Increasing, Decreasing, and f

When a Function is Increasing or Decreasing

Example 117. Consider the function f R R defined by x x2 . It is decreasing on R0 ,


increasing on R+0 , strictly decreasing on R , and strictly increasing on R+ .

y
Decreasing on
Strictly decreasing on

Increasing on
Strictly decreasing on

Both increasing and decreasing at x = 0.

Note: At x = 0, f is both decreasing and increasing, but neither strictly decreasing nor
strictly increasing. This follows from the formal definitions (below).

Definition 41. Given a function f and a set of points S, we say that f is ...
1. ... increasing on S if for any x1 , x2 S with x2 > x1 , we have f (x2 ) f (x1 );
2. ... strictly increasing on S if for any x1 , x2 S with x2 > x1 , we have f (x2 ) > f (x1 );
3. ... decreasing on S if for any x1 , x2 S with x2 > x1 , we have f (x2 ) f (x1 );
4. ... strictly decreasing on S if for any x1 , x2 S with x2 > x1 , we have f (x2 ) < f (x1 );
Of course, if a function is strictly increasing on a set of points, then it is also increasing on
that set. And if it is strictly decreasing, then it is also decreasing.

Exercise 63. Let g R R defined by x sin x. Identify the sets on which which g is
increasing, decreasing, strictly increasing and/or strictly decreasing. (Answer on p. 1027.)

Page 133, Table of Contents

www.EconsPhDTutor.com

10.2

The First Derivative Increasing/Decreasing Test

The derivative is the slope of the tangent. And so not surprisingly, the derivative is intimiately related to whether a function is increasing or decreasing. Formally:
Fact 10. Let f R R be a differentiable function. Let a, b R with b > a. Then
1. f is decreasing on (a, b) f (x) 0, for all x (a, b).
2. f is increasing on (a, b) f (x) 0, for all x (a, b).

3. f is strictly decreasing on (a, b) f (x) < 0, for all x (a, b).


4. f is strictly increasing on (a, b) f (x) > 0, for all x (a, b).
5. f is both increasing and decreasing at a f (a) = 0.

Proof. Optional, see p. 962 in the Appendices.

Example 133 (revisited). Consider again f R R defined by x x2 .


1. f is decreasing on R0 , and so f (x) 0 for x 0.

2. f is increasing on R+0 , and so f (x) 0 for x 0.


3. f is strictly decreasing on R0 , and so f (x) < 0 for x 0.
4. f is strictly increasing on R+0 , and so f (x) > 0 for x 0.

5. f is both increasing and decreasing at x = 0, and so f (x) = 0.

, for

, for

Both increasing and decreasing at x = 0:

Page 134, Table of Contents

, for

, for

www.EconsPhDTutor.com

11

Extreme, Stationary, and Turning Points


11.1

Maximum and Minimum Points

Let f D R and x D. Informally:21


1. If f (x) f (a) for all a D that are close to x, then we call x a maximum point of
f and f (x) a maximum value.
2. If f (x) f (a) for all a D that are close to x, then we call x a minimum point of
f and f (x) a minimum value.
3. If f (x) > f (a) for all a D that are close to x, then we call x a strict maximum
point of f and f (x) a strict maximum value.
4. If f (x) < f (a) for all a D that are close to x, then we call x a strict minimum
point of f and f (x) a strict minimum value.
Of course, a strict maximum point is also a maximum point. And a strict minimum point
is also a minimum point.
Any maximum or minimum point is also known as an extremum (plural: extrema) or an
extreme point.

21

See p. 962 in the Appendices for the formal definitions.

Page 135, Table of Contents

www.EconsPhDTutor.com

Example 118. Below is graphed f R R defined by f (x) = (x1)2 . x = 1 is a maximum


point and a strict maximum point of f . The corresponding maximum value (and also strict
maximum value) is f (1) = 0.
Also graphed is g R R defined by g(x) = (x + 1)2 . x = 1 is a minimum point and a strict
minimum point of g. The corresponding minimum value (and also strict minimum value)
is g(1) = 0.

x = -1
minimum
point for g

x=1
maximum
point for f

Page 136, Table of Contents

www.EconsPhDTutor.com

A function can have multiple maximum and multiple minimum points:


Example 119. Graphed below is h R R defined by x 6x5 15x4 10x3 + 30x2 .
x = 1 is a maximum point and a strict maximum point of h. The corresponding
maximum value (and also strict maximum value) is h(1) = 19.
x = 1 is a maximum point and a strict maximum point of h. The corresponding maximum
value (and also strict maximum value) is h(1) = 11.
x = 0 is a minimum point and a strict minimum point of h. The corresponding minimum
value (and also strict minimum value) is h(0) = 0.
x = 2 is a minimum point and a strict minimum point of h. The corresponding minimum
value (and also strict minimum value) is h(2) = 8.

y
x = 1
maximum points

x
-2

-1

x = 0, 2
minimum points

Page 137, Table of Contents

www.EconsPhDTutor.com

The next example highlights the fact that a maximum point is sometimes not a strict
maximum point. Likewise with minimum points.
Example 120. Below is graphed i R R defined by x 3. (This is a constant
function.)
Every point x R is a maximum point of i. The corresponding maximum value is always
i(x) = 3.
But no point is a strict maximum point.
Every point x R is a minimum point of i. The corresponding minimum value is always
i(x) = 3.
But no point is a strict minimum point.

Every point is a
maximum point.

Every point is a
minimum point.
x
-2

Page 138, Table of Contents

-1

www.EconsPhDTutor.com

11.2

Global Maximum and Minimum Points

Definition 42. Let f D R and a D.


1. If f (a) f (x) for all x D, we call a the global maximum point of f and f (a) the global
maximum value.
2. If f (a) f (x) for all x D, we call a the global minimum point of f and f (a) the global
minimum value.
3. If f (a) > f (x) for all x D/{a}, we call a the strict global maximum of f and f (a) the
strict global maximum value.
4. If f (a) < f (x) for all x D/{a}, we call a the strict global minimum of f and f (a) the
strict global minimum value.

The next fact is perhaps obvious:


Fact 11. There cannot be more than one strict global maximum point of a function. (Similarly, there cannot be more than one strict global minimum point of a function.)

Proof. Suppose for contradiction that two distinct points x1 and x2 are strict global maximum points of f . Then since x1 is a strict global maximum point, we have f (x1 ) > f (x2 ).
Similarly, since x2 is a strict global maximum point, we have f (x2 ) > f (x1 ). The two
inequalities are contradictory. So it is impossible that two distinct points x1 and x2 are
strict global maximum points of f .

Page 139, Table of Contents

www.EconsPhDTutor.com

Example 119 (revisited). Consider again the function h R R defined by x


6x5 15x4 10x3 + 30x2 . (Graph reproduced below for convenience.)
x = 1 are maximum points. However, they are not global maximum points. Indeed, h
has no global maximum point because lim h(x) = (as x increases without bound, h(x)
x
also increases without bound). In other words, there is no x such that h(x) h(a) for all
a R.
Similarly, x = 0, 2 are minimum points. However, they are not global minimum points.
Indeed, h has no global minimum point because lim h(x) = (as x decreases without
x
bound, h(x) also decreases without bound). In other words, there is no x such that
h(x) h(a) for all a R.
y
x = 1
maximum points

x
-2

-1

x = 0, 2
minimum points

We next restrict the domain of h in two ways to create two new functions i and j:

Page 140, Table of Contents

www.EconsPhDTutor.com

Example 119 (revisited). Graphed below (left) is the function i [1.5, 2.5] R defined
by x 6x5 15x4 10x3 + 30x2 .
i has three maximum points in total, namely 1, 2.5. However, only 2.5 is a global maximum
point of i because only i(2.5) i(x) for all x [1.5, 2.5]. Of course, it is also a strict global
maximum point because i(2.5) > i(x) for all x [1.5, 2.5].
i has three minimum points in total, namely 1.5, 0, 2. However, only 1.5 is a global
maximum point of i because only i(1.5) i(x) for all x [1.5, 2.5]. Of course, it is also
a strict global minimum point because i(1.5) < i(x) for all x [1.5, 2.5].

x = 1
max

x = 2.5
max and
global max

x = -1
max and
global max x = 1, 1.2
max

x
-2

-1

0
1
2
x = -1.5
min and
global min x = 0, 2 min

x
-2

-1

x = -1.2, 0 min

x = 2 min and
global min

Also graphed above (right) is the function j [1.2, 2.2] R defined by x 6x5 15x4
10x3 + 30x2 .
Again, there are three maximum points in total, namely 1, 2.2. However, only 1 is a
global maximum point of j because only j(1) j(x) for all x [1.2, 2.2]. Of course, it is
also a strict global maximum point because j(1) > i(x) for all x [1.2, 2.2].
And again, there are three minimum points in total, namely 1.2, 0, 2. However, only 2 is
a global minimum point of j because only j(2) j(x) for all x [1.2, 2.2]. Of course, it is
also a strict global minimum point because j(2) < j(x) for all x [1.2, 2.2].

Page 141, Table of Contents

www.EconsPhDTutor.com

Note that the A-level syllabuses and exams only ever talk about maximum and minimum
points. They do not ever talk about
1. Strict maximum points;
2. Strict minimum points;
3. Global maximum points;
4. Global minimum points;
5. Strict global minimum points; and
6. Strict global maximum points.
Nonetheless, these concepts are not difficult to grasp. It is thus well worth learning them,
just so you have a better understanding of how to find maximum and minimum points.
Note also that what we simply call maximum and minimum points are sometimes instead
called local maximum and minimum points, so that they are better contrasted with global
maximum or minimum points.

Exercise 64. (Answer on p. 1027.) For each of the following functions, write down,
if any of these exist, the (i) maximum points, (ii) minimum points, (iii) strict maximum
points, (iv) strict minimum points, (v) global maximum points, (vi) global minimum points,
(vii) strict global maximum points, (viii) strict global minimum points; and also all the
corresponding values of the function at these points.
(a) f R R defined by x 100.
(b) g R R defined by x x2 .
(c) h [1, 2] R defined by x x2 .

Page 142, Table of Contents

www.EconsPhDTutor.com

11.3

Stationary and Turning Points

Definition 43. A point x is a stationary point of f if f (x) = 0.


Graphically, a stationary point is where the slope of the tangent is 0 (flat).
Definition 44. A turning point is any point that is both a stationary point and a maximum
or minimum point.

So every turning point is both a stationary point and an extreme point.


But the converse is not true: A stationary point need not always be a turning point. And
an extreme point need not always be a turning point.

Page 143, Table of Contents

www.EconsPhDTutor.com

Example 121. Graphed below is the function f [1.5, 0.5] R defined by x x5 +2x4 +x3 .
Five points are labelled. The table below classifies each point.
D is a stationary point but not a turning point. (As we shall learn in Section 151, D is an
example of an inflexion point.)
A is a minimum point and E is a maximum point. But neither is a turning point.
Type
Max
Min
Strict Max
Strict Min
Global Max
Global Min
Strict Global Max
Strict Global Min
Stationary
Turning

A B C D E

y
E

C
f (x) = x5 + 2x4 + x3
A

Exercise 65. Is each of the following statements true or false? To show that a statement
is false, simply give a counterexample from the above example. If it is true, explain why.
(Answer on p. 1028.)
(a) Every maximum point or minimum point is a stationary point.
(b) Every maximum point or minimum point is a turning point.
(c) Every stationary point is a maximum point or minimum point.
(d) Every turning point is a maximum point or minimum point.
(e) Every turning point is a stationary point.
(f) Every stationary point is a turning point.

Page 144, Table of Contents

www.EconsPhDTutor.com

11.4

The Interior Extremum Theorem

Informally, a point x S is in the interior of a set S if x is not at the edge of S. Formally:


Definition 45. x S is in the interior of S if there exists such that (x , x + ) S.
x S is a non-interior point of S if it is not in the interior of S.

Example 122. Consider the set S = [0, 1]. The points 0.2, 1/3, and 0.775 are all in the
interior of S. Indeed, every point x (0, 1) is in the interior of S.
In contrast, the points 0 and 1 are non-interior points of S.
Example 123. Consider the set S = [0, 0.5) (0.5, 1]. The points 0.2, 1/3, and 0.775 are all
in the interior of S. Indeed, every point x (0, 0.5) (0.5, 1) is in the interior of S.
In contrast, the points 0 and 1 are non-interior points of S.
The point 0.5 is not in the interior of S. It is not even a non-interior point of S, because
it is not in the set S to begin with.

Page 145, Table of Contents

www.EconsPhDTutor.com

The Interior Extremum Theorem (IET) is the fundamental reason why we lurrrve taking
derivatives and setting them equal to zero this is a great way to find maxima and minima!
Theorem 2. (Interior Extremum Theorem [IET].) Let f D R be a differentiable
function. If a is a maximum or minimum point AND in the interior of D, then f (a) = 0
(i.e. c is a stationary point).

Proof. Optional, see p. 963 in the Appendices.

Heres a non-rigorous explanation of the intuition behind the IET:


Example 136 (revisited). Graphed below is f R R defined by x (x 1)2 . Heres
the intuition for why f (0) = 0:
In order for 1 to be a maximum point of f , it must be that to its left, f is increasing; while
to its right, f is decreasing. In other words, to the left of 1, f (x) 0. While to the right of
1, f (x) 0. Altogether then, we must have f (1) = 0 at the maximum point, the slope
of the function must be 0.

x = -1
minimum
point for g

x=1
maximum
point for f

Exercise 66. Refer to the above Example. Explain the intuition for why g (1) = 0.
(Answer on p. 1028.)

Exercise 67. True or false: Let f D R be a differentiable function. If c is a maximum


or minimum point AND in the interior of D, then x is a turning point. (Answer on p.
1028.)

Page 146, Table of Contents

www.EconsPhDTutor.com

11.5

How to Find Maximum and Minimum Points

In secondary school, you may have been taught that to find the maximum and minimum
points of f , simply follow this procedure:

The Incorrect Recipe for Finding Maximum and Minimum Points.


Given a differentiable function f D R,
1. Compute f (x). Find the points x at which f (x) = 0.
2. These points are also the maximum and minimum points. (If we also want to know
which are maximum and which are minimum points, then simply employ some method
like sketch-the-graph or the Second Derivative Test.)

Unfortunately, the above procedure (lets call it the Incorrect Recipe) may sometimes
fail. It rests on the false belief that f (x) = 0 x is an extremum. This is false
because
1. The IET does NOT say, f (x) = 0 x is an extremum. It is perfectly possible that
f (x) = 0 without x being an extremum.

2. The IET does NOT say, x is an extremum f (x) = 0 . Instead, it says, x is an


extremum AND an interior point f (x) = 0. Thus, it is perfectly possible that x
is an extremum without f (x) = 0.
Here is an example to illustrate these two failings of the Incorrect Recipe.

Page 147, Table of Contents

www.EconsPhDTutor.com

Example 144 (revisited). Graphed below is the function f [1.5, 0.5] R defined by
x x5 + 2x4 + x3 . Five points are labelled.
According to the Incorrect Recipe,
1. Compute f (x) = 5x4 + 8x3 + 3x2 = x2 (5x2 + 8x + 3) = x2 (5x + 3)(x + 1). We see that
3
f (x) = 0 x = , 1, 0.
5
3
2. So , 1, 0 are the maximum and minimum points of f .
5
The Incorrect Recipe does correctly identify the points B = (1, f (1)) and C =
3
3
( , f ( )) as maximum and minimum points, respectively. But it makes two mistakes.
5
5
Mistake #1: D = (0, 0) is neither a maximum nor a minimum point, contrary to the
Incorrect Recipe.
Mistake #2: A and E are respectively a minimum and a maximum point, but neither is
detected by the Incorrect Recipe.

y
E

C
f (x) = x5 + 2x4 + x3
A

We now give the Correct Recipe for finding maximum and minimum points:

Page 148, Table of Contents

www.EconsPhDTutor.com

The Correct Recipe for Finding Maximum and Minimum Points.


Given a differentiable function f D R,
1. Identify all the stationary points (i.e. x where f (x) = 0).
2. Identify all the non-interior points.
3. Check to see if each stationary point and each non-interior point is a maximum point,
a minimum point, or neither. (To do so, employ some method like sketch-the-graph or
the Second Derivative Test.)

The Correct Recipe rectifies the Incorrect Recipe in two ways:


1. The Correct Recipe demands that you also check the non-interior points, which may
possibly be maximum or minimum points, but may not be detected by the Incorrect
Recipe.
2. The Correct Recipe does not assume that every single one of our shortlist of points (the
stationary points and the non-interior points) is either a maximum point or a minimum
point. It allows for the possibility that some of these points could be neither.
By the way, the condition that f is differentiable is very important. If f is not
differentiable, then the above Correct Recipe might not work. But not to worry,
since most functions on the A-levels are usually differentiable.
Example 124. Consider f [1, 1] R defined by x x3 . Lets apply the Correct Recipe.
1. Identify all the stationary points (i.e. x where f (x) = 0).
f (x) = 3x2 . So f (x) = 0 x = 0. The only stationary point is x = 0.
2. Identify all the non-interior points.
Every point x (1, 1) is in the interior of [1, 1]. The only non-interior points are 1 and
1.
3. Check if each of these points is a maximum point, a minimum point, or neither.
From a sketch of the graph, we see that the stationary point x = 0 is neither a maximum
nor a minimum point. The non-interior point 1 is a minimum point. The non-interior
point 1 is a maximum point.
Altogether, we conclude that 1 is the only minimum point and 1 is the only maximum
point.

Page 149, Table of Contents

www.EconsPhDTutor.com

Exercise 68. For each of the following functions, find all the maximum and minimum
points using the Correct Recipe. (Answer on p. 1029.)
(a) f R R defined by x x.
(b) g [0, 1] R defined by x x.
(c) h R R defined by x x4 2x2 . Identify also the global minimum point(s) of h (if
any exist).

Page 150, Table of Contents

www.EconsPhDTutor.com

12

Concavity, Inflexion Points, and the 2DT

A function is concave downwards (or simply concave) on an interval if the line


segment connecting any two points of the graph in this interval is below the graph.
A function is concave upwards (or simply convex) on an interval if the line segment
connecting any two points of the graph in this interval is above the graph.
An inflexion point is any point where the concavity of the function changes, either
from downwards to upwards, or upwards to downwards.22

Example 125. Graphed below is f R R defined by x x3 .


f is concave downwards on R0 because there, the line segment connecting any two points
on f is below the graph of f .

y
Tangent line at x = 0
is concave upwards on
x
-2

-1

is concave downwards on

In contrast, f is concave upwards on R+0 because there, the line segment connecting any
two points on f is above the graph of f .
0 is an inflexion point because this is where the function f changes from being concave
downwards to being concave upwards.
A test for whether a point is an inflexion point is this: Draw the tangent line to the graph
at that point. The point is an inflexion point The line is above the graph on one side
of the point and below the graph on the other side (see Fact 95 in the Appendices).
The tangent line to the graph at the point 0 is drawn in green (it coincides with the
horizontal axis). We indeed see that the line is above the graph on the left side of the point
and below the graph on the right side of the point. Therefore, 0 is an inflexion point.
22

These are informal definitions. For the formal definitions, see p. 964 in the Appendices (optional).

Page 151, Table of Contents

www.EconsPhDTutor.com

For a graph to be concave downwards, its slope must be decreasing. Conversely, to be


concave upwards, its slope must be increasing. Altogether then, the following proposition
is intuitively plausible.
Proposition 2. Let f D R be a twice-differentiable function.
(a) f is concave downwards on an interval f (x) 0 for every x in this interval.
(b) f is convex upwards on an interval f (x) 0 for every x in this interval.
(c) x is an inflexion point f (x) = 0.

Proof. Optional, see p. 967 in the Appendices.

Example 151 (revisited). Consider f R R defined by x x3 . f is concave downwards


on R0 , concave upwards on R+0 , and has an inflexion point at x = 0.
We can verify that, as per the above proposition:

< 0,

f (x) = 3x2 = 0,

> 0,

Page 152, Table of Contents

for x R0 ,
for x = 0,
for x R+0 .

www.EconsPhDTutor.com

It is very tempting to believe that the converse of part (c) of the above proposition is true.
That is, it is very tempting to believe that
f (x) = 0 x is an inflexion point.
But this is wrong! It is perfectly possible that f (x) = 0 without x being an inflexion
point! Heres an example:
Example 126. Consider g R R defined by x x4 . We have g (x) = 4x3 and g (x) =
12x2 , so that g (x) = 0 x = 0.
We might thus be tempted to conclude that 0 is an inflexion point. However, this is not
the case. Although g (0) = 0, we have g (x) > 0 for x > 0 and we also have g (x) > 0 for
x < 0, and so the concavity of g does not change at the point 0.
To qualify as an inflexion point, the concavity of the function must change. At 0,
the concacivty of g does not change. Therefore, 0 is NOT an inflexion point.

y
g is concave upwards everywhere

.
However, is not an
inflexion point of .
x

Page 153, Table of Contents

www.EconsPhDTutor.com

Inflexion points may be further sub-divided into stationary points of inflexion and
non-stationary points of inflexion.
Definition 46. A stationary point of inflexion is simply any point that is both an inflexion
point and a stationary point.
A non-stationary point of inflexion is simply any point that is an inflexion point, but not
a stationary point.

The A-level syllabuses explicitly exclude non-stationary points of inflexion. Nonetheless,


there is the temptation to believe that every inflexion point must also be a stationary
point. Heres a quick counter-example that dispels this belief:
Example 127. The graph below is for the function f R R defined by x x3 + x.
We have f (x) = 3x2 + 1 and f (x) = 6x. The point 0 is not a stationary point because
f (0) = 1 0.
However, 0 is an inflexion point, because to the left of 0, f is concave downwards; and to
the right, f is concave upwards. So 0 is a point of inflexion. Indeed, it is a non-stationary
point of inflexion.
Also illustrated is the tangent line at y = x (whose slope is indeed non-zero). Observe that
indeed, to the left of 0, the tangent line is above the graph; while to the right of 0, the
tangent line is below the graph. This serves as a second way to verify that 0 is a point of
inflexion.

Concave upwards on

x
Tangent line at 0
Concave downwards on

Page 154, Table of Contents

www.EconsPhDTutor.com

12.1

The Second Derivative Test (2DT)

f must be concave upwards


around the minimum
turning point 0.
So

f must be concave downwards


around the maximum
turning point 0.

for all x near 0.

So

for all x near 0.

From graphs, it looks like around a maximum turning point a, f must be concave downwards, i.e. f (a) < 0. Similarly, around a minimum turning point b, f must be concave
upwards, i.e. f (b) > 0. The next proposition is thus intuitively plausible.

Proposition 3. (Second Derivative Test [2DT].) Let f be a twice-differentiable function. Let a be a stationary point (i.e. f (a) = 0).
1. If f (a) < 0, then a is a maximum point.
2. If f (a) > 0, then a is a minimum point.

3. If f (a) = 0, then the 2DT is uninformative. That is, a could be a maximum point, a
minimum point, an inflexion point, or something else altogether!

Proof. Optional, see p. 968 in the Appendices.

The third part of the above Proposition must be heavily emphasised: If f (a) = 0 and
f (a) = 0, then the 2DT tells us absolutely nothing about a! a could be a maximum
point, a minimum point, an inflexion point, or something else altogether!
We previously gave the Correct Recipe for finding maximum and minimum points. Lets
now add the 2DT to this recipe:

Page 155, Table of Contents

www.EconsPhDTutor.com

The Enriched Recipe for Finding Maximum and Minimum Points.


Given a twice-differentiable function f D R,
1. Identify all the stationary points (i.e. a where f (a) = 0).
(a) Evaluate f at each of these points.
(b) f (a) < 0 a is a maximum point. Conversely, f (a) > 0 x is a minimum
point. If f (a) = 0, then we need to determine the nature of a using some other
method (e.g. sketch-the-graph).
2. Identify all the non-interior points.
(a) Check if each of these points is a maximum point, a minimum point, or neither.

If f is not twice-differentiable, then the Enriched Recipe may not work. Fortunately, most functions in A-levels are twice-differentiable.
Example 121 (revisited). Consider f [1.5, 0.5] R defined by x x5 + 2x4 + x3 .
1. Identify all the stationary points. f (x) = 5x4 + 8x3 + 3x2 = x2 (5x2 + 8x + 3) = 0
x = 0 or x = 1, 0.6 (quadratic formula).
(a) f (x) = 20x3 + 24x2 + 6x = 2x(10x2 + 12x + 3).
(b) f (0.6) > 0 0.6 is a minimum point. f (1) < 0 1 is a maximum
point. But f (0) = 0, so the 2DT tells us nothing. By sketching the graph (nonrigorous method that will suffice for the A-levels), we see that 0 is an inflexion
point.
2. The only two non-interior points are 1.5 and 0.5. Again by sketching the graph, we see
that 1.5 is a minimum point and 0.5 is a maximum point.
Altogether, we conclude that there are two maximum points 1 and 0.5 and two
minimum points 0.6 and 1.5.
Exercise 69. Use the Enriched Recipe to find the maximum and minimum points of each
of the following functions. (Answer on p. 1031.)
(a) g R R defined by x x8 + x7 x6 .

(b) h ( , ) R defined by x tan x.
2 2
(c) i [0, 2] R defined by x sin x + cos x.

Page 156, Table of Contents

www.EconsPhDTutor.com

12.2

Summary of Points and Venn Diagram

The Venn diagram below depicts the five types of points you need to know for the A-levels:
Inflexion, maximum, minimum, stationary, and turning points. To its right is a graph of a
rather-arbitrary function t D R designed to illustrate these various points. The x- and
y-coordinates of a are denoted ax and ay ; similarly for other points.

a
b

Inflexion

All
points

y
e

Stationary

i
f
h

Turning

g
c

f
h

Max

Min

b
a
x

For most functions youll ever encounter, most points are like a. For lack of a better
name, we can call such points boring points a boring point is simply any point that
is not an inflexion, maximum, minimum, stationary, or turning point.
b is a non-stationary point of inflexion (explicitly excluded from the A-levels).
c is a stationary point of inflexion.
A point like d (not illustrated) a stationary point that is not a maximum, minimum,
or inflexion point is extremely unusual. You can find an exotic example on p. 968.
f is both a maximum and minimum point because for all x D that are close to
fx D, we have t(x) t (fx ) t(x).
The set of turning points is simply the intersection of the set of stationary points and
the set of maximum and minimum points.
h is a maximum point because t(x) t (hx ) for all x D that are close to hx .
j is a minimum point because t(x) t (jx ) for all x D that are close to jx .
i is both a maximum and minimum point because there are simply no x D that are
close to ix D, and thus it is trivially or vacuously true that t(x) t (ix ) t(x) for x
that are close to x.23 i is not a stationary point because t (ix ) 0 indeed, t (ix ) is
undefined.24
23
24

A point like ix D that is not close to any other x D is, aptly enough, called an isolated point.
ix is an example of a critical point. A critical point is any point that is either stationary or where the derivative is
undefined. Dont worry, not something you need to know for the A-levels.

Page 157, Table of Contents

www.EconsPhDTutor.com

Exercise 70. For each of the following equations, (i) sketch its graph. (ii) Write down the
points at which it intersects the axes. (iii) Identify any turning points. (iv) Write down the
equations of any lines of symmetry and also (v) asymptotes. (a) y = 2ex + x. (b) x = 3x + 2.
(c) y = 2x2 + 1. (Answers on pp. 1032, 1033, and 1034.)

Page 158, Table of Contents

www.EconsPhDTutor.com

13

Relating the Graph of f to that of f

Given the graph of f , you are required to know how to figure out what f looks like. Lets
start with a very simple example.
Example 128. Let f R R be some differentiable function. Graphed below in blue is
its derivative f . You are told also that f (0) = 2. What does the graph of f look like?
(Pretend for a moment that you cant see the red graph.)

The derivative simply gives the slope of f . Since f (x) = 1 for all x, this means that f has
constant slope of 1. We are given moreover that f (0) = 2 (i.e. the vertical intercept is 2).
Altogether then, f (x) = x + 2 and is graphed in red above.

Page 159, Table of Contents

www.EconsPhDTutor.com

Example 129. Let g R R+ R be some differentiable function. Graphed below in blue


is its derivative g . You are told also that lim g(x) = 2. What does the graph of g look
x0
like? (Pretend for a moment that you cant see the red graph.)

-1

The derivative simply gives the slope of g. Since g (x) = 1 for all x < 0 and g (x) = 1 for
all x > 0, this means that g has constant slope of 1 for x < 0 and constant slope of 1 for
all x > 0. We are given moreover that lim g(x) = 2, so the two branches of g nearly meet
x0

at (0, 2), with a hole there. Altogether then,

x 2,
g(x) =

x 2,

for x < 0,
for x > 0.

Or more concisely, g(x) = x 2. Graphed above in red is g.

Page 160, Table of Contents

www.EconsPhDTutor.com

Example 130. Let h R R be some differentiable function. Graphed below in blue is its
derivative h defined by h (x) = x. You are told also that h(x) = 0. What does the graph
of h look like? (Pretend for a moment that you cant see the red graph.)

The derivative simply gives the slope of h. Since h (x) < 0 for all x < 0, h (0) = 0, and
h (x) > 0 for all x > 0, this means that h is strictly decreasing on R , a turning point at 0,
and strictly increasing on R+ .
Moreover, the derivative (slope) is increasing (indeed it is increasing at a constant rate)
so the graph of h is concave upwards throughout.
Altogether then, even if we dont know how to figure out what h(x) is, we can at least
roughly sketch the graph of h (in red above below). (Of course, you probably already know
x2
from secondary school that h(x) = , but were not supposed to know this until we learn
2
about integration later in this textbook.)

Page 161, Table of Contents

www.EconsPhDTutor.com

14

Quick Revision: Quadratic Equations y = ax2 + bx + c

Quadratic equations show up very often in various contexts. So here is a fairly complete if
brisk review of quadratic equations, which you were supposed to have completely mastered
in secondary school.
Example 131. Below are the graphs of the equations y = x2 + 3x + 1 (red), y = x2 + 2x + 1
(blue), y = x2 +x+1 (green), y = x2 +x+1 (red dotted), y = x2 2x1 (blue dotted),
and y = x2 x 1 (green dotted).

6
y=

x2

+x+1

y = x2 + 2x + 1

y = x2 + 3x + 1
2
y = - x2 + x + 1

0
-4

-3

-2

-1

-2
y=-

x2

1
y = - x2 - x - 1

- 2x - 1

-4

Page 162, Table of Contents

www.EconsPhDTutor.com

Heres a whirlwind study of the quadratic equation y = ax2 + bx + c. Assume that a 0,


otherwise we are in the trivial case of a linear equation. First, write:
ax2 + bx + c =

1 2 b
c
(x + x + ) .
a
a
a

To complete the square, observe that (x + k)2 = x2 + 2kx + k 2 and so


b
b 2 b2
x + x = (x + ) .
a
2a
4a
2

b 2 b2
c 1
b 2 b2 4ac
1
].
Hence, ax + bx + c = [(x + ) 2 + ]= [(x + )
a
2a
4a
a a
2a
4a2
2

What we just did above is called completing the square. We can use this to compute the
zeros or roots of the equation ax2 + bx + c = 0.
ax2 + bx + c = 0

b 2 b2 4ac
1
b 2 b2 4ac
] = (x + )
= [(x + )
a
2a
4a2
2a
4a2

b 2 b2 4ac
(x + ) =
2a
4a2
x=

b2 4ac
.
2a

This last expression give the roots of the equation ax2 + bx + c = 0. This expression will
NOT be printed in the A-Level List of Formulae! So be sure you remember it!

b b2 4ac
x=
.
2a

Page 163, Table of Contents

www.EconsPhDTutor.com

We can distinguish between six categories of quadratic equations, based on the signs of
a (the coefficient of x2 ) and b2 4ac (the discriminant). Each of these six categories was
illustrated in the figure above.
Category
1. a > 0, b2 4ac > 0
2. a > 0, b2 4ac = 0
3. a > 0, b2 4ac < 0
4. a < 0, b2 4ac > 0
5. a < 0, b2 4ac = 0
6. a < 0, b2 4ac < 0

Features
-shaped.
Intersects the horizontal axis at two points.
-shaped.
Just touches the horizontal axis at the minimum point.
-shaped.
Doesnt intersect the horizontal axis.
-shaped.
Intersects the horizontal axis at two points.
-shaped.
Just touches the horizontal axis at the maximum point.
-shaped.
Doesnt intersect the horizontal axis.

The vertical intercept (the value of f at 0) is simply c.


The Sign of a. If a > 0, then the graph is -shaped and has a minimum turning point
b
at x = . Conversely, if a < 0, then the graph is -shaped and has a maximum turning
2a
b
point at x = .
2a
The Discriminant. The term b2 4ac is called the discriminant. This name makes sense,
because it helps us discriminate between several possible cases of the equation ax2 +bx+c = 0:
If b2 4ac > 0, then:
There are two real roots (or zeros or horizontal intercepts), namely
b

b2 4ac
.
2a

Moreover, we can write


ax2 + bx + c = (x

b +

b2 4ac
b + b2 4ac
) (x +
).
2a
2a

What we have just done is to factorise the expression ax2 + bx + c. Factorisation is often
a useful trick to play.
Notice that if you plug in either of the roots into the right hand side (RHS) of the above
equation, we do indeed get zero, as expected.
Page 164, Table of Contents

www.EconsPhDTutor.com

If b2 4ac = 0, then:
There is only one real root (or zero or horizontal intercept), namely

b
.
2a

Moreover, we can write


b 2
b 2
ax + bx + c = (x ) = (x + ) .
2a
2a
2

b
Notice that if you plug x = into the RHS of the above equation, we do indeed get
2a
zero, as expected.
If b2 4ac < 0, then:
There are no real roots (or zeros or horizontal intercepts).
There is no way to factorise the expression ax2 +bx+c (unless we use complex numbers,
which well learn about only in Part IV).

Exercise 71. For each of the following equations, sketch its graph and identify its intercepts
and turning points (if these exist). (a) y = 2x2 + x + 1. (b) y = 2x2 + x + 1. (c) y = x2 + 6x + 9.
(Answer on p. 1035.)

Page 165, Table of Contents

www.EconsPhDTutor.com

15

Transformations
15.1

y = f (x) + a

The graph of y = f (x) + a is simply the graph of y = f (x) translated (moved) upwards by
a units.
Example 132. Define the function f R R by x x3 1. The graphs of f (red) and
y = f (x) + 2 (blue) are shown below.
Notice the blue curve is simply the red curve translated upwards by 2 units.

f(x), f(x) + 2

10

x
0
-3.0

-1.5

0.0

1.5

3.0

-2

-4

-6

-8

-10

Page 166, Table of Contents

www.EconsPhDTutor.com

15.2

y = f (x + a)

The graph of y = f (x + a) is simply the graph of y = f (x) translated leftwards by a units.


Why leftwards (and not rightwards)? The reason is that in order for f (x1 ) and f (x2 + a)
to hit the same value, we must have x2 = x1 a. That is, every x value is moved to the left
by a units.
Example 133. Define the function f R R by x x3 1. The graphs of f (red) and
y = f (x + 2) (blue) are shown below. (The latter equation is simply y = (x + 2)3 1.)
Notice the blue curve is simply the red curve translated leftwards by 2 units.

f(x), f(x+2)

10

x
0
-4

-2

-2

-4

-6

-8

-10

Page 167, Table of Contents

www.EconsPhDTutor.com

15.3

y = af (x)

The graph of y = af (x) is simply the graph of f (x) vertically-stretched (outwards from the
horizontal axis) by a stretching factor of a.
Example 134. Define the function f R R by x x3 1. The graphs of f (red) and
y = 2f (x) (blue) are shown below.
Notice the blue curve is simply the red curve stretched vertically (outwards from the
horizontal axis) by a factor of 2.
10

f(x), 2f(x)

x
0
-3.0

-1.5

0.0

1.5

3.0

-2

-4

-6

-8

-10

Page 168, Table of Contents

www.EconsPhDTutor.com

15.4

y = f (ax)

The graph of y = f (ax) is simply the graph of f (x) horizontally-stretched (outwards from
the vertical axis) by a stretching factor of 1/a. Or equivalently, the graph of y = f (ax) is
simply the graph of f (x) horizontally-compressed (inwards towards the vertical axis) by a
compression factor of a.
Why a stretching factor of 1/a (and not a)? The reason is that in order for f (x1 ) and
f (ax2 ) to hit the same value, we must have x2 = x1 /a. That is, every x value is scaled by
a factor of 1/a.
Example 135. Define the function f R R by x x3 1.The graphs of f (red) and
y = f (2x) (blue) are shown below. (The latter equation is simply y = (5x)3 1 = 125x3 1.)
Notice the blue curve is simply the red curve stretched horizontally (outwards from the
1
vertical axis) by a factor of . (Again, the A-level exams might instead word this as a
2
stretch with scale factor 0.5 parallel to the y-axis.)
Equivalently, the blue curve is simply the red curve compressed horizontally (inwards
towards from the vertical axis) by a factor of 2.
8

f(x), f(2x)

x
0
-2

-1

-2

-4

-6

-8

Page 169, Table of Contents

www.EconsPhDTutor.com

15.5

Combinations of the Above

Example 136. Define the function f R R by x x3 1. The graphs of f (red),


y = 1.1f (x 1) (blue), and y = f (1.1x) 1 (green) are shown below.
Notice the blue curve is simply the red curve translated rightwards by 1 unit and then
stretching it vertically (outwards from the vertical axis) by a factor of 1.1.
Notice the green curve is simply the red curve stretched horizontally (outwards from
the vertical axis) by a factor of 1/1.1 and then translated downwards by 1 unit.
f(x), 1.1f(x-1), f(1.1x)-1 8

x
0
-2

-1

-2

-4

-6

-8

Page 170, Table of Contents

www.EconsPhDTutor.com

15.6

y = f (x)

The graph of y = f (x) is simply the graph of f (x), but with all points for which f (x) < 0
reflected in the horizontal axis.
Example 137. Define the function f R R by x x3 1. The graphs of f (red) and
y = f (x) (blue) are shown below.

f(x), |f(x)|

x
0
-2

-1

-2

-4

-6

-8

Page 171, Table of Contents

www.EconsPhDTutor.com

15.7

y = f (x)

The graph of y = f (x) is simply the graph of f (x), but with all points for which x < 0
reflected in the vertical axis.
Example 138. Define the function f R R by x x3 1. The graphs of f (red) and
y = f (x) (blue) are shown below.

f(x), f(|x|)

x
0
-2

-1

-2

-4

-6

-8

Page 172, Table of Contents

www.EconsPhDTutor.com

15.8

y=

1
f (x)

Example 139. Define the function f R R by x x3 1. The graphs of f (red) and


1
(blue) are shown below.
y=
f (x)
1
. So
f (x)
in this case, x = 1 f (x) = 0 and thus x = 1 is a vertical asymptote for the graph of
1
1
y=
. As x approaches 1 from the left,
. And as x approaches 1 from the
f (x)
f (x)
1
right,
.
f (x)
1
Also, if as x , f (x) , then we also have
0, so that y = 0 is a horizontal
f (x)
asymptote. So here, as x , f (x) approaches 0 from above and as x , f (x)
approaches 0 from below.
Notice that wherever f (x) = 0, we have a vertical asymptote for the graph of y =

f(x), 1/f(x)

x
0
-2

-1

-2

-4

-6

-8

Page 173, Table of Contents

www.EconsPhDTutor.com

15.9

y 2 = f (x)

Three observations about the graph of y 2 = f (x):


1. It is symmetric in the horizontal axis. This is because if y1 satisfies y 2 = f (x), then so
too does y1 .

2. If f (x) < 0, then there is no value of y for which y 2 = f (x). And so the graph of y 2 = f (x)
is empty wherever f (x) < 0.

3. The graph of y 2 = f (x) intersects the horizontal axis at the same point as the graph of
y = f (x). Moreover, at any such point, the tangent to the graph of y 2 = f (x) is vertical.

Example 140. Define the function f R R by x x3 1.The graphs of f (red) and


y 2 = f (x) (blue) are shown below.

7
y = f(x)

6
5
4
3
2
1

0
-1

-1 0
-2

y2 = f(x)

-3
-4
-5
-6
-7
-8

Page 174, Table of Contents

www.EconsPhDTutor.com

Exercise 72. The graph of the function f R R is drawn below in red. Graph each of
the following equations. (a) y = 2f (3x). (b) y = f (x 1). (c) y 2 = f (x) + 4. (Answer on
p. 1036.)

-5

-4

-3

-2

-1

30 f(x), y
29
28
27
26
25
24
23
22
21
20
19
18
17
16
15
14
13
12
11
10
9
8
7
6
5
4
3
2
1
0
1
-1 0
-2
-3
-4
-5
-6
-7
-8
-9
-10

x
2

1
Exercise 73. Describe a series of transformations that would transform the graph of y =
x
1
to y = 3
. (Answer on p. 1037.)
5x 2

Page 175, Table of Contents

www.EconsPhDTutor.com

16

Conic Sections

Conic sections are formed from the intersection of a double cone and a 2D cartesian plane.
Take an infinitely large double cone (it goes upwards and downwards forever). Use a 2D
cartesian plane to slice the double cone from all conceivable positions and at all conceivable
angles. The intersection of the plane and the surface of the double cone form curves which,
aptly enough, are called conic sections.
The figure below25 doesnt show the upper half of the double cone, but you can easily
imagine it. Of the four curves depicted, only the hyperbola also cuts the upper half of the
double cone.

25

Taken from Wikipedia, which has an excellent page on conic sections.

Page 176, Table of Contents

www.EconsPhDTutor.com

The three types of conic sections are the ellipse (plural: ellipses), the parabola (parabolae), and the hyperbola (hyperbolae). The circle is regarded as a special case of the
ellipse.26 Here are the distinguishing characteristics of each:
Type
Ellipse
Parabola
Hyperbola

Description
Formed from only one half of the double cone.
A closed curve.27
Formed from only one half of the double cone.
Not a closed curve.
Formed from both halves of the double cone and is
thus composed of two distinct branches.
Not a closed curve.

Arises when
B 2 4AC < 0
B 2 4AC = 0
B 2 4AC > 0

We can prove (but do not do so in this textbook) that in general, a conic section is the
graph of the equation
1

Ax2 + Bxy + Cy 2 + Dx + Ey + F = 0,
where A, B, C, D, E, F are real constants and x and y are the two variables (on the
cartesian plane).
We refer to the expression B 2 4AC as the discriminant of the above equation. It is so
named because it discriminates between the three possible types of conic sections. We
can prove (but do not do so in this textbook) that if B 2 4AC > 0, then we have an ellipse;
if B 2 4AC = 0, then we have a parabola; and if B 2 4AC < 0, then we have a hyperbola.
In secondary school, we already learnt in some detail a special case of conic sections the
1
quadratic y = ax2 + bx + c. This is the special case of the equation = where
A = a, B= 0, C = 0, D= b, E = 1, and F = c.
The quadratic y = ax2 + bx + c is indeed a parabola, because B 2 4AC = 02 4(a)(0) = 0.
We already reviewed qudratic equations in section 14 and so we wont talk any more about
them in this chapter.

26

Strictly speaking, there are also the so-called degenerate conic sections, but we shall ignore these.

Page 177, Table of Contents

www.EconsPhDTutor.com

For A-levels, we are only required to learn about five more special cases of conic sections,
listed below. And so thats the plan for this chapter.
1.

x2 y 2
+
= 1,
a2 b2

2.

x2 y 2

= 1,
a2 b2

3.

y 2 x2

= 1,
b2 a2
ax + b
,
cx + d

4.

y=

5.

ax2 + bx + c
y=
.
dx + e
1

Exercise 74. As per the general form given in =, state for each of the above five equations,
what A, B, C, D, E, and F are. Compute the discriminant for each equation. Hence,
conclude that first equation is of an ellipse and the remaining four are of hyperbolae.
(Answer on p. 1038.)

Page 178, Table of Contents

www.EconsPhDTutor.com

The Ellipse x2 + y 2 = 1 (The Unit Circle)

16.1

x2 y 2
+
= 1 describes an ellipse. In this section, well study a special case of
a2 b2
this equation, where a = b = 1. The equation then becomes x2 + y 2 = 1, which is the unit
circle centred on the origin.
The equation

By unit circle, we mean that it has radius of unit length, i.e. length 1.

1
f(x), g(x), y
p = (x, y)

x2

y2

=1

y
x

-1.0

-0.5

0.0

0.5

1.0

(-0, 0)
Centre

-1
Why does this equation describe a circle?
You can easily see that (1, 0), (0, 1), (1, 0), and (0, 1) all satisfy the equation and are
thus part of its graph. Indeed, these are the horizontal and vertical intercepts. What about
elsewhere on the circle?
Consider any point p on the unit circle. It forms a triangle the line connecting it to the
origin is the hypothenuse; that connecting it to the horizontal axis is the side; and that
Page 179, Table of Contents

www.EconsPhDTutor.com

connecting it to the vertical axis is the base. By the Pythagorean Theorem, x2 + y 2 = 12 = 1.


We have just proven that every point (x, y) on the unit circle satisfies the equation x2 +y 2 = 1.
We now examine some of its characteristics.
1. Intercepts. The graph intersects the vertical axis at the points (0, 1) and (0, 1) and
the horizontal axis at the points (1, 0) and (1, 0).
2. Turning points. In this case, it is easy to see that there is a maximum turning point
at (0, 1) and a minimum turning point at (0, 1). But just as an exercise, lets also try
to find these turning points more rigorously, i.e. through calculus.
Exercise 49 showed that although it is impossible to rewrite the equation x2 + y 2 = 1 into
the form of a single function, it is nonetheless possibleto and rewrite it into the form of
two functions.
Namely, f [1, 1] R defined by x 1 x2 and g [1, 1] R defined

by x 1 x2 . Above, the graph of the function f is the upper semicircle (red) and
the graph of the function g is the lower semicircle (blue).
Lets compute the first derivative of f and set it equal to 0:
f (x) = 0.5(1 x2 )0.5 (2x)
= x(1 x2 )0.5
x(1 x2 )0.5 = 0
x = 0.
So the only stationary point of the function f is 0. We must now determine whether it is
a maximum, minimum, or inflexion point.
Compute the second derivative and evaluate it at the stationary point:
f (x) = (1 x2 )0.5 x [0.5(1 x2 )1.5 (2x)] .
This second derivative is messy and can be further simplified, but in this case there is no
need to simplify it, since all we want is to evaluate it at 0. We have
f (0) = (1 02 )0.5 0 [0.5(1 02 )1.5 (2 0)] = 1 < 0.
Hence, the point x = 0 is a maximum turning point of f . We should make it a habit to
write out the point in full, as (0, f (0)) = (0, 1).
Since g = f , it follows that g (0) = 0 and g (0) = 1 > 0. That is, the only stationary point
of the function g is (0, g(0)) = (0, 1). and it is a minimum point.
3. Asymptotes. By observation, there are no asymptotes.
4. Symmetry. The graph is a perfect circle centred on the origin. So by observation,
every line that passes through the origin is a line of symmetry!

Page 180, Table of Contents

www.EconsPhDTutor.com

16.2

x2 y 2
The Ellipse 2 + 2 = 1
a
b

f(x), g(x), y

b
Vertical Intercept

a
Horizontal Intercept

x2 / a2 + y2 / b2 = 1

(-0, 0)
Centre
y=0
Line of Symmetry

x=0
Line of Symmetry
-b
Vertical Intercept

-a
Horizontal Intercept

Squares are a proper subset of rectangles. Similarly, circles are a proper subset of ellipses.
The ellipse can be regarded as the generalisation of the circle.
Why does the equation x2 /a2 + y 2 /b2 = 1 describe an ellipse? Rewrite the equation as
y 2
x 2
( ) + ( ) = 1.
a
b
Hence, going from x2 + y 2 = 1 to x2 /a2 + y 2 /b2 = 1 involves two transformations:
1. First, stretch the graph horizontally, outwards from the vertical axis, by a factor of a.
2. Then stretch the graph vertically, outwards from the horizontal axis, by a factor of b.
This gives us an elongated circle that we call an ellipse.
1. Intercepts. The graph intersects the vertical axis at the points (0, b) and (0, b), and
the horizontal axis at the points (a, 0) and (a, 0).
2. Turning points. Clearly, there are maximum and minimum turning points at (0, b)
and (0, b). Lets find these rigorously using calculus.
Lets again break the equationup and rewrite it into the form of two functions.
Namely, f
[a, a] R defined by x b 1 x2 /a2 and g [a, a] R defined by x b 1 x2 /a2 .
These are graphed above. Lets compute the first derivative of f and set it equal to 0:

f (x) = 0.5b

Page 181, Table of Contents

0.5

2x
x2
( 2 ) = bx (1 2 )
a
a

0.5

a2 = 0 x = 0.

www.EconsPhDTutor.com

So the only stationary point of the function f is 0. We can show that it is a maximum
point, by computing the second derivative and evaluating it at 0:
d
x2
f (x) =
[bx (1 2 )
dx
a

0.5

d
x2
a ] = a b [x (1 2 )
dx
a
2

0.5

0.5

1.5

x2
x2
2x
= a b [(1 2 )
0.5 (1 2 )
( 2 )]
a
a
a
2
2
0
0
0x
f (0) = a2 b [(1 2 ) 0.5 0.5 (1 2 ) 1.5 ( 2 )] = a2 b < 0.
a
a
a
2

So (0, f (0)) = (0, b) is a maximum turning point of f .


And since g = f , g (0) = 0 and g (0) = a2 b > 0. That is, the only stationary point of g is
(0, b) and it is a minimum point.
3. Asymptotes. By observation, there are no asymptotes.
4. Symmetry. By observation, there are only two lines of symmetry, namely y = 0 and
x = 0 (the horizontal and vertical axes).

Exercise 75. (Answer on p. 1039.) Let a, b, c, d be constants with a, b non-zero. Consider


the equation
2

(x + c)
(y + d)
+
= 1.
a2
b2
(i) Sketch its graph. (ii) Write down the points at which it intersects the axes. (iii) Identify
any turning points. (iv) Write down the equations of any lines of symmetry and also (v)
asymptotes.

Page 182, Table of Contents

www.EconsPhDTutor.com

16.3

The Hyperbola: y = 1/x

y = 1/x (graphed) is the first hyperbola well study. It is also the simplest possible hyperbola.

y = -x
line of
symmetry

y
The graph of
y = 1 / x has
two branches.

4
3

y=x
line of
symmetry

2
1
x
0
-5

-4

-3

-2

-1

-1
-2

(0, 0)
Centre

4
y=0
horizontal
asymptote

-3
-4

x=0
vertical
asymptote

-5
It turns out that all hyperbolae well study have some common features. They have two
branches. In the case of y = 1/x, one branch is top-right and the other is bottom-right.

Page 183, Table of Contents

www.EconsPhDTutor.com

Moreover, for all hyperbolae well study,


1. Intercepts. Hyperbolae may or may not cross the axes. It depends.
y = 1/x is an example of a hyperbola that crosses neither the vertical nor the horizontal
axis. (But this is not true of all hyperbolae.)
2. Turning points. Hyperbolae may or may have turning points. It depends.
y = 1/x is an example of a hyperbola that has no turning points. (But this is not true of
all hyperbolae.)
3. Asymptotes. Hyperbolae always have two asymptotes.
In the case of y = 1/x, they are y = 0 and x = 0.
An interesting feature here is that the two asymptotes are perpendicular. A rectangular hyperbola is any hyperbola whose two asymptotes are perpendicular. And so
y = 1/x is an example of a rectangular hyperbola.
(But as well see, not all hyperbolae are rectangular.)
4. The centre is the point at which the two asymptotes intersect.
In the case of y = 1/x, the centre is (0, 0).
5. Two lines of symmetry. Both pass through the centre. Moreover, each line of symmetry bisects an angle formed by the two asymptotes.
In the case of y = 1/x, they are y = x and y = x.

Page 184, Table of Contents

www.EconsPhDTutor.com

The Hyperbola x2 y 2 = 1

16.4

x2 y 2 = 1 is a hyperbola and so it has two distinct branches. Notice also that if x (1, 1),
then there is no value of y for which x2 y 2 = 1. Hence, the graph of this equation is empty
in the region where x (1, 1).

f(x), g(x), y

x=0
Line of
Symmetry

(-0, 0)
Centre 3

y = f(x)

2
x2 - y2 = 1

y=0
Line of
Symmetry

0
-5

-4

-3

-2

-1

-1
Horizontal
Intercept

-2

Horizontal
Intercept

-3
y = g(x)
y = x -4
Linear
Asymptote
-5

y = -x
Linear
Asymptote

1. Intercepts. The graph crosses the horizontal axis at the points (1, 0) and (1, 0), but
does not intersect the vertical axis.
2. The two turning points there is a minimum turning point at (0, b) and a maximum
turning point at (0, b).

3. Asymptotes. We have y = x2 1. So as x , y = x2 1 x2 = x.
(Informally, as x , the 1 becomes negligible and we can simply ignore it). And so
the two asymptotes are y = x and y = x. The two asymptotes are perpendicular and
so this is a rectangular hyperbola.
4. The centre (point at which the two asymptotes intersect) is (0, 0).
5. We know that the two lines of symmetry bisect the angles formed by the asymptotes.
So they must have slope 1 and 1. Moreover, both pass through the centre (0, 0).
Altogether, we can work out that the lines of symmetry are y = x and y = x.

Page 185, Table of Contents

www.EconsPhDTutor.com

16.5

x2 y 2
The Hyperbola 2 2 = 1
a
b

To get from the equation x2 y 2 = 1 to the equation


x 2
y 2
( ) ( ) =1
a
b
involves two simple transformations:
1. First stretch the graph horizontally, outwards from the vertical axis, by a factor of a.
2. Then stretch the graph vertically, outwards from the horizontal axis, by a factor of b.

f(x), g(x), y

y = f(x)

(-0, 0)
Centre

x=0
Line of
Symmetry

x2 / a2 - y2 / b2 = 1

y=0
Line of
Symmetry

a
Horizontal
Intercept

-a
Horizontal
Intercept
y = g(x)
y = bx / a
Linear
Asymptote

Page 186, Table of Contents

y = -bx / a
Linear
Asymptote

www.EconsPhDTutor.com

y 2
x 2
The graphs characteristics are similar to before. Again, ( ) ( ) = 1 is a hyperbola
a
b
and so it has two distinct branches. Notice also that if x (a, a), then there is no value
x2 y 2
of y for which 2 2 = 1. Hence, the graph of this equation is empty in the region where
a
b
x (a, a).
1. Intercepts. The graph crosses the horizontal axis at the points (a, 0) and (a, 0), but
does not intersect the vertical axis.
2. There are no turning points.

x
x 2
3. Asymptotes. We have y = b ( ) 1. So as x , y = b ( ) 1
a
a

x 2
x
x
x
b ( ) = b . And so the two asymptotes are y = b and y = b . The two
a
a
a
a
asymptotes are perpendicular and so this is a rectangular hyperbola.
4. The centre (point at which the two asymptotes intersect) is (0, 0).
5. We know that the two lines of symmetry bisect the angles formed by the asymptotes.
So they must have slope 1 and 1. Moreover, both pass through the centre (0, 0).
Altogether, we can work out that the lines of symmetry are y = x and y = x.

Exam Tip
On the A-level exams, they typically only ask for (i) the intercepts; (ii) the asymptotes;
and (iii) turning points.
Nonetheless, you might as well know about the centre and the two lines of symmetry,
because these concepts are not difficult and will help you to sketch better graphs.

Page 187, Table of Contents

www.EconsPhDTutor.com

16.6

y 2 x2
The Hyperbola 2 2 = 1
b
a

SYLLABUS ALERT
y 2 /b2 x2 /a2 = 1 is explicitly in the 9758 (revised) but not the 9740 (old) syllabus.
But even if youre taking 9740, you might as well learn to draw y 2 /b2 x2 /a2 = 1, because
its really simple (since you now know how to draw x2 /a2 y 2 /b2 = 1).
y 2 x2
The graph of the equation 2 2 = 1 is simply the graph we studied in the previous section,
b a

but rotated clockwise (or anticlockwise).


2

y = f(x)

x=0
Line of
symmetry
(-0, 0)
Centre

f(x), g(x), y
b
Vertical
Intercept

y2 / b2 - x2 / a2 = 1

y=0
Line of
symmetry

y = -bx / a
Linear
asymptote

y = bx / a
Linear
asymptote
y = g(x)

-b
Vertical
intercept

Lets summarise the graphs characteristics. This is a hyperbola and so there are two
distinct branches. Notice also that if y (b, b), then there is no value of x for which

Page 188, Table of Contents

www.EconsPhDTutor.com

y 2 x2
= 1. Hence, the graph of this equation is empty in the region where y (b, b). The
b2 a2
range of y is thus (, b] [b, ).
1. Intercepts. The graph crosses the vertical axis at the points (0, b) and (0, b), but does
not intersect the horizontal axis.
2. The two turning points are (0, b) (minimum) and (0, b) (maximum).

2
x
x 2
3. Asymptotes. We have y = b 1 + ( ) . So as x , y = b 1 + ( )
a
a

2
x
x
x
x
b ( ) = b . And so the two asymptotes are y = b and y = b . The two
a
a
a
a
asymptotes are perpendicular and so this is a rectangular hyperbola.
4. The centre (point at which the two asymptotes intersect) is (0, 0).
5. We know that the two lines of symmetry bisect the angles formed by the asymptotes.
So they must have slope 1 and 1. Moreover, both pass through the centre (0, 0).
Altogether, we can work out that the lines of symmetry are y = x and y = x.

y 2 x2
If wed like, we can also find the turning points of 2 2 = 1 more rigorously, that is,
b
a
through calculus. As with the circle, although it is not possible to rewrite this equation
into the form of a single function, it is possible to rewrite
it into the form of two functions.
x 2
Namely, f (, a] [a, ) R defined by x b ( ) + 1 and g (, a) (a, )
a

2
x
R defined by x b ( ) + 1. The graph of the function f is entirely above the horia
zontal axis, while that of g is entirely below the horizontal axis.
Lets compute the first derivative of f :
x 2
f (x) = 0.5b [( ) + 1]
a

0.5

2x b
x

)=
.
a2
a x2 + a2

Hence, the only stationary point of f is (0, b). Lets check what sort of a stationary point
this is.
b
f (x) =
a
And so f (0) =

0.5
x2 + a2 x(0.5) (x2 + a2 )
(2x)
.
x2 + a2

b
> 0. Hence, this is a minimum point.
a2

Similarly, by computing the first derivative of g and doing the work, we can find that the
only stationary point of g is (0, b) and that this is a maximum point.

Page 189, Table of Contents

www.EconsPhDTutor.com

16.7

Long Division of Polynomials

Remember long division? Turns out itll be useful for dividing polynomials. Here are a
couple primary school examples to jog your memory.
Example 141. Whats 83 7? By long division, the quotient is 11 with a remainder of 6.
So, 83 7 = 116/7.
11
7 83
77
6
The quotient is the integer portion of the solution and the remainder is the left-over
integer.
Example 142. Whats 470 17? By long division, the quotient is 27 with a remainder of
11. So, 470 17 = 2711/17.
27
17 470
459
11

Page 190, Table of Contents

www.EconsPhDTutor.com

Long division can be used to divide one polynomial by another. But first of all, in case you
dont remember what a polynomial is...
Definition 47. An nth-degree polynomial in one variable is any expression a0 xn + a1 xn1 +
a2 xn2 + + an1 x + an where each ai is a constant and x is the variable.
In this textbook, well almost always consider only polynomials in one variable. So when
I say polynomial, Ill always mean a polynomial in one variable, unless otherwise stated.28
Example 143. The expressions 7x 3 and 4x + 2 are 1st-degree polynomials (in one variable). These are also called linear polynomials. (Polynomials of low degree are often also
called by such special names.)

Example 144. The expressions 3x2 + 4x 5 and x2 + 2x + 1 are 2nd-degree polynomials.


These are also called quadratic polynomials.

Example 145. The expressions 2x3 + 2x2 + 3x 1 and 3x3 + 2x2 + 3x + 1 3rd-degree polynomials. These are also called cubic polynomials.

Example 146. The expressions 5x4 2x3 + 2x2 + 3x 1 and 9x4 + 3x3 + 2x2 + 3x + 1 are
4th-degree polynomials. These are also called quartic polynomials.

28

Actually, weve already secretly studied an example of a polynomial in two variables the expression on the LHS of the
equation of the conic section: Ax2 + Bxy + Cy 2 + Dx + Ey + F = 0.

Page 191, Table of Contents

www.EconsPhDTutor.com

Now lets do some polynomial division.


x2 + 3
. We might be perfectly content with
Example 147. Say you have an expression
x1
this expression. Or we might try to simplify it through long division:
x +1
x 1 x2 +0x +3
x2 x +0
x +3
x 1
4
The quotient is x + 1 and the remainder is 4. Hence,
4
x2 + 3
=x+1+
x1
x1
4x3 + 2x2 + 1
Example 148. Lets simplify
through long division:
2x2 x 1
2x
+2
2x2 x 1 4x3 +2x2
4x3 2x2
4x2
4x2

+0x
2x
+2x
2x
4x

+1
+0
+1
+3
+3

The quotient is 2x + 2 and the remainder is 4x + 3. Hence,


4x3 + 2x2 + 1
4x + 3
=
2x
+
2
+
.
2x2 x 1
2x2 x 1

Exercise 76. Simplify each of the following fractions through long division. (a)
(b)

4x2 3x + 1
x2 + x + 3
. (c)
. (Answer on p. 1040.)
x+5
x2 2x + 1

Page 192, Table of Contents

16x + 3
.
5x 2

www.EconsPhDTutor.com

16.8

The Hyperbola y =

bx + c
dx + e

In the next section, well study this equation:


ax2 + bx + c
.
y=
dx + e
In this section, as a warm-up, well study the special case of the above equation, where
a = 0:
y=

Page 193, Table of Contents

bx + c
.
dx + e

www.EconsPhDTutor.com

Example 149. Graphed below is the equation y = (2x + 1)/(x + 1). This is the case where
b = 2, c = 1, d = 1, and e = 1. Do the long division:
2
x + 1 2x +1
2x +2

y=

1
7
y = -x + 1
line of
symmetry

(-1, 2)
Centre
-6

-4

2x + 1
1
=2
.
x+1
x+1

y
y=x+3
line of
symmetry
y=2
horizontal
asymptote

1
x
-2
0
-1
x = -1
vertical
asymptote

-3

As usual, this is a hyperbola with two distinct branches. Other features:


1. Intercepts. The graph intersects the vertical axis at the point (0, 1) and the horizontal
axis at the point (0.5, 0).
2. There are no turning points.
3. Asymptotes. As x 1, y . And so x = 1 is a vertical asymptote. As x ,
y 2. And so y = 2 is a horizontal asymptote. The two asymptotes are perpendicular
and so this is a rectangular hyperbola.
4. The centre (point at which the two asymptotes intersect) is (1, 2).
5. We know that the two lines of symmetry bisect the angles formed by the asymptotes.
So they must have slope 1 and 1. Moreover, both pass through the centre (1, 2).
Altogether, we can work out that the lines of symmetry are y = x + 3 and y = x + 1.

Page 194, Table of Contents

www.EconsPhDTutor.com

Example 150. Graphed below is the equation y = (7x + 3)/(2x + 4). This is the case where
b = 7, c = 3, d = 2, and e = 4. Do the long division:
3.5
2x + 4 7x

+3

7x +14
11

y=

7x + 3
11
= 3.5
.
2x + 4
2x + 4

Lets summarise the graphs characteristics. This is a hyperbola and so there are two
distinct branches.
1. Intercepts. The graph intersects the vertical axis at the point (0, 0.75) and the horizontal axis at the point (3/7, 0).
2. There are no turning points.
3. Asymptotes. As x 2, y . And so x = 2 is a vertical asymptote. As x ,
y 3.5. And so y = 3.5 is a horizontal asymptote. The two asymptotes are perpendicular
and so this is a rectangular hyperbola.
4. The centre (point at which the two asymptotes intersect) is (2, 3.5).
5. We know that the two lines of symmetry bisect the angles formed by the asymptotes.
So they must have slope 1 and 1. Moreover, both pass through the centre (2, 3.5).
Altogether, we can work out that the lines of symmetry are y = x + 5.5 and y = x + 1.5.

Page 195, Table of Contents

www.EconsPhDTutor.com

Lets now look, more generally, at the equation y =


b/d
dx + e bx
bx

bx + c
. By long division, we have:
dx + e

+c
+be/d
c be/d

The quotient is b/d and the remainder is c be/d. Lets further simplify this so that x
has no coefficient.
bx + c b c be/d
= +
dx + e d dx + e
=

b c be/d 1
+
d
d
x + e/d

b cd be 1
+
d
d2 x + e/d

We can thus get from y = 1/x to the above equation, through these transformations:
1. Shift the graph leftwards by

1
e
units to get the graph of y =
.
d
x + e/d

2. Stretch the graph vertically, outwards from the horizontal axis, by a factor of
cd be
get the graph of y = 2
.
d (x + e/d)
3. Finally, shift the graph upwards by b/d units to get the final graph.

cd be
to
d2

Exam Tip
The A-level exams often ask you to list down a series of transformations that will get you
from one graph to another, as was just done.

Page 196, Table of Contents

www.EconsPhDTutor.com

bx + c
Lets now summarise the characteristics of the graph of the equation y =
. This is
dx + e
a hyperbola with two distinct branches.
1. Intercepts. If e = 0, then the graph does not cross the vertical axis. If e 0, then the
graph intersects the vertical axis at the point (0, c/e). If b = 0, then the graph does not
cross the horizontal axis. If b 0, then the graph intersects the vertical axis at the point
(c/b, 0).
2. There are no turning points.29
3. Asymptotes. As x e/d, y . And so x = e/d is a vertical asymptote. As
x , y b/d. And so y = b/d is a horizontal asymptote. The two asymptotes are
perpendicular and so this is a rectangular hyperbola.
4. The centre (point at which the two asymptotes intersect) is (e/d, b/d).
5. We know that the two lines of symmetry bisect the angles formed by the asymptotes.
So they must have slope 1 and 1. Moreover, both pass through the centre (e/d, b/d).
Altogether, we can work out that the lines of symmetry are y = x + e/d + b/d and y =
x e/d + b/d.

Exercise 77. For each of the following equations, sketch its graph and identify its intercepts, turning points, asymptotes, centre, and lines of symmetry (if there are any of these).
x2
3x + 1
3x + 2
(a) y =
. (b) y =
. (c) y =
. (Answers on pp. 1041, 1042, and 1043.)
x+2
2x + 1
2x + 3

29

See p. 926 in the Appendices (optional) for a proof that y =

Page 197, Table of Contents

bx + c
has no turning points.
dx + e

www.EconsPhDTutor.com

16.9

ax2 + bx + c
The Hyperbola y =
dx + e

We now study the more general equation


ax2 + bx + c
.
y=
dx + e
Well rule out the following cases.
a = 0, because in that case
section.

ax2 + bx + c bx + c
=
and this was already studied in the last
dx + e
dx + e

ax2 + bx + c ax2 + bx + c
d = 0, because in that case
=
, which is a quadratic and which
dx + e
e
we already studied in secondary school.
ax2 + bx a
b
Both c and e are 0, because in that case
= x + , which is a linear expression.
dx
d
d
Well start with the simplest possible case (a = 1, b = 0, c = 1, d = 1, and e = 0). This is the
equation
y=

Page 198, Table of Contents

x2 + 1
.
x

www.EconsPhDTutor.com

Example 151. Graphed below is the equation y = (x2 + 1) /x.

10

y = (x2 + 1) / x

8
y=x
Oblique
Asymptote

6
(0, 0)
Centre

Minimum
Turning Point

2
0

-10

-6
Maximum
Turning Point

-2

-2
-4

x=0
vertical
asymptote

-8
-10
x
x x2 +1

Do the long division:

10

y = (1 - 2) x
Line of Symmetry

-6
y = (1 + 2) x
Line of Symmetry

x2

y=

x2 + 1
1
=x+ .
x
x

1
As usual, this is a hyperbola that has two distinct branches. Other features:
1. Intercepts. The graph intersects neither the vertical axis nor the horizontal axis.
2. There are two turning points (1, 2) is a maximum turning point and (1, 2) is a
minimum turning point. (To find these, compute the first derivative dy/dx = 1 1/x2 .
Set these equal to 0 for find two stationary points: x = 1. Use the 2DT to determine
that x = 1 and x = 1 are, respectively maximum and minimum turning points.)
By observation, y can take on any value except those between these two turning points.
The range of y is thus (, 2] [2, ).
3. Asymptotes. As x 0, y . Hence, there is one vertical asymptote: x = 0. As
x , y x. Hence, there is one oblique asymptote: y = x. The two asymptotes are
not perpendicular and so this is not a rectangular hyperbola.
4. The centre (point at which the two asymptotes intersect) is (0, 0).
5. We know that the two lines of symmetry bisect the angles formed by the asymptotes
and pass through the centre. You dont need to learn how to figure out their
equations (but see pp. 927ff. in the Appendices if youre interested).

Page 199, Table of Contents

www.EconsPhDTutor.com

x2 + 3x + 1
. Do the long division:
Example 152. Graphed below is the equation y =
x+1
x +2
x + 1 x2 +3x +1
x2 +x

2x
2x

+2
1

y = (1 - 2) x + 2 - 2
Line of Symmetry

-11

y=

-7

y = (1 + 2) x + 2 + 2
Line of Symmetry

10
8
6
4
2
0
-3 -2
-4
-6
-8
-10

x2 + 3x + 1
1
=x+2
.
x+1
x+1

y=x+2
Oblique
Asymptote

x
1

(-1, 1)
Centre

x = -1
vertical
asymptote

As usual, this is a hyperbola that has two distinct branches. Other features:
1. Intercepts. The graph intersects the vertical axis at the point (0, 1) and the horizontal

axis at the points (0.5(3 + 5), 0) and (0.5(3 5), 0). (The horizontal intercepts
are simply the zeros of the quadratic x2 + 3x + 1.)
2. There are no turning points. (Compute dy/dx = 1 + 1/(x + 1)2 . Set this equal to 0
there are no stationary points and thus no turning points either.)
By observation, y can take on any value. The range of y is thus R.
3. Asymptotes. As x 1, y . Hence, there is one vertical asymptote: x = 1. As
x , y x+2. Hence, there is one oblique asymptote: y = x+2. The two asymptotes
are not perpendicular and so this is not a rectangular hyperbola.
4. The centre (point at which the two asymptotes intersect) is (1, 1).
5. We know that the two lines of symmetry bisect the angles formed by the asymptotes
and pass through the centre. Again, you dont need to know how to find their equations.

Page 200, Table of Contents

www.EconsPhDTutor.com

2x2 + 2x + 1
. Do the long division:
Example 153. Graphed below is the equation y =
x + 1
2x 4
x + 1 2x2 +2x +1
2x2 2x
4x

2x2 + 2x + 1
5
5
= 2x 4 +
= 2x 4 +
.
x + 1
x + 1
x + 1

4x
4
5

14
10
Minimum
Turning
Point
-9

6
2

-5
y = (-2 + 5) x - 4 - 5
Line of Symmetry

-1-2
-6
-10
-14

Maximum
Turning
Point

-18

y = (-2 - 5) x - 4 + 5 Line
of Symmetry
x=1
vertical
asymptote
x
3

11

(1, -6)
Centre y = -2x - 4
Oblique
Asymptote

-22
-26

As usual, this is a hyperbola that has two distinct branches. Other features:
1. Intercepts. The graph intersects the vertical axis at the point (0, 1), but not the
horizontal axis, because there are no real zeros for the quadratic 2x2 + 2x + 1.

2. There are two turning points (1 2.5, 0.325) and (1 + 2.5, 12.325) are the
minimum and maximum turning points. (Verify this.)
By observation, y can take on any value except those between these two turning points. The
range of y is thus (, 12.325] [0.325, ).
3. Asymptotes. As x 1, y . Hence, there is one vertical asymptote: x = 1. As
x , y 2x 4. Hence, there is one oblique asymptote: y = 2x 4. The two
asymptotes are not perpendicular and so this is not a rectangular hyperbola.
4. The centre (point at which the two asymptotes intersect) is (1, 6).
5. We know that the two lines of symmetry bisect the angles formed by the asymptotes
and pass through the centre. Again, you dont need to know how to find their equations.
Page 201, Table of Contents

www.EconsPhDTutor.com

ax2 + bx + c
We will not look more generally at the equation y =
, because it gets rather
dx + e
messy. But if you want, you can read about it on pp. 927ff. of the Appendices (optional).

Exercise 78. For each of the following equations, sketch its graph and identify its intercepts, turning points, asymptotes, centre, and lines of symmetry (if any of these exist). (a)
x2 + x 1
2x2 2x 1
x2 + 2x + 1
. (b) y =
. (c) y =
. (Answers on pp. 1044, 1046, and
y=
x4
x+1
x+4
1048.)

Page 202, Table of Contents

www.EconsPhDTutor.com

17

Simple Parametric Equations

A graph (or curve) is simply a set of points. Parametric equations give us an alternative
method to describing the same graph (or curve).
Example 154. Recall that the graph of the equation x2 + y 2 = 1 i.e. the set S = {(x, y)
x2 + y 2 = 1} is the unit circle centred on the origin.

t = 3 / 4, x = - 2 / 2, y = 2 / 2
vx = - 2 / 2 ms-1, vy = - 2 / 2 ms-1
ax = 2 / 2 ms-2, ay = - 2 / 2 ms-2

Arrows indicate the


instantaneous
direction of travel.
x2 + y2 = 1
x
t = 0, x = 1, y = 0
vx = 0 ms-1, vy = 1 ms-1
ax = -1 ms-2, ay = 0 ms-2

t = 3 / 2, x = 0, y = -1
vx = 1 ms-1, vy = 0 ms-1
ax = 0 ms-2, ay = 1 ms-2

Observe that if x = cos t and y = sin t, then by a trigonometric identity, x2 + y 2 = 1. As it


turns out, this gives us a second way of writing the set S:
S = {(x, y) x = cos t, y = sin t, t R}.
The variable t is called a parameter, hence the name parametric equations. As t
increases from 0 to 2, we trace out, anti-clockwise, a unit circle centred on the origin.
t = 0 (x, y) = (1, 0),

t = /4 (x, y) = ( 2/2, 2/2) ,

t = 4/4 (x, y) = (1, 0),

t = 5/4 (x, y) = ( 2/2, 2/2) ,

t = 2/4 (x, y) = (0, 1),

t = 3/4 (x, y) = ( 2/2, 2/2) ,

t = 6/4 (x, y) = (0, 1),

t = 7/4 (x, y) = ( 2/2, 2/2) .

Page 203, Table of Contents

www.EconsPhDTutor.com

A nice interpretation is of the parameter t as time.


Example 203 (continued from above). The set S = {(x, y) x = cos t, y = sin t, 0 t <
2} can also be interpreted as tracing the motion of a particle as it moves anti-clockwise
around a circle. x and y give the distances of the particle (in metres) from the origin, in
the x- and y-directions.
We have x = cos t and y = sin t. This says that at any instant of time t, the particle is cos t
metres to the east of the origin and sin t metres to the north of the origin. (Note that if
cos t < 0, then the particle is to the west of the origin. And if sin t < 0, then the particle is
to the south of the origin.)
At time t = 0 s, the particle is at the position (x, y) = (1, 0). At time t = 1 s, the particle
has moved to the position (x, y) = (0.54, 0.84). At time t = /2 1.07 s, the particle has
moved to position (x, y) = (0, 1).

Having interpreted t as time, we can now also easily talk about the velocity and acceleration of the particle at different instants in time.
Example 203 (continued from above). We have x = cos t and y = sin t. From this,
we can easily compute the particles velocity in each direction: vx = dx/dt = sin t and
vy = dy/dt = cos t.
This says that at any instant of time t, the velocity of the particle is sin t ms-1 in the
x-direction and cos t ms-1 in the y-direction. (Note that if sin t < 0, then the particle is
moving westwards. And if cos t < 0, then the particle is moving southwards.)

So for example,
at
time
t
=
7/4,
its
velocity
is

sin
(7/4)
=
2/2 ms-1 rightwards and

cos (7/4) = 2/2 ms1 upwards.


Similarly, we can compute the particles acceleration in each direction: ax = d2 x/dt2 =
cos t and ay = d2 y/dt2 = sin t.

So for example,at time t = 7/4, its acceleration is cos (7/4) = 2/2 ms-1 rightwards and
cos (7/4) = 2/2 ms1 upwards. That is, the particle is travelling rightwards (because
its velocity rightwards at this instant in time is positive); however, its rightwards velocity
is slowing down.

Exercise 79. (Answer on p. 1050.) Let P be the particle whose position (in metres) is described by the set {(x, y) x = cos t, y = sin t, t R}, where t is time (seconds). Let Q be the
particle whose position (in metres) is described by the set {(x, y) x = sin t, y = cos t, t R}.
(a) How does the starting point (when t = 0) of Q differ from that of P ? (b) What about
the direction of travel?

Page 204, Table of Contents

www.EconsPhDTutor.com

Example 155. Recall that the graph of the equation x2 /a2 + y 2 /b2 = 1 i.e. the set
T = {(x, y) x2 /a2 + y 2 /b2 = 1} is the ellipse centred on the origin, with horizontal
intercepts a and vertical intercepts b.

y
t = 3 / 4

Arrows indicate the


instantaneous
direction of travel.
x
t = 0, x = 1, y = 0

t = 3 / 2

Observe that if x = a cos t and y = b sin t, then by the same trigonometric identity as before,
x2 /a2 + y 2 /b2 = 1. As it turns out, this gives us a second way of writing the set T :
T = {(x, y) x = a cos t, y = b sin t, t R} .
Similar to before, as t increases from 0 to 2, we trace out, anti-clockwise, an ellipse centred
on the origin.
At any instant in time t, the particles position, velocity, and acceleration are (x, y) =
(a cos t, b sin t), (vx , vy ) = (a sin t, b cos t), and (ax , ay ) = (a cos t, b sin t).

Exercise 80. Let P be the particle whose position (in metres) is described
{(x, y) x = a cos t, y = b sin t, t R}, where t is time (seconds). At each of the following
times, state the particles position and also its velocity and acceleration in both the x- and

y- directions. (a) t = ; (b) t = ; (c) t = 2. (Answer on p. 1050.)


4
2

Page 205, Table of Contents

www.EconsPhDTutor.com

Example 156. Recall that the graph of the equation x2 y 2 = 1 i.e. the set U = {(x, y)
x2 y 2 = 1} is the rectangular east-west hyperbola centred on the origin, with
horizontal intercepts 1 and no vertical intercepts.

Arrows indicate 5 y
the instantaneous 4
direction of travel. 3
x2 - y2 = 1
2
1
t=4
0
t=3
-5
-4
-3
-2
-1 -1 0
-2
t=2
-3
-4
-5

t=1
x

t=0
1

t=5

Observe that if x = sec t and y = tan t, then by a trigonometric identity, x2 y 2 = 1. As it


turns out, this gives us a second way of writing the set U :
U = {(x, y) x = sec t, y = tan t, t R, t k/2} .
Note that t cannot be a half-integer multiple of , because then tan t would be undefined.
Again, lets interpret this as the movement of a particle. Interestingly, the particle always
moves upwards, as we can easily prove vy = dy/dt = sec2 t > 0 for all t.
At t = 0, the particle is at (x, y) = (1, 0). During t [0, /2), the particle moves northeast
along the green segment and flies off towards infinity as t /2 1.57.
An instant after /2 seconds, the particle magically reappears near infinity in the southwest. During t (/2, ], the particle moves northeast along the blue segment. At t = ,
the particle is at (1, 0).
During t [, 3/2), the particle moves northwest along the red segment and flies off
towards infinity as t 3/2 4.71.
An instant after 3/2 seconds, the particle magically reappears near infinity in the southeast. During t (3/2, 2], the particle moves northwest along the pink segment.

Page 206, Table of Contents

www.EconsPhDTutor.com

Exercise 81. (Answer on p. 1051.) Suppose that the position of a particle is described by
the set {(x, y) x = tan t, y = sec t, t R}, where t is time, measured in seconds.
(a) Rewrite the set using a single cartesian equation.
(b) Compute dx/dt. And hence make an observation about how the particle travels in the
x-direction.
The graph below indicates six positions of the particle A, B, C, D, E, and F . (Also
indicated are the directions of travel.) The particle is at these positions at times t = 0, 1,
2, 3, 4, and 5 but not necessarily in that order.
(c) Using only the graphs of s = tan t and s = sec t (above) to guide you and without using
a calculator, state where the particle is, at each of the the times t = 0, 1, 2, 3, 4, and 5.

3
C

2
B
1
{(x, y): x = tan t, y = sec t, t
-5

-4

-3

-2

-1

}
0

-1
E
-2
D

-3

Arrows indicate -4
the instantaneous
direction of travel.
-5

Page 207, Table of Contents

www.EconsPhDTutor.com

17.1

Eliminating the Parameter t

Given a pair of parametric equations that describes a set of points, we can often go in
reverse: We can eliminate the parameter t and describe the same set of points using a
single equation.
Example 157. The set {(x, y) x = t2 + t, y = t 1, t R} describes the position (metres) of
a particle at time t (seconds).

y
Instantaneous
Direction of
Travel

Instantaneous
Direction of
Travel

Instantaneous
Direction of Travel

t = 1, x = 2, y = 0
vx = (2t + 1) ms-1 = 3 ms-1
vy = 1 ms-1, ax = 2 ms-2, ay = 0 ms-2
t = 0, x = 0, y = - 1
vx = (2t + 1) ms-1 = 1 ms-1
vy = 1 ms-1, ax = 2 ms-2, ay = 0 ms-2
t = - 1, x = 0, y = - 2
vx = (2t + 1) ms-1 = - 1 ms-1
vy = 1 ms-1, ax = 2 ms-2, ay = 0 ms-2

x = y2 + 3y + 2

Write t = y + 1 and x = (y + 1)2 + (y + 1) = y 2 + 3y + 2. And so the same set can be rewritten


as: {(x, y) x = y 2 + 3y + 2}.
As an exercise, lets also compute the velocity and acceleration of the particle.
vx = dx/dt = 2t + 1 and vy = dy/dt = 1. This says that at any instant in time t, the particle
has velocity 2t + 1 ms1 rightwards and 1 ms1 upwards.
ax = d2 x/dt2 = 2 and ay = d2 y/dt2 = 0. This says that the particle is always accelerating
rightwards at the rate 2 ms2 . Moreover, it is never accelerating upwards (this is consistent
with the above finding that its upwards velocity is a constant 1 ms1 ).

Page 208, Table of Contents

www.EconsPhDTutor.com

Example 158. The set {(x, y) x = 2 cos t 4, y = 3 sin t + 1, t R} describes the position
(metres) of a particle at time t (seconds).

5
t = / 2, x = - 4, y = 4
vx = - 2 sin (t) ms-1 = - 2 ms-1
vy = 3 cos (t) ms-1 = 0 ms-1
ax = - 2 cos (t) ms-2 = 0 ms-2
ay = - 3 sin (t) ms-2 = -3 ms-2
t = , x = - 6, y = 1
vx = - 2 sin (t) ms-1 = 0 ms-1
vy = 3 cos (t) ms-1 = - 3 ms-1
ax = - 2 cos (t) ms-2 = 2 ms-2
ay = - 3 sin (t) ms-2 = 0 ms-2

4
3
2
1
x
0

-7

-5

-3

-1

1
-1

t = 3 / 2 , x = - 4, y = - 2
vx = - 2 sin (t) ms-1 = 2 ms-1 -2
vy = 3 cos (t) ms-1 = 0 ms-1
ax = - 2 cos (t) ms-2 = 0 ms-2
ay = - 3 sin (t) ms-2 = 3 ms-2 -3
Write (x + 4) /2 = cos t and (y 1) /3 = sin t. Using the trigonometric identity cos2 t+sin2 t =
2
2
1, we can rewrite the set as {(x, y) x = [(x + 4) /2] + [(y 1) /3] = 1}. This is the ellipse
centred on (4, 1).
As an exercise, lets also compute the velocity and acceleration of the particle.
vx = dx/dt = 2 sin t and vy = dy/dt = 3 cos t. This says that at any instant in time t, the
particle has velocity 2 sin t ms1 leftwards and 3 cos t ms1 upwards.
ax = d2 x/dt2 = 2 cos t and ay = d2 y/dt2 = 3 sin t. This says that at any instant in time
t, the particle is accelerating leftwards at the rate 2 cos t ms2 and upwards at the rate
3 sin t ms2 .

Page 209, Table of Contents

www.EconsPhDTutor.com

Exercise 82. Each of the following sets describes the position (metres) of a particle at
time t (seconds). Rewrite each set into a form where the parameter t is eliminated. Sketch
the graph of each. Indicate the particles position and direction of travel at t = 0. (Answers
on pp. 1052, 1053, and 1054.)
(a) {(x, y) x = 2 sin t 1, y = 3 cos2 t, t R}.
(b) {(x, y) x =

1
, y = t2 + 1, t R}.
t1

(c) {(x, y) x = t 1, y = ln(2t + 1), t > 0.5}.

Page 210, Table of Contents

www.EconsPhDTutor.com

18

Equations and Inequalities

N
N
Given any fraction
(where N and D are real numbers with D non-zero), we have
>0
D
D
if and only if one of the following is true:
1. N > 0 AND D > 0; OR
2. N < 0 AND D < 0.
The expressions that are in the numerator (N ) and denominator (D) can get pretty complicated. So here are some very simple examples just to warm you up.

Example 159.

4
> 0 because both the numerator and denominator are positive.
7

Example 160.

5
> 0 because both the numerator and denominator are negative.
3

Example 161.

9
< 0 because the numerator is negative but the denominator is positive.
2

Example 162.

1
> 0 because the numerator is positive but the denominator is negative.
8

Page 211, Table of Contents

www.EconsPhDTutor.com

18.1

Example 163.

ax + b
>0
cx + d

x+3
> 0 one of the following is true:
3x + 2

1. x + 3 > 0 AND 3x + 2 > 0; OR


2. x + 3 < 0 AND 3x + 2 < 0.
Notice that (1) x + 3 > 0 AND 3x + 2 > 0 x > 3 AND x > 2/3 , which in turn is
equivalent to the single inequality x > 2/3.
Notice that (1) x + 3 < 0 AND 3x + 2 < 0 x < 3 AND x < 2/3 , which in turn is
equivalent to the single inequality x < 3.
x+3
> 0 x > 2/3 OR x < 3 (equivalently, x (, 3)
Altogether then,
3x + 2
2
( , )).
3
Note that I use quotation marks , but these are not necessary. Instead, they merely help
to make especially clear which groups of conditions corresponds to each other.

Example 164.

4x 1
> 0 one of the following is true:
x+2

1. 4x 1 > 0 AND x + 2 > 0 x > 1/4 AND x > 2 x > 1/4 ; OR


2. 4x 1 < 0 AND x + 2 < 0 x < 1/4 AND x < 2 x < 2.
Altogether then,
(1/4, )).
Example 165.

4x 1
> 0 x > 1/4 and x < 2 (equivalently, x (, 2)
x+2

5x + 4
> 0 one of the following is true:
2x + 1

1. 5x + 4 > 0 AND 2x + 1 > 0 x > 4/5 AND x < 1/2 x (4/5, 1/2) ; OR
2. 5x + 4 < 0 AND 2x + 1 < 0 x < 4/5 AND x > 1/2, but these are mutually
contradictory and thus impossible.
Altogether then,

5x + 4
> 0 x (4/5, 1/2).
2x + 1

When given any inequality that is of a slightly different form, be sure to always convert it
N
into what Ill call the standard form
> 0. Strictly speaking, this is not necessary, but
D
Page 212, Table of Contents

www.EconsPhDTutor.com

if you always do this, youll make a habit of solving inequalities in this form, and thus be
less likely to make a careless mistake.

Example 166. Consider the inequality

3x 2
< 3. This inequality is equivalent to
5x + 1

3x 2
>0
5x + 1

15x + 3 (3x 2)
>0
5x + 1

18x + 5
> 0.
5x + 1

This last inequality is in turn true one of the following is true:


1. 18x + 5 > 0 AND 5x + 1 > 0 x < 5/18 AND x < 1/5 x < 5/18 ; OR
2. 18x + 5 < 0 AND 5x + 1 < 0 x > 5/18 AND x > 1/5 x > 1/5.
Altogether then,
(1/5, )).

3x 2
< 3 x < 5/18 OR x > 1/5 (equivalently, x (, 5/18)
5x + 1

2x + 1
Exercise 83. For what values of x is each of the following inequalities true? (a)
> 0.
3x + 2
1
1
3x 18
2x + 3
x1
> 0. (c)
> 0. (d)
> 0. (e)
> 0. (f)
< 9. (Answers on p.
(b)
4
4
4
9x 14
x + 7
1055.)

Page 213, Table of Contents

www.EconsPhDTutor.com

18.2

ax2 + bx + c
>0
dx2 + ex + f

ax2 + bx + c
. But
dx2 + ex + f
ax2 + bx + c
> 0. This is
you are required to know how to find the values of x for which
dx2 + ex + f
just the same game as before, albeit slightly more complicated.

Dont worry, you are not required to know how to graph the equation y =

2x2 + x + 3
> 0 one of the following is true:
x2 + 3x + 2

Example 167.

1. 2x2 + x + 3 > 0 AND x2 + 3x + 2 > 0; OR


2. 2x2 + x + 3 < 0 AND x2 + 3x + 2 < 0.

y = 2x2 + x + 3 is a -shaped quadratic and has no real roots (because the discriminant is
negative). Hence, it is always positive. It is thus impossible that 2x2 + x + 3 < 0 AND
x2 + 3x + 2 < 0 (Case 2).
We need thus only examine Case 1. As we just said, it is always true that 2x2 + x + 3 > 0.
So we need only examine when it is true that x2 + 3x + 2 > 0.
The equation y = x2 + 3x + 2 has a -shaped graph and has two real zeros given by:

32 4(1)(2) 3 17 3 17
=
=
= 0.5 (3 17) .
2(1)
2
2

Hence, the expression x2 + 3x + 2 > 0 x (0.5 (3 17) , 0.5 (3 + 17)).


3

Altogether then,

2x2 + x + 3
>
0

(0.5
(3

17)
,
0.5
(3
+
17)).
x2 + 3x + 2

A dirty trick is to use your TI84 to do a quick check that this answer is correct:

Page 214, Table of Contents

www.EconsPhDTutor.com

x2 + 4x 1
> 0 one of the following is true:
Example 168.
2x2 + x + 2
1. x2 + 4x 1 > 0 AND 2x2 + x + 2 > 0; OR
2. x2 + 4x 1 < 0 AND 2x2 + x + 2 < 0.
The equation y = 2x2 + x + 2 has a -shaped graph and has no real zeros (because the
discriminant is negative). Hence, it is always positive. It is thus impossible that x2 +
4x 1 < 0 AND 2x2 + x + 2 < 0 (Case 2).
We need thus only examine Case 1. As we just said, it is always true that y = 2x2 +x+2 > 0.
So we need only examine when it is true that x2 + 4x 1 > 0.
The equation y = x2 + 4x 1 has a -shaped graph and has two real zeros given by:

42 4(1)(1) 4 12 4 12
=
=
= 2 3.
2(1)
2
2

Hence, the expression x2 + 4x 1 > 0 x (2 3, 2 + 3).


4

x2 + 4x 1
Thus,
>
0

(2

3,
2
+
3) (0.268, 3.732). As usual, lets check
2x2 + x + 2
using our TI84:

Page 215, Table of Contents

www.EconsPhDTutor.com

x2 + 5x + 4
> 0 one of the following is true:
Example 169.
x2 2x + 1
1. x2 + 5x + 4 > 0 AND x2 2x + 1 > 0; OR
2. x2 + 5x + 4 < 0 AND x2 2x + 1 < 0.
The equation y = x2 + 5x + 4 has a -shaped graph and has two real zeros given by:
5

(5)2 4(1)(4) 5 9 5 3
=
=
= 4, 1.
2(1)
2
2

Hence, the expression x2 + 5x + 4 > 0 x < 4 OR x > 1.


The equation y = x2 2x + 1 has a -shaped graph and has two real roots given by:

(2)2 4(1)(1) 2 8
=
= 1 2.
2(1)
2

Hence, the expression x2 + 4x 1 > 0 x (1 2, 2 1). Thus:
2


1. x2 +5x+4 > 0 AND x2 2x+1 > 0 x < 4 OR x > 1 AND x (1 2, 2 1).

Since 1 2 < 1, this is equivalent to x (1, 2 1).

2. x2 + 5x + 4 < 0 AND x2 2x + 1 < 0 x (4, 1) AND x < 1 2 or x > 2 1.

Since 1 2 < 1, this is equivalent to x (4, 1 2).

x2 + 5x + 4
Altogether then,
>
0

(4,
1

2)

(1,
2 1). As usual, lets
x2 2x + 1
check using our TI84:

Page 216, Table of Contents

www.EconsPhDTutor.com

x2 4x + 3
> 0 one of the following is true:
Example 170.
x2 2x
1. x2 4x + 3 > 0 AND x2 2x > 0; OR
2. x2 4x + 3 < 0 AND x2 2x < 0.
The equation y = x2 4x + 3 has a -shaped graph and has two real zeros given by:
4

(4)2 4(1)(3) 4 4
=
= 1, 3.
2(1)
2

Hence, the expression x2 4x + 3 > 0 x (1, 3).


The equation y = x2 2x has a -shaped graph and has two real roots given by:
2

(2)2 4(1)(0) 2 4
=
= 0, 2.
2(1)
2

Hence, x2 2x > 0 x (0, 2). Thus:


1. x2 4x + 3 > 0 AND x2 2x > 0 x (1, 3) AND x (0, 2) x (1, 2).
2. x2 4x + 3 < 0 AND x2 2x < 0 x < 1 OR x > 3 AND x < 0 OR x > 2
x < 0 or x > 3.
Altogether then,
using our TI84:

x2 4x + 3
> 0 x (, 0) (1, 2) (3, ). As usual, lets check
x2 2x

Exercise 84. Without using a calculator, find the values of x for which each of the following
x2 + 2x + 1
x2 1
x2 3x 18
2x + 5
inequalities is true. (a) 2
> 0. (b) 2
> 0. (c)
>
0.
(d)
>
x 3x + 2
x 4
x2 + 9x 14
x + 4
3x + 1
. (Answers on pp. 1057, 1058, 1059, and 1060.)
6x 7

Page 217, Table of Contents

www.EconsPhDTutor.com

18.3

Solving Inequalities by Graphical Methods

Example 171. For what values of x is x > sin (0.5x)?


Rewrite the inequality as x sin(0.5x) > 0. Graph y = x sin(0.5x) on your graphing
calculator. Our goal is to first find the horizontal intercepts of this equation; this will let
us solve for x > sin (0.5x).
After Step 1.

After Step 2.

After Step 3.

After Step 4.

After Step 5.

After Step 6.

In the TI84:
1. Press ON to turn on your calculator.
2. Press Y= to bring up the Y= editor.
3. Press X,T,,n SIN 0 . 5 . To enter , press the blue 2ND button and then
(which corresponds to the button). Now press X,T,,n ) and altogether you will
have entered x sin(0.5x).
4. Now press GRAPH and the calculator will graph y = x sin(0.5x).
It looks like the horizontal intercepts are close to the origin. Lets zoom in to see better.
5. Press the (ZOOM) button to bring up a menu of ZOOM options.
6. Press 2 to select the Zoom In option. Nothing seems to happen. But now press ENTER
and the TI will zoom in a little for you.
It looks like there are 3 horizontal intercepts. To find out what precisely they are, well use
the TI84s zero option.

(... Example continued on the next page ...)

Page 218, Table of Contents

www.EconsPhDTutor.com

(... Example continued from the previous page ...)


After Step 7.

After Step 8.

After Step 9.

After Step 11.

After Step 12.

After Step 13.

After Step 10.

3. Press the blue 2ND button and then CALC (which corresponds to the TRACE
button). This brings up the CALCULATE menu.
4. Press 2 to select the zero option. This brings you back to the graph, with a cursor
flashing. Also, the TI84 prompts you with the question: Left Bound?
TI84s ZERO function works by you first specifying a Left Bound and a Right Bound
for x. TI84 will then check to see if there are any horizontal intercepts (i.e. values of x for
which y = 0) within those bounds.
5. Using the < and > arrow keys, move the blinking cursor until it is where you want your
first Left Bound to be. For me, I have placed it a little to the left of where I believe
the leftmost horizontal intercept to be.
6. Press ENTER and you will have just entered your first Left Bound.
TI84 now prompts you with the question: Right Bound?.
7. So now just repeat. Using the < and > arrow keys, move the blinking cursor until it is
where you want your first Right Bound to be. For me, I have placed it a little to the
right of where I believe the leftmost horizontal is.
8. Again press ENTER and you will have just entered your first Right Bound.
TI84 now asks you: Guess? This is just asking if you want to proceed and get TI84 to
work out where the horizontal intercept is. So go ahead and:
9. Press ENTER . TI84 now informs you that there is a Zero at x = 1, y = 0 and
places the blinking cursor at precisely that point. This is the first horizontal intercept
weve found.
To find each of the other 2 horizontal intercepts, just repeat steps 3 through 9. You
should be able to find that they are at x = 0 and x = 1. Altogether, the 3 intercepts are
x = 1, 0, 1. Based on these and what the graph looks like, we conclude: x > sin (0.5x)
x (1, 0) (1, ).

Page 219, Table of Contents

www.EconsPhDTutor.com

Example 172. For what values of x is x > e + ln x?


For this example, I wont give the full detailed instructions of what to do on the TI84; Ill
only show a few screenshots. First, rewrite the inequality as x e ln x > 0 and so graph
y = x e ln x on your graphing calculator:
After Graphing. Zoom In, Adjust Window.

Look for the values of x for which x e ln x = 0. They are x = 0.7083, 4.1387:
Leftmost horizontal intercept. Rightmost horizontal intercept.

Based on these horizontal intercepts and what the graph looks like, we conclude: x > e+ln x
if and only if x (0, 0.7083) (4.1387, ).

Exercise 85. Use a graphing calculator to find the values of x for which each of the

1
> x3 + sin x.
following inequalities is true. (a) x3 x2 + x 1 > ex . (b) x > cos x. (c)
2
1x
(Answers on pp. 1061, 1061, and 1062.)

Page 220, Table of Contents

www.EconsPhDTutor.com

18.4

Systems of Equations

Warm-up questions:
Exercise 86. (PSLE-style question.) When Apu was 40 years old, Beng was twice as old
as Caleb. Today, Caleb is 28 years old and Apu is twice as old as Beng. What are the ages
of Apu and Beng today? (If necessary, assume that the age of a person is always an integer
and is fixed between January 1st and December 31st of each year.) (Answer on p. 1063.)
Exercise 87. (O-Level style question.) Planes A and B leave the same point at 12pm.
Plane A travels northeast at a constant speed of 100 km/h. Plane B travels south at a
constant speed of 200 km/h. At 3pm, both planes make an instant turn and start flying
directly towards each other at the same speed. At what time will the two planes collide?
(Answer on p. 1063.)

Definition 48. Given an equation involving a single variable x, a real solution to the
equation is any value of x R such that the equation is true.
Example 173. The equation x + 5 = 8 has one real solution: 3. The equation x2 1 = 0
has two real solutions: 1 and 1. The equation x2 1 = 8 has two real solutions: 3 and 3.
The equation x3 4x = 0 has three real solutions: 2, 0, and 2.
Example 174. The equation x2 + 1 = 0 has no real solution.

Definition 49. Given an equation involving a single variable x, a real solution set is the
set of values of x R such that the equation is true.
Example 175. The real solution set of the equation x + 5 = 8 is {3}. The real solution
set of the equation x2 1 = 0 is {1, 1}. The real solution set of the equation x2 1 = 8 is
{3, 3}. The real solution set of the equation x3 4x = 0 is {2, 0, 2}.
Example 176. The real solution set of the equation x2 + 1 = 0 is = {}.

Page 221, Table of Contents

www.EconsPhDTutor.com

Definition 50. Given a system of equations (or more simply a set of equations) involving
two variables x and y, a real solution to the set of equations is any point (or ordered pair)
(x, y) with x, y R for which the system of equations is true; and a real solution set is the
set of ordered pairs (x, y) for which the system of equations is true.
Example 177. Consider the system of equations y = x + 1, y = x + 3. To solve this system
of equations, plug in the second equation into the first to get: x + 3 = x + 1. Now solve:
x = 1. And so y = x + 1 = 2. Altogether, this system of equations has one real solution (1, 2).
Its real solution set is thus {(1, 2)}.
Example 178. Consider the system of equations y = 0.5x2 1.5 and y = x. To solve this
system of equations, plug in the second equation into the first to get: x = 0.5x2 1.5.
Rearranging: x2 2x 3 = 0. Now solve: x = 3, 1. Correspondingly, y = 3, 1. Altogether,
this system of equations has two real solutions: (3, 3) and (1, 1). Its real solution set is
thus {(3, 3), (1, 1)}.
A system of equations can have no real solutions.
Example 179. Consider the system of equations y = ln x and y = x. Observe that for all
x (0, 1), ln x < 0 and hence x > ln x. Moreover, for x = 1, ln x = 0 < x. Also, for x > 1,
1
d
d
ln x = < 1 <
x = 1, so the slope of y = x is steeper than that of y = ln x. Altogether
dx
x
dx
then, for all x > 0, x > ln x. Hence, this system of equations has no real solutions. Its real
solution set is thus = {}.
A system of equations can also have infinitely many real solutions.
Example 180. Consider the system of equations y = x and 2y = 2x. Observe that this
system of equations has infinitely many real solutions, e.g. (1, 1), (2, 2), (2.74, 2.74). There
is thus no way to explicitly list out all its real solutions. However, using set-builder notation,
we can write down its real solution set as {(x, y} y = x}. This says that every ordered pair
(x, y) such that y = x is a real solution to the given system of equations.

Solve these problems without using a calculator.


Exercise 88. The points (1, 2), (3, 5), and (6, 9) satisfy the equation y = ax2 +bx+c. What
are a, b, and c? (Answer on p. 1064.)
Exercise 89. The point (1, 2) satisfies the equation y = ax2 + bx + c. Moreover, the
minimum point of the equation y = ax2 + bx + c is (0, 0). What are a, b, and c? (Answer on
p. 1064.)

Page 222, Table of Contents

www.EconsPhDTutor.com

You are required to know how to use a graphing calculator to find the numerical solution
of equations (including system of linear equations).
Example 181. Solve the system of equations y = x4 x3 5, y = ln x.
One method is to graph both equations on your graphing calculator and then find their
intersection points.
Here Ill use another method: First rewrite the two equations as a third equation y =
x4 x3 5 ln x. Our goal is to find the horizontal intercepts of this equation, which will
in turn also be the solutions to the above set of equations.
Briefly, in the TI84:
1. Graph the equation y = x4 x3 5 ln x.
It looks like there is only one horizontal intercept.
2. Zoom in.
3. Find the horizontal intercept using the zero option.
Conclusion: There is one solution to this set of equations and its x-coordinate is 1.8658. To
find the y-coordinate, we need merely plug in this value of x into either of the equations in
the original set of equations: y = ln x = ln 1.8658 0.6237. Altogether, this set of equations
has one solution: (1.8658, 0.6237).
After Step 1.

After Step 2.

After Step 3.

Exercise 90. Using your graphing calculator, solve the following systems of equations. (a)
1
1
, y = x5 x3 + 2. (c) y =
x2 + y 2 = 1, y = sin x. (b) y =
, y = x3 + sin x. (Answers
2
1

x
1+ x
on pp. 1065, 1066, and 1067.)

Page 223, Table of Contents

www.EconsPhDTutor.com

Part II

Sequences and Series

Page 224, Table of Contents

www.EconsPhDTutor.com

19

Finite Sequences

Recall that an ordered pair (of real numbers) was simply any pair of real numbers, enclosed
by parentheses, and whose order matters (and this was the only difference between an
ordered pair and a set of two objects).
Example 182. (1, 2) and (2, 1) are both ordered pairs with (1, 2) (2, 1).
We can analogously define ordered triples, quadruples, quintuples, etc.
Example 183. (1, 2, 3) and (2, 1, 3) are both ordered triples with (1, 2, 3) (2, 1, 3).
(1, 1, 1, 1) and (2, 4, 1, 3) are both ordered quadruples with (1, 1, 1, 1) (2, 4, 1, 3).
(2, 2, 3, 2, 2) and (2, 4, 1, 5, 3) are both ordered quintuples with (2, 2, 3, 2, 2) (2, 4, 1, 5, 3).
Well simply call all of these ordered n-tuples or even simply tuples. Hence,
Example 184. (1, 2, 3), (2, 1, 3), (1, 1, 1, 1), (2, 4, 1, 3), (2, 2, 3, 2, 2), and (2, 4, 1, 5, 3) are
all ordered n-tuples. (1, 2, 3) and (2, 1, 3) are ordered 3-ples or triples. (1, 1, 1, 1) and
(2, 4, 1, 3) are ordered 4-tuples or quadruples. (2, 2, 3, 2, 2) and (2, 4, 1, 5, 3) are ordered
5-tuples or quintuples.
In fact, when talking about tuples, it will be understood that they are ordered, so well
drop the word ordered and simply call them tuples (instead of ordered tuples).
Definition 51. A finite sequence of length n is any n-tuple.

Example 185. (1, 2, 3) and (2, 1, 3) are 3-ples or, equivalently, finite sequences of length
3.
(1, 2, 3, 4) and (2, 4, 1, 3) are 4-tuples or, equivalently, finite sequences of length 4.
(1, 2, 3, 4, 5) and (2, 4, 1, 5, 3) are 5-tuples or, equivalently, finite sequences of length 5.
We refer to the objects in a sequence as terms.
Example 186. Given the sequence (2, 1, 3), 2 is its first term, 1 is its second term, and
3 is its third term.

Page 225, Table of Contents

www.EconsPhDTutor.com

19.1

A Corresponding Function for a Sequence

Another perspective is to think of a finite sequence of length n as a function whose domain


is {1, 2, 3, . . . , n} and whose codomain is R.30
Example 187. (2, 4, 6, 8, 10, 12, 14) is a finite sequence of length 7, consisting of the first
seven even positive integers. A corresponding function f for this sequence has
Domain {1, 2, 3, 4, 5, 6, 7};
Codomain R; and
Mapping rule f (n) = 2n, for all n.
Indeed, the values of the function f (1) = 2, f (2) = 4, f (3) = 6, ..., f (7) = 14 exactly list out
the terms in the finite sequence (2, 4, 6, 8, 10, 12, 14).
Example 188. (2, 5, 12, 23, 38, 57, 80, 107, 138, 173) is a finite sequence of length 10. A
corresponding function f for this sequence has
Domain {1, 2, 3, 4, 5, 6, 7, 8, 9, 10};
Codomain R; and
Mapping rule : f (n) = 2n2 3n + 3, for all n.
Indeed, the values of the function f (1) = 2, f (2) = 5, f (3) = 12, f (4) = 23, ..., f (10) = 173
exactly list out the terms in the finite sequence (2, 5, 12, 23, 38, 57, 80, 107, 138, 173).

Exercise 91. (Answer on p. 1068.) For each of the following finite sequences, write down
a corresponding function.
(a) (1, 4, 9, 16, 25, 36, 49, 64, 81, 100).
(b) (2, 5, 8, 11, 14, 17, 20).
(c) (0.5, 4, 13.5, 32, 62.5, 108, 171.5).
(d) (2, 6, 6, 12, 10, 18, 14, 24, 18, 30, 22, 36, 26, 42).
(e) (18, 14.5).

30

Indeed, this is how a sequence is usually formally defined.

Page 226, Table of Contents

www.EconsPhDTutor.com

19.2

Recurrence Relations

SYLLABUS ALERT
Recurrence relations are included in the 9740 (old) syllabus, but not in the 9758 (revised)
syllabus. So you can skip this section if youre taking 9758.

Example 189. (1, 2, 4, 8, 16, 32, 64, 128, 256, 512, 1024) is a finite sequence of length 10. A
corresponding function f for this sequence has
Domain {1, 2, 3, 4, 5, 6, 7, 8, 9, 10};
Codomain R; and
Mapping rule : f (1) = 1 and f (n) = 2f (n 1) (the recurrence relation), for all n 2.
The equation f (n) = 2f (n1) is an example of a recurrence relation. That is, it describes
how each term in the sequence is generated, depending on what previous terms were.
In this particular example of a sequence, we can easily write down another corresponding
function that does not involve a recurrence relation:
Example 190. (1, 2, 4, 8, 16, 32, 64, 128, 256, 512, 1024) is a finite sequence of length 10. A
corresponding function g for this sequence has
Domain {1, 2, 3, 4, 5, 6, 7, 8, 9, 10};
Codomain R; and
Mapping rule : g(n) = 2n1 (not a recurrence relation), for all n.

If we can describe a sequence without using a recurrence relation, then we can immediately
compute what each term in the sequence is. So in the case of the finite sequence just given,
we prefer to use the function g rather than the function f as a corresponding function.
In contrast, with a recurrence relation, we need to know what some of the previous terms
are, in order to compute each term. So if possible, we prefer to describe sequences without
using recurrence relations.
But sometimes, it is difficult to describe a sequence without using a recurrence relation.

Page 227, Table of Contents

www.EconsPhDTutor.com

Example 191. (1, 4, 10, 22, 46, 94, 190, 382, 766, 1534) is a finite sequence of length 10. A
corresponding function f for this sequence has
Domain {1, 2, 3, 4, 5, 6, 7, 8, 9, 10};
Codomain R; and
Mapping rule : f (1) = 1 and f (n) = 2f (n 1) + 2 (the recurrence relation), for all n 2.
It is possible to describe the sequence just given without using a recurrence relation, but it
does not come obviously (at least to the untrained eye) and takes a little work, as well see.

A recurrence relation can certainly involve more than just the previous term. In the Fibonnaci sequence, each term (from the third term onwards) is the sum of the previous
two terms: f (n) = f (n 2) + f (n 1). This equation is again a recurrence relation.
But in the past ten years exams, I havent seen a question where the recurrence relation
involves more than just the previous term. So we shall not bother doing much of these.
Example 192. (1, 1, 2, 3, 5, 8, 13, 21, 34, 55, 89) is a finite sequence of length 11, consisting
of the first 11 Fibonacci numbers. A corresponding function f for this sequence has
Domain {1, 2, 3, 4, 5, 6, 7, 8, 9, 10, 11};
Codomain R; and
Mapping rule : f (n) = 1, for n = 1, 2; and f (n) = f (n 2) + f (n 1) (the recurrence
relation), for all n 3.

Exercise 92. Each of the following finite sequences involves a recurrence relation. (Hint:
Each involves only the previous term and also a squared term.) Write down a corresponding
function for each. (a) (3, 4, 9, 64, 3969). (b) (1, 2, 10, 290, 252010). (Answer on p. 1069.)

Page 228, Table of Contents

www.EconsPhDTutor.com

19.3

Creating New Sequences

Notation. Consider the finite sequence of length k (a1 , a2 , a3 , . . . , ak ). A shorthand piece of


notation for this sequence is (an )nk . We call n the index variable or dummy variable.
Well assume the index variable always starts from 1, unless otherwise specified.
Example 193. (1, 1, 1, 3, 5, 9, 17, 31, 57, 105, 193) is a finite sequence of length 11. We can
also write it as (an )n11 = (a1 , a2 , a3 , . . . , a11 ), where a1 = 1, a2 = 1, a3 = 1, a4 = 3, a5 = 5, ...,
a11 = 193.
Example 194. (1, 1, 1, 2, 2, 3, 4, 5, 7, 9, 12, 16, 21, 28, 37, 49, 65, 86, 114, 151) is a finite sequence of length 20. We can also write it as (bn )n20 = (b1 , b2 , b3 , . . . , b20 ), where b1 = 1,
b2 = 1, b3 = 1, b4 = 2, b5 = 2, ..., b20 = 151.
Example 195. (2, 4, 6, 8, 10, 12, 14) is a finite sequence of length 7. We can also write it as
(cn )n7 = (c1 , c2 , c3 , . . . , c7 ), where c1 = 2, c2 = 4, c3 = 6, ..., c7 = 14.

Example 196. (1, 1, 3, 5, 11, 21, 43, 85, 171, 341, 683) is a finite sequence of length 11. We
can also write it as (dn )n11 = (d1 , d2 , d3 , . . . , d11 ), where d1 = 1, d2 = 1, d3 = 3, d4 = 5, d5 = 11,
..., d11 = 683.
We can create new sequences out of old ones, in the obvious fashion:
Example 197. Using the sequence (an )n11 = (1, 1, 1, 3, 5, 9, 17, 31, 57, 105, 193), here are
some new sequences we can create:
(zn )n11 = (an + 1)n11 = (a1 + 1, a2 + 1, a3 + 1, . . . , a11 + 1)
= (2, 2, 2, 4, 6, 10, 18, 32, 58, 106, 194) = (z1 , z2 , z3 , . . . , z11 ) ,
(yn )n11 = (2an )n11 = (2a1 , 2a2 , 2a3 , . . . , 2a11 )
= (2, 2, 2, 6, 10, 18, 34, 62, 114, 210, 386) = (y1 , y2 , y3 , . . . , y11 ) ,
(xn )n11 = (an 1)n11 = (a1 1, a2 1, a3 1, . . . , a11 1)
= (0, 0, 0, 2, 4, 8, 16, 30, 56, 104, 192) = (x1 , x2 , x3 , . . . , x11 ) ,
(wn )n11 = (an /2)n11 = (a1 /2, a2 /2, a3 /2, . . . , a11 /2)
= (1/2, 1/2, 1/2, 3/2, 5/2, 9/2, 17/2, 31/2, 57/2, 105/2, 193/2) = (w1 , w2 , w3 , . . . , w11 ) .

Page 229, Table of Contents

www.EconsPhDTutor.com

Moreover, using two (or more) finite sequences that are of the same length, we can
likewise create a new finite sequence (also of the same length), in the obvious fashion:
Example 198. Using the sequences (an )n11 = (1, 1, 1, 3, 5, 9, 17, 31, 57, 105, 193) and
(dn )n11 = (1, 1, 3, 5, 11, 21, 43, 85, 171, 341, 683), here are some new sequences we can create:
(en )n11 = (an + dn )n11 = (a1 + d1 , a2 + d2 , a3 + d3 , . . . , a11 + d11 )
= (2, 2, 4, 8, 16, 30, 60, 116, 228, 446, 876) = (e1 , e2 , e3 , . . . , e11 ) ,
(fn )n11 = (an dn )n11 = (a1 d1 , a2 d2 , a3 d3 , . . . , a11 d11 )
= (1, 1, 3, 15, 55, 189, . . . , 131819) = (f1 , f2 , f3 , . . . , f11 ) ,
(gn )n11 = (an dn )n11 = (a1 d1 , a2 d2 , a3 d3 , . . . , a11 d11 )
= (0, 0, 2, 2, 6, 12, 26, . . . , 490) = (g1 , g2 , g3 , . . . , g11 ) ,
(hn )n11 = (an /dn )n11 = (a1 /d1 , a2 /d2 , a3 /d3 , . . . , a11 /d11 )
= (1, 1, 1/3, 3/5, 5/11, 9/21, . . . , 193/683) = (h1 , h2 , h3 , . . . , h11 ) .

There are of course many other new sequences we can create, whether using only one
sequence, using two sequences, or even using three or more sequences.
Remark 6. You cannot create a new sequence using two finite sequences that are of different
lengths. For example, given two finite sequences (an )n11 = (1, 1, 1, 3, 5, 9, 17, 31, 57, 105, 193)
and (cn )n7 = (2, 4, 6, 8, 10, 12, 14), there is no such sequence as (an + cn )n11 or even
(an + cn )n7 . Either of these supposed sequences is simply undefined.
It turns out that we are rarely interested in finite sequences. Instead, we are much more
interested in infinite sequences, which is a simple extension of the concept of finite sequences.

Page 230, Table of Contents

www.EconsPhDTutor.com

20

Infinite Sequences

We can easily extend the concept of finite sequences to infinite sequences, which have
domain Z+ = {1, 2, 3, 4, . . . } (the entire set of positive integers).
Example 199. (2, 4, 6, 8, 10, 12, 14, 16, 18, . . . ) is the infinite sequence consisting of all the
even positive integers. A corresponding function f for this sequence has
Domain Z+ ;
Codomain R; and
Mapping rule f (n) = 2n for all n.

Example 200. (1, 3, 6, 10, 15, 21, 28, 36, 45, 55, . . . ) is the infinite sequence consisting of the
triangular numbers. A corresponding function f for this sequence has
Domain Z+ ;
Codomain R; and
Mapping rule f (1) = 1 and f (n) = 1 + 2 + + n for all n 2.

Example 201. The infinite sequence (1, 2, 6, 24, 120, 720, 5040, ...) has the corresponding
function f with
Domain Z+ ;
Codomain R; and
Mapping rule f (n) = 1 2 n = n! for all n.

Exercise 93. For each of the following infinite sequences, write down a corresponding function. (a) (1, 4, 9, 16, 25, 36, 49, 64, 81, 100, . . . ). (b) (2, 5, 8, 11, 14, 17, 20, . . . ). (c)
(0.5, 4, 13.5, 32, 62.5, 108, 171.5, . . . ). (d) (2, 6, 6, 12, 10, 18, 14, 24, 18, 30, 22, 36, 26, 42, . . . ).
(Answer on p. 1070.)

Page 231, Table of Contents

www.EconsPhDTutor.com

20.1

Creating New Sequences

(an ) is our shorthand notation for an infinite sequence, where (an ) = (a1 , a2 , a3 , . . . ).
As stated, we are rarely interested in finite sequences. And so whenever we talk about
a sequence, it should be assumed that we are talking about an infinite sequence, unless
otherwise clearly stated.
The idea of creating new sequences carries over from the finite case in the obvious fashion.
Example 202.

Let
and
Then

(an ) = (1, 1, 2, 3, 5, 8, 13, 21, 34, 55, . . . )


(bn ) = (2, 4, 6, 8, 10, 12, 14, 16, 18, 20, . . . ) .
(an + bn ) = (3, 5, 8, 11, 15, 20, 27, 37, 52, 75, . . . ) .

Analogous to Remark 6, you cannot create a new sequence using a finite sequence and an
infinite sequence. Instead, you can only create one using two infinite sequences.
Example 203.

Let
and
Then

Page 232, Table of Contents

(an ) = (1, 1, 2, 3, 5, 8, 13, 21, 34, 55, . . . )


(bn )n7 = (2, 4, 6, 8, 10, 12, 14) .
(an + bn ) is undefined.

www.EconsPhDTutor.com

21

Series

Definition 52. Given a finite sequence (an )nk , its series is the expression
a1 + a2 + a3 + + ak .
We refer to a1 as the first term of the sequence and also as the first term of the series.
Similarly, a2 is the second term of both the sequence and the series. Etc.
Definition 53. Given a finite sequence (an )nk , its sum of series is the number S such
that S = a1 + a2 + a3 + + ak .

Example 204.

(an )n8 = (1, 1, 1, 3, 5, 9, 17, 31) ,


1 + 1 + 1 + 3 + 5 + 9 + 17 + 31
68.

Example 205.

(bn )n11 = (2, 4, 6, 8, 10, 12, 14) ,


2 + 4 + 6 + 8 + 10 + 12 + 14
56.

Given the sequence


its series is the expression
and its sum of series is the number

Given the sequence


its series is the expression
and its sum of series is the number

It may seem strange and unnecessary to distinguish between a series and a sum of series.
Arent they exactly the same thing?
It turns out that expressions like a1 + a2 + a3 + + ak play an important role in maths and
so we want to reserve a special name for the expression itself and distinguish it from the
sum of series. For example, we might be specifically interested in the series 1 + 2 + 3, rather
than just the sum of series 6.
Clearly, every finite sequence has a well-defined sum of series simply add up all the terms
in the finite sequence!
Definition 54. Given an infinite sequence (an ), its series is the expression a1 + a2 + a3 + . . . .
A series that corresponds to a finite sequence is called a finite series, while a series that
corresponds to an infinite sequence is called an infinite series.

Page 233, Table of Contents

www.EconsPhDTutor.com

21.1

Convergent and Divergent Sequences and Series

Every finite sequence has a sum of series. In contrast, not all infinite sequences do:
Example 206. Consider the sequence (an ) = (1, 1, 1, 1, 1, 1, . . . ). Its series is the expression
1 + 1 + 1 + 1 + 1 + . . . . There is no number equal to 1 + 1 + 1 + 1 + 1 + . . . and so a sum of series
does not exist for this sequence.
But some infinite sequences do have sums of series:
Example 207. Consider the sequence (bn ) = (0, 0, 0, 0, 0, 0, . . . ). Its series is the expression
0 + 0 + 0 + 0 + 0 + . . . . The sum of series for this sequence exists and is 0.

Definition 55. An infinite sequence for which a sum of series exists is said to have a
convergent series.
An infinite sequence for which no sum of series exists is said to have a divergent series.
So in the above examples, we say that the sequence (an ) has a divergent series (because its
sum of series does not exist), while the sequence (bn ) has a convergent series (because its
sum of series exists).

Page 234, Table of Contents

www.EconsPhDTutor.com

But what exactly is a convergent series? When exactly is a series convergent?


These are actually fascinating questions, which means, of course, that theyre not in the
syllabus. Here is a simple example that gives you a glimpse of the difficulties involved.
Chapter 85 in the Appendices (optional) gives the precise definitions of when a series
converges or diverges.
Example 208. Consider the sequence (cn ) = (1, 1, 1, 1, 1, 1, . . . ), where the terms simply alternate between 1 and 1. Its series is the expression 1 1 + 1 1 + 1 1 + . . . . Is there
any number that is equal to 1 1 + 1 1 + 1 1 + . . . ? Its actually not obvious. On the one
hand, we can pair together every two terms like so:
1 1 + 1 1 + 1 1 + . . . = (1 1) + (1 1) + (1 1) + . . .

0

= 0 + 0 + 0 + ...

and happily conclude that the sum of series is 0. But wait a minute ... what if we instead
pair together every two terms like so:
1 1 + 1 1 + 1 1 + 1 . . . = 1 + (1 + 1) + (1 + 1) + (1 + 1) + . . .

0

= 1 + 0 + 0 + 0 + ...

Then wed have to conclude that the sum of series is 1!


It turns out that the sequence (cn ) = (1, 1, 1, 1, 1, 1, . . . ) is divergent. Or equivalently,
a sum of series simply does not exist for this sequence.

Page 235, Table of Contents

www.EconsPhDTutor.com

22

Summation Notation

is the upper-case Greek letter sigma. An enlarged version of that letter , read aloud
as sum, is used to express series in compact notation:
Example 209. Consider the series 1 + 2 + 3 + 4 + 5 + 6 + 7 + 8 + 9. Another way to write it
is to use summation notation:
9

1 + 2 + 3 + 4 + 5 + 6 + 7 + 8 + 9 = n.
n=1

Lets examine the expression on the RHS.


The variable n below the is called the index variable or dummy variable. We could
have named it p or z or x or any other letter (instead of n) and it wouldnt have mattered.
Hence the name dummy.
The = 1 below the says that we start counting the index variable from n = 1. We call
the number 1 the starting point.
The 9 above the is called the stopping point. It says that we should stop adding
once we hit n = 9.
9

Altogether, the notation says that we are adding up 9 terms, namely a1 , a2 , ..., a9 .
n=1

The expression to the right of the tells us what each an is. In this example, it is n, which
simply says that for every n, an = n.
9

Altogether, n says that we add up a1 through a9 , where each an is simply equal to n.


n=1

Example 210. The series 1 + 1 + 1 + 1 + 1 + 1 + 1 + 1 + 1 can be written as


9

1.

n=1

This says that the starting point is 1 and the ending point is 9. In other words, we add
up a1 , a2 , . . . , a9 , where for each n, an = 1. And so a1 = 1, a2 = 1, etc. Altogether:
9

1 = a1 + a2 + + a9 = 1 + 1 + 1 + 1 + 1 + 1 + 1 + 1 + 1.

n=1

Page 236, Table of Contents

www.EconsPhDTutor.com

Example 211. The series 2 + 4 + 6 + 8 + 10 + 12 + 14 can be written as


7

2n.

n=1

This says that the starting point is 1 and the ending point is 7. In other words, we add
up a1 , a2 , . . . , a7 , where for each n, an = 2n. And so a1 = 2, a2 = 4, etc. Altogether:
7

2n = a1 + a2 + + a7 = 2 1 + 2 2 + + 2 7 = 2 + 4 + 6 + 8 + 10 + 12 + 14.

n=1

The series 3 + 5 + 7 + 8 + 11 + 13 + 15 can be rewritten as (2n + 1) the parentheses help


n=1

to clarify that we are not talking about 1 + 2n.


n=1

This says that the starting point is 1 and the ending point is 7. In other words, we add
up a1 , a2 , . . . , a7 , where for each n, an = 2n + 1. And so a1 = 3, a2 = 5, etc. Altogether:
3

15



(2n + 1) = a1 + a2 + + a7 = (2 1 + 1) + (2 2 + 1) + + (2 7 + 1).
7

n=1

Page 237, Table of Contents

www.EconsPhDTutor.com

Example 212. The series 2 + 4 + 8 + 16 + 32 + 64 + 128 + 256 + 512 + 1024 can be written as
10

2n .

n=1

This says that the starting point is 1 and the ending point is 10. In other words, we
add up a1 , a2 , . . . , a10 , where for each n, an = 2n . And so a1 = 2, a2 = 4, etc. Altogether:
10

2n = a1 + a2 + + a10 = 21 + 22 + 23 + + 210 = 2 + 4 + 8 + + 1024.

n=1

Its nice to have 1 as the starting point, but theres no reason why this must always be so.
Example 213. The series 1 + 2 + 4 + 8 + 16 + 32 + 64 + 128 + 256 + 512 + 1024 can be written
as
10

2n .

n=0

This says that the starting point is 0 and the ending point is 10. In other words, we
add up a0 , a1 , a2 , . . . , a10 , where for each n, an = 2n . And so a0 = 1, a1 = 2, a2 = 4, etc.
Altogether:
10

2n = a0 + a1 + a2 + + a10 = 20 + 21 + 22 + + 210 = 1 + 2 + 4 + + 1024.

n=0

Page 238, Table of Contents

www.EconsPhDTutor.com

Exercise 94. Rewrite each of the following in summation notation. (Answer on p. 1071.)
(a) 1 + 4 + 9 + 16 + 25 + 36 + 49 + 64 + 81 + 100.
(b) 2 + 5 + 8 + 11 + 14 + 17 + 20 + 23.
(c) 0.5 + 4 + 13.5 + 32 + 62.5 + 108 + 171.5.
Exercise 95. Find the sum of each of the following series. (Answer on p. 1071.)
5

(a) (2 n) .
n=2
17

(b) (4n + 5).


n=16
33

(c) (x 3).
x=31

Heres the general definition of the summation notation:


Definition 56. Let s, k be integers with s k. Let f be a function whose domain contains
s, s + 1, . . . , k and whose codomain is R. Then
k

f (n) = f (s) + f (s + 1) + + f (k).

n=s

Page 239, Table of Contents

www.EconsPhDTutor.com

23

Arithmetic Sequences and Series

Example 214. Consider the finite sequence (4, 7, 10, 13, 16, 19, 22). A corresponding function f for this sequence has
Domain {1, 2, 3, 4, 5, 6, 7} (a subset of Z+ );
Codomain R; and
Mapping rule f (1) = 4 and f (n) f (n 1) = 3 for all n 2.
This is an example of a finite arithmetic sequence.
Example 215. Consider the infinite sequence (4, 7, 10, 13, 16, 19, 22, 25, 28, 31, 34, . . . ). A
corresponding function f for this sequence has
Domain Z+ ;
Codomain R; and
Mapping rule f (1) = 4 and f (n) f (n 1) = 3 for all n 2.
This is an example of an infinite arithmetic sequence.

Definition 57. An arithmetic sequence (or an arithmetic progression) is any finite or infinite sequence (an ) where an+1 an is a constant for all n = 1, 2, 3, . . . . We call an+1 an
the common difference. We call the series for an arithmetic sequence an arithmetic series.
And its sum of series (if it exists at all) is called an arithmetic sum of series.

Example 216. The sequence (an ) = (1, 4, 7, 10, 13, 16, 19, . . . ) is an arithmetic sequence
because an+1 an is constant for n = 1, 2, 3, . . . .
But the sequence (bn ) = (1, 1, 4, 7, 10, 13, 16, 19, . . . ) is not an arithmetic sequence because
a2 a1 = 0 a3 a2 = 3.

The next fact is intuitively obvious. Clearly, there is no number for which, for example,
4 + 7 + 10 + 13 + 16 + 19 + 22 + . . . is equal to.
Fact 12. The infinite arithmetic sequence (an ) has no sum of series, except in the trivial
case where (an ) = (0, 0, 0, 0, 0, 0, . . . ).

Page 240, Table of Contents

www.EconsPhDTutor.com

23.1

Finite Arithmetic Sequences and Series

Every finite sequence, including arithmetic ones, has a sum of series.


Example 217. Youve probably heard of the apocryphal story about an eight-year-old
Gauss adding up the numbers from 1 to 100 in an instant. The trick is to pair the first
number with the last, the second number with the second last, etc. then use multiplication.
Like this:
50 terms

1 + 2 + 3 + 4 + + 100 = (1 + 100) + (2 + 99) + (3 + 98) + + (50 + 51)


101

101

101

101

= 101 50 = 5050.
In general, there is a simple formula for the sum of a finite arithmetic series: (First Term
+ Last Term) (Number of Terms) 2.
k
Fact 13. The finite arithmetic series a1 + a2 + + ak has sum of series (a1 + ak ) .
2
(We will only prove Fact 13 on p. 249.)
Example 218. Consider the arithmetic sequence (7, 17, 27, 37, . . . , 837). Its common difference is 10. The difference between the first and last terms is 830. And so the last term
is 830 10 = 83 terms after the first. Hence, there are in total 84 terms. By Fact 13, its
84
sum of series is (7 + 837)
= 35488.
2
Example 219. Consider the arithmetic sequence (1, 5, 9, 13, 17, 21, 25, 29, 33, . . . , 393). Its
common difference is 4. The difference between the first and last terms is 392. And so the
last term is 392 4 = 98 terms after the first. Hence, there are in total 99 terms. By Fact
99
13, its sum of series is (1 + 393)
= 19503.
2

Exercise 96. Rewrite each of the following arithmetic series in summation notation and
compute their sums. (a) 2+7+12+17+22+27+32+ +997. (b) 3+20+37+54+71+ +1703.
(c) 81 + 89 + 97 + 105 + 113 + + 8081 (Answer on p. 1072.)

Page 241, Table of Contents

www.EconsPhDTutor.com

24

Geometric Sequences and Series

Example 220. Consider the finite sequence (1, 2, 4, 8, 16, 32, 64, 128). A corresponding
function f for this sequence has
Domain {1, 2, 3, 4, 5, 6, 7, 8} (a subset of Z+ );
Codomain R; and
Mapping rule f (1) = 1 and f (n + 1) f (n) = 2 for all n = 1, 2, 3, . . . .
This is an example of a finite geometric sequence.
Example 221. Consider the finite sequence (1, 2, 4, 8, 16, 32, 64, 128, 256, 512, . . . ). A corresponding function f for this sequence has
Domain Z+ ;
Codomain R; and
Mapping rule f (1) = 1 and f (n + 1) f (n) = 2 for all n = 1, 2, 3, . . . .
This is an example of a infinite geometric sequence.
Definition 58. A geometric sequence (or a geometric progression) is any sequence (an )
where an+1 an is constant for all n = 1, 2, 3, . . . . We call an+1 an the common ratio. We
call the series for a geometric sequence a geometric series. And its sum of series (if it exists
at all) is called a geometric sum of series.

Example 222. The sequence (an ) = (1, 2, 4, 8, 16, 32, . . . ) is a geometric sequence because
an+1 an is constant for all n = 1, 2, 3, . . . .
But the sequence (bn ) = (1, 1, 2, 4, 8, 16, 32, . . . ) is not a geometric sequence because a2 a1 =
1 a3 a2 = 2.

Page 242, Table of Contents

www.EconsPhDTutor.com

24.1

Finite Geometric Sequences and Series

It turns out that just like with finite arithmetic series, there is a nice formula for the finite
geometric series. Lets start with the simple case first where the first term is simply 1.

Fact 14. 1 + r + r + r + + r

n1

1 rn
.
=
1r

Proof. Let S = 1 + r + r2 + + rn1 . Then rS = r + r2 + r3 + + rn .


Now take the difference: S rS = 1 rn .
1 rn
Hence, S =
.
1r
The trick used in the above proof is called the method of differences and the A-level
syllabus requires you to know it. The general case of a geometric series follows immediately
from the above:

Fact 15. a1 + a1 r + a1 r + a1 r + + a1 r

n1

1 rn
= a1
.
1r

Example 223. Consider the geometric sequence (1, 2, 4, 8, 16, . . . , 1024). Its common ratio
is 2. The ratio of the last term to the first is 1024 1 = 1024 = 210 . And so the last term
is 10 terms after the first. Hence, there are in total 11 terms. Thus, its sum of series is
1 211 2047
1
=
= 2047.
12
1
Example 224. Consider the geometric sequence (4, 12, 36, 108, . . . , 8748). Its common
ratio is 3. The ratio of the last term to the first is 8748 4 = 2187 = 37 . And so the last
term is 7 terms after the first. Hence, there are in total 8 terms. Thus, its sum of series is
1 38
6560
4
=4
= 4 3280 = 13120.
13
2

Exercise 97. Rewrite each of the following geometric series into summation notation and
compute their sums. (a) 7 + 14 + 28 + 56 + + 448 + 896. (b) 20 + 10 + 5 + + 5/8. (c)
1 + 1/3 + 1/9 + + 1/243. (Answer on p. 1073.)

Page 243, Table of Contents

www.EconsPhDTutor.com

24.2

Infinite Geometric Sequences and Series

Perhaps surprisingly, it turns out that under a certain condition, an infinite geometric
sequence can have a sum of series. Again, lets start with the simple case:

Fact 16. If r < 1, then 1 + r + r2 + r3 + =

1
.
1r

Proof. Write the series as S = 1 + r + r2 + r3 + . . . . Then rS = r + r2 + r3 + r4 + . . . . (By the

way, we can also use summation notation for infinite series: S = r and S = rn+1 .)
n=0

n=0

Since r < 1, it follows that as n , rn 0. Hence, if we take the difference, we have


simply S rS = 1.
And so, S =

1
.
1r

The general case follows immediately:


Fact 17. If r < 1, then a1 + a1 r + a1 r2 + a1 r3 + =

a1
.
1r

The converse is also true:


Fact 18. If r 1, then a1 + a1 r + a1 r2 + a1 r3 + . . . diverges.

Proof. Optional, see p. 929 in the Appendices.

Exercise 98. Rewrite each of the following infinite geometric series in summation notation
and compute its sum. (a) 6 + 9/2 + 27/8 + . . . . (b) 20 + 10 + 5 + . . . . (c) 1 + 1/3 + 1/9 + . . . . (Answer
on p. 1073.)

Page 244, Table of Contents

www.EconsPhDTutor.com

25

Proof by the Method of Mathematical Induction

SYLLABUS ALERT
Proof by the method of mathematical induction is included in the 9740 (old) syllabus, but
not in the 9758 (revised) syllabus. So you can skip this Chapter if youre taking 9758.

Well now learn a new technique called proof by the method of mathematical induction. Its pretty difficult, so go real slow.31
Imagine an infinite chain of dominos. Our goal is to knock all of them down. Suppose we
manage to do two things:
1. Knock down the 1st domino (the base case).
2. Prove that if the jth domino is knocked down, then so too is the (j + 1)th domino
(the inductive step).
Then we will have succeeded. Because once the 1st domino is knocked down, the inductive
step implies that the 2nd domino is also knocked down, and now again by the inductive
step the 3rd domino is also knocked down, and now again by the inductive step the 4th
domino is also knocked down, ..., ad infinitum (to infinity).

31

Which is perhaps why they decided to drop it from the revised 9758 syllabus! It does appear though as the first topic of
Further Maths, which will be revived in 2017 and for which a free textbook will soon be appearing!

Page 245, Table of Contents

www.EconsPhDTutor.com

The metaphor of dominos is an apt description of the method of mathematical induction,


which Ill standardise into a three-step recipe:

The Method of Mathematical Induction


Step #1. Let P(k) be (shorthand for) the proposition to be proven. Our goal is to show
that P(k) is true for all k = 1, 2, 3, . . .
Step #2 (the base case). Verify that P(1) is true.
Step #3 (the inductive step). Show that P(j) implies P(j + 1) (for all j = 1, 2, 3, . . . ).

Step #1 rarely involves much work. Step #2 is usually, but not always, very easy. Step
#3 is usually the hardest part on the A-level exams, it usually just involves some (or a
lot of) algebra.
Why does the method of mathematical induction work? Step #2 (the base case) shows
that P(1) is true (knock down the 1st domino). Step #3 (the inductive step) then
implies that P(2) is also true (the falling 1st domino knocks down the 2nd domino).
Step #3 (the inductive step) then implies that P(3) is also true (the falling 2nd domino
knocks down the 3rd domino).
Step #3 (the inductive step ) then implies that P(4) is also true (the falling 3rd domino
knocks down the 4th domino).
Ad infinitum (to infinity). Thus, we have proven that P(k) is true for all k = 1, 2, 3, . . . , as
desired.
Too abstract? Work through all the examples and exercises and you should find that it is
not very difficult. For our first example, well reprove an earlier fact, but now using the
method of mathematical induction.

Page 246, Table of Contents

www.EconsPhDTutor.com

Fact 14 (reproduced from p. 243). 1 + r + r2 + r3 + + rn1 =

1 rn
.
1r

Proof. Step #1. Let P(k) be (shorthand for) the proposition that
1 + r + r2 + r3 + + rk1 =

1 rk
.
1r

Our goal is to show that P(k) is true for all k = 1, 2, 3, . . .

Step #2. Verify that P(1) is true.


1=

1 r1
.
1r

Step #3. Show that P(j) implies P(j + 1) (for all j = 1, 2, 3, . . . ).


Assume that P(j) is true. That is,
1

1 + r + r2 + r3 + + rj1 =

1 rj
.
1r

Our goal is to show that P(j + 1) is true. That is,


1 + r + r2 + r3 + + rj =
To this end, write:

1 rj+1
.
1r

1 + r + r2 + r3 + + rj = (1 + r + r2 + r3 + + rj1 ) + rj
j
1 rj + (1 r)rj 1 rj+1
1 1r
j
=
+r =
=
, as desired.
1r
1r
1r

In this particular instance, the method of mathematical induction was terribly cumbersome,
compared to our earlier four-sentence proof (p. 243). But it turns out that in many other
instances, this method is the best and sometimes the only tool to use.
Lets try more examples.

Page 247, Table of Contents

www.EconsPhDTutor.com

Example 225. Prove that r2 =


r=1

n(n + 1)(2n + 1)
.
6
k

Step #1. Let P(k) be (shorthand for) the proposition that r2 =


r=1

k(k + 1)(2k + 1)
.
6

Our goal is to show that P(k) is true for all k = 1, 2, 3, . . .


1

Step #2. Verify that P(1) is true: r2 = 1 =


n=1

1(1 + 1)(2 1 + 1)
.
6

Step #3. Show that P(j) implies P(j + 1) (for all j = 1, 2, 3, . . . ).


j

Assume that P(j) is true. That is, r2 =


n=1

j(j + 1)(2j + 1)
.
6

Our goal is to show that P(j + 1) is true. That is,


j+1

r2 =

n=1

(j + 1) [(j + 1) + 1] [2(j + 1) + 1]
.
6

To this end, write:


j+1

r2 = r2 + (j + 1)2

n=1

n=1

=
6

=
7

=
5

=
4

j(j + 1)(2j + 1)
+ (j + 1)2
6
j+1
[j(2j + 1) + 6(j + 1)]
6
j+1
(2j 2 + 7j + 1)
6
(j + 1)(j + 2)(2j + 3)
6
(j + 1) [(j + 1) + 1] [2(j + 1) + 1]
,
6

(Using =)

as desired.

I just used the backwards-forwards method. The order in which I wrote down each line
is given by the numbers above each = sign.
Another trick is to exploit the fact that it has got to work out right. So for example,
it might not immediately be obvious that 2j 2 + 7j + 1 = (j + 2)(2j + 3), but you know it
has got to work out right and thus this must surely be true (unless of course you made
some mistake with the algebra somewhere). And if you expand the RHS, you find that this
equation is indeed true.

Page 248, Table of Contents

www.EconsPhDTutor.com

Fact 13 (reproduced from p. 241). The finite arithmetic sequence (an )nk has sum of
k
series (a1 + ak ) .
2
Proof. Step #1. Let P(k) be (shorthand for) the proposition that
a1 + a2 + + ak =

k(a1 + ak )
.
2

Our goal is to show that P(k) is true for all k = 1, 2, 3, . . .

Step #2. Verify that P(1) is true: a1 =

1(a1 + a1 )
.
2

Step #3. Show that P(j) implies P(j + 1) (for all j = 1, 2, 3, . . . ).


Assume that P(j) is true. That is,
1

a1 + a2 + + aj =

j(a1 + aj )
.
2

Our goal is to show that P(j + 1) is true. That is,


a1 + a2 + + aj+1 =

(j + 1)(a1 + aj+1 )
.
2

Lets first observe that aj a1 = (j 1) (aj+1 aj ). In words, this equation says: Consider
the difference between the j th term and the first term; it is equal to j 1 times the difference
2 (j 1)aj+1 + a1
between two consecutive terms. Rearranging, we have aj =
.
j
Now write:
a1 + a2 + + aj+1

j(a1 + aj )
+ aj+1
2
j {a1 + [(j 1)aj+1 + a1 ] /j}
ja1 + (j 1)aj+1 + a1
=
+ aj+1 =
+ aj+1
2
2
(j + 1)a1 + (j 1)aj+1
(j + 1)a1 + (j 1)aj+1 + 2aj+1
=
+ aj+1
=
2
2
(j + 1)a1 + (j + 1)aj+1
(j + 1)(a1 + aj+1 )
=
=
, as desired.
2
2
= (a1 + a2 + + aj ) + aj+1

Page 249, Table of Contents

www.EconsPhDTutor.com

n(n + 1)
Exercise 99. Prove that r = [
] . (Answer on p. 1074.) By the way, this shows
2
r=1
n

r=1

r=1

that r3 = ( r) .
Exercise 100. (Answer on p. 1075.) Let a R. Prove that
1 (n + 1)an + nan+1
,
ra = a
(1 a)2
r=1
n

Exercise 101. Prove that r4 =


r=1

Page 250, Table of Contents

n(n + 1)(2n + 1)(3n2 + 3n 1)


. (Answer on p. 1076.)
30

www.EconsPhDTutor.com

Part III

Vectors

Page 251, Table of Contents

www.EconsPhDTutor.com

26

Quick Revision of Some O-Level Maths


26.1

Lines vs. Line Segments vs. Rays

A line is infinite, while a line segment is finite.


Example 226. The line running through points a and b goes forever, in both directions
(red dotted line). In contrast, the line segment ab is finite. The line ab is a different
mathematical object from the line segment ab.

The length of the line segment ab is thus a well-defined concept. In contrast, it makes no
sense to talk about the length of the line ab.
A ray is a portion of a line, beginning at some point along the line, then going towards
infinity. You can think of a ray as a half-infinite-line. The figure above illustrates in grey
the ray that starts from the point a and goes in the direction b.

This textbook will strictly reserve the word ray to mean a half-infinite-line. But you should
know that some other writers use ray to mean a (finite) line segment.

Page 252, Table of Contents

www.EconsPhDTutor.com

26.2

Angles - Acute, Right, Obtuse, Straight, Reflex

We will not use the degree as a unit of measurement for angles. In this textbook, the
unit of measurement for angles is the radian. As well see in a moment, the radian is
actually a unitless unit. So well always write, for example, /3 instead of /3 rad.

rad= 45 ,
rad= 90 , rad= 180 , and
4
2
2 rad= 360 . (This last sentence is the one and only time in this textbook that well use
degrees as a unit of measurement for angles.)

But just to refresh your memory, 0 rad= 0 ,

Angles are given different names, depending on their size.


is the zero angle if = 0,

is an obtuse angle if ( , ),
2

is an acute angle if (0, ),


2

is a straight angle if = ,

is a right angle if =

,
2

is a reflex angle if (, 2).

In the figure below, the angle A is acute, R is right, O is obtuse, S is straight, and X is
reflex. The zero angle is not depicted.
By convention, every angle is depicted as a sector of a circle, unless it is a right angle, in
which case it is depicted by a square.

Page 253, Table of Contents

www.EconsPhDTutor.com

26.3

Triangles - Acute, Right, Obtuse

Triangles are also given different names, depending on the size of their largest angle. A
triangle is:
Acute if its largest angle is acute;
Right if its largest angle is right; and
Obtuse if its largest angle is obtuse.
In the figure below, the largest angle of each triangle is highlighted.

Obtuse triangle

Acute triangle
Right triangle

Page 254, Table of Contents

www.EconsPhDTutor.com

26.4

Sine, Cosine, Tangent - Definitions

Both the sine and cosine functions have domain R and codomain [1, 1].
The tangent function has domain (1.5, 0.5)(0.5, 0.5)(0.5, 1.5). . . i.e.
all reals except half-integer multiples of . And the tangent functions codomain is R.
Draw a unit circle. Then given any point p = (px , py ) on the unit circle and the angle A
that the line segment op makes with the positive x-axis, we define sin A = py , cos A = px ,
py
and tan A = . Note that the line segment op has length 1.
px

y
p

py

A
px

In the case where A is acute (the point p is in the top-right quadrant of the cartesian
plane), one mnemonic is SOH, CAH, TOA Sine is Opposite over Hypothenuse, Cosine
is Adjacent over Hypothenuse, and Tangent is Opposite over Adjacent.

Page 255, Table of Contents

www.EconsPhDTutor.com

26.5

Sine, Cosine, Tangent - Values and Graphs

Sine and cosine fluctuate between 1 and 1. We describe their fluctuations as being sinusoidal. In contrast, tangent fluctuates between and . At half-integer multiples of ,
the tangent function is undefined.

y = tan x

y = cos x

y = sin x

0
3

x
0

-2

You dont need to memorise the following (because you have a calculator). But you will
solve problems a little more quickly if you have these memorised.
x

sin x 0
cos x 1
tan x 0

Page 256, Table of Contents

6
1/2

3/2
3/3

2/2
2/2

3/2

1/2

2
3

3/2

1/2

Undefined 3

3
4

2/2

5
6

1/2

3/3

2/2 3/2 1

www.EconsPhDTutor.com

26.6

Formulae for Sine, Cosine, and Tangent

For all x for which all expressions are well defined, we have:
tan x =

sin x
,
cos x

sin(x) = sin x,
sin(x + 2) = sin x,

cos(x) = cos x,
cos(x + 2) = cos x,

tan(x) = tan x,
tan(x + 2) = tan x.

The following formulae will appear in the List of Formulae youll get during exams, so
you dont need to memorise them. Exam Tip: Whenever you see a question with
trigonometric functions, make sure you have this list right next to you! For all
A, B, P, Q for which all expressions are well-defined, we have:
sin(A B) = sin A cos B cos A sin B,
cos(A B) = sin A cos B cos A sin B,
tan(A B) =

tan A tan B
,
1 tan A tan B

sin 2A = 2 sin A cos A,


cos 2A = cos2 A sin2 A = 2 cos2 A 1 = 1 2 sin2 A,
tan 2A =

2 tan A
,
1 tan2 A

sin P + sin Q = 2 sin (

P Q
P +Q
) cos (
),
2
2

sin P sin Q = 2 cos (

P +Q
P Q
) sin (
),
2
2

cos P + cos Q = 2 cos (

P +Q
P Q
) cos (
),
2
2

cos P cos Q = 2 sin (

Page 257, Table of Contents

P +Q
P Q
) sin (
).
2
2

www.EconsPhDTutor.com

26.7

Arcsine, Arccosine, Arctangent

We define sin2 A to be the square of sin A. One might thus suppose that analogously,
sin1 x = 1/ sin x, but this is not so! Instead:
Definition 59. The arcsine function, denoted sin1 , has domain [1, 1], codomain (and
range) [0.5, 0.5], and rule x y where sin y = x.
Below is the graph of the arcsine function. The endpoints (1, 0.5) and (1, 0.5) are
marked with red dots.

y
0.5
y = sin-1 x

x
-1.0

-0.6

-0.2

0.2

0.6

1.0

-0.5

Page 258, Table of Contents

www.EconsPhDTutor.com

A remark about principal values.


We refer to [0.5, 0.5] as the principal values of the arcsine function. What does this
mean?

Angles come full circle every 2 radians. And so for example, sin = 0.5. But also
6

sin ( + 2) = 0.5. And also sin ( + 4) = 0.5. And also sin ( 2) = 0.5. Indeed,
6
6
6

sin ( + 2k) = 0.5 for any k Z. We say that the sine function is periodic.
6

Yet we do not say that sin1 (0.5) = + 2k for any k Z because this would mean that
6
sin1 maps each element in the domain to more than one (indeed infinitely many) elements
in the codomain. And so sin1 wouldnt be a function.
Instead, we define the arcsine function so that its principal values are [0.5, 0.5]. That

is, the codomain of the arcsine function is [0.5, 0.5]. And thus, sin1 (0.5) = .
6
Note that the choice of [0.5, 0.5] as the principal values of the arcsine function is a
somewhat arbitrary convention. We could equally well have chosen, say, [0.5, 1.5] as our
principal values. Its nicer though that our principal values are centred on 0.

Page 259, Table of Contents

www.EconsPhDTutor.com

Definition 60. The arccosine function, denoted cos1 , has domain [1, 1], codomain (and
range) [0, ], and rule x y where cos y = x.
Below is the graph of the arccosine function. The endpoints (1, ) and (1, 0) are marked
with blue dots.
Note that [0, ] are the principal values of the arccosine function. Why cant we select
[0.5, 0.5] as the principal values for the arccosine function, like we did for the arcsine
function?32

y = cos-1 x

-1.0

32

-0.6

-0.2

0.2

0.6

1.0
x

Because then cos1 (1), for example, would be undefined.

Page 260, Table of Contents

www.EconsPhDTutor.com

Definition 61. The arctangent function, denoted tan1 , has domain R, codomain (and
range) (0.5, 0.5), and rule x y where tan y = x.
Below is the graph of the arctangent function. There are two horizontal asymptotes, namely
y = 0.5 and y = 0.5. That is, as x , y 0.5.
Note that (0.5, 0.5) are the principal values of the arctangent function.

y
y = 0.5
horizontal
asymptote
x
-10

-6

y=

tan-1

-2

10

y = -0.5
horizontal
asymptote

Remark 7. This notation can be tremendously confusing, which is why many writers prefer
to write arcsin x, arccos x, and arctan x instead of sin1 x, cos1 x, tan1 x. But the Singapore
Cambridge A-level syllabus does not use the arcsin x, arccos x, or arctan x notation and so
neither shall this textbook.
Page 261, Table of Contents

www.EconsPhDTutor.com

26.8

The Law of Sines and the Law of Cosines

Consider a triangle with sides of lengths a, b, and c and angles A, B, and C.

a
a sin C

A
b - a cos C

a cos C
b

Proposition 4. A triangle with sides of lengths a, b, and c and angles A, B, and C has
area is 0.5ab sin C.

Proof. The triangle has base b and height a sin C. Hence, its area is 0.5ab sin C.

Page 262, Table of Contents

www.EconsPhDTutor.com

Proposition 5. (The Law of Sines.) For a triangle with sides of lengths a, b, and c and
angles A, B, and C,
a
b
c
=
=
.
sin A sin B sin C

Proof. The area of the above triangle is 0.5ab sin C. By symmetry, it is also 0.5bc sin A and
0.5ac sin B. Equate these and divide by 0.5abc:
0.5ab sin C = 0.5bc sin A = 0.5ac sin B

b
c
a
=
=
.
sin A sin B sin C

Proposition 6. (The Law of Cosines.) For a triangle with sides of lengths a, b, and c
and angles A, B, and C, c2 = a2 + b2 2ab cos C.

Proof. (Optional.) By the Pythagorean Theorem,


2

c2 = (a sin C)2 + (b a cos C)


= a2 sin2 C + b2 2ab cos C + a2 cos2 C
= a2 (sin2 C + cos2 C) + b2 2ab cos C
= a2 + b2 2ab cos C,
where the last line uses the identity sin2 C + cos2 C = 1.

One perhaps-obvious implication of the Law of Cosines is that the length of any one side
of a triangle is always less than the sum of the lengths of the other two sides.
Corollary 3. For a triangle with sides of lengths a, b, and c, a < b + c.
Proof. c2 = a2 +b2 2ab cos C = a2 +b2 2ab+2ab2ab cos C = (ab)2 +2ab(1cos C) > (ab)2 .
Hence, c > a b or a < b + c.

Page 263, Table of Contents

www.EconsPhDTutor.com

27

Vectors in Two Dimensions (2D)

Recall that a point is simply an ordered pair of real numbers.


Example 227. The points a = (1, 2), b = (3, 1), c = (1, 1), and d = (3, 2) can be
illustrated graphically on the cartesian plane. The origin (0, 0) is usually named o.

a
2
c
1

0
-3

-2

-1

o
0

b
-1

-2

-3

We now introduce an entirely new mathematical object, called a vector.


We will not formally define vectors, because to do so would require more maths than is
covered at A-level. But informally, a vector is an arrow with two properties: direction
and length.

Page 264, Table of Contents

www.EconsPhDTutor.com

Example 228. In the figure, ab, cd, and u are all vectors. (As well see, there are multiple
ways to denote vectors.)

y
4

The vector ab = v = v
3
Length = 5
c 1

0
-3

-2

-1

-1
The vector cd = v = v
-2

5
x

b
The vector u
d

-3

Given two points a and b, ab denotes the vector from point a to point b. The word vector
means carrier (in Latin). You may have learnt in biology that mosquitoes are vectors,
because they carry diseases (to humans). In mathematics likewise, a vector carries us
from one point to another.

Example 229. The vector ab carries us from point a to point b. The vector cd carries us
from point c to point d.

Page 265, Table of Contents

www.EconsPhDTutor.com

Like a point, a vector can be described as being an ordered pair of real numbers

Example 230. The vector ab = (4, 3) carries us 4 units to the right and 3 units down. The

vector cd = (4, 3) carries us 4 units to the right and 3 units down. The vector u = (2, 1.5)
carries us 2 units to the right and 1.5 units down.
Note that were now using the (x, y) ordered set notation for the third time!33
Do not confuse a point with a vector!
Example 231. The point (4, 3) is a zero-dimensional object. In contrast, the vector
(4, 3) is a two-dimensional object.

The vector (x, y) can also be written as

Example 232. We can write (4, 3) =

x
.
y

4
2
and u = (2, 1.5) =
.
3
1.5

a
notation for vectors is very useful, because as well see shortly, well be doing a
b
lot of addition and multiplication with vectors, and this notation can help us see better (in
a literal sense). But in print, Ill often prefer using the (a, b) notation, simply because this
takes up less space.
The

The point a is called the vectors tail and the point b is called the vectors head. This is
potentially confusing, so always remember: a vector carries us from tail to head and
not the other way round!
A vector is defined by two characteristics: direction and length.
It must be stressed that the tail and head of a vector do not matter. Only the
direction and length do. So long as two vectors have the same direction and length, they
are considered to be exact same vector. Examples to illustrate

33

So far, we have used (x, y) to denote (i) an open interval specifically, the set of real numbers greater than x but smaller
than y; (ii) the ordered pair of real numbers x and y; and now also (iii) the vector that carries us x units to the right and
y units up.

Page 266, Table of Contents

www.EconsPhDTutor.com

Example 233. Informally, ab, cd, and u all point in the same direction. ab
and cd have
the same length, which we can compute using the Pythagorean Theorem as 32 + 42 = 5.

Hence, ab and cd are considered to be exactly the same: ab = cd. Even though they have

different heads and tails, both ab and cd carry us 4 units right and 3 units down. The

vector (4, 3) can carry us from a to b or from c to d. Thus, cd = (4, 3) = ab = (4, 3).
They are one and the same vector.

In contrast, the vector u has only half the length of ab and so u ab. (Indeed, as we shall

learn later, we can write u = 0.5ab.)


Example 234. The vector (0, 1) can carry us from a to c or from b to d. Thus,

= (0, 1).
Thus, bd = (0, 1) =
ac
But,
and

= (0, 1)
= (0, 1),
ca
ac
= (0, 1).
(0, 0.5)
ac

Yet another way of denoting vectors is by a single letter, either with a right arrow overhead

or in bold font. For example, in the figure above, the vector ab or cd is also named using

the letter v, either as


v or as boldfont v.

Example 235. So altogether, I can write the vector ab in five different ways:
4


ab =
v = v = (4, 3) =
.
3

Given a choice between writing


v or v, the bold font v is preferred in print publications.

But in handwriting, most people prefer


v (because writing in bold font is hard).

Exercise 102. Using a, b, c, or d from the above figure as the tail and a distinct point as
the head, there are 12 possible vectors. Weve already written out 4 of these in the last two
examples. Write out the other 8 in ordered set notation. (Answer on p. 1077.)

Page 267, Table of Contents

www.EconsPhDTutor.com

The position vector of a point a is simply the vector from the origin o = (0, 0) to the
point a. Formally:
Definition 62. Given a point a = (a1 , a2 ), its position vector is the vector a = (a1 , a2 ).
The position vector of the point a carries us from the origin o to the point a and so it
Take care not to confuse the point a = (a , a ) with the vector
can also be denoted
oa.
1 2
a = (a1 , a2 ) they are different objects!

Informally, the zero vector is the vector that carries us nowhere. Formally:

Definition 63. The zero vector is the vector (0, 0) and can be denoted 0 or 0 .

Page 268, Table of Contents

www.EconsPhDTutor.com

27.1

Sum and Difference of Points and Vectors

Here is a quick summary of what youll learn in this section.


(1)
(2)
(3)
(4)

Point
Point
Point
Point

+ Point = Undefined,
Point =
Vector,
+ Vector =
Point,
Vector =
Point.

1. Point + Point = Undefined


If a and b are points, then there is no such thing as a + b.34
The analogy is to points in the real world it makes no sense to talk about the sum of
two locations:
Example 236. Consider the points Paris and Tokyo. The sum Paris + Tokyo = ?? is
undefined. It makes no sense to talk about the sum of two locations.

p+v

v
u
ba
p

34

b
qu

At least in this textbook (and in the A-levels).

Page 269, Table of Contents

www.EconsPhDTutor.com

2. Point Point = Vector

Definition 64. Given two points a = (a1 , a2 ) and b = (b1 , b2 ), their difference ba is defined
to be the vector from a to b, i.e., b a = (b1 a1 , b2 a2 ).
Example 237. Paris Tokyo = The journey that carries us from Tokyo to Paris. We might
write Paris Tokyo =(9000 km, 1000 km), meaning that to get from Tokyo to Paris, we
must travel 9, 000 km west and 1, 000 km north.
It makes sense to talk about the distance of the journey from Tokyo to Paris. Shortly, well
see that it similarly makes sense to talk about the length of the vector from a to b.
Example 238. (See figure on p. 265.) Given the points a = (1, 2) and b = (3, 1), their
difference b a is the vector from a to b, i.e., b a = (3 (1), 1 2) = (4, 3).

3. Point + Vector = Point

Definition 65. Given the point p = (p1 , p2 ) and the vector v = (v1 , v2 ), their sum p + v is
defined to be the point p + v = (p1 + v1 , p2 + v2 ).
Geometrically, if the vector v has tail p, then it also has head p + v.
Example 239. Tokyo + (9000 km, 1000 km) = Paris. This says that starting from Tokyo,
if we embark on a journey that carries us 9, 000 km west and 1, 000 km north, then well
end up in Paris.

Example 240. (See figure on p. 265.) Consider the vector (4, 3). If its tail is a = (1, 2),
then its head is (1, 2) + (4, 3) = (3, 1) = b. And if its tail is c = (1, 1), then its head is
(1, 1) + (4, 3) = (3, 2) = d.

Page 270, Table of Contents

www.EconsPhDTutor.com

4. Point Vector = Point

Definition 66. Given the point q = (q1 , q2 ) and the vector u = (u1 , u2 ), their difference
q u is defined to be the point q u = (q1 u1 , q2 u2 ).
Geometrically, if the vector u has head q, then it also has tail q u.
Example 241. Paris (9000 km, 1000 km) = Tokyo. This says that starting from Paris,
if we embark on a journey that is the exact opposite of going 9, 000 km west and 1, 000 km
north (equivalently, we embark on a journey that goes 9, 000 km east and 1, 000 km south),
then well end up in Tokyo.

Example 242. (See figure on p. 265.) Consider again the vector (4, 3). If its head is
b = (3, 1), then its tail is (3, 1) (4, 3) = (1, 2) = a. And if its head is d = (3, 2), then
its tail is (3, 2) (4, 3) = (1, 1) = c.

Exercise 103. Consider the vector (4, 3). (a) If it has tail (0, 0), then what is its head?
(b) If it has head (0, 0), then what is its tail? (c) If it has tail (5, 2), then what is its head?
(d) If it has head (5, 2), then what is its tail? (Answer on p. 1077.)

Page 271, Table of Contents

www.EconsPhDTutor.com

27.2

Sum, Additive Inverse, and Difference of Vectors

Here is a quick summary of what youll learn in this section.


(1) Vector + Vector = Vector,
(2)
Vector = Vector, (additive inverse)
(3) Vector Vector = Vector.

1. Vector + Vector= Vector

Definition 67. If u = (u1 , u2 ) and v = (v1 , v2 ) are vectors, then their sum, denoted u + v,
is the vector defined by u + v = (u1 + v1 , u2 + v2 ).
Geometrically, if the tail of v is the head of u, then u + v is the vector from the tail of u
to the head of v.

u+v

Example 243. (See figure on p. 265.) ab + bc = (4, 3) + (4, 2) = (0, 1) =


ac.

Example 244. (See figure on p. 265.) ad + cb = (4, 4) + (4, 2) = (8, 6).

Page 272, Table of Contents

www.EconsPhDTutor.com

2. Vector= Vector (Additive inverse)

Definition 68. If v = (v1 , v2 ), then its additive inverse, denoted v, is defined by


v = (v1 , v2 ).
Geometrically, if the vector v is from point a to point b, then v is the vector from point
b to point a. And so informally, the additive inverse is simply the same vector but flipped
in the opposite direction.

Example 245. The additive inverse of ab is ba. That is, ab = ba.

Example 246. The additive inverse of bc is cb. That is, bc = cb.

3. Vector Vector= Vector

Definition 69. Given two vectors u and v, their difference, denoted u v, is defined to
be the sum of the vectors u and v. Or equivalently, if u = (u1 , u2 ) and v = (v1 , v2 ), then
u v is the vector defined by u v = (u1 v1 , u2 v2 ).
Geometrically, if we place the heads of u and v at the same point, then u v is the vector
from the tail of u to the tail of v.

u-v

Page 273, Table of Contents

www.EconsPhDTutor.com

could be written as the


In the previous section, we learnt that by definition, the vector
pq
= q p. Now, well prove that
can also be written as the
difference of two points:
pq
pq
difference of two vectors:
= q p.
Fact 19. Let p and q be two points with position vectors p and q. Then
pq
+ (
=
+
=
+
This is thus the vector that carries
Proof. q p = q + (p) =
oq
op)
oq
po
po
oq.
us first from p to o, then from o to q; in short, it carries us from p to q. So it is simply the

vector
pq.

Example 247. (See figure on p. 265.) b a = (3, 1) (1, 2) = (4, 3) = ab.

Example 248. (See figure on p. 265.) d c = (3, 2) (1, 1) = (4, 3) = cd.


Interpreting u v as the sum of the vectors u and v is often convenient:

Example 249. (See figure on p. 265.) Without any numbers, we can compute: ab cb =

We can verify with numbers that this is correct:


ab + ( cb) = ab + bc =
ac.
ab cb =
.
(4, 3) (4, 2) = (0, 1) =
ac

Example 250. (See figure on p. 265.) ad cb = (4, 4) (4, 2) = (0, 2).

+
Exercise 104. Write down what
ac
cb, dc +
ca, bd + da, ad cd, dc bd, and bd + db are,
without writing out any numbers.(Answer on p. 1077.)

Exercise 105. Using the figure on p. 265, compute each of the following:
ac
cb, dc
ca,

bd da, ad + cd, dc + bd, and bd db? (Answer on p. 1077.)

Page 274, Table of Contents

www.EconsPhDTutor.com

27.3

Displacement Vectors

Definition 70. If a moving particle starts at point a and ends at point b, we call ab its
displacement vector.

Example 251. A particle is travelling along the red arc, along the path shown. Its starting
point is in blue and its ending point is in purple. Its displacement vector is thus (2, 2).

y
x

0
-1

0
-1
-2

2 Ending
point

Displacement
vector (2, 2)
Starting point

-3
-4

Page 275, Table of Contents

www.EconsPhDTutor.com

27.4

Length (or Magnitude) of a Vector

The Pythagorean Theorem says that if c is the length of hypothenuse of a right-angled


triangle and a, b are the lengths of the other two sides, then a2 + b2 = c2 .

c
a

As you learnt in secondary school we can calculate the distance between two points using
the Pythagorean Theorem:
Example
be two points. Then the distance between p
252. Let p = (1, 1) and q =(1, 1)
2
2
and q is [1 (1)] + [1 (1)] = 4 + 4 = 8.

1 - (-1)

1 - (-1)

0
-2

-1

-1

-2

Page 276, Table of Contents

www.EconsPhDTutor.com

The vector v = (v1 , v2 ) goes v1 units right and v2 units up. We are thus motivated to define
its length (or magnitude) as:
Definition
71. The length (or magnitude) of a vector v = (v1 , v2 ) is denoted v and defined
by v = v12 + v22 .

Example 252 (continued). Another way to find the distance between p and q is to first
= (2, 2). The distance between p
find the vector that carries us from p to q. This is
pq

2
2

and q is thus simply the length (or magnitude) of this vector: pq = (2) + (2) = 8.
Of course, the distance from p to q is the same as the distance from q to p. So we could
= (2, 2) and gotten the same answer
=
just as well have calculated the length of
qp
qp

22 + 22 = 8.

Exercise 106. Using the figure on p. 265, compute each of the following:
ac
cb, dc
ca,

bd da, ad + cd, dc + bd, and bd db. Also, find the distance between (18, 4) and
(1, 2). (Answer on p. 1077.)
Exercise 107. In general, given any two vectors u and v, is it true that u + v = u + v?
(Answer on p. 1077.)

Page 277, Table of Contents

www.EconsPhDTutor.com

27.5

Scalar Multiplication of a Vector

Definition 72. A scalar is simply any real number.


A scalar is often contrasted with a vector. A vector has both magnitude (or length) and
direction. In contrast, a scalar has magnitude but no direction.
Definition 73. If v = (v1 , v2 ) is a vector and c R is a scalar, then cv denotes the vector
defined by cv = (cv1 , cv2 ). We call this operation scalar multiplication of a vector.
Graphically, cv is simply the vector that has the same direction as v, but with c times the
length. This is formally shown in the next fact.

v
cv

Fact 20. If v = (v1 , v2 ) is a vector and c R, then cv = c v.

Proof.

2
2
cv = (cv1 , cv2 ) = (cv1 ) + (cv2 )

2
2
2
2
= c v1 + c v2 = c v12 + v22
= c (v1 , v2 ) = c v .

Exercise 108. Using the figure on p. 265, write down 2ab, 3


ac, and 4ad in ordered set

= 3
and 4
notation. Verify that 2ab = 2 ab, 3
ac
ac,
ad = 4 ad. (Answer on p. 1078.)

Page 278, Table of Contents

www.EconsPhDTutor.com

27.6

Unit Vectors

Definition 74. A unit vector is any vector of length 1.

2 2
Example 253. Lets verify that the vectors (1, 0), (0, 1), and (
,
) are all unit vectors:
2 2

(1, 0) = 12 + 02 = 1,

(0, 1) = 02 + 12 = 1,


2
2

2
2 2
2
(
(
,
) =
) +(
) = 2/4 + 2/4 = 1.
2 2
2
2
Example 254. Lets verify that the vectors (1, 1) and (1, 1) are not unit vectors:

12 + 12 = 2 1,

2
2
(1, 1) = (1) + (1) = 2 1.
(1, 1) =

We specially reserve the name i (or i ) for the unit vector (1, 0), which is the unit vector
that is purely in the direction of the x-axis. Similarly, we specially reserve the name j (or

j ) for the unit vector (0, 1), which is the unit vector that is purely in the direction of the
y-axis.
And so, using also what we learnt about the sum of and scalar multiplication of vectors,
we can rewrite any vector into the sum of is and js:

Page 279, Table of Contents

www.EconsPhDTutor.com

Example 255. The position vectors for the points a, b, and c (illustrated below) are
a = (1, 2) = i + 2j, b = (4, 3) = 4i 3j, and c = (0, 6) = 6j.

y
c

6
j
5
j
4
j
3
j

2
ji
1
j

0
-3

-2

-1
-1
-2
-3

0-j

-j
-j
i

Page 280, Table of Contents

b
i

www.EconsPhDTutor.com

points in the same direction


Informally, the unit vector in the direction v denoted v
v, but has length 1. Formally:

Definition 75. The unit vector in the direction v is defined by v


=

1
v.
v


Exercise 109. In the figure on p. 265, what are the unit vectors in the directions ab,
ac,

and ad? What are the unit vectors in the directions 2ab, 3ac, and 4ad? (Answer on p.
1078.)

The following fact is an obvious corollary to Fact 20.


Fact 21. If c is a scalar and v
is a unit vector, then the vector c
v has length c.

Informally, two vectors have the same unit vector they both point in the same
direction. Formally:
a can be written as a scalar
=b
Fact 22. Let a and b be any two vectors. Then a
multiple of b.

Proof. Optional, see p. 930 in the Appendices.


Informally, any vector in the plane can be written as the linear combination of any other
two vectors. Formally:
Fact 23. Let a and b be any two vectors in the same plane with distinct directions (i.e.
Then every vector in the same plane can be written as a + b for some , R.
b).
a
Proof. Optional, see p. 930 in the Appendices.
See TYS Exercise 338 (i) for an application of the above fact.
Exercise 110. Given the vectors a = (1, 3) and b = (7, 5), show that each of the following
vectors can be written in the form a + b for some , R. (i) (0, 1). (ii) (1, 0). (iii)
(1, 1). (Answer on p. 1078.)

Page 281, Table of Contents

www.EconsPhDTutor.com

27.7

The Ratio Theorem


a

b
o

Theorem 3. Ratio Theorem. Let a, b, and p be points, where p is on the line segment
ab. Let a, b, and p be the corresponding position vectors. Then

bp

ap
p=
a+
b.
+
+

ap
ap
bp
bp
Proof. Optional, see p. 932 (Appendices).

and =
Or if we let =
ap
bp, then the above can be rewritten in a form that is perhaps
easier to remember:
p=

a
b
a + b
+
=
.
+ +
+

By the way, the List of Formulae (p. 4) contains this statement:


The point dividing AB in the ratio has position vector

Page 282, Table of Contents

a + b
.
+

www.EconsPhDTutor.com

Example 256. Consider the points a = (3, 4) and b = (1, 2). Find the point p that divides
the line segment ab into the ratio 3 2.
2
3
2
3
3 14
3 14
We have p = a + b = (3, 4) + (1, 2) = ( , ). Hence, the point is p = ( , ).
5
5
5
5
5 5
5 5

Example 257. Consider the points a = (8, 3) and b = (2, 6). Find the point p that divides
the line segment ab into the ratio 3 7.
We have p = 0.7a +0.3b = 0.7(8, 3)+0.3(2, 6) = (6.2, 0.3). Hence, the point is p = (6.2, 0.3).

Exercise 111. (a) Consider the points a = (1, 2) and b = (3, 4). Find the point p that
divides the line segment ab into the ratio 5 6. (b) Consider the points a = (1, 4) and
b = (2, 3). Find the point p that divides the line segment ab into the ratio 5 1. (c)
Consider the points a = (1, 2) and b = (3, 4). Find the point p that divides the line
segment ab into the ratio 2 3. (Answer on p. 1078.)

Page 283, Table of Contents

www.EconsPhDTutor.com

28

Scalar Product

Definition 76. Given two 2D vectors u = (u1 , u2 ) and v = (v1 , v2 ), their scalar product (or
dot product), denoted u v, is defined by u v = u1 v1 + u2 v2 .

And so to get the scalar product, simply multiply each term of each vector with the corresponding term of the other, then add these up. Its that simple!
The scalar product is itself simply a scalar (i.e. a real number). Hence the name.
Example 258. (5, 3) (2, 1) = 5 2 + (3) 1 = 7.
Example 259. (0, 17) (1, 3) = 0 (1) + 17 3 = 51.
Ordinary multiplication is distributive:
Example 260. 3 (5 + 11) = 3 5 + 3 11 and 18 (7 31) = 18 7 18 31.
It turns out that the scalar product is likewise distributive:
Fact 24. Let a, b, and c be vectors. Then a (b + c) = a b + a c and (a + b) c = a c + b c.

Proof. Optional, see p. 931 in the Appendices.


Here is one use of the scalar product: the length of a vector is simply the square root of its
scalar product with itself. Formally:
Fact 25. Given a vector v, v =

v v.

Proof. By Definition 71 (length of vector), v = v12 + v22 . By Definition 76 (scalar product),

v v = v1 v1 + v2 v2 = v12 + v22 . Hence, v = v v.


Next up is a more important use of the scalar product:

Page 284, Table of Contents

www.EconsPhDTutor.com

28.1

The Angle between Two Vectors

Fact 26. Let [0, ] be the angle between two non-zero vectors u and v. Then
u v = u v cos .

Proof. Optional, see p. 931 (Appendices).


The above fact35 gives us a very convenient way to calculate the angle between two vectors,
because rearranging, we have:
= cos1 (

35

uv
).
u v

We have two possible interpretations of the scalar product that are entirely equivalent. We can use either of these
interpretations as our definition and then prove that the other interpretation is true.
(1) In this textbook, we first define the scalar product by u v = u1 v1 + u2 v2 , then prove that u v = u v cos . That is,
we start with the algebraic definition, then prove a geometric property.
(2) In contrast, others may prefer to first define the scalar product by uv = u v cos , then prove that uv = u1 v1 +u2 v2 .
That is, we start with the geometric definition, then prove an algebraic property.
Either way, we first define the scalar product one way or the other. We then prove that the alternative statement is
equivalent.
(It is possible that your JC teachers take the second approach, rather than the first, as is done in this textbook.
Or worse, your teachers simply leave you confused as to why the hell u v = u v cos and at the same time, magically
enough, u v = u1 v1 + u2 v2 . This was my experience as a JC student a number of years ago. If this is also your current
experience, hopefully this textbook has helped to clear things up!)

Page 285, Table of Contents

www.EconsPhDTutor.com

Example 261. The vector i = (1, 0) points east. The vector (1, 1) points northeast. We

know the angle between these two vectors is . Lets check and verify that the formula
4
works:
= cos1 (

i (1, 1)
(1, 0) (1, 1)
) = cos1 (
)
i (1, 1)
(1, 0) (1, 1)

11+01

= cos

( 12 + 02 ) ( 12 + 12 )
1

= cos1 (

1+0
1

) = cos1 ( ) = .
4
1 2
2

Example 262. The vector i = (1, 0) points east. The vector j = (0, 1) points north. We

know the angle between these two vectors is right (i.e. ). Lets check and verify that the
2
formula works:
= cos1 (

ij
(1, 0) (0, 1)
) = cos1 (
)
i j
(1, 0) (0, 1)

10+01

= cos

( 12 + 02 ) ( 02 + 12 )
1

= cos1 (

Page 286, Table of Contents

0+0

) = cos1 0 =
11
2

www.EconsPhDTutor.com

Example 263. The angle between the vectors (3, 2) and (1, 4) is
= cos1 (

(3, 2) (1, 4)
)
(3, 2) (1, 4)

(1)
+
2

(4)

= cos1

2
2
2
2
( 3 + 2 ) ( (1) + (4) )
11
3 8
) = cos1 (
) 2.404
= cos1 (
13 17
221

This is an example where the angle is obtuse, i.e. between /2 and .

y
(3, 2)

x
2.404 rad

(-1, -4)

Page 287, Table of Contents

www.EconsPhDTutor.com

Recall that the arccosine function is defined to have range [0, ]. That is, cos1 x [0, ].
Moreover,

x > 0 cos1 x [0, ), i.e. cos1 x is an acute (or zero) angle.


2

x = 0 cos1 x = , i.e. cos1 x is a right angle.


2

x < 0 cos1 x ( , ], i.e. cos1 x is an obtuse (or straight) angle.


2

These three observations, together with Fact 26, imply the following Fact, which by the
way was already illustrated by the previous three examples:
Fact 27. Let u and v be vectors. The angle between u and v is
(i) acute (or zero) if u v < 0;
(ii) right if u v = 0; and
(iii) obtuse (or straight) if u v > 0.
Well use the words perpendicular, orthogonal, and normal interchangeably:
Definition 77. Two vectors are orthogonal (or perpendicular or normal) if the angle be
tween them is right (i.e. equal to ).
2
I will sometimes write u v to mean u is orthogonal (or perpendicular or normal) to v.

Exercise 112. First write down the angle between each of the following pairs of vectors
without using the above formula. Then verify that the formula does indeed
give you
these correct angles: (a) (2, 0) and (0, 17); (b) (5, 0) and (3, 0); (c) i and (1, 3/3); (d) i

and (1, 3). (Answers on pp. 1079 and 1080.)

Exercise 113. Verify that i and j are orthogonal, by computing their scalar product.
(Answer on p. 1081.)

Page 288, Table of Contents

www.EconsPhDTutor.com

28.2

Projection of One Vector on Another

The scalar product also gives a convenient way of computing the length of the projection
of one vector on another.
Say we have a right triangle (left diagram) where the angle and the length a are known.
What is the length b? It is simply a cos .

Now suppose a (blue) and b (green) are vectors (right diagram). The projection of the
vector a on the vector b is denoted ab (red). Note that ab is itself a vector.
What is the length of the projection? Well, if a is the length of the vector a and is the
angle between the two vectors, then the length of the projection is ab simply a cos .
Nicely enough, we actually have a quick alternative method of computing this length. Let
be the unit vector for b. Then
b
= ab
cos = a 1 cos = a cos = ab .
ab
or more correctly a b,
since a b
may sometimes
So we have a nice interpretation for a b
be negative:

is simply the length of the projection of a on b!


a b
Page 289, Table of Contents

www.EconsPhDTutor.com

Example 264. The length of the projection of (3, 2) on (1, 1) is


1
1
(1, 1)] = (3, 2) (1, 1)
(1, 1)
2

5
5 2
1
= (3 1 + 2 1) = =
.
2
2
2

(3, 2) (1,
1) = (3, 2) [

You should
verify for yourself that the length of the projection of (3, 2) on (1000, 1000) is

5 2
. The length of the vector to be projected (3, 2) matters, but the length of
also
2
the vector onto which it is projected be it (1, 1) or (1000, 1000) doesnt matter.

Example 265. The length of the projection of (6, 1) on (2, 0) is


1
1
(2, 0)] = (6, 1) (2, 0)
(2, 0)
2
12
1
= 6.
= (6 2 + 1 0) =
2
2

(6, 1) (2,
0) = (6, 1) [

Again, you can verify for yourself that the length of the projection of (6, 1) on (50000, 0)
is also 6. Again, the length of the vector to be projected (6, 1) matters, but the
length of the vector onto which it is projected be it (2, 0) or (50000, 0) doesnt matter.

Exercise 114. What are the lengths of the projections of (a) (1, 0) on (33, 33) and (b)
(33, 33) on (1, 0)? (Answer on p. 1081.)

Page 290, Table of Contents

www.EconsPhDTutor.com

28.3

Direction Cosines

The angle between a vector v and the x-axis is simply the angle between v and i = (1, 0).
Similarly, the angle between v and the y-axis is simply the angle between v and j = (0, 1).
Example 266. Consider the angle a between the vector (3, 2) and the x-axis. We have:
= cos a =

31+20
3
(3, 2) (1, 0)
=
= .

(3, 2) (1, 0) ( 32 + 22 ) ( 12 + 02 )
13

We refer to = 3/ 13 as the x-direction cosine of the vector (3, 2). By computing

cos1 = cos1 (3/ 13) 0.588, we find that the angle a between the vector (3, 2) and the
x-axis is 0.588.

y
(3, 2)
0.983 rad
x
2.404 rad

(-1, -4)

Page 291, Table of Contents

www.EconsPhDTutor.com

Example 267. Consider the angle b between the vector (3, 2) and the y-axis. We have:
= cos b =

(3, 2) (0, 1)
2
2
30+21
= .
=
=

(3, 2) (0, 1) ( 32 + 22 ) ( 02 + 12 )
13 1
13

We refer to = 2/ 13 as the y-direction cosine of the vector (3, 2). By computing

cos1 = cos1 (2/ 13) 0.983, we find that the angle b between the vector (3, 2) and the
y-axis is 0.983.
Definition 78. Given a vector v, its x-direction cosine is simply the length of the
on the x-axis.
projection of v
on the y-axis.
Similarly, its y-direction cosine is simply the length of the projection of v
The next Fact is immediate from the above definition:
= (, ).
Fact 28. Let v be a vector and and be its x- and y-direction cosines. Then v
Example 268. The x- and y-direction cosines of the vector (3, 2) are
3
=
13

2
and = .
13

3
2
Hence, the unit vector in the direction (3, 2) is ( , ).
13 13

Exercise 115. For each of the following vectors, find their x- and y-direction cosines.
Hence write down their unit vectors. (a) (1, 3). (b) (4, 2). (c) (1, 2). (Answer on p.
1081.)

Page 292, Table of Contents

www.EconsPhDTutor.com

29

Vectors in 3D

In two dimensions, we had the cartesian (or two-dimensional) plane with x- and y-axes.
Informally, the x-axis goes to the right and the y-axis goes up. A point was any ordered
pair of real numbers. The origin o = (0, 0) was the intersection point of the two axes. And
relative to the origin, the generic point a = (a1 , a2 ) was the point a1 units to the right and
a2 units up.
In three dimensions, we now instead have the three-dimensional space (3D space).
The x- and y-axes are as before. There is an additional z-axis that, informally, comes
out of the paper, perpendicular to the plane of the paper, straight towards your face.
We call this the right hand coordinate system, because if you take your right hand,
stick out your thumb, forefinger, and middle finger so that they are perpendicular, your
thumb represents the x-axis, your forefinger the y-axis, and your middle finger the z-axis.
(Try it!)
(If instead the z-axis goes into the paper, then wed have a left hand coordinate system.
Can you explain why?)

a2

x
a1

a3
z

Page 293, Table of Contents

www.EconsPhDTutor.com

In the context of 3D space, a point is any ordered triple of real numbers. The origin
o = (0, 0, 0) is the point where the x-, y-, and z-axes intersect. And relative to the origin,
the generic point a = (a1 , a2 , a3 ) is the point a1 units to the right, a2 units up, and a3 units
out of the paper.
Everything we learnt about 2D vectors finds its analogy in three-dimensional (3D)
vectors. Most of the time, the analogy is obvious. Try these exercises.

Exercise 116. (Answer on p. 1082.) (a) Fill in the blanks. A 3D vector is an arrow
that has two characteristics: __________ and __________. Just like a
point, it can be described by an __________ of __________. The vector
a = (a1 , a2 , a3 ) carries us from the origin to _______________.
(b) What other ways are there to denote the vector a = (a1 , a2 , a3 )? (Hint. The unit vector
in the z-axis is now called k.)


(c) Let a = (a1 , a2 , a3 ) and b = (b1 , b2 , b3 ) be points. What are (i) a+b; (ii) a+ ob; (iii)
oa+
ob;

ba?
and (iv)
oa

Page 294, Table of Contents

www.EconsPhDTutor.com


The length (or magnitude) of a 2D vector v = (v1 , v2 ) was defined by v12 + v22 . What then
is the length (or magnitude) of a 3D vector? This is the one instance where the analogy
from the 2D case to the 3D case is perhaps less than obvious. So lets explore this issue.
Consider the blue point a in the figure below. What is its distance from the origin (0, 0, 0)?
In other words, what is the length of the green dotted line?

a2

x
a1

a3
z
First lets calculate the distance of the red point from the origin,
in other words the length

of the red dotted line. By the Pythagorean Theorem, it is a22 + a23 .

Now, notice the the green dotted line, the red dotted line (length a22 + a23 ), and the
blue dotted line (length a1 ) form a right-angled triangle, with the hypothenuse being the
green dotted line. Thus, the length of the green dotted line is (again by the Pythagorean
Theorem):

a21

Page 295, Table of Contents

2
2
2
+ ( a2 + a3 ) = a21 + a22 + a23 .

www.EconsPhDTutor.com

We are thus motivated to define the length (or magnitude) of a 3D vector as follows:
Definition 79.
The length (or magnitude) of a vector a = (a1 , a2 , a3 ) is denoted a and
defined by a = a21 + a22 + a23 .
This is very much analogous to the definition of the length (or magnitude) of a 2D vector.

Lets continue with our exercises for 3D vectors:

Exercise 117. (Answer on p. 1083.) (a) Compute the lengths of the vectors a = (1, 2, 3),
b = (4, 5, 6), and a b.
(b) Compute the lengths of the vectors 2a = (2, 4, 6), 3b = (12, 15, 18), and 4(a b).
(c) Compute the unit vectors in the directions a = (1, 2, 3), b = (4, 5, 6), and a b.
(d) Compute (1, 2, 3) (4, 5, 6) and (2, 4, 6) (1, 2, 3).
(e) Compute the angles (i) between the vectors a = (1, 2, 3) and b = (4, 5, 6); and (ii)
between the vectors u = (2, 4, 6) and v = (1, 2, 3). (iii) Are the vectors (2, 4, 6) and
(1, 2, 3) orthogonal?
(f) Compute the length of the projection of a = (1, 2, 3) on b = (4, 5, 6).
(g) Find the point that divides the line segment ab in the ratio 2 3.
(h) For each of the following vectors, find their x-, y-, and z-direction cosines. And then
write down their unit vectors. (i) (1, 3, 2). (ii) (4, 2, 3). (iii) (1, 2, 4).

Page 296, Table of Contents

www.EconsPhDTutor.com

30
30.1

Vector Product
Vector Product in 2D

Recall that given two 2D vectors u = (ux , uy ) and v = (vx , vy ), their scalar product was the
scalar defined by u v = ux vx + vx vy . We now define a very similar concept.
Definition 80. Given two 2D vectors u = (ux , uy ) and v = (vx , vy ), their vector product (or
cross product), denoted u v, is the scalar defined by
u v = ux vy uy vx .

Example 269. If u = (1, 2) and v = (3, 4), then u v = 1 4 2 3 = 2.


Example 270. If p = (3, 5) and q = (6, 1), then p q = 3 1 5 6 = 33.
Example 271. If u = (1, 4) and v = (2, 3), then u v = (1) (3) 4 2 = 5.
Ordinary multiplication is commutative. This simply means that given any real numbers
a, b, we have a b = b a. For example,
Example 272. 4 7 = 7 4 and 3 5 = 5 3.
In contrast, the vector product is not commutative because u v v u. This might be
the first time in your life that youre encountering a product that isnt commutative.
In fact, the vector product is anticommutative because u v = v u! For example,
Example 273. If u = (1, 2) and v = (3, 4), then u v = 1 4 2 3 = 2, but v u =
2 3 1 4 = 2.
Example 274. If u = (1, 4) and v = (2, 3), then u v = (1) (3) 4 2 = 5, but
v u = 4 2 (1) (3) = 5.

Page 297, Table of Contents

www.EconsPhDTutor.com

Recall that if [0, ] is the angle between two vectors, then based on our definition that
u v = ux vx + uy vy , we could prove that u v = u v cos .
It turns out based on our definition that u v = ux vy uy vx , we can prove a very similar
result:36
Fact 29. Let u and v be two non-zero 2D vectors and [0, ] be the angle between them.
Then the scalar u v is equal to either u v sin or u v sin .

Proof. Optional, see p. 933 in Appendices.

Earlier we already had one formula for calculating the angle between two vectors. Let
[0, ] be the angle between u and v. Then
= cos1 (

uv
).
u v

The above Fact now gives us a second formula. Let [0, ] be the acute or right angle
between u and v. Then
= sin1

uv
.
u v

However, well stick with using only the first cosine formula. We wont use the second
sine formula, mainly because, as well see, computing the vector product is very tedious,
especially in the 3D case, where it is a different creature altogether.

36

Footnote 36 explained that the scalar product could be defined in one of two equivalent ways.
Similarly, the vector product can be defined in one of two equivalent ways. We can use either definition and then prove
that the other is true.
(1) In this textbook, we first define the vector product by u v = ux vy uy vx ; we then prove that u v = u v sin ,
where is the angle between the two vectors. That is, we start with the algebraic definition, then prove a geometric
property.
The alternative approach is this:

(2) Define the vector product by u v = u v sin if [0, ] or u v = u v sin if ( , ] ; then prove that
2
2
u v = ux vy uy vx . That is, we start with the geometric definition, then prove an algebraic property.

Page 298, Table of Contents

www.EconsPhDTutor.com

30.2

Areas of Triangles and Parallelograms

SYLLABUS ALERT
Calculation of the area of a triangle or parallelogram is included in the 9740 (old) syllabus,
but not in the 9758 (revised) syllabus. So you can skip this section if youre taking 9758.

The vector product is also helpful for computing the area of triangles and parallelograms.
Fact 30. The triangle with sides of lengths u, v, and v u has area 0.5u v.

Case #1.

|v| sin ( )
v
|v| sin

vu

vu

Case #2.

Proof. Case #1. If the vectors u and v form an acute or right angle , then the area of the
triangle is simply 0.5 Base Height or 0.5 u v sin . And by Fact 29, 0.5 u v sin =
0.5u v.
Case #2. And if the vectors u and v form an obtuse angle , then the area of the triangle is
again simply 0.5 Base Height or 0.5 u v sin( ). Recall that sin( ) = sin cos
sin cos = sin . So again the area of the triangle is 0.5 u v sin or 0.5u v.

Example 275. Consider the triangle formed by the points (0, 0), (3, 4), and (5, 6). Its
area is simply 0.5 (3, 4) (5, 6) = 0.53 6 4 5 = 1.

Page 299, Table of Contents

www.EconsPhDTutor.com

Fact 31. The parallelogram with sides of lengths u and v, and diagonal of length v u
has area u v.

vu
u

Proof. Such a parallelogram is simply composed of two of the triangles from Fact 30. And
so its area is simply twice the area of the triangle, or 2 0.5u v = u v.

Page 300, Table of Contents

www.EconsPhDTutor.com

30.3

Vector Product in 3D

The 3D vector product is very different from the 2D vector product. The latter was simply
a scalar (real number); in contrast, the 3D vector product is instead a VECTOR!
Also previously, we first started with the algebraic definitions. For example, the 3D scalar
product was defined as uv = u1 v1 +u2 v2 +u3 v3 and the 2D vector product as uv = u1 v2 u2 v1 .
We then showed that these algebraic definitions were equivalent to some geometric
interpretations.
For the vector product in 3D, I will go the other way round. That is, I will start with
the (very long) geometric definition, then show that it is equivalent to some algebraic
interpretation.

Page 301, Table of Contents

www.EconsPhDTutor.com

Definition 81. Given two distinct 3D vectors u = (ux , uy , uz ) and v = (vx , vy , vz ), their
vector product (or cross product), denoted u v, is the (unique) vector that satisfies 3
properties:
1. u v is orthogonal (perpendicular) to both u and v.
Lets see what this first property means. Recall that it doesnt matter where we put the
heads and tails of vectors. So lets put u and v on the same plane, with their heads at the
same point.

uv

Plane

We see that there are exactly two vectors that are orthogonal to both u and v the vector
pointing up (green) and the vector pointing down (purple).
There is thus an ambiguity. Which of these two vectors is u v?
To resolve this ambiguity, we also require that u v satisfy a second property:
2. u v satisfies the right-hand rule: Take your right hand, stick out your thumb,
forefinger, and middle finger so that they are perpendicular, your thumb represents the
vector u v, your forefinger the vector u, and your middle finger the vector v. Hence,
in the figure, u v points up (green). (Try it yourself!)
Note that the right-hand rule is a mere convention, but one that everyone has agreed upon.
There is no especially compelling reason for using it, other than the fact that left-handed
people are an oppressed minority! (If instead we used the left-hand rule, then u v would
point down (purple) (try it!), but for better or worse, we dont use the left-hand rule.)
The third and last property specifies the length (or magnitude) of u v.
3. u v = u v sin , where [0, ] is the angle between them.

Page 302, Table of Contents

www.EconsPhDTutor.com

Note that [0, ] sin 0, so that u v sin is never negative. (Otherwise, wed
have the distressing possibility that the length of u v is sometimes negative!)
One implication of this third and last property is that if both u and v point in the same
direction (so that = 0), then u v is the zero vector (i.e. u v = 0).
Fact 32. (a) i j = k; (b) j k = i; (c) k i = j; (d) j i = k; (e) k j = i; and (f)
i k = j.
Proof. In each case, use the right-hand rule to show that properties #1 and #2 of Definition
81 are satisfied.

In each case, the length of the cross product is u v sin = 1 1 sin = 1. So indeed,
2
property #3 is also satisfied.

Ordinary multiplication is distributive:


Example 276. 3 (5 + 11) = 3 5 + 3 11 and 18 (7 31) = 18 7 18 31.
It turns out that the vector product is likewise distributive:
Fact 33. Let a, b, and c be vectors. Then a (b + c) = a b + a c. Moreover, (a + b) c =
a c + b c.
Proof. Optional, see p. 934 in the Appendices.

The next proposition gives the promised algebraic interpretation of the vector product.
Proposition 7. Given two 3D vectors u = (ux , uy , uz ) and v = (vx , vy , vz ), their vector
product is given by:
uy vz uz vy
uv=
uz vx ux vz
ux vy uy vx

Proof. Optional, see p. 935 in Appendices. (The proof is actually quite simple. It just
involves some tedious algebra.)

Page 303, Table of Contents

www.EconsPhDTutor.com

Example 277. If u = (1, 2, 3) and v = (4, 5, 6), then


u v = (2 6 3 5, 3 4 1 6, 1 5 2 4) = (3, 6, 3) .
Lets verify that u v is orthogonal to u, by computing (u v) u = (3, 6, 3) (1, 2, 3) =
3 + 12 9 = 0 . Similarly, lets verify that u v is orthogonal to v, by computing
(u v) v = (3, 6, 3) (4, 5, 6) = 12 + 30 18 = 0 .
Example 278. If u = (1, 3, 5) and v = (2, 4, 6), then
u v = (3 6 (5) (4), (5) 2 (1) 6, (1) (4) 3 2) = (2, 4, 2) .
Lets verify that uv is orthogonal to u, by computing (u v)u = (2, 4, 2)(1, 3, 5) =
2 12 + 10 = 0 . Similarly, lets verify that u v is orthogonal to v, by computing
(u v) v = (2, 4, 2) (2, 4, 6) = 4 + 16 12 = 0 .
As in the 2D case of the vector product, here again in the 3D case, the vector product is
anticommutative, i.e. u v = v u (see Exercise 120).
Exercise 118. For each of the following pairs of vectors, compute the vector product and
verify that it is orthogonal to each of the two vectors. (a) u = (0, 1, 2) and v = (3, 4, 5). (b)
u = (1, 2, 3) and v = (1, 0, 5). (Answer on p. 1085.)
Exercise 119. Verify that in general, u v is orthogonal to u and v by showing that
(u v) u = 0 and that (u v) v = 0. (Answer on p. 1085.)
Exercise 120. (a) Given u = (1, 2, 3) and v = (4, 5, 6), show that v u = u v. (b) Prove
that in general, i.e. the 3D vector product is anti-commutative, i.e. u v = v u. (Answer
on p. 1085.)

Ordinary multiplication is associative. This simply means that (a b) c = a (b c).


Example 279. (2 3) 7 = 2 (3 7) and (8 13) 2 = 8 (13 2).
In contrast, the vector product is not associative.
Example 280. If u = (1, 2, 3), v = (4, 5, 6), and w = (1, 0, 1), then (u v)w = (3, 6, 3)
(1, 0, 1) = (6, 0, 6), but u (v w) = (1, 2, 3) (5, 2, 5) = (16, 20, 8).

Page 304, Table of Contents

www.EconsPhDTutor.com

31
31.1

Lines

Lines on a 2D Plane: Cartesian to Vector Equations

In general, a line on a 2D plane can be described by the cartesian equation


ax + by + c = 0.
This says that the line consists of exactly those points (x, y) that satisfy the equation
ax + by + c = 0.
(You may be more familiar with describing lines in the form y = mx+d. This simply involves
a rearrangement of the above equation. But the above equation is preferred because it is
more general it allows for the possibility that the coefficient on y is 0.)
Example 281. Consider the line described by the cartesian equation 3x y + 2 = 0.
Rearranging, we get a more familiar-looking equation: y = 3x + 2.
For convenience (but at the cost of some sloppiness), we may even simply identify the line
with the cartesian equation.
Example 282. Consider the line 3x y + 2 = 0.

Describing lines using cartesian equations is secondary school stuff. Well now learn a
second method of describing lines through vector equations. In general, any line can be
described in the form
r = p + v, R,
where r is a generic point on the line, p is some known point on the line, v is a direction
vector of the line, and is a parameter that can take any real value.
Here are some examples to make sense of this.

Page 305, Table of Contents

www.EconsPhDTutor.com

Example 283. Consider the line (on a 2D plane) described by the cartesian equation
3x y + 2 = 0. It runs through the point (0, 2). A vector that points in the same direction
as this line is (1, 3). Hence, we can also describe it using the vector equation
r = (0, 2) + (1, 3), R.
This says that the line consists of every point r that can be written as (0, 2) + (1, 3) for
some real number . We call a parameter. As varies, we get different points of
the line. So for example, corresponding to = 0, 1, and 1, the line contains the points
(0, 2) + 0(1, 3) = (0, 2), (0, 2) + 1(1, 3) = (1, 5), and (0, 2)1(1, 3) = (1, 1). Of course, it
also contains infinitely many other points, one for each value of R.
We call (1, 3) a direction vector of the line. Note that this direction vector is not unique,
any scalar multiple thereof, i.e. c(1, 3) with c R, is also a direction vector of the line!
Again, we can either say the line is described by the vector equation r = (0, 2) +
(1, 3). OR, for convenience (but at the cost of some sloppiness), we can also say the line
is the very equation r = (0, 2) + (1, 3).

y 4

Line

The point (0, 2)

Cartesian equation
3x - y + 2 = 0

The vector (1, 3)


x
0

-4

-2

Vector equation
r = (0, 2) + (1, 3)

-2

-4
Page 306, Table of Contents

www.EconsPhDTutor.com

Example 284. Consider the line (on a 2D plane) described by the cartesian equation
x + y 1 = 0. It runs through the point (0, 1). A vector that points in the same direction
as this line is (1, 1). Hence, we can also describe it using the vector equation
r = (0, 1) + (1, 1), R.
This says that the line consists of every point r that can be written as (0, 1) + (1, 1)
for some real number . Corresponding to = 0, 1, and 1, the line contains the points
(0, 1) + 0(1, 1) = (0, 1), (0, 1) + 1(1, 1) = (1, 0), and (0, 1)1(1, 1) = (1, 2).

Line

Cartesian equation
x+y-1=0

y 4

2
The point (0, 1)
x
0

-4

-2

-2

Vector equation
r = (0, 1) + (1, -1)

The vector (1, -1)

-4

Page 307, Table of Contents

www.EconsPhDTutor.com

Example 285. Consider the line (on a 2D plane) described by the cartesian equation
y 3 = 0. It runs through the point (0, 3). A vector that points in the same direction as
this line is (1, 0). Hence, we can also describe it using the vector equation
r = (0, 3) + (1, 0), R.
This says that the line consists of every point r that can be written as (0, 3) + (1, 0)
for some real number .Corresponding to = 0, 1, and 1, the line contains the points
(0, 3) + 0(1, 0) = (0, 3), (0, 3) + 1(1, 0) = (1, 3), and (0, 3)1(1, 0) = (1, 3).

y 4

Vector equation
r = (0, 3) + (1, 0)

Line
Cartesian equation
y-3=0

The point (0, 3)


2

x
0
-4

-2

The vector (1, 0)


-2

-4

Exercise 121. Rewrite each of the following lines into vector equation form. (a) 5x+y+1 =
0. (b) x 2y 1 = 0. (c) y 4 = 0. (d) x 4 = 0. (Answer on p. 1086.)

Page 308, Table of Contents

www.EconsPhDTutor.com

We just learnt how to describe a line using the vector equation


1

r = p + v, R.
Notice that the LHS of this equation is the generic Point r. And the RHS of the equation
is the Point p minus the Vector v, which equals Point (see p. 270). So LHS and RHS
do indeed match up.
There is another way to describe a line using a vector equation. We can instead write:
2

r = p + v, R,
where now r is the position vector of a generic point r on the line and p is the position
vector of some known point p on the line. So now LHS is a Vector and so too is RHS.
1

Equation = said that the line consists of those points r that could be written as p + v. In
2
contrast, equation = says that the line consists of those points whose position vector r can
be written as p + v. But both equations can equally well describe the very same line. The
difference is a fine and pedantic one and really doesnt matter much.

What matters is that you take care not to write either


3

r = p + v, R; WRONG!
4

or r = p + v, R. WRONG!
3

The LHS of = is a Point while the RHS of = is a Vector. Therefore = cannot possibly be
true.
4

The LHS of = is a Vector while the RHS of = is a Point. Therefore = cannot possibly be
true.
As usual, this is all very pedantic, but can serve as a useful test of your understanding.

Page 309, Table of Contents

www.EconsPhDTutor.com

31.2

Lines on a 2D Plane: Vector to Cartesian Equations

In the previous section, given the cartesian equation of a line, we worked out its vector
equation. Now given its vector equation, well work out its cartesian equation.
Suppose a line (on a 2D plane) can be described by the vector equation
r = p + v = (p1 , p2 ) + (v1 , v2 ).
where R and v is a non-zero vector.37 And so any point (x, y) on this line must satisfy
x = p1 + v1

and y = p2 + v2 .

The above are the cartesian equations for a line (on a 2D plane)! But wait a minute ...
isnt there supposed to be just one equation? Well, if wed like, we can quite easily combine
them into a single equation by eliminating the parameter . In general:

Fact 34. The line with vector equation r = (p1 , p2 ) + (v1 , v2 ) (for R) is the line with
cartesian equations as given by the 3 cases below.
(1)

x p1 y p2
=
,
v1
v2

if v1 , v2 0;

(2) x = p1 , y is free,

if v1 = 0, v2 0;

(3) x is free, y = p2 ,

if v1 0, v2 = 0;

Note that Case (1) is the most common situation.


Proof. Optional, see p. 936 in the Appendices.

Some examples:

37

Otherwise wed simply be describing the single point p!

Page 310, Table of Contents

www.EconsPhDTutor.com

Example 286. The line described by the vector equation r = (1, 2) + (1, 1), where R
has cartesian equations x = 1 + and y = 2 + .
As varies between and , this pair of equations gives us the points that are on the
line. For example, when = 1, 17, 33, we have the points (2, 3), (18, 20), and (34, 36).
We can eliminate and reduce the above pair of equations into the single cartesian equation
y = x + 1 or
y1 x
= .
1
1
Example 287. Consider the line described by the vector equation r = (0, 0)+(4, 5), where
R has cartesian equations x = 4 and y = 5.
As varies between and , this pair of equations gives us the points that are on the
line. For example, when = 1, 17, 33, we have the points (4, 5), (68, 85), and (132, 165).
Eliminating , we can reduce the above pair of equations to y = 1.25x or
x
y
= .
1.25 1
Example 288. Consider the line described by the vector equation r = (3, 1)+(0, 2), where
R has cartesian equations x = 3 and y = 1 + 2.
As varies between and , this pair of equations gives us the points that are on the
line. So in fact, the above equations say that x must always be 3 and y is free to vary along
with . For example, when = 1, 17, 33, we have the points (3, 3), (3, 25), and (3, 67).
Hence, the above pair of equations can be reduced to x = 3.
Exercise 122. Rewrite each of the following lines into cartesian equation form. (a) r =
(1, 1) + (3, 2), where R. (b) r = (5, 6) + (7, 8), where R. (c) r = (0, 3) + (3, 0),
where R. (Answer on p. 1086.)

Page 311, Table of Contents

www.EconsPhDTutor.com

31.3

Lines in 3D Space: Vector Equations

Lines in 2D space are described by the cartesian equation ax + by + c = 0. A reasonable


guess might be that lines in 3D space are analogously described by the cartesian equation
ax + by + cz + d = 0. Turns out this is wrong! The equation ax + by + cz + d = 0 actually
describes a plane, as well see later (Chapter 32).
In the 3D case, its easier to start by looking at the vector equation of a line. It turns out
to be exactly analogous to the 2D case. It can be written as r = a + v, where R and v
is a non-zero 3D vector.
This vector equation says that the line contains every point r can be expressed as (a1 , a2 , a3 )+
(v1 , v2 , v3 ), where R is a parameter.
Example 289. Consider the line described by the vector equation r = (1, 2, 3) + (0, 1, 1),
where R. Corresponding to = 0, 1, and 1, the line contains the points (1, 2, 3),
(1, 3, 4), and (1, 1, 2).

y
3
(1, 2, 3)

2
1
x
1

(1, 3, 4)

1
2
3 (1, 1, 2)

Line
r = (1, 2, 3) + (0, 1, 1)

Page 312, Table of Contents

www.EconsPhDTutor.com

Example 290. Consider the line described by the vector equation r = (0, 0, 0) + (1, 0, 0),
where R. Corresponding to = 0, 1, and 1, the line contains the points (0, 0, 0),
(1, 0, 0), and (1, 0, 0).

y
2
(-1, 0, 0)
1

Line
r = (0, 0, 0) + (1, 0, 0)
x
1
(1, 0, 0)

1
2 (0, 0, 0)

Page 313, Table of Contents

www.EconsPhDTutor.com

31.4

Lines in 3D Space: Vector to and from Cartesian Equations

We now try to work out the cartesian equation of a line in 3D space. Suppose a line can
be described by the vector equation
r = p + v = (p1 , p2 , p3 ) + (v1 , v2 , v3 ).
where R and v is a non-zero vector.38 And so any point (x, y, z) on this line must satisfy
x = p1 + v1 , y = p2 + v2 , and z = p3 + v3 .
The above are the cartesian equations for a line (in 3D space)! These are exactly analogous
to the cartesian equations (p. 31.2) in the 2D case.
Unlike in the 2D case, it is generally impossible to reduce these equations into a single
cartesian equation. However, we can reduce them into two equations.
Fact 35. The line with vector equation r = (p1 , p2 , p3 ) + (v1 , v2 , v3 ) where R is the line
with cartesian equations as given by the 7 cases below.

(1)

x p 1 y p2 z p3
=
=
v1
v2
v3

if v1 , v2 , v3 0;

(2) x = p1 ,

y p2 z p3
=
,
v2
v3

if v1 = 0, v2 , v3 0;

(3) y = p2 ,

x p1 z p 3
=
,
v1
v3

if v2 = 0, v1 , v3 0;

(4) z = p3 ,

x p1 y p2
=
,
v1
v2

if v3 = 0, v1 , v2 0;

(5) x = p1 , y = p2 , z is free,

if v1 , v2 = 0, v3 0;

(6) x = p1 , z = p3 , y is free,

if v1 , v3 = 0, v2 0;

(most common case)

(7) y = p2 , z = p3 , x is free, if v2 , v3 = 0, v1 0.
Proof. Optional, see p. 937 in the Appendices.

38

If v is a zero vector, then we are simply describing the single point p!

Page 314, Table of Contents

www.EconsPhDTutor.com

The first two examples are where v1 , v2 , and v3 are non-zero (Case 1 of Fact 35).
Example 291. Consider the line described by the vector equation r = (1, 2, 3) + (4, 5, 6),
where R. It can be described by the cartesian equations x = 1 + 4, y = 2 + 5, and
z = 3 + 6.
As varies between and , these 3 equations give us the points that are on the line.
For example, when = 1, 3, 17, we have the points (5, 7, 9), (13, 17, 21), and (69, 87, 105).
By rearranging each equation so that is on one side, we can reduce these three equations
to just two:
x1 y2 z3
=
=
.
4
5
6
That is, this is the line that contains the points (x, y, z) which satisfy the above cartesian
equations.

Example 292. Consider the line described by the vector equation r = (0, 0, 0) + (2, 3, 5),
where R. It can be described by the cartesian equations x = 2, y = 3, and z = 5.
As varies between and , these 3 equations give us the points that are on the line.
For example, when = 1, 3, 17, we have the points (2, 3, 5), (6, 9, 15), and (34, 51, 85).
By rearranging each equation so that is on one side, we can reduce these three equations
to just two:
x y z
= = .
2 3 5
That is, this is the line that contains the points (x, y, z) which satisfy the above cartesian
equations.

Page 315, Table of Contents

www.EconsPhDTutor.com

We now look at examples where exactly one of v1 , v2 , or v3 is non-zero (Cases 2, 3, and 4


of Fact 35).

In the case where v1 = 0 (but v2 0 and v3 0), then this is a line that is on the 2D yz
plane where x = p1 .
Example 293. Consider the line described by the vector equation r = (1, 2, 3) + (0, 5, 6),
where R. It can be described by the cartesian equations x = 1, y = 2 + 5, and z = 3 + 6.
As varies between and , these 3 equations give us the points that are on the line.
For example, when = 1, 3, 17, we have the points (1, 7, 9), (1, 17, 21), and (1, 87, 102).
We see that x must always be equal to 1.
By rearranging the second and third equations so that is on one side, we can reduce these
three equations to just two:
y2 z3
=
.
x = 1,
5
6
That is, this is the line that contains the points (x, y, z) which satisfy the above cartesian
equations.

Similarly, in the case where v2 = 0 (but v1 0 and v3 0), then this is a line that is on the
2D xz plane where y = p2 .
Example 294. Consider the line described by the vector equation r = (1, 2, 3) + (4, 0, 6),
where R. It can be described by the cartesian equations x = 1 + 4, y = 2, and z = 3 + 6.
As varies between and , these 3 equations give us the points that are on the line.
For example, when = 1, 3, 17, we have the points (5, 2, 9), (13, 2, 21), and (69, 2, 105).
We see that y must always be equal to 2.
By rearranging the first and third equations so that is on one side, we can reduce these
three equations to just two:
x1 z3
=
.
y = 2,
4
6
That is, this is the line that contains the points (x, y, z) which satisfy the above cartesian
equations.

Page 316, Table of Contents

www.EconsPhDTutor.com

Finally, in the case where v3 = 0 (but v2 0 and v3 0), then this is a line that is on the
2D xy plane where z = p3 .
Example 295. Consider the line described by the vector equation r = (1, 2, 3) + (4, 5, 0),
where R. It can be described by the cartesian equations x = 1 + 4, y = 2 + 5, and z = 3.
As varies between and , these 3 equations give us the points that are on the line.
For example, when = 1, 3, 17, we have the points (5, 7, 3), (13, 17, 3), and (69, 87, 3).
We see that z must always be equal to 3.
By rearranging the first and second equations so that is on one side, we can reduce these
three equations to just two:
x1 y2
=
.
z = 3,
4
5
That is, this is the line that contains the points (x, y, z) which satisfy the above cartesian
equations.

We now look at examples where exactly two of v1 , v2 , or v3 are zero (Cases 5, 6, and 7 of
Fact 35).
In the case where v1 = 0 and v2 = 0, but v3 0, then this is a line that runs through the
points (p1 , p2 , ) for R.
Example 296. Consider the line described by the vector equation r = (1, 2, 3) + (0, 0, 6),
where R. It can be described by the cartesian equations x = 1, y = 2, and z = 3 + 6.
As varies between and , these 3 equations give us the points that are on the line.
For example, when = 1, 3, 17, we have the points (1, 2, 9), (1, 2, 21), and (1, 2, 105).
We see that x and y must always be equal to 1 and 2. Hence, the above equations simply
reduce to:
x = 1,

y = 2.

That is, this is the line that contains the points (x, y, z) which satisfy the above cartesian
equations. These are the points (1, 2, ), where can be any real.

Page 317, Table of Contents

www.EconsPhDTutor.com

Similarly, in the case where v1 = 0 and v3 = 0, but v2 0, then this is a line that runs
through the points (p1 , , p3 ) for R.
Example 297. Consider the line described by the vector equation r = (1, 2, 3) + (0, 5, 0),
where R. It can be described by the cartesian equations x = 1, y = 2 + 5, and z = 3.
As varies between and , these 3 equations give us the points that are on the line.
For example, when = 1, 3, 17, we have the points (1, 7, 3), (1, 17, 3), and (1, 87, 3).
We see that x and z must always be equal to 1 and 3. Hence, the above equations simply
reduce to:
x = 1,

z = 3.

That is, this is the line that contains the points (x, y, z) which satisfy the above cartesian
equations. These are the points (1, , 3), where can be any real.
In the case where v2 = 0 and v3 = 0, but v1 0, then this is a line that runs through the
points (, p2 , p3 ) for R.
Example 298. Consider the line described by the vector equation r = (1, 2, 3) + (4, 0, 0),
where R. It can be described by the cartesian equations x = 1 + 4, y = 2, and z = 3.
As varies between and , these 3 equations give us the points that are on the line.
For example, when = 1, 3, 17, we have the points (5, 2, 3), (13, 2, 3), and (69, 2, 3).
We see that y and z must always be equal to 2 and 3. Hence, the above equations simply
reduce to:
y = 2,

z = 3.

That is, this is the line that contains the points (x, y, z) which satisfy the above cartesian
equations. These are the points (, 2, 3), where can be any real.

Exercise 123. Rewrite each of the following vector equation descriptions of lines into
cartesian equations describing the same line. (a) r = (1, 1, 1) + (3, 2, 1), where R.
(b) r = (5, 6, 1) + (7, 8, 1), where R. (c) r = (0, 3, 1) + (3, 0, 1), where R. (d)
r = (9, 9, 9) + (1, 0, 0), where R. (Answer on p. 1087.)

Page 318, Table of Contents

www.EconsPhDTutor.com

SYLLABUS ALERT
The 9740 (old) List of Formulae contains the following statement, but not the 9758 (revised)
List of Formulae!
If A is the point with position vector a = a1 i + a2 j + a3 k and the direction vector b is given
by b = b1 i + b2 j + b3 k, then the straight line through A with direction vector b has cartesian
equation
x a1 y a2 z a3
=
=
(= ).
b1
b2
b3

By the way, the above statement printed in the 9740 (old) List of Formulae is false (*gasp*),
because it fails to specify that b1 , b2 , b3 must be non-zero. (The correct statement was just
given as Fact 35.)
Consider for example, the point (0, 0, 0) and the direction vector b given by b = j + k. Then
contrary to the above statement, the straight line through A with direction vector b does
not have cartesian equation
x y z
= = ,
0 1 1
x
is undefined. This is the common mistake to which I devoted an entire chapter
0
(Chapter 2) earlier in this book. This seems like a very pedantic point to make, but dividing
by zero has been the cause of the downfall of many a student (and in this case some folks
at MOE).
because

Page 319, Table of Contents

www.EconsPhDTutor.com

In the examples and exercise we just went through, we started with a vector equation of a
line and then wrote down the lines cartesian equations. Now well go the other way round,
starting with the lines cartesian equations, then write down the vector equations.
Example 299. Consider a line described by the cartesian equations
3x 4 2y 18 z 1
=
=
.
6
5
3
In order to directly apply Fact 35, you must make sure that the coefficients on x, y,
and z are all 1! So first rewrite the above into
x 4/3 y 9 z 1
=
=
.
2
2.5
3
And now by Fact 35, we can immediately describe this line by the vector equation r =
(4/3, 9, 1) + (2, 2.5, 3), for R.
Example 300. Consider a line described by the cartesian equations
5x y 13 3z 14
=
=
.
2
6
8
Rewrite the above equations into
x y 13 z 14/3
=
= 8
.
2/5
6
/3
And so by Fact 35, we can immediately describe this line by the vector equation r =
(0, 13, 14/3) + (2/5, 6, 8/3), for R.
Example 301. Consider a line described by the cartesian equations 2x = 17 and 3z = 4.
Rewrite them into x = 8.5 and z = 4/3. And so by Fact 35, we can immediately describe
this line by the vector equation r = (8.5, 0, 4/3) + (0, 1, 0), for R.

Exercise 124. Rewrite each of the following cartesian equation descriptions of lines into
7x 2 0.3y 5 8z
a vector equation describing the same line. (a)
=
= . (b) 2x = 3y = 5z. (c)
5
7
7
3y 1
x 3 5z 2
17x 4 =
= 3z. (d)
=
, 3y = 11. (Answer on p. 1088.)
2
2
7

Page 320, Table of Contents

www.EconsPhDTutor.com

31.5

Collinearity

Definition 82. A set of points are said to be collinear if there is some line that contains
all of these points.
Any two points are always collinear simply take the line that passes through both of
them.
In contrast, three points may not be collinear. To check whether three points are collinear,
1. First take the line that passes through two of the points.
2. Then check whether the third point is on this line.

a
a

a, b, and c are
not collinear.

a, b, and c
are collinear.

Page 321, Table of Contents

www.EconsPhDTutor.com

Example 302. Are the points a = (1, 2, 3), b = (4, 5, 6), and c = (7, 8, 9) collinear?
First take the line through a and b. The vector from a to b is (3, 3, 3) and the line passes
through a. Hence, the line can be written as r = (1, 2, 3) + (3, 3, 3) ( R).
Then check whether c is on the line: Is there such that c = (7, 8, 9) = (1, 2, 3) + (3, 3, 3)?
Rearranging, we have (6, 6, 6) = (3, 3, 3), which we can write out as:
6 = 3,

6 = 3.

6= 3,

Clearly, all three of the above equations are true if = 2. And so c is also on the line.
Hence, the three points are collinear.

a, b, and c are collinear.

d, e, and f are not collinear.


f = (0, 0, 1)

c = (7, 8, 9)

= (-1, 1, 0)

= (3, 3, 3)

e = (0, 1, 0)

b = (4, 5, 6)
a = (1, 2, 3)

d = (1, 0, 0)

Example 303. Are the points d = (1, 0, 0), e = (0, 1, 0), and f = (0, 0, 1) collinear?
First take the line through d and e . The vector from d to e is (1, 1, 0) and the line passes
through d. Hence, the line can be written as r = (1, 0, 0) + (1, 1, 0) ( R).
Then check whether f is on the line: Is there such that f = (0, 0, 1) = (1, 0, 0)+(1, 1, 0)?
Rearranging, we have (1, 0, 1) = (1, 1, 0), which we can write out as:
1 = ,

0= ,

1 = 0.

Clearly, there is no such that the above three equations can be true. And so the point f
is not on the line through d and e. Hence, the three points are not collinear.

Exercise 125. Determine whether each of the following set of three points are collinear. (a)
a = (3, 1, 2), b = (1, 6, 5), and c = (0, 1, 0). (b) a = (1, 2, 4), b = (0, 0, 1), and c = (3, 6, 10).
(Answer on p. 1088.)

Page 322, Table of Contents

www.EconsPhDTutor.com

32

Planes

Informally, a plane is a flat 2D surface in 3D space. How do we describe it using equations?


Here are two very useful clues:
1. u v u v = 0.
In words: Two vectors are orthogonal if and only if their scalar product is 0.
2. Since the plane is a flat surface, there must be some vector n that is orthogonal (perpendicular) to this plane.
That is, n is orthogonal to every vector on the plane. We call n the planes normal
vector (hence the use of the letter n).
Is the normal vector unique? No, because any other vector cn (where c is any scalar) serves
equally well as a normal vector. In the figure below, n is a normal vector to the illustrated
plane. So too is 0.5n. And so too is n.
But otherwise, besides cn, there are no other vectors that are also orthogonal to
the plane. That is, any vector that cannot be written in the form cn is not orthogonal to
the plane.

Black vectors
Plane are on the plane
n (a normal
0.5n (Also a
vector)
normal vector)
-n (Also a
normal vector)

Page 323, Table of Contents

www.EconsPhDTutor.com

Suppose a plane contains some point p = (p1 , p2 , p3 ) and has a normal vector n = (a, b, c).
Now consider any point r on the plane. We can construct the vector r p.
This vector r p lies on the plane and must therefore be orthogonal to n, the planes normal
vector. So, for any point r that lies on the plane, we have
(r p) n = 0.

q (point not
on the plane)

n is normal to r p ,
but not to q p.

q p (vector not
on the plane)
n (a normal
vector)

p (point on
the plane)
r p (vector
on the plane)

Plane
r1 (point on
the plane)

Now consider any point q that is not on the plane. We can construct the vector q p.
This vector q p does not lie on the plane and must therefore not be orthogonal to n, the
planes normal vector. So, for any point q that does not lie on the plane, we have
(q p) n 0.
Altogether then, we conclude: A point r is on the plane if and only if
Fact 36. Suppose a plane contains point p and has normal vector n. Then the plane
contains exactly those points r such that
(r p) n = 0.

Page 324, Table of Contents

www.EconsPhDTutor.com

Recall that a line can be described by the vector equation r = p + v where r is a generic
point on the line and p is a known point on the line. Alternatively, it can also be described
by the vector equation r = p + v, where r is the position vector of r and p is the position
vector of p.
On the previous page, we proved that if a plane that contains point p and has normal vector
n, then it may be described by the vector equation (r p) n = 0, where r is a generic
point on the plane. Similar to a line, the same plane can also be described by the vector
equation (r p) n = 0.
By the distributivity of the scalar product, we have:
(r p) n = 0 r n p n = 0 r n = p n.
Now, p is known (it is the position vector of a point p known to be on the plane). So too
is n (it is the planes normal vector). Thus, p n is simply some known number.
So we can describe the plane even more simply by the vector equation

r n = d,
where d = p n.

Page 325, Table of Contents

www.EconsPhDTutor.com

Example 304. Consider a plane that contains the point p = (1, 2, 3) and has normal vector
(1, 1, 0). We compute
1 1

d=pn=
2 1
3 0

= 1 1 + 2 1 + 3 0 = 3.

We thus conclude that the plane may be described by the vector equation r (1, 1, 0) = 3.
This says that the plane contains exactly every point r, whose position vector r satisfies the
above equation. For example, the points r1 = (3, 0, 0), r2 = (0, 3, 5), and r3 = (1, 2, 1) are
on the plane, because their position vectors r1 = (3, 0, 0), r2 = (0, 3, 5), and r3 = (1, 2, 1)
satisfy the above equation, as we can easily verify:
3 1

r1 n =
0 1
0 0

= 3 1 + 0 1 + 0 0 = 3.

0 1

r2 n =
3 1
5 0

= 0 1 + 3 1 + 5 0 = 3.

1 1

r3 n =
2 1
1 0

= 1 1 + 2 1 + (1) 0 = 3.

Lest you be sceptical that a plane could be described so simply, lets verify that two vectors
on the plane are indeed orthogonal to the normal vector n. First consider r2 r1 = (0, 3, 5)
(3, 0, 0) = (3, 3, 5) this is a vector on the plane. We can verify that indeed
3 1

(r2 r1 ) n =
3 1
5 0

= 3 1 + 3 1 + 5 0 = 0.

Next consider p r3 = (1, 2, 3) (1, 2, 1) = (0, 0, 4) this is also a vector on the plane. We
can verify that indeed
0 1

(p r3 ) n =
0 1
4 0
Page 326, Table of Contents

= 0 1 + 0 1 + 4 0 = 0.

www.EconsPhDTutor.com

Example 305. Consider a plane that contains the point p = (0, 0, 1) and has normal vector
(2, 1, 1). We compute
0 2

d=pn=
0 1
1 1

= 0 2 + 0 (1) + 1 1 = 1.

We thus conclude that the plane may be described by the vector equation r (2, 1, 1) = 1.
This says that the plane contains exactly every point r, whose position vector r satisfies
the above equation. For example, the points r1 = (1, 1, 0), r2 = (0, 1, 2), and r3 = (1, 2, 1)
are on the plane, because their position vectors r1 = (1, 1, 0), r2 = (0, 1, 2), and r3 = (1, 2, 1)
satisfy the above equation, as we can easily verify:
1 2

r1 n =
1 1
0 1

= 1 2 + 1 (1) + 0 1 = 1.

0 2

r2 n =
1 1
2 1

= 0 2 + 1 (1) + 2 1 = 1.

1 2

r3 n =
2 1
1 1

= 1 2 + 2 (1) + 1 1 = 1.

Lest you be sceptical that a plane could be described so simply, lets verify that two vectors
on the plane are indeed orthogonal to the normal vector n. First consider r2 r1 = (0, 1, 2)
(1, 1, 0) = (1, 0, 2) this is a vector on the plane. We can verify that indeed
1 2

(r2 r1 ) n =
0 1
2 1

= (1) 2 + 0 (1) + 2 1 = 0.

Next consider p r3 = (0, 0, 1) (1, 2, 1) = (1, 2, 0) this is also a vector on the plane.
We can verify that indeed
1 2

(p r3 ) n =
2 1
0 1
Page 327, Table of Contents

= (1) 2 + (2) (1) + 0 1 = 0.

www.EconsPhDTutor.com

We just learnt how to write down the vector equation of a plane, given a point on the
plane and its normal vector.
We now do the same, but given instead three points on a plane.
Example 306. A plane contains the points a = (1, 2, 3), b = (4, 5, 8), and c = (2, 3, 5).

= (1, 1, 2) are on the plane. Hence, a normal vector to the


Both vectors ab = (3, 3, 5) and
ac


plane is ab
ac = n = (1, 1, 0).
Since a n = 1, the plane can be described by the vector equation r (1, 1, 0) = 1.
Example 307. A plane contains the points a = (1, 0, 0), b = (0, 1, 0), and c = (0, 0, 1).

= (1, 0, 1) are on the plane. Hence, a normal vector to


Both vectors ab = (1, 1, 0) and
ac


the plane is ab
ac = n = (1, 1, 1).
Since a n = 1, the plane can be described by the vector equation r (1, 1, 1) = 1.

Page 328, Table of Contents

www.EconsPhDTutor.com

We jnow write down the vector equation of a plane, given two points and a vector on
the plane.
Example 308. A plane contains the points a = (0, 0, 3) and b = (1, 4, 5), and the vector
v = (3, 2, 1).

Both vectors ab = (1, 4, 2) and v = (3, 2, 1) are on the plane. Hence, a normal vector to the

plane is ab v = n = (0, 5, 10).


Since a n = 30, the plane can be described by the vector equation r (0, 5, 10) = 30.
Example 309. A plane contains the points a = (8, 2, 0) and b = (3, 6, 9), and the vector
v = (0, 1, 1).

Both vectors ab = (5, 8, 9) and v = (0, 1, 1) are on the plane. Hence, a normal vector to

the plane is ab v = n = (1, 5, 5).


Since a n = 18, the plane can be described by the vector equation r (1, 5, 5) = 18.

Page 329, Table of Contents

www.EconsPhDTutor.com

32.1

Planes: Vector to Cartesian Equations

Let n = (a, b, c) be the normal vector of a plane. Let p = (p1 , p2 , p3 ) be a point on the plane.
Then the plane can be described by the vector equation
r n = p n,
where r = (x, y, z) is the position vector of a generic point on the plane. Writing out the
vectors in the above equation explicitly, we have:
x a
y b

z c
or

p1 a
= p b
2
p3 c

ax + by + cz = ap1 + bp2 + cp3 .

This last equation is the cartesian equation description of the same plane. Note, once
again, that d = ap1 + bp2 + cp3 is simply some known number. So this cartesian equation
simply says that the plane contains exactly those points (x, y, z) that satisfy the equation
ax + by + cz = d.
Example 310. The plane with vector equation r (1, 1, 0) = 3 has cartesian equation
x + y = 3.
Example 311. The plane with vector equation r (2, 1, 1) = 1 has cartesian equation
2x y + z = 1.
Example 312. The plane with vector equation r (1, 1, 0) = 1 has cartesian equation
x y = 1.
Example 313. The plane with vector equation r (1, 1, 1) = 1 has cartesian equation
x + y + z = 1.
Example 314. The plane with vector equation r (0, 5, 10) = 30 has cartesian equation
5y 10z = 30.
Example 315. The plane with vector equation r (1, 5, 5) = 18 has cartesian equation
x + 5y 5z = 18.

Page 330, Table of Contents

www.EconsPhDTutor.com

Its thus surprisingly easy to go back and forth between a planes vector and cartesian
equations:
r (a, b, c) = d

ax + by + cz = d.

Example 316. Given a plane with cartesian equation 2x + 3y + 5z = 7, we immediately


know that it has vector equation r (2, 3, 5) = 7.
Example 317. Given a plane with cartesian equation 2x + 3z = 5, we immediately know
that it has vector equation r (2, 0, 3) = 5.

Heres a nice observation: Every plane that contains the origin (0, 0, 0) can be written in
the form ax + by + cz = 0. Conversely, every plane that does not contain the origin can be
written in the form ax + by + cz = 1. Formally:
Fact 37. A plane r n = d contains the origin d = 0.
Proof. Given a plane r n = d, the origin is on the plane (and thus satisfies this equation)
0 n = d = 0.

SYLLABUS ALERT
The 9740 (old) List of Formulae contains the following statement, but not the 9758 (revised)
List of Formulae!
The plane through A with normal vector n = n1 i + n2 j + n3 k has cartesian equation
n1 x + n2 y + n3 z + d = 0

where

d= a n.

Exercise 126. Find the vector and cartesian equations that describe the planes containing
each of the following set of three points: (a) a = (7, 3, 4), b = (8, 3, 4), and c = (9, 3, 7). (b)
a = (8, 0, 2), b = (4, 4, 3), and c = (2, 7, 2). (c) a = (8, 5, 9), b = (8, 4, 5), and c = (5, 6, 0).
(Answer on p. 1089.)

Exercise 127. Write down the vector equations of the planes whose cartesian equations
are as given: (a) 3x + 2y + 5z = 3. (b) 2y + 5z = 3. (c) 5z = 3. (Answer on p. 1090.)

Page 331, Table of Contents

www.EconsPhDTutor.com

32.2

Planes: Hessian Normal Form

Example 318. The plane with vector equation r (1, 0, 1) = 11 or cartesian equation x + z =
11 can be described in an infinite number of ways. For example, the same plane can also
be described by any of the following four equations: r (2, 0, 2) = 22, r (1/11, 0, 1/11) = 1,
2x + 2z = 22, and x/11 + z/11 = 1.
If you talk about the plane r (2, 0, 2) = 22 and I talk about the plane x/11x + z/11z = 1, it
make take us a moment to realise that we are talking about the exact same plane. To save
ourselves such trouble, it may be desirable to describe planes in a standardised form, called
the Hessian normal form.
as our normal vector. However,
This involves simply picking the unit normal vector n
there are two possible unit normal vectors, one pointing up and the other pointing down.
0, so that the RHS of our
We will choose the unit normal vector that ensures that p n
vector or cartesian equation in Hessian normal form is always non-negative.

Example 319. Consider the plane r (1, 0, 1) = 11 or x + z = 11. We have (1,


0, 1) =

( 2/2, 0, 2/2). And so the plane can be rewritten in Hessian normal form as r

( 2/2, 0, 2/2) = 11 2/2 or ( 2/2) x + ( 2/2) z = 11 2/2.


is uniquely defined. Indeed,
Notice that in the Hessian normal form, the number d = p n
it is the distance of the plane from the origin! (Well prove this in section 33.2.)

Example 320. Consider the plane r (8, 1, 3) = 3 or 8x + y + 3z = 3. We have (8,


1, 3) =

(8/ 74, 1// 74, 3// 74). Note though that right now, the RHS is negative. So in order
to ensure that d 0 (as required by the Hessian normal form), we need simply reverse the

sign of our unit normal vector that is, we should pick (8/ 74, 1/ 74, 3/ 74) as our
unit normal
then,
can be rewritten
vector.
Altogether

the plane
in Hessian
normal form
as
r (8/ 74, 1/ 74, 3/ 74) = 3/ 74 or (8/ 74) x (1/ 74) y (3/ 74) z = (3/ 74).

Exercise 128. Rewrite each of the following planes vector equation into Hessian normal
form. (a) r (3, 6, 2) = 4. (b) r (1, 2, 2) = 1. (c) r (8, 1, 4) = 0. (Answer on p. 1090.)

Page 332, Table of Contents

www.EconsPhDTutor.com

33

Distances

Before we proceed, here are some useful things to remember. A line can be fully determined
by
1. Any two distinct points.
2. Any vector and a point.
Similarly, a plane can be fully determined by
1. Any three distinct points.
2. Any two distinct points and a distinct vector.39
3. Two distinct vectors and a point.

39

If the two points are a and b, then the vector must be distinct from cab, for any c R.

Page 333, Table of Contents

www.EconsPhDTutor.com

33.1

Distance of a Point from a Line

Definition 83. The foot of the perpendicular from a point a to a line l is the point b on
the line l that is closest to the point a. The distance between the point a and the line l is
the length of the line segment ab.

Distance
between
a and b

b
p

Note that the line ab must be perpendicular to the line l. Hence the name foot of the
perpendicular.

It is easier to remember how the proof of the following proposition works, than to try to
memorise the proposition itself:
Proposition 8. Given a point a and a line r = p +
v (for R),
2
2 (
v
) ; and
(a) The distance between the point and the line is
pa
pa

v
) v
.
(b) The foot of the perpendicular from the point to the line is the point p + (
pa

Proof. Let b be the foot of the perpendicular from the point to the line.
(a) Pick any known point on the line here the obvious choice is p. Consider the right and base of length
v
(refer
angled triangle bpa it has hypothenuse of length
pa
pa
Page 334, Table of Contents

www.EconsPhDTutor.com

to the diagram above). Hence, by the Pythagorean Theorem, the length of line segment ab
(or the distance between the point a and the line l) is:

2
2 (
v
) , as desired.

pa
pa

v
away from the point p, heading in the direction pb.
pa
(b) The point b is a distance

v
pb. There are two possible cases to examine.
pa
Hence b = p +

is pointing in the same direction as pb.


Case #1 : v

v
v
v
and
> 0, so that
=
. Altogether then, = becomes b =
Then pb = v
pa
pa
pa
v
v
v
= p + (
) v
, as desired.
p +
pa
pa

and pb are pointing in opposite directions.


Case #2 : v

v
v
v
=
. Altogether then, = becomes b =
< 0, so that
pa
pa
Then pb =
v and
pa
v
v
) (
) (
p + (
pa
v ) = p + (
pa
v), as desired.

On p. 938 in the Appendices (optional), I give another proof of the above Proposition using
calculus. The idea of this second proof will be illustrated in the last two examples of this
section.

Page 335, Table of Contents

www.EconsPhDTutor.com

Example 321. Consider the point a = (1, 2, 3) and the line r = (0, 1, 2) + (9, 1, 3) ( R).
= (1, 1, 1) and so
2 = 12 +12 +12 = 3.
Pick a point on the line say p = (0, 1, 2). We have
pa
pa
Also,
(1, 1, 1) (9, 1, 3)
13

v
=
pa
= .
91
92 + 12 + 32
2
v
) = 169/61. Hence, the length of the side is
pa
And so (

169
=
91

104
=
91

8
1.069.
7

This is the distance between point a and the line l. Moreover,


13 (9, 1, 3) 1

b = (0, 1, 2) +
= (9, 8, 17) .
7
91
91

Not to scale.

a = (1, 2, 3)
Distance between
a and b is 1.069

l
b=

(9, 8, 17)

p = (0, 1, 2)

v
> 0.
Note that in this example, v and pb do point in the same direction and we have
pa

In contrast, in the next example, v and pb will point in opposite directions and we will
v
< 0.
have
pa
Page 336, Table of Contents

www.EconsPhDTutor.com

Example 322. Consider the point a = (1, 0, 1) and the line r = (3, 2, 1)+(5, 1, 2) ( R).
= (4, 2, 0) and so
2 = 42 +22 +02 =
Pick a point on the line say p = (3, 2, 1). We have
pa
pa
20. Also,
(4, 2, 0) (5, 1, 2)
22

v
=

pa
= .
30
52 + 12 + 22

v
< 0 and sure enough, v and pb point in the opposite directions.) So
(As noted,
pa
2
v
) = 484/30. Hence, the length of the side is
(
pa

20

484
=
30

116
=
30

58
2.823.
15

This is the distance between point a and the line l. Moreover,


22 (5, 1, 2) 1

(10, 19, 7) .
=
b = (3, 2, 1)
15
30
30

Not to scale.

a = (-1, 0, 1)
Distance between
a and b is 2.823

l
b=

(-10, 9, -7)

p = (3, 2, 1)

Page 337, Table of Contents

www.EconsPhDTutor.com

Exercise 129. For each of the following, find (i) the distance between the given point a
and the given line l; and also (ii) the point b on the line that is closest to a. (a) The point
a = (7, 3, 4) and the line l described by r = (8, 3, 4) + (9, 3, 7). (b) The point a = (8, 0, 2)
and the line l described by r = (4, 4, 3) + (2, 7, 2). (c) The point a = (8, 5, 9) and the line l
described by r = (8, 4, 5) + (5, 6, 0). (Answers on pp. 1091, 1092, and 1093.)

Page 338, Table of Contents

www.EconsPhDTutor.com

We now learn a second method for finding the foot of a perpendicular and hence the
distance of a point to a line. This second method involves calculus and finding the minimum
point. It also occasionally features on the A-level exams.
Example 323. Consider the point a = (1, 2, 3) and the line r = (0, 1, 2) + (9, 1, 3) ( R).
The distance between a and a generic point r on the line is
RRR
RRR 1 9
a r = RRRR
2 1+
RRR
RRR 3 2 + 3
=

R R
RRRR RRRR 1 9
RRRR = RRRR 1
RR RR
RRRR RRRR 1 3
R R

R
RRRR
RRRR
RR
RRRR
R

(1 9)2 + (1 )2 + (1 3)2 =

912 26 + 3.

Recall what means. It is a parameter as varies, the vector equation r = (0, 1, 2) +


(9, 1, 3) gives us another point of the line.

So now the expression 912 26 + 3 tells us:


As varies, the distance between the point
a and the corresponding point r on the line is 912 26 + 3.
Our goal is to find the point
r on the line that is closest to the point a. In other
2
words,
our goal is to minimise 91 26 + 3. So we can look for the minimum point
of 912 26 + 3.

To simplify matters, note that minimising 912 26 + 3 is the same as minimising 912
26 + 3. So we might as well look for the minimum point of 912 26 + 3. To this end:
d
set
(912 26 + 3) = 182 26 = 0
d

26 1
= .
182 7

Altogether then, the point b on the line l that is closest to the point a has parameter = 1/7.
So b = (0, 1, 2) + 1/7(9, 1, 3) = 1/7(9, 8, 17).
And the distance between a and l (or equivalently, the length of the line segment ab) is

912 26 + 3 =

1 2
1
91 ( ) 26 ( ) + 3 =
7
7

8
.
7

Of course, these are the same as what we found in Example 321 a few pages ago.

Page 339, Table of Contents

www.EconsPhDTutor.com

Example 324. Consider the point a = (1, 0, 1) and the line described by the vector
equation r = (3, 2, 1) + (5, 1, 2) ( R). The distance between a and a generic point r on
the line is
RRR
RRR 1 3 + 5

a r = RRRR
0
2+
RRR
RRR 1 1 + 2
=

R R
RRRR RRRR 4 5
RRRR = RRRR 2
RR RR
RRRR RRRR 2
R R

R
RRRR
RRRR
RR
RRRR
R

(4 5)2 + (2 )2 + (2)2 = 302 + 44 + 20.

Now look for the minimum point of 302 + 44 + 20:


d
set
(302 + 44 + 20) = 60 + 44 = 0
d

44 11
=
.
60
15

So b = (3, 2, 1) 11/15(5, 1, 2) = 1/15(10, 19, 7).


And the distance between a and l (or equivalently, the length of the line segment ab) is

302 + 44 + 20 =

11 2
11
30 (
) + 44 (
) + 20 =
15
15

58
.
15

Of course, these are the same as what we found in Example 322 a few pages ago.

Exercise 130. For each of the following, use the second method (calculus) to find (i) the
distance between the given point a and the given line l; and also (ii) the point b on the
line that is closest to a. (a) The point a = (7, 3, 4) and the line l described by r = (8, 3, 4) +
(9, 3, 7). (b) The point a = (8, 0, 2) and the line l described by r = (4, 4, 3) + (2, 7, 2). (c)
The point a = (8, 5, 9) and the line l described by r = (8, 4, 5) + (5, 6, 0). (Answers on p.
1094.)

Page 340, Table of Contents

www.EconsPhDTutor.com

33.2

Distance of a Point from a Plane

This is very much analogous to the distance of a point from a line.


Definition 84. The foot of the perpendicular from a point a to a plane P is the point b on
the plane P that is closest to the point a. The distance between the point a and the plane
P is the length of the line segment ab.

Distance
between
a and b

Plane
p
b

Proposition 9. Given a point a (with position vector a) and a plane given in Hessian

= d,
normal form r n
; and
(a) The distance between the point and the plane is d a n
) n
.
(b) The foot of the perpendicular from the point to the plane is the point a + (d a n

Proof. Let b be the foot of the perpendicular from the point to the line.
(a) Pick any point p on the plane. The length of the line segment ab and hence also the
on
distance between the point and the plane is simply the length of the projection of
ap
n
= (p a) n
= d a n
, as desired.
the planes normal vector, which is simply
ap

away from a, heading in the direction ab. Hence,


(b) The point b is a distance d a n

ab. There are two possible cases to examine.


b = a + d a n

n
is pointing in the same direction as pb, then n
= ab. Moreover
=
Case #1 : If n
ap

> 0, so that d a n
= d a n
. Altogether then, = becomes b = a + (d a n
) n
, as
dan
desired.

n
and pb are pointing in opposite directions, then n
= ab. Moreover
= d
Case #2 : If n
ap

< 0, so that d a n
= (d a n
). Altogether then, = becomes b = a(d a n
) (
a n
n) =
) n
, as desired.
a + (d a n
Page 341, Table of Contents

www.EconsPhDTutor.com

Example 325. Consider the point a = (1, 2, 3) and the plane described by the vector
equation r (1, 1, 1) = 3.
Convert the vector equation of the plane to Hessian normal form:

3
1 1 1
r ( , , ) = = 3.
3 3 3
3

=
So n

3
= 2 3.
(1, 1, 1), d = 3, and a n
3

= 3 2 3 =
Altogether then, the distance between the point and the plane is d a n

3 and the foot of the perpendicular is

) n
= (1, 2, 3) + ( 3 2 3)
a + (d a n

3
(1, 1, 1) = (0, 1, 2).
3

By the way, notice that in this example, n points in the opposite direction from ab. And

< 0.
n. And moreover, d a n
so ab =

a = (1, 2, 3)
Not to scale.

Plane
p = (0, 1, 2)

Page 342, Table of Contents

Distance
between
a and b
b = (0, 1, 2)

www.EconsPhDTutor.com

Example 326. Consider the point a = (0, 0, 0) and the plane described by the vector
equation r (1, 2, 3) = 32.
Convert the vector equation of the plane to Hessian normal form:
2
3
32
1
r ( , , ) = .
14 14 14
14
32
1
= 0.
= (1, 2, 3), d = , and a n
So n
14
14
32
= 0 =
Altogether then, the distance between the point and the plane is d a n
14
32
and the foot of the perpendicular is
14
32
1
16
) n
= (0, 0, 0) + ( 0) (1, 2, 3) = (1, 2, 3).
a + (d a n
7
14
14

By the way, notice that in this example, n points in the same direction as ab. And so

. And moreover, d a n
> 0.
ab = n

a = (0, 0, 0)
Not to scale.

Distance
between
a and b

Plane
p = (4, 5, 6)

Page 343, Table of Contents

b=

(1, 2, 3)

www.EconsPhDTutor.com

Exercise 131. For each of the following, find (i) the distance between the given point
a and the given plane P ; and also (ii) the point b on the line that is closest to a. (a)
a = (7, 3, 4), P r (9, 3, 7) = 109. (b) a = (8, 0, 2), P r (2, 7, 2) = 42. (c) a = (8, 5, 9),
P r (5, 6, 0) = 64. (Answers on pp. 1095, 1096, and 1097.)

Page 344, Table of Contents

www.EconsPhDTutor.com

34
34.1

Angles

Angle between Two Lines (2D)

Consider two lines on the 2D cartesian plane that are parallel (and thus either do not
intersect or are identical). We define the angle between them to be 0.

We define the angle between


two parallel lines to be 0.

Now consider two lines that intersect (see diagram below). Taking their intersection point
to be the vertex, A and B are, respectively, the acute and obtuse angles between the two
lines. Of course, there is the possibility that the two lines are perpendicular, in which case
A and B are both right (i.e. equal to /2).

So when talking about the angle between two lines, there is some potential for confusion.
Are we talking about angle A or angle B?
By convention, the angle between two lines is the smaller angle. (Also, on the A-level
exams, they are usually quite careful to specifying that they want the acute angle, so that
there is no confusion.)
Page 345, Table of Contents

www.EconsPhDTutor.com

If we have the direction vectors of the lines, then we can simply use what we learnt about
the scalar product to compute the angle between them.
Example 327. Consider the lines (on the 2D cartesian plane) r = (1, 3) + (2, 1) and
r = (1, 1) + (1, 3) ( R). The angle between their direction vectors v1 = (2, 1) and
v2 = (1, 3) is given by
= cos1 (

v1 v2
(2, 1) (1, 3)
5
) = cos1 (
) = cos1 ( ) 0.785.
v1 v2
(2, 1) (1, 3)
5 10

So the acute angle between the two lines is 0.785.

y 4
A = 0.785
Vector equation
r = (1, 3) + (2, 1)
2

x
0
-4

-2

Vector equation
r = (-1, -3) + (1, 3)

2
The vector (1, 3)

-2
A = 0.785
The vector (2, 1)
-4

Page 346, Table of Contents

www.EconsPhDTutor.com

Example 328. Consider the lines (on the 2D cartesian plane) r = (0, 0) + (2, 3) and
r = (1, 0) + (3, 1) ( R). The angle between their direction vectors v1 = (2, 3) and
v2 = (3, 1) is given by
= cos1 (

(2, 3) (3, 1)
3
v1 v2
) = cos1 (
) = cos1 ( ) 1.837.
v1 v2
(2, 3) (3, 1)
13 10

This is the obtuse angle between the two lines. So the acute angle between the two lines is
A = 1.837 = 1.305.

y 4

The vector (-2, 3)


B = 1.837

Vector equation
r = (0, 0) + (-2, 3)

The vector (3, 1)

2
A = 1.305
B = 1.837
x
0

-4

-2

A = 1.305
Vector equation
r = (1, 0) + (3, 1)

-2

-4

Page 347, Table of Contents

www.EconsPhDTutor.com

Example 329. Consider the lines (on the 2D cartesian plane) r = (2, 2) + (3, 3) and
r = (1, 1) + (1, 1) ( R). The angle between their direction vectors v1 = (3, 3) and
v2 = (1, 1) is given by
= cos1 (

(3, 3) (1, 1)
6
6
v1 v2
) = cos1 (
) = cos1 ( ) = cos1 ( ) = .
v1 v2
(3, 3) (1, 1)
6
18 2

So the two vectors are parallel. Which means that the two lines are parallel and so by
definition, the angle between the two lines is 0.

y 4

The lines are parallel


and thus the angle
between them is 0.

The vector (3, 3)

The vector (-1, -1)

x
0

-4

-2

Vector equation
r = (2, -2) + (3, 3)
-2
Vector equation
r = (1, 1) + (-1, 1)
-4

Page 348, Table of Contents

www.EconsPhDTutor.com

Exercise 132. Find the acute angle between each of the following pairs of lines. (a)
r = (1, 2) + (1, 1) and r = (0, 0) + (2, 3) ( R). (b) r = (1, 2) + (1, 5) and
r = (0, 0) + (8, 1) ( R). (c) r = (1, 2) + (2, 6) and r = (0, 0) + (3, 2) ( R). (Answer
on p. 1098.)

Page 349, Table of Contents

www.EconsPhDTutor.com

34.2

Angle between Two Lines (3D)

Visualising lines in 3D space is difficult. Which is why we tackled the 2D case first.
It turns out that we compute angles between two lines in 3D space in exactly the same way
as in the 2D case.
1. If two lines are parallel, then again we define the angle between them to be 0.
2. If two lines intersect, then again we take their intersection point to be the vertex and
take the smaller angle formed to be the angle between the two lines.
On the 2D cartesian plane, the above were the only two possibilities two lines either are
parallel or intersect. In contrast, in 3D space, there is the third possibility that two lines
neither are parallel nor intersect! As well learn in section 35.1, any two lines that
neither are parallel nor intersect are called skew lines.
What is the angle between two skew lines, given that they do not intersect?
3. Given two skew lines, translate one of them so that they intersect. Examine the angle
between the two now-intersecting lines. This is defined to be the angle between the two
skew lines.
The next example illustrates.

Page 350, Table of Contents

www.EconsPhDTutor.com

Example 330. Below, the red line and pink line are skew lines, i.e., they neither intersect
nor are parallel. To find the angle between them, translate the red line upwards so that the
new red dotted line intersects the pink line at the purple dot. The angle A is then defined
to be the angle between the two skew lines.

Skew lines (lines that neither


intersect nor are parallel)
y
Translate one of
the lines so that
they intersect

2
1
x
1
1

2
3
z

So once again, given any two lines, the angle between them is simply the angle between
their direction vectors. So again the scalar product comes in handy.

Page 351, Table of Contents

www.EconsPhDTutor.com

Example 331. Consider the lines r = (0, 1, 2)+(9, 1, 3) and r = (4, 5, 6)+(3, 2, 1) ( R).
The angle between their direction vectors v1 = (9, 1, 3) and v2 = (3, 2, 1) is given by
= cos1 (

(9, 1, 3) (3, 2, 1)
32
v1 v2
) = cos1 (
) = cos1 ( ) 0.459.
v1 v2
(9, 1, 3) (3, 2, 1)
91 14

So the acute angle between the two lines is 0.459.


Example 332. Consider the lines r = (1, 2, 3) + (0, 1, 0) and r = (0, 0, 0) + (8, 3, 5)
( R). The angle between their direction vectors v1 = (0, 1, 0) and v2 = (8, 3, 5). Thus,
= cos1 (

(0, 1, 0) (8, 3, 5)
3
v1 v2
) = cos1 (
) = cos1 ( ) 1.879.
v1 v2
(0, 1, 0) (8, 3, 5)
1 98

So the obtuse angle between the two lines is 1.879. And the angle between the two lines is
1.263.
Example 333. Consider the lines r = (1, 3, 3) + (1, 5, 3) and r = (7, 4, 7) + (7, 2, 1)
( R). The angle between their direction vectors v1 = (1, 5, 3) and v2 = (7, 2, 1). Thus,
= cos1 (

v1 v2
(1, 5, 3) (7, 2, 1)
) = cos1 (
) = cos1 (0) 0.5.
v1 v2
(1, 5, 3) (7, 2, 1)

So the two lines are perpendicular and the angle between them is right (i.e. /2).

Exercise 133. Find the angle between each of the following pairs of lines. (a) r = (1, 2, 3)+
(1, 1, 0) and r = (0, 0, 0) + (2, 3, 4) ( R). (b) r = (1, 2, 3) + (1, 5, 6) and r =
(0, 0, 0) + (8, 1, 1) ( R). (c) r = (1, 2, 3) + (2, 6, 7) and r = (0, 0, 0) + (3, 2, 1) ( R).
(Answer on p. 1098.)

Page 352, Table of Contents

www.EconsPhDTutor.com

34.3

Angle between A Line and a Plane

Fact 38. The angle A between the line r = p + v and the plane r n = d is given by
A = sin1

vn
.
v n

Line

Line

is obtuse

is acute

A = 0.5

Plane

A = 0.5

Plane

Direction
vector of line

Direction
vector of line

Proof. Consider the angle between the lines direction vector and the planes normal
vn
.
vector. By the scalar product, we have cos =
v n
Case #1: If is acute or right, i.e. (0, /2], then the angle A between the line and the
plane is A = /2 (see figure). And so

vn
sin A = sin ( ) = sin ( ) cos sin cos ( ) = cos =
.
2
2
2
v n
Note that if (0, /2], then v n 0, so that v n = v n. Altogether, we indeed have
sin A =

vn

v n

or A = sin1

vn
.
v n

Case #2: If is obtuse or straight, i.e. (/2, ], then the angle A between the line and
the plane is A = 0.5 (see figure). And so

v n
sin A = sin ( ) = sin cos ( ) sin ( ) cos = cos =
.
2
2
2
v n
Note that if (/2, ], then v n < 0, so that v n = v n. Altogether, we indeed have
sin A =

Page 353, Table of Contents

vn

v n

or A = sin1

vn
.
v n

www.EconsPhDTutor.com

Example 334. The angle between the line r = (0, 1, 2) + (9, 1, 3) ( R) and the plane
r (1, 1, 1) = 3 is

13
v

n
(9,
1,
3)

(1,
1,
1)
13
sin1
= sin1
= sin1 = sin1 0.906.
v n
(9, 1, 3) (1, 1, 1)
91 3
7 3
Example 335. The angle between the line r = (4, 2, 3) + (1, 0, 1) ( R) and the plane
r (1, 1, 1) = 5 is
sin1

(1, 0, 1) (1, 1, 0)
1
vn
= sin1
= sin1 = sin1 (1/2) = /6.
v n
(1, 0, 1) (1, 1, 0)
2 2

Example 336. The angle between the line r = (5, 5, 5) + (1, 0, 1) ( R) and the plane
r (0, 1, 0) = 3 is
sin1

(1, 0, 1) (0, 1, 0)
vn
= sin1
= sin1 0 = 0.
v n
(1, 0, 1) (0, 1, 0)

Exercise 134. Find the angle between the given line and plane. (a) r = (1, 2, 3) +
(1, 1, 0) ( R) and r (3, 4, 5) = 0. (b) r = (1, 2, 3) + (0, 2, 6) ( R) and r (1, 3, 5) = 2.
(c) r = (1, 2, 3) + (1, 9, 8) ( R) and r (2, 8, 2) = 3. (Answer on p. 1099.)

Page 354, Table of Contents

www.EconsPhDTutor.com

34.4

Angle between Two Planes

Given two planes P1 and P2 , the angle between them is simply the angle between any two
vectors v1 and v2 on the two planes.

n1
v2

n2

Angle between
the two planes
normal vectors

Angle between
the two planes

P2

v1

P1

But the normal vector n1 of the first plane is orthogonal to v1 ; similarly, the normal vector
n2 of the second plane is orthogonal to v2 . And so the angle between v1 and v2 is equal to
the angle between n1 and n2 .
Altogether then, the angle between two planes is simply the angle between their normal
vectors.
Again, there are two possible angles by convention, we take the smaller one.

Page 355, Table of Contents

www.EconsPhDTutor.com

Example 337. Consider the planes r (1, 1, 1) = 12 and r (1, 1, 0) = 1. The angle
between the two planes is:
= cos1 (

= cos1 (

n1 n2
)
n1 n2
(1, 1, 1) (1, 1, 0)
)
(1, 1, 1) (1, 1, 0)

2
2
= cos1 ( ) = cos1 ( )
3 2
6
2.526.
This is the obtuse angle. So the acute angle between the two planes is 2.526 = 0.615
radian.
Example 338. Consider the planes r (2, 1, 3) = 26 and r (3, 0, 5) = 25. The angle
between the two planes is
= cos1 (

= cos1 (

n1 n2
)
n1 n2
(2, 1, 3) (3, 0, 5)
)
(2, 1, 3) (3, 0, 5)

9
= cos1 ( ) 1.146.
14 34

Exercise 135. Find the angle between the two given planes. (a) r (1, 2, 3) = 1 and
r (3, 4, 5) = 2. (b) r (1, 2, 3) = 3 and r (5, 1, 1) = 4. (c) r (1, 1, 8) = 5 and r (3, 0, 10) = 6.
(Answer on p. 1100.)

Page 356, Table of Contents

www.EconsPhDTutor.com

35

Relationships between Lines and Planes


35.1

Relationship between Two Lines

Definition 85. Two lines are parallel if their direction vectors can be written as scalar
multiples of each other.

Example 339. The lines r = (0, 0, 0) + (0, 1, 0) and r = (4, 17, 0) + (1, 0, 0) ( R) are
not parallel, because (0, 1, 0) cannot be written as a scalar multiple of (1, 0, 0).
Example 340. The lines r = (8, 1, 1) + (3, 6, 9) and r = (4, 5, 6) + (1, 2, 3) ( R) are
parallel, because (3, 6, 9) = 3(1, 2, 3).

Page 357, Table of Contents

www.EconsPhDTutor.com

Definition 86. A set of points are coplanar if there is some plane that contains all of these
points.

Any two points are always coplanar indeed, they are collinear (p1 and p2 in the figure
below). Three points are also always coplanar, although they may not be collinear (p1 , p2 ,
and p3 in the figure below). But four points may not be coplanar (p1 , p2 , p3 , and p4 in the
figure below).

Line 2
Two points are coplanar. They
also lie on the same line.

Three points are coplanar, though


not necessarily on the same line.

p1
Plane

p3
p2
Line 1

p4

Four points may


not be coplanar.

Definition 87. Two lines are coplanar if there is some plane on which both lie. Two lines
that are not coplanar are called skew lines.

Example 341. In the figure above, Line 1 and Line 2 are skew lines. Line 1 lies on the
plane illustrated. Line 2 cuts through the plane and does not intersect Line 1.

Page 358, Table of Contents

www.EconsPhDTutor.com

How do we determine whether two lines l1 and l2 are coplanar or skew? Well,
1. If they are parallel, then obviously we can construct a plane that contains both lines.
And so the two lines are coplanar.
2. If they are not parallel and they lie on the same plane, then they must intersect. This
is just the familiar fact you learnt in primary school two non-parallel lines on the
plane must definitely intersect.
Altogether we conclude:
Fact 39. Two lines are coplanar if and only if they (i) are parallel; OR (ii) intersect.
Equivalently, two lines are skew if and only if they (i) are not parallel; AND (ii) do not
intersect.

Example 342. Consider the lines r = (8, 1, 1)+(3, 6, 9) and r = (4, 5, 6)+(1, 2, 3) ( R).
The direction vector of one can be written as the scalar multiple of the other, so they are
parallel. Hence, they are also coplanar; or equivalently, they are not skew.

Example 343. Consider the lines r = (0, 0, 0) + (0, 1, 0) and r = (4, 17, 0) + (1, 0, 0)
( R). The direction vector of one cannot be written as the scalar multiple of the
other, so they are not parallel. If they intersect, then there are reals and such that
(0, 0, 0) + (0, 1, 0) = (4, 17, 0) + (1, 0, 0), or
0 = 4 + , = 17, and 0 = 0.
= 17, = 4 solves the above equations. (What does this mean? This means that the
first line goes through the point (0, 0, 0) + (0, 1, 0) = (0, 17, 0) and the second line also goes
through the same point (4, 17, 0) + (1, 0, 0) = (0, 17, 0).)
The two lines intersect at (0, 17, 0). And so they are coplanar or equivalently, they are
not skew.
If wed like, we can easily find the plane on which these two lines lie. Remember: All
we need are two distinct vectors and a point to determine a plane. We already have two
distinct vectors, namely the direction vectors of the two lines. Using these, we can find
a normal vector for the plane namely (0, 1, 0) (1, 0, 0) = (0, 0, 1). Noting also that
the origin is on the first line and therefore on the plane, we conclude that the plane is
r (0, 0, 1) = 0.

Page 359, Table of Contents

www.EconsPhDTutor.com

Example 344. Consider the lines r = (0, 1, 2)+(9, 1, 3) and r = (4, 5, 6)+(3, 2, 1) ( R).
They are not parallel. Lets see if they have an intersection point. If they intersect, then
there are reals and such that (0, 1, 2) + (9, 1, 3) = (4, 5, 6) + (3, 2, 1), or
1

9 = 4 + 3, 1 + = 5 + 2, and 2 + 3 = 6 + .
3

Take 2 = minus = to get (4 + 6) (1 + ) = (12 + 2) (5 + 2) or 3 + 5 = 7 or = 0.8.


2
Now from =, this means that = 1.6. These do not work if we try plugging them into
1
=. Hence, there are no reals and that solve the above system of equations. In other
words, the two lines do not intersect.
And so the two lines are not coplanar or equivalently, they are skew.

Exercise 136. Determine whether each of the following pairs of lines is coplanar or skew.
If they are coplanar, find the plane that contains both of them. (a) r = (8, 1, 5) + (3, 2, 1)
and r = (1, 2, 3) + (5, 6, 7) ( R). (b) r = (0, 0, 6) + (3, 9, 0) and r = (1, 1, 1) + (1, 3, 0)
( R). (c) r = (6, 5, 5) + (1, 0, 1) and r = (8, 3, 6) + (0, 1, 1) ( R). (Answer on p. 1101.)

Page 360, Table of Contents

www.EconsPhDTutor.com

35.2

Relationship between a Line and a Plane

Definition 88. A line with direction vector v and a plane with normal vector n are parallel
if v n = 0 (i.e. v and n are perpendicular).
The above definition makes sense, because if the line is perpendicular to the planes normal
vector, then the line must be parallel to the plane itself.
Fact 40. Given a plane and a line, there are three possible cases (illustrated below):
1. The line and plane are parallel and do not intersect at all.
2. The line and plane are parallel and the line lies completely on the plane.
3. The line and plane are not parallel and intersect at exactly one point.

Line 1

Line 3

Plane
Line 2

Proof. Optional, see p. 939 in the Appendices.


Note that if a line and a plane are parallel, then either (i) they do not intersect at all; or
(ii) the line lies completely on the plane.
So if a line and a plane are parallel and you can prove that they share at least one
intersection point, then it must be that the line lies completely on the plane.
Conversely, if a line and a plane are parallel and you can prove that there is at least one
point on the line that is not on the plane (or that there is at least one point on the plane
that is not on the line), then it must be that they do not intersect at all.

Page 361, Table of Contents

www.EconsPhDTutor.com

Example 345. Consider the line r = (3, 5, 5)+(9, 1, 3) ( R) and the plane r(1, 1, 1) = 3.
We have (9, 1, 3) (1, 1, 1) = 13 0 and so they are not parallel.
They must therefore intersect at exactly one point. Lets find it.
Plug in a generic point of the line into the equation for the plane: [(3, 5, 5) + (9, 1, 3)]
(1, 1, 1) = 3 3 + 9 + 5 + + 5 + 3 = 3 13 + 13 = 3 = 10/13. So the
intersection point is (3, 5, 5) 10/13(9, 1, 3).
Example 346. Consider the line r = (3, 5, 5) + (9, 1, 3) ( R) and the plane r (1, 0, 3) =
6. We have (9, 1, 3) (1, 0, 3) = 0 and so they are parallel.
There are two possibilities. Either they do not intersect at all OR the line lie completely
on the plane.
The point (3, 5, 5) is on the line but is not on the plane, as we can easily verify (3, 5, 5)
(1, 0, 3) = 12 6. And so the line and plane do not intersect at all.
Example 347. Consider the line r = (3, 5, 3) + (9, 1, 3) ( R) and the plane r (1, 0, 3) =
6. We have (9, 1, 3) (1, 0, 3) = 0 and so they are parallel.
There are two possibilities. Either they do not intersect at all OR the line lie completely
on the plane.
The point (3, 5, 3) on the line is also on the plane: (3, 5, 3) (1, 0, 3) = 6. Since they are
parallel and share at least one intersection point, it must be that the line lies completely
on the plane.

Exercise 137. Determine whether the given line and plane are (i) parallel but do not
intersect; (ii) parallel with the line lying completely on the plane; or (iii) intersect at
exactly one point. (a) r = (4, 5, 6) + (2, 3, 5) ( R) and r (10, 0, 4) = 26. (b) r =
(5, 5, 6) + (2, 3, 5) ( R) and r (10, 0, 4) = 26. (c) r = (4, 5, 6) + (2, 3, 5) ( R) and
r (10, 0, 3) = 26. (Answer on p. 1102.)

Page 362, Table of Contents

www.EconsPhDTutor.com

35.3

Relationship between Two Planes

Definition 89. Two planes are parallel if their normal vectors can be written as scalar
multiples of each other.
(Note that an alternative definition is this: Two planes are parallel if they do not intersect.
We will show that these two definitions are equivalent.)
Imagine that two planes intersect at some line, which well call the intersection line.
Since this intersection line is on both planes, it must also be perpendicular to the normal
vectors of both planes. In other words, it must have direction vector n1 n2 . The next
fact is thus not surprising (although actually proving it takes a little work).
Fact 41. Two non-parallel planes with normal vectors n1 and n2 intersect at all if and only
if they intersect along a line with direction vector n1 n2 (i.e. the line is perpendicular to
both n1 and n2 ).

Proof. Optional, see p. 940 in the Appendices.

Fact 42. Given two planes, there are three possible cases:
1. The two planes are parallel and exactly identical.
2. The two planes are parallel and do not intersect at all.
3. The two planes are not parallel and share an intersection line with direction vector
n1 n2 (where n1 , n2 are the normal vectors of the plane).

Proof. Optional, see p. 942 in the Appendices.

Page 363, Table of Contents

www.EconsPhDTutor.com

Example 348. In the figure below, planes P1 and P2 are parallel and do not intersect at
all. Planes P2 and P3 are not parallel and share an intersection line with direction vector
n2 n3 .

n2
P3

n3

P2
Intersection line
of P2 and P3

n2 n3

P1

Note that analogous to our study of two lines, if two planes are parallel, then either (i) they
do not intersect at all; or (ii) they are identical.
So if two planes are parallel and you can prove that they share at least one intersection
point, then it must be that the two planes are identical.
Conversely, if two planes are parallel and you can prove that there is at least one point
on one plane that is not on the other plane, then it must be that they do not intersect
at all.
And in the case where they are not parallel, to find the intersection line, simply find a point
p where the two planes intersect. Then the intersection line is simply r = p + (n1 n2 )
( R).

Page 364, Table of Contents

www.EconsPhDTutor.com

Example 349. Consider the planes r (7, 1, 1) = 42 and r (1, 1, 2) = 6.


Clearly, (7, 1, 1) cannot be written as a scalar multiple of (1, 1, 2). So the two planes are
not parallel and share an intersection line whose direction vector is (7, 1, 1) (1, 1, 2) =
(1, 13, 6).
Find a point p = (x, y, z) where the two planes intersect:
1

7x + y + z = 42,

x + y + 2z = 6.

There are infinitely many points where the two planes intersect. So why not we look for an
intersection point where x = 0. Ill call this the plug in x = 0 trick.
2

In which case = minus = yields z = 36 and y = 78. Hence, the intersection line is r =
(0, 78, 36) + (1, 13, 6) ( R).

Example 350. Consider the planes r (1, 1, 1) = 12 and r (1, 1, 0) = 1.


Clearly, (1, 1, 1) cannot be written as a scalar multiple of (1, 1, 0). So the two planes are
not parallel and share an intersection line whose direction vector is (1, 1, 1) (1, 1, 0) =
(1, 1, 0).
Find a point p = (x, y, z) where the two planes intersect:
1

x + y + z = 12,

x y = 1.
2

Again, we can play the plug in x = 0 trick. In which case = says that y = 1 and now from
1
=, we have z = 11. And so (0, 1, 11) is an intersection point of the two planes. Hence, the
intersection line is r = (0, 1, 11) + (1, 1, 0) ( R).

Page 365, Table of Contents

www.EconsPhDTutor.com

You can use the plug in x = 0 trick whenever the intersection line has direction vector
with a x-coordinate that is not equal to 0.
But the plug in x = 0 trick may not work if the intersection line has direction vector
with x-coordinate equal to 0.

Example 351. Consider the planes r (0, 1, 3) = 0 and r (1, 1, 3) = 2.


Clearly, (0, 1, 3) cannot be written as a scalar multiple of (1, 0, 5). So the two planes are
not parallel and share an intersection line whose direction vector is (0, 1, 3) (1, 1, 3) =
(0, 3, 1).
Find a point p = (x, y, z) where the two planes intersect:
1

y + 3z = 0,

x + y + 3z = 2.

Here the direction vector of the intersection line has x-coordinate 0. So the plug in x = 0
1
2
trick might not work. And indeed it doesnt, because if we plug in x = 0, then = and = are
contradictory.
So lets try the plug in y = 0 trick instead, which I know will work because the ycoordinate of the direction vector of the interesction line is non-zero (its 3). Then from
1
2
= we have z = 0 and now from = we have x = 2. And so (2, 0, 0) is an intersection point
of the two planes. Hence, the intersection line is r = (2, 0, 0) + (0, 3, 1) ( R).
Alternatively, we could also have used the plug in z = 0 trick instead, which again I
know will work because the z-coordinate of the direction vector of the interesction line is
1
2
non-zero (its 1). Then from = we have y = 0 and now from = we have x = 2. And so again
we find that (2, 0, 0) is an intersection point of the two planes. And so again we would
have concluded that the intersection line is r = (2, 0, 0) + (0, 3, 1) ( R).

Page 366, Table of Contents

www.EconsPhDTutor.com

Example 352. Consider the planes r (4, 0, 3) = 5 and r (8, 0, 6) = 64.


Clearly, (4, 0, 3) can be written as a scalar multiple of (8, 0, 6) and so the two planes are
parallel.
Lets check if they are identical. The point (5/4, 0, 0) is on the first plane. However, it is
not on the second plane because (5/4, 0, 0) (8, 0, 6) = 10 64.
So the two planes are not identical and do not intersect at all.

Example 353. Consider the planes r (4, 0, 3) = 32 and r (8, 0, 6) = 64.


Clearly, (4, 0, 3) can be written as a scalar multiple of (8, 0, 6) and so the two planes are
parallel.
Lets check if they are identical. The point (8, 0, 0) is on the first plane. And it is also on
the second plane because (8, 0, 0) (8, 0, 6) = 64.
Since the two planes are parallel and share at least one intersection point, it must be that
the two planes are exactly identical.

Exercise 138. Determine whether the given pair of planes are parallel and identical, parallel and do not intersect, or are not parallel. If they are not parallel, determine also their
intersection line. (a) r(4, 9, 3) = 61 and r(1, 1, 2) = 19. (b) r(1, 1, 0) = 4 and r(1, 6, 8) = 60.
(c) r (4, 4, 8) = 56 and r (1, 1, 2) = 12. (d) r (4, 4, 8) = 48 and r (1, 1, 2) = 12. (Answer on
p. 1103.)

Page 367, Table of Contents

www.EconsPhDTutor.com

35.4

Relationship between Three Planes

SYLLABUS ALERT
The relationship between three planes is included in the 9740 (old) syllabus, but not in the
9758 (revised) syllabus. So you can skip this section if youre taking 9758.

Given 3 planes P1 , P2 , and P3 , we can form 3 pairs of planes:


P1 and P2 ;
P1 and P3 ; and
P2 and P3 .
To find the relationship between the 3 planes is simply to find the relationships between
each of these 3 pairs of planes. This can be insanely tedious, but there is nothing new here.
Everything follows from what you learnt in the previous sections.
Lets nonetheless give a summary of the possibilities. Given three planes, we have 3 possible
cases, each of which can be broken up into several sub-cases, for a total of 8 distinct
possibilities.

Page 368, Table of Contents

www.EconsPhDTutor.com

1. All 3 planes are parallel to each other. There are 3 possible sub-cases:
(a) All 3 planes are identical.
(b) Only 2 planes are identical.
(c) No 2 planes are identical.

P1 , P2 , P3

3 parallel,
identical planes

P3

P2
P3
P1
P1 , P2

3 parallel, nonintersecting planes

3 parallel planes,
where P1 and P2
are identical

Page 369, Table of Contents

www.EconsPhDTutor.com

2. Two planes are parallel to each other, but the 3rd plane is not parallel to either
of the first 2 planes. There are 2 possible sub-cases:
(a) The first 2 planes are identical. And so here we are really back to the situation of
two non-parallel planes, which we already covered in detail in the previous section.
They intersect along a line.
(b) The first 2 planes are not identical. And so the non-parallel plane intersects each
of the other two planes along a separate line of intersection.

P3

P1 and P2 are parallel and


identical. P3 intersects
both at the same line.
P1 , P2

P3
P2

P1 and P2 are parallel but


non-identical. P3 intersects
both at separate lines.

Page 370, Table of Contents

P1

www.EconsPhDTutor.com

3. No 2 planes are parallel. Each pair of planes intersects along a line. There are thus
three intersection lines (though possibly some may be identical). It is possible to prove
(but we wont do so in this book) that there are only 3 possible sub-cases:
(a) None of the intersection lines intersect with each other. That is, each pair of planes
simply intersects along some distinct intersection line.
(b) All 3 intersection lines are identical. So all 3 planes intersect along the same intersection line.
(c) The 3 intersection lines and thus all 3 planes intersect at a single point.
To determine which of the above sub-cases were in, we must determine the relation between
each pair of intersection lines. This is tedious, but nothing new.

P3

3 non-parallel planes that


intersect at different lines
P2

P1

3 non-parallel planes that


intersect at the same line
P3
P2

P1
P3
P1

P2

Page 371, Table of Contents

3 non-parallel planes that


intersect at only one point

www.EconsPhDTutor.com

Example 354. Consider the planes P1 , P2 , and P3 , given by r (1, 0, 0) = 0, r (0, 1, 0) = 0,


and r (0, 0, 1) = 0.
Step #1. Check if any two planes are parallel.
By observation, no planes normal vector can be written as a scalar multiple of another
planes normal vector. So no two planes are parallel. (So we are in Case 3.)
Step #2. Find the 3 intersection lines along which each pair of planes intersect.
By observation, all three planes contain the origin.
The planes P1 and P2 share an intersection line with direction vector (1, 0, 0) (0, 1, 0) =
(0, 0, 1) and so their intersection line is r = (0, 0, 0) + (0, 0, 1) ( R). Call this line l1 .
The planes P1 and P3 share an intersection line with direction vector (1, 0, 0) (0, 0, 1) =
(0, 1, 0) and so their intersection line is r = (0, 0, 0) + (0, 1, 0) ( R). Call this line l2 .
The planes P2 and P3 share an intersection line with direction vector (0, 1, 0) (0, 0, 1) =
(1, 0, 0) and so their intersection line is r = (0, 0, 0) + (1, 0, 0) ( R). Call this line l3 .
Step #3. Determine where, if at all, the 3 intersection lines intersect.
l1 and l2 are not parallel, but they do intersect at the point (0, 0, 0) and so that is also their
only intersection point.
l1 and l3 are not parallel, but they do intersect at the point (0, 0, 0) and so that is also their
only intersection point.
l2 and l3 are not parallel, but they do intersect at the point (0, 0, 0) and so that is also their
only intersection point.
Conclusion.
Altogether, we conclude that the 3 intersection lines intersect at a single point. Hence, the
3 planes also intersect at a single point. (So we are in Case 3c.)

Page 372, Table of Contents

www.EconsPhDTutor.com

Example 355. Consider the planes P1 , P2 , and P3 , given by r(1, 1, 0) = 1, r(2, 2, 0) = 4,


and r (0, 1, 1) = 1.
Step #1. Check if any two planes are parallel.
By observation, P1 s normal vector (1, 1, 0) can be written as a scalar multiple of P2 s
normal vector (2, 2, 0). And so these two planes are parallel.
P3 is not parallel to either of the first two planes, since (0, 1, 1) cannot be written as a
scalar multiple of (1, 1, 0) or (2, 2, 0).
Altogether then, we are in Case 2.
Step #2. Check if the two parallel planes are identical.
They are not, because (1, 0, 0) is on P1 but is not on P2 , as we can easily verify (1, 0, 0)
(2, 2, 0) = 2 4. So we are in Case 2b.
Step #3. Find the intersection lines. There are two one shared by P1 and
P3 and the other shared by P2 and P3 . (P1 and P2 are distinct, parallel planes
and thus do not intersect at all.)
The intersection line of P1 and P3 has direction vector (1, 1, 0) (0, 1, 1) = (1, 1, 1). Lets
1
find a point (x, y, z) at P1 and P3 intersect: the equations for the planes are x + y = 1 and
2
y + z = 1.
Using the plug in x = 0 trick, we see that they intersect at (0, 1, 0). Hence, their intersection line is r = (0, 1, 0) + (1, 1, 1) ( R). Call this line l1 .
The intersection line of P2 and P3 must also have direction vector (1, 1, 1). Lets find a
1
point (x, y, z) at P2 and P3 intersect: the equations for the planes are 2x 2y = 4 and
2
y + z = 1.
Using the plug in x = 0 trick, we see that they intersect at (0, 2, 1). Hence, their
intersection line is r = (0, 2, 1) + (1, 1, 1) ( R). Call this line l2 .
The lines l1 and l2 are parallel.

Exercise 139. What is the relationship between the 3 planes P1 , P2 , and P3 , given by
r (1, 0, 1) = 1, r (0, 1, 1) = 1, and r (1, 1, 0) = 2? (Answer on p. 1104.)

Page 373, Table of Contents

www.EconsPhDTutor.com

Part IV

Complex Numbers

Page 374, Table of Contents

www.EconsPhDTutor.com

36

Complex Numbers: Introduction

Heres a brief motivation of complex numbers:


1. Solve x 1 = 0. Easy; the answer is a natural number: x = 1.
2. To solve x + 1 = 0, we must invent negative numbers. The answer is x = 1.

3. To solve x2 = 2, we must invent irrational numbers. The answer is x = 2.

4. To solve x2 = 1, we must invent complex numbers.

Well start by defining the imaginary unit, then work our way to complex numbers.
Definition 90. The imaginary unit, denoted i, is a number that satisfies i2 = 1.
Using the imaginary unit, we can construct other purely imaginary numbers:
Definition 91. A purely imaginary number is any real, non-zero multiple of the imaginary
unit. That is, a purely imaginary number is any bi, where b R with b 0.
(We specify that b 0 because 0i = 0 is not a purely imaginary number, but a real number.)

Example 356. i + i = 2i = 2 1 is purely imaginary. So too are i = 1 and i = 1.


i is both the imaginary unit and a purely imaginary number.
We can add real numbers to purely imaginary numbers to form imaginary numbers:
Definition 92. An imaginary number is any a + bi, where a, b R with b 0.
Again, we specify that b 0 because otherwise a + 0i would not be an imaginary number,
but a real number.
Example 357. 3 + 2i is imaginary, but not purely imaginary. In contrast, 2i is both
imaginary and purely imaginary.

Page 375, Table of Contents

www.EconsPhDTutor.com

A complex number is simply any real number or imaginary number.


Definition 93. A complex number is any a + bi, where a, b R.
Notice that here in contrast, we do not specify that b 0. The reason is that complex
numbers include all real numbers.
Example 358. 10 and 17 are complex and real. 2+9i and 32i are complex and imaginary.
2i is complex, imaginary, and purely imaginary. i is complex , imaginary, purely imaginary,
and also the imaginary unit.
We denoted the set of real numbers by the symbol R. We now denote the set of complex
numbers by the symbol C.
Definition 94. The set of all complex numbers, denoted C, is defined as {a + bia, b R}.

The set of reals is a proper subset of the set of complex numbers formally,
Fact 43. R C.
Proof. Every element a R can be written as a + 0i and is thus an element of C. So R is a
subset of C. Moreover R is not equal to C, because for example 3 + 7i C but 3 + 7i R.
Altogether then, R is a proper subset of C.

Complex numbers are thus the extension of the concept of real numbers. On the next page
is a modified version of our taxonomy of numbers from p. 39, with the complex numbers
fleshed out:

Page 376, Table of Contents

www.EconsPhDTutor.com

Complex Numbers

Real Numbers

Impure Imaginary
Numbers

Imaginary Numbers

Purely Imaginary
Numbers
The Imaginary Unit

Note: there is no such thing as a positive or negative complex number. To fully


appreciate why is beyond the scope of the A-levels. But for now here is a very simple
example just to illustrate the point.
Example 359. 1, 1, i, and i are all complex numbers. We do say that 1 is a positive
real number and 1 is a negative real number.
But we do not say that i is a positive complex number or that i is a negative complex
number.
In fact, we do not even say that 1 is a positive complex number or that 1 is a negative
complex number.

Exercise 140. Fill in the following table. The first column has been done for you. (Answer
on p. 1105.)
Is this ...
13 2i
A complex number?
Yes
A real number?
No
An imaginary number?
Yes
A purely imaginary number?
No
The imaginary unit?
No

Page 377, Table of Contents

3i 0 4 4 + 2i i
3

www.EconsPhDTutor.com

36.1

The Real and Imaginary Parts of Complex Numbers

Definition 95. Given a complex number z = a + bi, its real part is a and is denoted Re(z).
Similarly, its imaginary part is b and is denoted Im(z).

Example 360. Re(3 + 2i) = 3 and Im(3 + 2i) = 2.


Example 361. Re(7) = 7 and Im(7) = 0.
Example 362. Re(19i) = 0 and Im(19i) = 19.
It is also often convenient to write complex numbers in ordered pair notation, with the first
term being the real part and the second term being the imaginary.
Example 363. Given z = 3 + 2i, we can also write z = (3, 2).
Example 364. Given z = 7, we can also write z = (7, 0).
Example 365. Given z = 19i, we can also write z = (0, 19).
Of course, two complex numbers z and w are equal if and only if (i) their real parts are
equal; AND (ii) their imaginary parts are equal.
Example 366. Suppose z = 3 + bi and w = a 17i are equal. Then it must be that a = 3
and b = 17.

Exercise 141. Exactly two of the following complex numbers are identical. Find out which
two. (Answer on p. 1105.)

1
2
a=
i,
2
2

Page 378, Table of Contents

3
1
b = i,
2
2

c = sin sin i,
3
3

d=

cos ( ) i.
2
4

www.EconsPhDTutor.com

37

Basic Arithmetic of Complex Numbers

The familiar arithmetic operations work the same way on imaginary numbers as they do
on real numbers. Addition and subtraction are especially simple.

37.1

Addition and Subtraction

Example 367. Let z = 2 + i and w = 3i. Then z + w = 2 + 4i and z w = 2 2i.


We can also write z = (2, 1) and w = (0, 3), so that z + w = (2 + 0, 1 + 3) = (2, 4) and
z w = (2 0, 1 3) = (2, 2).
Example 368. Let z = 7 i and w = 2 + 5i. Then z + w = 9 + 4i and z w = 5 6i.
We can also write z = (7, 1) and w = (2, 5), so that z + w = (7 + 2, 1 + 5) = (7 + 2, 1 + 5)
and z w = (7 2, 1 5) = (5, 6).

In general,
Fact 44. If z = (a, b) and w = (c, d), then
z + w = (a + c, b + d),
z w = (a c, b d).

Exercise 142. For each of the following, compute


z +w and z w. (a) z = 5+2i, w = 7+3i.
(b) z = 3 i, w = 11 + 2i. (c) z = 1 + 2i, w = 3 2i. (Answer on p. 1106.)

Page 379, Table of Contents

www.EconsPhDTutor.com

37.2

Multiplication

Below are listed the powers of i. Note that the cycle repeats after every fourth power,
because i4 = 1.
i = i,

i2 = i i = 1,

i3 = i i2 = i,

i4 = i i3 = 1,

i5 = i i4 = i,

i6 = i i = 1,

i7 = i i2 = i,

i8 = i i3 = 1,

i9 = i i8 = i,

i10 = i i = 1,

i11 = i i2 = i,

i12 = i i3 = 1,

etc.
Example 369. Let z = i and w = 1 + i. Then zw = i (1 + i) = i 1 + i i = i 1.
Example 370. Let z = 2 + i and w = 3i. Then
zw = (2 + i) (3i) = (2) (3i) + i (3i)
= 6i + 3i2 = 6i + 3(1) = 3 6i.
Example 371. Let z = 2 i and w = 1 + i. Then
zw = (2 i)(1 + i) = 2 + 2i + i i2
= 2 + 3i i2 = 2 + 3i (1) = 1 + 3i.
Example 372. Let z = 3 + 2i and w = 7 + 4i. Then
zw = (3 + 2i)(7 + 4i) = 21 + 12i 14i + (2i)(4i)
= 21 2i + 8i2 = 21 2i + 8 (1) = 29 2i.

In general,
Fact 45. If z = (a, b) and w = (c, d), then
zw = (ac bd, ad + bc) .

Page 380, Table of Contents

www.EconsPhDTutor.com

Exercise 143. Prove Fact 45. (Answer on p. 1106.)

Exercise 144. For each of the following,


compute zw. (a) z = 5 + 2i, w = 7 + 3i. (b)
z = 3 i, w = 11 + 2i. (c) z = 1 + 2i, w = 3 2i. (Answer on p. 1106.)
Exercise 145. Given that z = 2 + i and az 3 + bz 2 + 3z 1 = 0, find a and b. (Answer on p.
1106.)

Page 381, Table of Contents

www.EconsPhDTutor.com

37.3

Division

Recall that to rationalise a surd in the denominator (section 5.2), we used a trick involving
conjugate pairs.
Example 373.

(
(1
3
3

5)
5 1)
3
3
1 5
=

=
=
.
15
4
1+ 5 1+ 5 1 5

We called a + b and a b a conjugate pair because (a + b)(a b) = a2 b2 . If b is the square


root of some number, then this is a rationalisation (make rational) that helps get rid
of an ugly surd.
Now, given z = a + bi, we call z = a bi its conjugate. And we call a + bi and a bi a
conjugate pair, because
(a + bi)(a bi) = a2 (bi) 2 = a2 b2 i2 = a2 + b2 .
This is a realisation (make real) that helps get rid of any complex numbers. Example:
Example 374. (1 + i) = 1 i, i = i, and (1 i) = 1 + i. Thus:
(a)

1i
1
1
1i
1i
=

= 2 2=
= 0.5 0.5i.
1+i 1+i 1i 1 i
1+1
1 1 i i i
=
=
=
= i.
i i i i2 1

(b)

(c)

1
1
1+i
1+i
1+i
=

= 2 2=
= 0.5 + 0.5i.
1i 1i 1+i 1 i
1+1

In general,
Fact 46. If z = (a, b), then

z = (a, b),

zz = a + b ,

1 1 z
z
a
b
= = 2 2 = ( 2 2, 2 2).
z z z
a +b
a +b a +b

Exercise 146. For each of the following z, write down its conjugate z and hence compute
its reciprocal (i.e. 1/z). (a) z = 5 + 2i. (b) z = 3 i. (c) z = 1 + 2i. (Answer on p. 1106.)

Page 382, Table of Contents

www.EconsPhDTutor.com

We now divide one complex number by another.


Example 375.
(a)

(b)

2 + i 2 + i 3i 6i 3i2 6i + 3
=

=
=
.
3i
3i
3i
9i2
9
3 + i 3 + i 1 + i (3 + i)(1 + i) 3 + 3i + i + i2 2 + 4i
=

=
=
=
= 1 + 2i.
1i 1i 1+i
12 i2
1+1
2

(c)

1+i
1 + i 3 + 2i 3 + 2i + 3i + 2i2 1 + 5i
=

=
=
.
3 2i 3 2i 3 + 2i
9+4
13

(d)

2 i 1 i 2 2i + i + i2 3 i
2i
=

=
=
= 1.5 0.5i.
1 + i 1 + i 1 i
1+1
2

(e)

3 + 2i
3 + 2i 7 4i 21 12i 14i 8i2 13 26i
=

=
=
= 0.2 0.4i.
7 + 4i 7 + 4i 7 4i
49 + 16
65

(f)

3 + 6i 3 + 6i 2 i 6 + 3i + 12i 6i2 6 6 + (3 + 12)i


=

=
=
.
2 + i
2 + i 2 i
22 2 i 2
4 + 2

In general,
Fact 47. If z = (a, b) and w = (c, d) with w 0, then
z z w
zw
ac + bd bc ad
= = 2
=
(
,
).
w w w
c + d2
c2 + d2 c2 + d2

Exercise 147. Rewrite each of the following fractions into the form a + bi. (a)

2 3i
11 + 2i
2 i
3
7 2i
. (d)
. (c)
. (e)
. (f)
. (Answer on p. 1107.)
1+i
i
2+i
5+i
3 2i

Page 383, Table of Contents

1 + 3i
. (b)
i

www.EconsPhDTutor.com

38
38.1

Solving Polynomial Equations


Complex Roots to Quadratic Equations

In section 14 (quadratic equations review), we saw that if ax2 + bx + c = 0 has non-negative


discriminant (i.e. b2 4ac 0), then its real roots are given by
x=

b2 4ac
.
2a

Example 376. Consider the equation x2 3x + 2 = 0. Its discriminant is positive: b2 4ac =


(3)2 4(1)(2) = 1 > 0. Hence, it has two real roots, given by
x=

b2 4ac 3 1
=
= 1, 2.
2a
2

Now, armed with our new concept of imaginary numbers, we can completely dispense with
the requirement that b2 4ac 0. We can simply say that ax2 + bx + c = 0 ALWAYS has
complex roots, given by
x=

b2 4ac
.
2a

Example 377. Consider the equation x2 2x+2 = 0. Its discriminant is negative: b2 4ac =
(2)2 4(1)(2) = 4 < 0. It has two imaginary (and thus also complex) roots, given by
x=


b2 4ac 2 4
4 1
2i
=
=1
= 1 = 1 i.
2a
2
2
2

Notice that 1 + i was a root to the given quadratic equation. And interestingly enough, so
too was 1 i.
It turns out that in general, a quadratic equation with real coefficients has roots that come
in conjugate pairs. That is, if x + yi is a root, then so too is its conjugate x yi. 40 More
examples:

40

This is not terribly surprising if you examine the general solution for the quadratic equation the b2 4ac bit corresponds precisely to the imaginary part.

Page 384, Table of Contents

www.EconsPhDTutor.com

Example 378. Consider the equation 3x2 +x+1 = 0. Its discriminant is negative: b2 4ac =
(1)2 4(3)(1) = 11 < 0. It has two imaginary (and thus also complex) roots, given by
x=

b2 4ac 1 11
11 1
11
1
1
=
=
=
i.
2a
6
6
6
6
6

Example 379. If 3 + 2i is a root to the quadratic equation x2 + bx + c = 0 (where b and c are


both real), then what are b and c? Well, we know that 3 2i is also a root to the equation.
And so
x2 + bx + c = [x (3 + 2i)] [x (3 2i)] = (x 3)2 (2i)2 = x2 6x + 13.
Hence, b = 6 and c = 13.

Exercise 148. Find the roots for each of the following quadratic equations. (a) x2 +x+1 = 0.
(b) x2 + 2x + 2 = 0. (c) 3x2 + 3x + 1 = 0. (Answer on p. 1108.)
Exercise 149. If 1 i is a root to the quadratic equation x2 + bx + c = 0 (where b and c are
both real), then what are b and c? (Answer on p. 1108.)

Page 385, Table of Contents

www.EconsPhDTutor.com

38.2

The Fundamental Theorem of Algebra

Recall from p. 47 that a polynomial of degree n in one variable is any expression


a0 xn + a1 xn1 + a2 xn2 + + an1 x + an where each ai is a constant and x is the variable.
Theorem 4. The Fundamental Theorem of Algebra. A polynomial of degree n in
one variable has exactly n zeros (though some may be repeated). That is, there are exactly
n (possibly repeated) solutions to the equation a0 xn + a1 xn1 + a2 xn2 + + an1 x + an = 0.
Proof. The proof of this theorem is way too advanced and so omitted from this book.41

Example 380. x2 1 is a polynomial of degree 2. And indeed, x2 1 = 0 has two solutions,


namely 1 and 1.
Example 381. x2 + 1 is a polynomial of degree 2. And indeed, x2 + 1 = 0 has two solutions,
namely i and i.
There are sometimes repeated solutions or what are more formally called multiple roots,
as the next example illustrates.
Example 382. x2 2x + 1 = 0 has two (repeated) solutions, namely 1 and 1. We call 1 a
multiple root (indeed a double root).

Example 383. x3 6x2 + 12x 8 = 0 has three (repeated) solutions, namely 2, 2, and 2.
We call 2 a multiple root (indeed a triple root).

41

But see this MathOverflow Q&A if youre interested.

Page 386, Table of Contents

www.EconsPhDTutor.com

The Fundamental Theorem of Algebra can be useful even if we have no idea how to find
the solutions to an equation.
Example 384. x17 + 3x4 2x + 1 is a polynomial of degree 17. I may not know what the
solutions to x17 + 3x4 2x + 1 = 0 are, but I know from the Fundamental Theorem of Algebra
that there MUST be 17 solutions (though some may possibly be repeated).

Example 385. x4 + x3 5x2 + x 6 is a polynomial of degree 4 and so it must have four


zeros. Suppose we are given as a hint that two of them are i and i. Then how would we
go about finding the other two zeros?
The problem of finding the zeros of a polynomial is really the same as the problem of
factorising a polynomial. This is because a is a zero of a polynomial if and only if (x a)
is a factor of the polynomial.
So (x i) and (x + i) are factors for the polynomial. Now, (x i)(x + i) = x2 i2 = x2 + 1.
So find (x4 + x3 5x2 + x 6) (x2 + 1) through long division:
x2 +x
6
x2 + 1 x4 +x3 5x2
x4 +0 +x2
x3 6x2
x3
+0
6x2
6x2

+x 6

+x
+0 6
+0 6
0.

Hence, (x4 + x3 5x2 + x 6) = (x2 + 1) (x2 + x 6).


By observation, x2 + x 6 = (x 2)(x + 3). Hence,
(x4 + x3 5x2 + x 6) = (x2 + 1) (x2 + x 6) = (x i)(x + i)(x 2)(x + 3).
Altogether, the four zeros of the given polynomial are i, 2, and 3.

Page 387, Table of Contents

www.EconsPhDTutor.com

Example 386. x3 3x2 5x 25 is a polynomial of degree 3 and so it must have three


zeros. As a hint, we are told that one of them is 5. What are the other two?
x2 +2x
+5
x 5 x3 3x2 5x 25
x3 5x2
2x2
2x2 10x
5x 25
5x 25
0.
So x3 3x2 5x 25i = (x 3) (x2 + 2x + 5). Im unable to easily see how x2 + 2x + 5 can be
factorised. So let me just use the quadratic formula:

x=

22 4(1)(5)
= 1 1 5 = 1 4 = 1 2i.
2

Altogether then,
x3 3x2 5x 25i = (x 5) (x2 + 2x + 5) = (x 5) [x (1 + 2i)] [x (1 2i)] .
So the three zeros of the polynomial are 5 and 1 2i.

Exercise 150. Each of the following polynomials has 1 as a zero. Find the other zeros.
(a) x3 + x2 2. (b) x4 x2 2x + 2. (Answer on p. 1109.)

Page 388, Table of Contents

www.EconsPhDTutor.com

38.3

The Complex Conjugate Roots Theorem

We saw above that if c + di is a root to a quadratic equation ax2 + bx + c = 0 (where a, b,


and c are real), then so too is its conjugate c di.
What is perhaps surprising is that this generalises to the case of any polynomial, provided
that all coefficients of the polynomial are real.
Example 387. If told that 2 i solves x3 x2 7x + 15 = 0, we know immediately that its
conjugate 2 + i also solves the same equation.
Example 388. If told that i solves 4x4 +5x2 +1 = 0, we know immediately that its conjugate
i also solves the same equation. Similarly, if told also that 0.5i solves the same equation,
we know immediately that its conjugate 0.5i also solves the same equation.

Theorem 5. (Complex Conjugate Roots Theorem.) Let a0 , a1 , . . . , ak be real. If


a + bi solves an xn + an1 xn1 + an2 xn2 + + a1 x + a0 = 0, then so does a bi.

Proof. Optional, see p. 944 in Appendices.


The condition that all coefficients ak are real is important. The above theorem
does not apply if any of the coefficients are imaginary.
Example 389. i solves x2 + ix + 2 = 0. However, its conjugate i does not solve the same
equation (verify this yourself!).

2
2
2
2
Example 390.
+
i solves x2 = i. However, its conjugate

i does not solve


2
2
2
2
the same equation (verify this yourself!).

Example 391. 2 + i solves x3 (i + 2)x2 + 2x 2(2 + i) = 0. However, its conjugate 2 i


does not solve the same equation (verify this yourself!).

Exercise 151. Each of the following polynomials has 2 3i as a zero. Find the other zeros.
(a) x4 6x3 + 18x2 14x 39. (b) 2x4 + 21x3 93x2 + 229x 195. (Answer on p. 1110.)

Page 389, Table of Contents

www.EconsPhDTutor.com

39

The Argand Diagram

The complex plane (or Argand diagram) gives us a nice geometric interpretation: The
complex numbers are simply points on the plane. The real axis is the horizontal
or x-axis. The imaginary axis is the vertical or y-axis.
Example 392. In the figure below, marked in red are the real numbers 3, 0, , and 2,
which may be written in ordered pair notation as (3, 0), (0, 0), (, 0), (2, 0). Points on
the horizontal axis are real numbers.
In blue are the purely imaginary numbers 4i and 3i, which may be written in ordered pair
notation as (0, 4) and (0, 3). Points on the vertical axis are purely imaginary numbers.
In green are the impure imaginary numbers 1 + i, 3 + 2i, 1 3i, and 4 i, which may
be written in ordered pair notation as (1, 1), (3, 2), (1, 3), and (4, 1). Points not on
either axis are impure imaginary numbers.

4
3
2
1
x
0
-5

-4

-3

-2

-1

-1
-2
-3
-4
-5

Page 390, Table of Contents

www.EconsPhDTutor.com

For our purposes, well regard the complex plane C as being exactly identical to the cartesian
plane {(x, y) x R, y R}. Both are represented graphically as a two-dimensional plane.
The only difference is that we interpret points on each plane differently: Points on the
complex plane are complex numbers, while points on the cartesian plane are ordered pairs
of real numbers.42

Exercise 152. Illustrate the complex numbers 1, 3, 2i, 1 + 2i, and 1 3i on a single
Argand diagram. (Answer on p. 1111.)

42

The differences between C and R2 in fact run deeper. See e.g. this discussion..

Page 391, Table of Contents

www.EconsPhDTutor.com

39.1

Complex Numbers in Polar Form

To write a complex number in standard form i.e. z = x + iy, we need only two pieces
of information: its real part (x) and its imaginary part (y).
We now write a complex number in polar form. Again, we need only two pieces of
information: the modulus, denoted z, and the argument, denoted arg z. Informally, the
modulus is the length of the position vector of z; the argument is the angle the position
vector of z makes with the positive x-axis.
Example 393. The complex number 3 = (3, 0) has modulus 3 = 3 and argument
arg 3 = . The complex number 4i = (0, 4) has modulus 4i = 4 and argument
arg
(4i) =

2
2
/2. The complex number 3 + 3i = (3, 3), has modulus 3 + 3i = 3 + 3 = 3 2 and
argument arg(3 + 3i) = /4.

4
3 + 3i = (3, 3)
3
2
1
-3 = (-3, 0)

x
0

-5

-4

-3

-2

-1

-1
-2
-3
-4

-4i = (0, -4)

-5

Page 392, Table of Contents

www.EconsPhDTutor.com

The formal definition of the modulus function is simple.


Definition
96. The modulus function has domain C, codomain R, and mapping rule z

x2 + y 2 . The modulus of z is denoted z.

In contrast, it is tricky to write down a formal definition of the argument function. One
problem is this: Angles are periodic.
Example 394. Consider again the complex number 3 + 3i = (3, 3). The angle it makes
with the positive x-axis is /4.
But angles are periodic. Equivalently, angles come full circle 2 radians. So it would make
just as much sense to say that the angle is 9/4. Or 17/4. Or 7/4. Or indeed any
/4 + 2k, where k is any inteer.
To overcome this problem, we shall somewhat arbitrarily choose (, ] as our principal
values. Thus, arg(3 + 3i) shall be uniquely defined to be the value /4 and nothing else.
Another problem is this: We are tempted to simply define arg(x + yi) = tan1 (y/x). Unfortunately, the tan1 function has codomain ge (/2, /2). Whereas, as we just decided,
arg should have codomain (, ]. To overcome this, altogether, the argument function is
defined as follows:
Definition 97. The argument function has domain C, codomain (, ], and mapping
rule as given below:

tan1 (y/x) ,

Undefined,

/2,
arg z =

/2,

tan1 (y/x) + ,

tan (y/x) ,

if x > 0 (top-right and bottom-right quadrants),


if x = 0 = y (the origin),
if x = 0, y > 0 (the positive y axis),
if x = 0, y < 0, (the negative y axis)
if x < 0, y 0 (top-left quadrant, including the negative x-axis),
if x < 0, y < 0 (bottom-left quadrant).

The argument of z is denoted arg z.


y
(My mnemonic for the above: arg z = tan1 . Top left +. Bottom left.)
x
We now illustrate and explain the above definition:

Page 393, Table of Contents

www.EconsPhDTutor.com

y
arg z =

arg z =
x
arg z =

If x > 0 (top-right and bottom-right quadrants), then define arg(x + yi) = tan1 (y/x).
The green point in the figure above illustrates. The angle that the position vector of the
green (x, y) makes with the positive x-axis is indeed simply tan1 (y/x).
If x = 0, y = 0 (the origin), then arg(x + yi) is undefined. In other words, we leave arg 0
undefined.43
If x = 0, y > 0 (positive vertical axis), then define arg(x + yi) = arg(yi) = /2.
If x = 0, y < 0 (negative vertical axis), then define arg(x + yi) = arg(yi) = /2.
If x < 0, y 0 (top-left quadrant plus the negative horizontal axis), then define arg(x +
yi) = tan1 (y/x) + .
The red point illustrates. The angle its position vector makes with the negative x-axis
is tan1 (y/x). And so arg(x + yi) = tan1 (y/x). Observe that tan1 (y/x) =
tan1 (y/ x) = tan1 (y/x). Thus, arg(x + yi) = tan1 (y/x) = tan1 (y/x) + .
If x < 0, y < 0 (bottom-left quadrant), then define arg(x + yi) = tan1 (y/x) .
The blue point illustrates. The angle its position vector makes with the negative x-axis is
tan1 (y/x). And so arg(x + yi) = tan1 (y/x) . Observe that (y/x) = (y/ x) =
(y/x). Thus, arg(x + yi) = tan1 (y/x) = tan1 (y/x) .
43

Some writers define arg 0 = 0, but we shall not do this.

Page 394, Table of Contents

www.EconsPhDTutor.com

The following fact is sometimes useful.


Fact 48. (a) z is purely imaginary (z is on the vertical axis) arg z = /2.
(a) z is real (z is on the horizontal axis) arg z = 0, .

Proof. Immediate from the definition of the arg function.

Exercise 153. Compute the modulus and argument of 4, 3, 2i, 1+2i, and 13i. Illustrate
these numbers and their arguments on a single Argand diagram. (Answer on p. 1112.)

Exercise 154. Where on the complex plane must a complex number be, if its argument is

... (a) Positive? (b) Negative? (c) 0? (d) ? (e) ? (f) > ? (g) < ? (Answer on p.
2
2
2
2
1113.)

Page 395, Table of Contents

www.EconsPhDTutor.com

Armed with the modulus and the argument, we have a nice geometric interpretation:
Fact 49. Let z be a complex number with z = r and arg z = . Then z = r (cos + i sin ).
We call r (cos + i sin ) the polar form representation of z.

y
z = r (cos + i sin ) = (r cos , r sin )

r
r sin
x
r cos

2
2

Example 395. For z = 5 2i = (5, 2), z = 52 + (2) = 29 and arg z = tan1


5

0.381. So in polar form, z = 29 [cos (0.381) + i sin (0.381)].

3
Example 396. For z = 1 + 3i = (1, 3), z = 12 + 32 = 10 and arg z tan1 1.249. So in
1

polar form, z = 10 [cos (1.249) + i sin (1.249)].

7
Example 397. For z = 4 + 7i = (4, 7), z = (4)2 + 72 = 65 and arg z = tan1
+
4

2.090. So in polar form, z = 65 [cos (2.090) + i sin (2.090)].

Exercise 155. Rewrite each of the following complex numbers in polar form: 1, 3, 2i,
1 + 2i, and 1 3i. (Answer on p. 1113.)

Page 396, Table of Contents

www.EconsPhDTutor.com

39.2

Complex Numbers in Exponential Form

Theorem 6. Euler Formula. For any R, ei = cos + i sin .

Proof. Optional, see p. 944 in the Appendices.

Fact 50. Let z be a complex number with z = r and arg z = . Then z = rei .

Proof. z = r and arg z = z = r(cos + i sin ) z = rei , where uses Fact


2
49 and uses the Euler Formula.
We call rei the exponential form representation of z.
Eulers identity is one of the most extraordinary and beautiful equations in all of mathematics. It links together five fundamental mathematical constants: e, i, , 1, and 0.
Corollary 4. (Eulers identity.) ei + 1 = 0.

Proof. By Theorem 6, ei = cos + i sin = 1 + 0 = 1. Hence, ei + 1 = 0.

2
Example 398. The number z = 5 2i = (5, 2) has modulus 52 + (2) = 29 and

argument tan1 (2/5) 0.381. Hence, we can also write z = 29ei(0.381) .

2 + 32 =
Example 399. The number z = 1 + 3i = (1, 3) has
modulus
1
10 and argument
i(1.249)
1
tan (3/1) 1.249. Hence, we can also write z = 10e
.

Example 400. The number z = 4 + 7i = (4, 7) has modulus (4)2 + 72 = 65 and


argument tan1 (7/ 4) + 2.090. Hence, we can also write z = 65ei(2.090) .

Exercise 156. Rewrite each of the following complex numbers in exponential form: 1, 3,
2i, 1 + 2i, and 1 3i. (Answer on p. 1113.)

Page 397, Table of Contents

www.EconsPhDTutor.com

40

More Arithmetic of Complex Numbers

Now that we know how to write complex numbers in polar and exponential forms, the
arithmetic of complex numbers becomes even easier.

40.1

The Product of Two Complex Numbers

Fact 51. Product of two complex numbers. Let z and w be complex numbers. Then
zw = z w ,

and

arg (zw) = arg z + arg w + 2k,

(where k = 1, 0, 1 ensures that arg z + arg w + 2k (, ]).


Proof. Let z = r(cos + i sin ) and w = s(cos + i sin ). Then
zw = rs(cos + i sin )(cos + i sin )
= rs(cos cos + i sin cos + i cos sin sin sin )
= rs [cos ( + ) + i sin ( + )] .
This is the complex number with modulus rs and which makes an angle + with the
positive x-axis.
Note though that + may not be in (, ]. Thus, rather than say that arg(zw) =
arg z + arg w, we instead say that arg (zw) = arg z + arg w + 2k (where k = 1, 0, 1 ensures
that arg z + arg w + 2k (, ]).

Here is an alternative quicker proof of the above fact, using the exponential form.
Proof. Let z = rei and w = sei . Then zw = rsei(+) . This is the complex number with
modulus rs and which makes an angle + with the positive x-axis.

Page 398, Table of Contents

www.EconsPhDTutor.com

Example 401. Let z = 5 2i and w = 1 + 3i. Then

2
z = 52 + (2) = 29, and arg z = tan1 (2/5),

w = 12 + 32 = 10,
and arg w = tan1 (3/1),

zw = 29 10 = 290, and
arg (zw) = tan1 (2/5) + tan1 (3/1) + 2k 0.869 + 2k = 0.869 (k = 0).
Notice that here arg z + arg w 0.869 (, ]. So arg z + arg w is already a principal
value and we can simply set k = 0 or arg(zw) = arg z + arg w.

So zw 290 (cos 0.869 + i sin 0.869) = 290ei(0.869) .


To get zw in standard form, use a calculator: Youll get

2
3
290 cos [tan1
+ tan1 ] = 11,
5
1

and

2
3
290 sin [tan1
+ tan1 ] = 13.
5
1

And indeed zw = (5 2i)(1 + 3i) = 11 + 13i.

Example 402. Let z = 4 + 7i and w = 1 6i. Then


z =

(4)2 + 72 =

65, and

arg z = tan1 [7/ (4)] + ,

w = 12 + (6)2 = 37, and arg w = tan1 (6/1),

zw = 65 37 = 2405, and
arg (zw) = tan1 [7/ (4)] + + tan1 (6/1) + 2k 0.684 + 2k = 0.684 (k = 0).
Notice that here arg z + arg w 0.684 (, ]. So arg z + arg w is already a principal
value and we can simply set k = 0 or arg(zw) = arg z + arg w.

So zw 2405 (cos 0.684 + i sin 0.684) = 2405ei(0.684) .


To get zw in standard form, use a calculator: Youll get

7
6
2405 cos [tan1
+ + tan1 ] = 38,
4
1

and

7
6
2405 sin [tan1
+ + tan1 ] = 31.
4
1

And indeed zw = (4 + 7i)(1 6i) = 38 + 31i.


Page 399, Table of Contents

www.EconsPhDTutor.com

Example 403. Let z = 3 + 4i and w = 5 + 2i. Then

(3)2 + 42 = 5,
and arg z = tan1 [4/ (3)] + ,

w = (5)2 + 22 = 29, and arg w = tan1 [2/ (5)] + .


z =

zw = 5 29,
arg (zw) = (tan1

and
2
4
+ ) + (tan1
+ ) + 2k 4.975 + 2k = 1.308 (k = 1).
3
5

Notice that here arg z + arg w 4.975 (, ]. So we need to set k = 1 to get


arg(zw) = arg z + arg w 2k 1.308 (, ], so that arg(zw) is indeed a principal
value.

So zw 5 29 [cos (1.308) + i sin (1.308)] = 5 29ei(1.308) .


To get zw in standard form, use a calculator: You use a calculator, youll get

2
2
4
4
5 29 cos [tan1
+ + tan1 ] = 7 and 5 29 sin [tan1
+ + tan1 ] 26.
3
5
3
5
And indeed zw = (3 + 4i)(5 + 2i) = 7 26i.

Exercise 157. Write down zw in polar and exponential forms, for each of the following
pair of z and w. (a) z = 1, w = 3. (b) z = 2i, w = 1 + 2i. (c) z = 1 3i, w = 3 + 4i. (Answer
on p. 1114.)

Page 400, Table of Contents

www.EconsPhDTutor.com

40.2

The Ratio of Two Complex Numbers

Fact 52. Ratio of two complex numbers. Let z and w be complex numbers. Then
z
z
=
,
w w

and

arg

z
= arg z arg w + 2k,
w

(where k = 1, 0, 1 ensures that arg z + arg w + 2k (, ]).


Proof. Let z = r(cos + i sin ) and w = s(cos + i sin ). Then
z r(cos + i sin ) r cos + i sin cos i sin
=
=

w s(cos + i sin ) s cos + i sin cos i sin


=

r cos cos + sin sin + i sin cos i cos sin


s
cos2 + sin2

r cos cos + sin sin + i sin cos i cos sin


s
1

r
[cos ( ) + i sin ( )] .
s

This is the complex number with modulus r/s and argument + 2k (where k is the
unique integer such that + 2k (, ]).

Here is an alternative quicker proof of the above fact, using the exponential form.
Proof. Let z = rei and w = sei . Then z/w = ei() (r/s). This is the complex number
with modulus r/s and argument + 2k (where k is the unique integer such that
+ 2k (, ]).

Ill recycle the examples used in the previous section.

Page 401, Table of Contents

www.EconsPhDTutor.com

Example 404. Let z = 5 2i and w = 1 + 3i. Then


z =

29,

arg z = tan1

z
= 2.9,
w

zw

2
,
5

w =

3
arg w = tan1 .
1

10,

2
3
z
tan1 + 2k 1.630 (k = 0).
arg ( ) = tan1
w
5
1

2.9 [cos (1.630) + i sin (1.630)] = 2.9ei(1.630) .

Example 405. Let z = 4 + 7i and w = 1 6i. Then


z =

z
=
w

65,

arg z = tan1

7
+ ,
4

w =

37,

arg w = tan1

6
.
1

z
65
7
6
, arg ( ) = tan1 +tan1 +2k 3.496+2k 2.788 (k = 1).
37
w
4
1

z
65
65 i(2.788)

[cos (2.788) + i sin (2.788)] =


e
.

w
37
37

Example 406. Let z = 3 + 4i and w = 5 + 2i. Then


z = 5,

z
5
= ,
w
29

arg z = tan1

4
+ ,
3

w =

29,

arg w = tan1

2
+ .
5

z
4
2
arg ( ) = tan1
tan1
+ 2k 0.547 + 2k = 0.547 (k = 0).
w
3
5
z
5
5
[cos (0.547) + i sin (0.547)] = ei(0.547) .
w
29
29

z
in polar and exponential forms, for each of the following
w
pairs of z and w. (a) z = 1, w = 3. (b) z = 2i, w = 1 + 2i. (c) z = 1 3i, w = 3 + 4i. (Answer
on p. 1114.)
Exercise 158. Write down

Page 402, Table of Contents

www.EconsPhDTutor.com

40.3

Sine and Cosine as Weighted Sums of the Exponential

Fact 53 expresses the sine and cosine functions as weighted sums of the exponential functions. It is not in the syllabus, but made a sudden first-time appearance on the 2015 A-level
exams (Exercise 356), just to screw students over.

Fact 53. cos =

ei + ei
ei ei
and sin =
.
2
2i

Proof. By the Euler Formula, ei = cos +i sin . Moreover, ei = cos ()+i sin () = cos
i sin , where the second equality uses the properties cos x = cos(x) and sin(x) = sin x.
Hence,

ei + ei cos + i sin + cos i sin


=
= cos , as desired.
2
2

ei ei cos + i sin cos + i sin


=
= sin , also as desired.
Similarly,
2i
2

The 2015 question was about the sum (or difference) of two complex numbers that
have the same modulus. Heres a similar example:

Page 403, Table of Contents

www.EconsPhDTutor.com

Example 407. Let z = 5ei and w = 5e0.4i . What, exactly, are the modulus and arguments
of z + w and z w?
Without the above fact, its not obvious. With the above fact, its easy. First, observe that
0.7 is the average of 1 and 0.4. Then factorise 5ei + 5e0.4i into a form where we can exploit
the above fact. Like so:
z + w = 5ei + 5e0.4i = 5e0.7i (e0.3i + e0.3i ) = 5e0.7i 2 cos(0.3),
where the last = uses Fact 53. And thus:
arg (z + w) = arg [5e0.7i 2 cos(0.3)]
= arg 5 + arg (e0.7i ) + arg 2 + arg [cos(0.3)] + 2k
= 0 + 0.7 + 0 + 0 + 2k = 0.7 (k = 0).
z + w = 5e0.7i 2 cos(0.3) = 5 e0.7i 2 cos(0.3)
= 5 1 2 cos(0.3) = 10 cos(0.3).
Altogether then, z + w = 10 cos(0.3)ei(0.7) .
We can play a similar trick to figure out the modulus and argument of z w:
z w = 5ei 5e0.4i = 5e0.7i (e0.3i e0.3i ) = 5e0.7i 2i sin(0.3).
where again the last = uses Fact 53. And thus:
arg (z w) = arg [5e0.7i 2i sin(0.3)]
= arg 5 + arg (e0.7i ) + arg 2 + arg i + arg [sin(0.3)] + 2k
= 0 + 0.7 + 0 + 0.5 + 0 + 2k = 0.8 (k = 1),
z w = 5e0.7i 2i sin(0.3) = 5 e0.7i 2 i sin(0.3)
= 5 1 2 1 sin(0.3) = 10 sin(0.3).
Altogether then, z w = 10 sin(0.3)ei(0.8) .

Page 404, Table of Contents

www.EconsPhDTutor.com

In general,
Fact 54.
ei + ei = 2 cos

i( + +2k)
e 2
2

and

ei ei = 2 sin

i( ++ +2m)
e 2
,
2

where k, m = 1, 0, 1 ensure that 0.5( + ) + 2k (, ] and 0.5( + + ) + 2m (, ].


Proof. See Exercise 160.

Exercise 159. Let z = 3ei(0.2) and w = 3ei(0.9) . By mimicking the steps in Example 407,
find z + w and z w in exact polar and exponential forms. (Answer on p. 1115.)

Exercise 160. Prove Fact 54. (Answer on p. 1116.)

SYLLABUS ALERT
If youre taking the 9758 (revised) exam, you are done with Part IV: Complex Numbers.
The remaining chapters in Part IV covers the following, which are on the 9740 (old) syllabus
but not on the 9758 (revised) syllabus:
geometrical effects of conjugating a complex number and of adding, subtracting, multiplying, dividing two complex numbers
loci such as z c r, z a z b and arg (z a) =
use of de Moivres theorem to find the powers and nth roots of a complex number.

Page 405, Table of Contents

www.EconsPhDTutor.com

41

Geometry of Complex Numbers

In secondary school, we learnt to do some geometry using cartesian equations. And in Part
III (Vectors), we learnt to do some geometry using vector equations. Now, well learn to
do some geometry using complex equations!

41.1

The Sum and Difference of Two Complex Numbers

Given two complex numbers z = x + iy and w = a + ib, their sum is simply the complex
number z + w = (x + a) + (y + b)i.
We already know how to interpret z = (x, y) and w = (a, b) as points on the plane. This
gives us a nice geometric interpretation: z +w = (x+a, y +b) is likewise a point on the plane.
We can also interpret z = (x, y) and w = (a, b) as position vectors. And thus as usual, the
sum of two vectors is itself a vector: z + w = (x + a, y + b).

z + w = (x + a, y + b)
z = (x, y)

z+w

w = (a, b)

Page 406, Table of Contents

www.EconsPhDTutor.com

Similarly, their difference z w is simply the point (x a, y b). This corresponds to the
vector z w = (x a)i + (y b)j.

z = (x, y)
z - w = (x - a, y - b)

z-w

w = (a, b)
x

Note that in general, z + w z + w or that z w z w. This is perhaps obvious


from the above figures and also bearing in mind Corollary 3 (the sum of the lengths of any
two sides of a triangle is always greater than the length of the third side).

Page 407, Table of Contents

www.EconsPhDTutor.com

41.2

The Product and Ratio of Two Complex Numbers

With sums and differences, there was an exact analogy to vectors. In contrast, with products
and ratios of complex numbers, there is no analogy to vectors. In particular, the
product of two complex numbers has nothing to do with the scalar product or vector
product of their position vectors.
Nonetheless, we do have nice geometric interpretations. We already know from Fact 51
that the product of two complex numbers z and w is simply the complex number zw with
1. zw = z w; and
2. arg (zw) = arg z + arg w + 2k, where k = 1, 0, 1 ensures that arg z + arg w + 2k (, ].
So geometrically, to get zw, we take z and
1. First multiply its length by a factor equal to the length of w;
2. Then rotate it anti-clockwise by the angle arg w.

y
zw = (ac - bd, ad + bc)

z = (a, b)

w = (c, d)
x

Page 408, Table of Contents

www.EconsPhDTutor.com

Similarly, we already know from Fact 51 that the ratio of complex numbers z to w is simply
the complex number z/w with
1. z/w = z / w; and
2. arg (z/w) = arg z arg w +2k, where k = 1, 0, 1 ensures that arg z arg w +2k (, ].
So geometrically, to get z/w, we take z and
1. First compress its length by a factor equal to the length of w;
2. Then rotate it anti-clockwise by the angle arg w. (Or equivalently, clockwise by the
angle arg w.)

z = (a, b)

w = (c, d)

Page 409, Table of Contents

www.EconsPhDTutor.com

41.3

Conjugating a Complex Number

If z = x + yi = (x, y), then z = x yi = (x, y). So the geometric effect of conjugating a


complex number is simply to reflect it in the horizontal axis.
Fact 55. Complex conjugate. Let z be a complex number. Then
z = z ,

and

arg (z ) = arg z + 2k,

where k = 1, 0, 1 ensures that arg z + 2k (, ].


Proof. Let z = r(cos + i sin ). Then
z = r(cos i sin ) = r [cos () i sin ()] .
This is the complex number with modulus r and angle with the positive x-axis.
So given z = r(cos + i sin ) = rei , its conjugate is simply
z = r [cos () + i sin ()] = rei() .
y
z* = (x, -y)

z = (x, y)

Page 410, Table of Contents

www.EconsPhDTutor.com

42

Loci Involving Cartesian Equations

A locus (plural: loci) is a set of points that satisfy some condition (or conditions). Weve
actually already encountered plenty of loci in Part I (Functions and Graphs), so this is
nothing new. This chapter reviews loci involving cartesian equations (and inequalities).
The goal is to prepare you for the next chapter, where we look at loci involving complex
equations (and inequalities).

42.1

Circles

Example 408. {(x, y) x2 + y 2 = 1} is the set of all points (x, y) in the cartesian plane that
satisfy the condition x2 + y 2 = 1. Graphically, this locus describes describing the unit circle
centred on the origin. (To be clear, it includes only the circumference of the circle.)

{(x, y): x2 + y2 = 1}

Page 411, Table of Contents

www.EconsPhDTutor.com

Example 409. The locus {(x, y) x2 + y 2 1} describes the entire interior of the unit circle
centred on the origin, including the circumference of the circle.

{(x, y):

x2

y2

1}

Page 412, Table of Contents

www.EconsPhDTutor.com

Example 410. The locus {(x, y) x2 + y 2 < 1} describes the entire interior of the unit circle
centred on the origin, excluding the circumference of the circle.

{(x, y):

x2

y2

< 1}

Page 413, Table of Contents

www.EconsPhDTutor.com

Example 411. The locus {(x, y) x2 + y 2 1} describes everything outside the unit circle
centred on the origin, including the circumference of the circle.

{(x, y): x2 + y2 1}
x

Page 414, Table of Contents

www.EconsPhDTutor.com

Example 412. The locus {(x, y) x2 + y 2 > 1} describes everything outside the unit circle
centred on the origin, excluding the circumference of the circle.

{(x, y): x2 + y2 > 1}


x

Exercise 161. Sketch the following loci:


(a) {(x, y) (x a)2 + (y b)2 = r2 }.
(b) {(x, y) (x a)2 + (y b)2 r2 }.
(c) {(x, y) (x a)2 + (y b)2 < r2 }.
(d)
2
2
2
2
2
2
{(x, y) (x a) + (y b) r }. (e) {(x, y) (x a) + (y b) > r }. (Answer on p.
1118.)

Page 415, Table of Contents

www.EconsPhDTutor.com

42.2

Lines

Example 413. The locus {(x, y) y = x} describes the line y = x.

{(x, y): y = x}

Page 416, Table of Contents

www.EconsPhDTutor.com

Example 414. The locus {(x, y) y x} describes the set of all points under the line
y = x, including the line itself. It contains literally half the plane, so we call this a halfplane. We can also specify that this is a closed half-plane the word closed means that
it includes also the line y = x.
Graphically, the locus {(x, y) y < x} describes the set of all points under the line y = x,
excluding the line itself. This is an open half-plane. The word open means that it
excludes the line y = x.
y

y
{(x, y): y x }

{(x, y): y < x }

Example 415. Graphically, the locus {(x, y) y x} describes the set of all points above
the line y = x, including the line itself. Again, this is a closed half-plane.
Graphically, the locus {(x, y) y > x} describes the set of all points above the line y = x,
but excluding the line itself. Again, this is an open half-plane.
y

{(x, y): y x}

Page 417, Table of Contents

{(x, y): y > x}

www.EconsPhDTutor.com

The locus of points that are equidistant to two points is simply a line.
Example 416. Let (a, b) and (c, d) be points. The locus of points that are equidistant to
(a, b) and (c, d) is the line illustrated below. This is because if you pick any point (e.g. P )
on the line, it is indeed equidistant to (a, b) and (c, d). And if you pick any point (e.g. Q)
not on the line, it must be either closer to (a, b) or closer to (c, d) in this case, Q is
closer to (a, b) than to (c, d).

Any point not on the


line must be closer to
one of the two points.
Q

y
P
Any point on the
line is equidistant
to the two points.

(a, b)
x

(c, d)

Page 418, Table of Contents

www.EconsPhDTutor.com

Let (a, b) and (c, d) be points. We now prove that the locus {(x, y) (x a, y b) =
(x c, y d)} simply describes a line:
(x a, y b) = (x c, y d)
(x a)2 + (y b)2 = (x c)2 + (y d)2
x2 2ax + a2 + y 2 2by + b2 = x2 2cx + c2 + y 2 2dy + d2
2ax + a2 2by + b2 = 2cx + c2 2dy + d2

2(d b)y + 2(c a)x + a2 + b2 (c2 + d2 ) = 0.

This last equation is of the form x + y + = 0 this is simply a line.44

Exercise 162. (a) Find the cartesian equation of the line that is equidistant to the points
(1, 4) and (5, 0).
(b) Describe in words the set {(x, y) (x 17, y 3) = (x + 2, y + 11)}. Then rewrite the
cartesian equation (x 17, y 3) = (x + 2, y + 11) into the form ay + bx + c = 0. (Answer
on p. 1120.)

44

If wed like, we can further simplify this equation. If d b 0, then it can be rewritten as

y=

c2 + d2 (a2 + b2 )
ac
x+
.
db
2(d b)

And if d b = 0, then this is a vertical line whose equation may be rewritten as

x=

Page 419, Table of Contents

c2 + d2 (a2 + b2 )
2(c a)

www.EconsPhDTutor.com

42.3

Intersection of Lines and Circles

Example 417. {(x, y) x2 + y 2 = 1, y = x} is the set of points satisfying two conditions,


namely the equation x2 +y 2 = 1 and the equation y = x. This locus describes the intersection
points of the circle x2 + y 2 = 1 and the line y = x. By plugging the equation of the line into
the equation of the circle, we can show that this locus consists of only two points:


2
2
2 2
{(x, y) x2 + y 2 = 1, y = x} = {(
,
),(
,
)}.
2
2
2 2

{(x, y): y = x}

{(x, y): x2 + y2 = 1}

{(x, y): y = x, x2 + y2 = 1}

Page 420, Table of Contents

www.EconsPhDTutor.com

Example 418. {(x, y) x2 + y 2 1, y = x} is the portion of the line y = x that is within the
interior of the circle, including the endpoints. It is illustrated in green in the figure below.

{(x, y): y = x}

{(x, y): x2 + y2 1}

{(x, y): y = x, x2 + y2 1}

Page 421, Table of Contents

www.EconsPhDTutor.com

Example 419. The locus {(x, y) x2 + y 2 > 1, y > x} describes the region above both the
circle x2 + y 2 = 1 and y = x, excluding the circumference of the circle and the line. It is
illustrated in green in the figure below.

{(x, y): y = x}
x

{(x, y): x2 + y2 = 1}

{(x, y): y > x, x2 + y2 > 1}

Exercise 163. Sketch on a cartesian plane the locus {(x, y) x2 + y 2 = 1, x > 0}. (Answer
on p. 1121.)

We now turn to loci involving complex equations (and inequalities).

Page 422, Table of Contents

www.EconsPhDTutor.com

43

Loci Involving Complex Equations


43.1

Circles

On an Argand diagram (or complex plane), the locus {z C z = 1} simply describes the
unit circle centred on the origin, as we now prove:

Let z = (x, y). Then z = x2 + y 2 and so the equation z = 1 is equivalent to x2 + y 2 = 1


or x2 + y 2 = 1. Hence,
{z C z = 1} = {(x, y) x2 + y 2 = 1} .
But we already saw in the previous chapter that the locus {(x, y) x2 + y 2 = 1} describes
the unit circle centred on the origin.
Loci involving complex equations (or inequalities) can usually be easily transformed into a
familiar cartesian equation (or inequality).

y
{z : |z | = 1} = {(x, y): x2 + y2 = 1}

Page 423, Table of Contents

www.EconsPhDTutor.com

Exercise 164. (a) Prove that the locus {z C z = r} describes the circle of radius r
centred on the origin.
(b) Let c be some fixed complex number. Prove that the locus {z C z c = r} is the
circle of radius r centred on the point c.
(c) What does the locus {z C z c r} describe?
(d) What does the locus {z C z c < r} describe? (Answer on p. 1122.)

Page 424, Table of Contents

www.EconsPhDTutor.com

43.2

Lines

Let b and c be fixed complex numbers. The equation z c = z b is simply the condition
that z is equidistant to b and c.
Hence, the locus {z C z c = z b} simply describes the points that are equidistant to
b and c. And as we showed earlier, such a locus is simply a line.

Any point not on the


line must be closer to
one of the two points.
e

y
d
Any point on the
line is equidistant
to the two points.

b
x

c
{z : |z b | = |z c |}

Exercise 165. Let b and c be fixed complex numbers. What is the locus of complex
numbers z that satisfy each of the following inequalities? (a) z c z b. (b) z c <
z b. (c) z c z b. (d)z c > z b. (Answer on p. 1122.)

Page 425, Table of Contents

www.EconsPhDTutor.com

43.3

Rays

The locus {z C arg z = } describes the set of points z whose argument is . It is thus
the ray (or half-line) which starts from but excludes the origin and which makes an angle
with the positive x-axis. The figure below illustrates.
The point A is in the locus, because indeed arg A = . In contrast, the point B is not in
the locus, because its argument is not arg B .
Note importantly that points along the dotted red ray, such as C, are not in the locus,
because arg C = .
Moreover, the origin is not in the locus, because arg 0 is undefined.

{z : arg z = }

x
C

If we really wanted to, we could rewrite the complex equation arg z = into cartesian form.
But it turns out that in this case, the cartesian form is more complicated. And so well
just stick with the equation arg z = .

Page 426, Table of Contents

www.EconsPhDTutor.com

Let a be a complex number. Then the graph of arg (z a) = is simply the translation of
the graph of arg z = . And so arg (z a) = is the ray (or half-line) which starts from
but excludes the point a and which makes an angle with the positive x-axis.
The point b is in the locus, because indeed arg(b a) = . In contrast, the point c is not in
the locus, because its argument is not arg(c a) .
Note importantly that points along the dotted red ray, such as d, are not in the locus,
because arg(d a) = .
Moreover, the point a is not in the locus, because arg(a a) = arg 0 is undefined.

{z : arg (z a) = }
b

a
d

Page 427, Table of Contents

www.EconsPhDTutor.com

The 9740 syllabus doesnt mention loci of the form arg z . Unfortunately, such loci
have occasionally appeared on the A-level exams,45 which means you have to learn it.
The locus arg z is simply the region bounded by (and including) the rays arg z =
and arg z = .

{z : arg z = }

{z : arg z = }

Exercise 166. What is {z C z = 1, < arg z < 0}? (Answer on p. 1122.)

45

See Exercises 360 (2013), 364 (2011), and 370 (2008).

Page 428, Table of Contents

www.EconsPhDTutor.com

43.4

Quick O-Level Revision: Properties of The Circle

Definition 98. A chord is a line segment connecting any two points on a circles circumference.
Here are a few properties of the circle (which you are supposed to still remember from
O-levels) and which would definitely have been useful in some complex loci questions in
the past ten years A-levels.
Fact 56. Let A be a point exterior to a circle. Let B and C be the points at which the
tangents from A touch the circle. Let O be the centre of the circle.
(a) The line through A and O (i) bisects the angle BAC; (ii) is the perpendicular bisector
of the chord BC; and (iii) passes through the points D and E, which are the points on the
circle that are respectively that closest to and furthest from A.
(b) The lengths AB and AC are equal.
(c) The angles OBA and OCA are right.

Perpendicular
bisector of chord
B

Chord
E
O

Tangents C

Proof. See p. ??? of my O-Level Maths Textbook (coming soon)!


Heres an example that illustrates the uses of the above properties of the circle.
Page 429, Table of Contents

www.EconsPhDTutor.com

Example 420. The complex number z satisfies the equation z + 4 + 2i = 1. (a) What are
the maximum and minimum possible values of z? (b) For what values of z is z maximised
and minimised?
z + 4 + 2i = 1 describes a unit circle centred on the point C = (4, 2). Even if not asked
for, you should make a quick sketch to help yourself see better.
By the above fact, z is maximised at F and minimised at N , where F and N lie on the
line through the origin and the circles centre.

(a) The maximum value of z is the length OF = OC + CF = (4)2 + (2)2 + 1 = 20 + 1.

The minimum value of z is the length ON = OC CN = (4)2 + (2)2 1 = 20 1.


(b) Consider CAN . The line through F , C, N , and the origin is y = 0.5x. So AN =
0.5CA. Moreover, CA2 + AN 2 = CN 2 = 12 = 1.
Altogether then, CA2 + 0.25CA2 = 1 or CA2 =

2
1
4
or CA = . And AN = . Hence,
5
5
5

2
1
N = (4 + , 2 + )
5
5
1
2
Symmetrically, F = (4 , 2 ).
5
5

y
O

|z + 4 + 2i | = 1

U
N

F
D

y = 0.5 x
(Line through the
origin and the
centre of the circle.)

(... Example continued on the next page ...)

Page 430, Table of Contents

www.EconsPhDTutor.com

(... Example continued from the previous page ...)


z satisfies z + 4 + 2i = 1. (c) What are the maximum and minimum possible values of arg z?
(d) For what values of z is arg z maximised and minimised?
(c) The points U and D at which arg z is maximised and minimised are also where the tangents OU and OD from the origin touch the circle. By the above fact, OU is perpendicular
to CU . Similarly, OD is perpendicular to CD.
The angle the lower half of theline y = 0.5x makes with the positive x-axis is = tan1 0.5.

The angle COU is sin1 (1/ 20). Hence, arg U = +COU = tan1 0.5sin1 (1/ 20).

Symmetrically, arg D = COD = COU = sin1 0.5 + tan1 (1/ 20).

(d) ODC is right. So OD2 +CD2 = OC 2 . OC = 20 and CD = 1. Hence, OD = 20 1 =

19. Altogether then D = 19 and arg D = tan1 0.5 + sin1 (1/ 20).

Symmetrically, we also have U = 19 and arg U = sin1 0.5 + tan1 (1/ 20).
(Figure reproduced for convenience.)

y
O

|z + 4 + 2i | = 1

U
N

F
D

y = 0.5 x
(Line through the
origin and the
centre of the circle.)

Exercise 167. The complex number z satisfies the equation z 2 2i = 1. (a) What are
the maximum and minimum possible values of z? (b) For what values of z is z maximised
and minimised? (c) What are the maximum and minimum possible values of arg z? (d)
For what values of z is arg z maximised and minimised? (Answer on p. 1123f.)

Page 431, Table of Contents

www.EconsPhDTutor.com

44

De Moivres Theorem
n

Theorem 7. De Moivres Theorem. (cos + i sin ) = cos (n) + i sin (n).


Proof. cos + i sin is the complex number with modulus 1 and argument . So by Fact 51,
n
(cos + i sin ) is the complex number with modulus 1n = 1 and argument n + 2k (where
k is the unique integer such that n + 2k is a principal value) this complex number can
be written as cos (n) + i sin (n).
Here is an alternative proof that uses the Euler Formula:
n 2

n 1

Proof. (cos + i sin ) = (ei ) = ei(n) = cos (n) + i sin (n), where = and = use the Euler
2

Formula (Theorem 6) and = uses the law of exponents (xa ) = xab , which applies even when
a is imaginary.

This is a totally free, bonus exercise on mathematical induction!


Exercise 168. Prove de Moivres Theorem using the method of mathematical induction.
(Answer on p. 1125.)

As stated, de Moivres Theorem applies only to complex numbers with modulus 1. It is


easy to rewrite it so that it applies more generally to any complex number with modulus r:
n

Corollary 5. [r (cos + i sin )] = rn [cos (n) + i sin (n)].


n

Or equivalently, (rei ) = rn ei(n) .


Or equivalently, if z = r and arg z = , then z n = rn and arg z n = n + 2k (where k is the
unique integer such that n + 2k is a principal value).

Page 432, Table of Contents

www.EconsPhDTutor.com

44.1

Powers of a Complex Number

On the Argand diagram, the powers of a complex number form a spiral.


Example 421. Let z = 1 + i. Then:
z =

2,

arg z =

+ 2k =
(k = 0),
4
4

2
z 2 = ( 2) = 2,

arg z 2 = 2 ( ) + 2k =
(k = 0),
4
2

z 3 = ( 2) = 2 2,

3
arg z 3 = 3 ( ) + 2k =
(k = 0),
4
4

4
z 4 = ( 2) = 4,

arg z 4 = 4 ( ) + 2k = (k = 0),
4

z 5 = ( 2) = 4 2,

(k = 1),
arg z 5 = 5 ( ) + 2k =
4
4

etc.

The powers of z = 1 + i, up to the 13th, are illustrated in the figure below.

z11

32 y z10
16

z12
-64

z9

z3 z2
z8
z
x
0
-16 z5 0
16
7
z
6
z
-16
z4

-48

-32

-32
-48
z13

Page 433, Table of Contents

-64

www.EconsPhDTutor.com

Example 422. Let z = 1 + 0.4i. Then:


z =

12 + 0.42 = 1.16,

arg z = tan1 (0.4) + 2k,

z 2 = 1.16,

arg z 2 = 2 tan1 (0.4) + 2k,

z 3 = 1.16 1.16,

arg z 3 = 3 tan1 (0.4) + 2k,

z 4 = 1.162 ,

arg z 4 = 4 tan1 (0.4) + 2k,

z 5 = 1.162 1.16,

arg z 5 = 5 tan1 (0.4) + 2k,

etc.

The powers of z = 1 + 0.4i, up to the 14th, are illustrated in the figure below.

z5

z6

z4

z3

y
z2
z1
z

z7

z8
z14
z9
z13
z10
z12
z10

Page 434, Table of Contents

z11

www.EconsPhDTutor.com

Exercise 169. (a) Given z = 3 4i, find z and arg z. Hence find z 7 and arg z 7 . Write
down (3 4i)7 in exponential form.
(b) Given z = 5 + 12i, find z and arg z. Hence find z 8 and arg z 8 . Write down (5 + 12i)8
in exponential form. (Answer on p. 1125.)
Exercise 170. For each of the given values of z, compute z 10 , expressing your answer in
all three forms (polar, exponential, and standard). (a) z = 1 i. (b) z = 2 + i. (c) z = 1 3i.
(Answer on p. 1126.)

Page 435, Table of Contents

www.EconsPhDTutor.com

44.2

Roots of a Complex Number

Example 423. What are the roots to the equation z 3 = 1 + i? That is, for what values of
z is the given equation true?
A nave application of de Moivres Theorem might suggest that
z 3 = 21/2 and arg z 3 = /4

1/3

z = (21/2 )

= 21/6 and arg z = (/4)/3 = /12.

This is not incorrect, but it gives us only one root to the equation z 3 = 1 + i, namely
z = 21/6 ei(/12) .
In contrast, the Fundamental Theorem of Algebra tells us that since the equation z 3 = 1 + i
involves a degree-3 polynomial, it should have 3 roots. Weve just found one root. How do
we find the other two?
The trick is to recognise that z 3 = 21/2 ei/4 can also be written as z 3 = 21/2 ei(/4+2k) ,
for any integer k. This is because if you plug in any integer k, you will always get
21/2 ei(/4+2k) = 21/2 ei(/4) . The reason is that ei(2) = 1.
1/3

1/3

We then have z = (z 3 ) = [21/2 ei(/4+2k) ] = 21/6 ei(/4+2k)/3 , for any integer k. Now in
contrast to before, different integers k will yield us distinct values for z = 21/6 ei(/4+2k)/3 .
In particular, if we pick values of k so that the values of (/4 + 2k) /3 are principal values,
that is, if we pick k = 0, 1, we have
z = 21/6 ei(/12) ,

21/6 ei(11/12) ,

21/6 ei(7/12) .

Observe that beautifully enough, the roots of the equation z 3 = 1 + i lie on a circle in
particular, the circle of radius 21/6 centred on the origin. Moreover, each root can be
2
obtained by rotating another root
radians about the origin.
3

Page 436, Table of Contents

www.EconsPhDTutor.com

z n
arg z n + 2k
In general, given z , we have z =
and arg z =
, where k are those integers
n
n
n
arg z
such that arg z =
+ 2k (, ].
n
n

The annoying part is to figure out the appropriate values of k. So heres how to do it:
1. If n is odd, then simply pick k = 0, 1, 2, . . . ,
0, 1, 2, . . . , 7.)

n1
. (E.g., if n = 15, then pick k =
2

n
2. If n is even AND arg z n > 0, then simply pick k = 0, 1, 2, . . . , . (E.g., if n = 16 and
2
arg z n > 0, then pick k = 0, 1, 2, . . . , 7, 8.)
n
3. If n is even AND arg z n 0, then simply pick k = 0, 1, 2, . . . , . (E.g., if n = 16 and
2
arg z n 0, then pick k = 0, 1, 2, . . . , 7, 8.)
You can easily verify that in each case, we do indeed have n roots (just count them). See
Fact 90 in the Appendices for a proof (or explanation) of why the above values of k ensure
that we have k distinct principal values for arg z.

More examples ...

Page 437, Table of Contents

www.EconsPhDTutor.com

Example 424. Consider z 4 = 5 + 12i.


1

We have z 4 = 13ei[tan (12/5)+2k] , for k Z. Hence, z = 131/4 ei[tan


Since 4 is even and arg z 4 > 0, we should pick k = 0, 1, 2 to get
1

131/4 ei[tan (12/5)+2k]/4 ,

131/4 ei[3tan (12/5)]/4 ,


z=
1

131/4 ei[tan (12/5)]/4 ,

1/4 i[3tan1 (12/5)]/4

,
13 e

(12/5)+2k]/4

, for k Z.

(k = 0),
(k = 1),
(k = 1),
(k = 2).

Page 438, Table of Contents

www.EconsPhDTutor.com

Example 425. Consider z 7 = 3 4i. We have z 7 = 5ei[tan (4/3)+2k] , for k Z. Hence,


1
z = 51/7 ei[tan (4/3)+2k]/7 , for k Z. Since 7 is odd, we should pick k = 0, 1, 2, 3.
1

Now consider w8 = 3 4i. We have w8 = 5ei[tan (4/3)+2m] , for m Z. Hence, w =


1
51/8 ei[tan (4/3)+2m]/8 , for m Z. Since 8 is even and arg w8 0, we should pick m =
0, 1, 2, 3, 4.
Altogether then, the possible values of z and w are given by:

1/8 i[tan1 (4/3)]/8

5
e
,

1/7 i[tan (4/3)]/7

5
e
,
(k
=
0),

51/8 ei[tan (4/3)+2]/8 ,


1

1/7
i[tan
(4/3)+2]/7

5 e
,
(k = 1),

1/8 i[tan1 (4/3)+4]/8

5
e
,

1/7 i[tan (4/3)+4]/7

5
e
,
(k
=
2),

51/8 ei[tan (4/3)+6]/8 ,


1/7 i[tan1 (4/3)+6]/7
and w=
z= 5 e
,
(k = 3),

1/8 i[tan1 (4/3)2]/8

5
e
,

1/7 i[tan (4/3)2]/7

5
e
,
(k
=
1),
1

51/8 ei[tan (4/3)4]/8 ,

1/7
i[tan
(4/3)4]/7

5 e
,
(k = 2),

1/8 i[tan1 (4/3)6]/8

5
e
,

1/7 i[tan (4/3)6]/7

5
e
,
(k
=
3),

1/8 i[tan (4/3)8]/8

,
5 e

(m = 0),
(m = 1),
(m = 2),
(m = 3),
(m = 1),
(m = 2),
(m = 3),
(m = 4).

x
/

Notice that the eight possible values of w are on a circle whose radius is just slightly shorter
than the red circle. (Only the red circle is illustrated.)

Page 439, Table of Contents

www.EconsPhDTutor.com

Exercise 171. Find the roots of each of the following equations. (a) z 10 = 1 i. (b)
z 11 = 2 + i. (c) z 12 = 1 3i. (Answer on p. 1127.)

Page 440, Table of Contents

www.EconsPhDTutor.com

Part V

Calculus

Page 441, Table of Contents

www.EconsPhDTutor.com

45

Solving Problems Involving Differentiation

Part I already covered differentiation. This chapter merely ties up some loose ends.

45.1

Inverse Function Theorem (IFT)

The Inverse Function Theorem (IFT) simply says that The change in y caused by a small
unit change in x (dy/dx) is the inverse of the change in x caused by a small unit change
in y (dx/dy).46 That is,
dy 1
= .
dx dx
dy
Example 426. Suppose that adding 1 g of Milo (the x-variable) to a cup of water increases
the volume of water by 2 cm3 (the y-variable). That is, dy/dx = 2 cm3 g-1 .
Then dx/dy = 0.5 g cm-3 . That is, if instead we had wanted to increase the volume of water
by 1 cm3 , we should have added 0.5 g of Milo to the water.

Heres a more typical use of the IFT:


Example 427. Let x [/2, /2]. Let y = sin x. Suppose we wish to find dx/dy in terms
of x.
Method #1 (longer method using Corollary 2 ). y = sin x x = sin1 y. So
1
dx d
1
1
=
sin1 y =
=
=
dy dy
1 y2
1 sin2 x cos x
Method #2 (quicker method using the IFT).

dy
dx
1
= cos x
=
.
dx
dy cos x

dy
dx
. Hence write down
. (You may leave
dx
dy
your answers expressed in terms of x and y.) (Answer on p. 1128.)

Exercise 172. Suppose x2 y + sin x = 0. Find

46

This is informal. For the formal statement of the IFT (optional), see p. 969 in the Appendices.

Page 442, Table of Contents

www.EconsPhDTutor.com

45.2

Differentiation of Simple Parametric Functions

Informal Fact.

dx
dy dy dx
=

(provided
0).
dx dt dt
dt

Here is an informal proof of the above informal fact. By the Chain Rule,
dy 1 dy dt
=
.
dx dt dx
By the IFT,
dt 2 1
= .
dx dx
dt
2

Plugging = into = yields the desired result:


dy dy dx
=
.
dx dt dt
See p. 970 in the Appendices for a formal version of the above Fact.

Example 428. Let x = t5 + t and y = t6 t. Find

dy
.
dx t=0

dy dy dx 6t5 1
=

=
.
dx dt dt 5t4 1
dy
= 1. It would be much more difficult (perhaps even impossible) if instead we first
dx t=0
dy
tried to express y in terms of x, then compute
.
dx

So

Exercise 173. Let x = cos t + t2 and y = et t3 . Find

Page 443, Table of Contents

dy
. (Answer on p. 1128.)
dx

www.EconsPhDTutor.com

45.3

Equations of Tangents and Normals

Recall from secondary school the following two facts:


Fact 57. The line with slope m through the point (a, b) has equation y b = m(x a).

Fact 58. Given a line with slope m, its perpendicular has slope

1
.
m

Example 429. The curve C has parametric equations x = t5 + t and y = t6 t, t R.


Consider the normal line at the point where t = 0. Find any point(s) at which the normal
line intersects the curve C again.
First, note that t = 0 (x, y) = (0, 0). Next,
R
R
R
dy dx RRRR
6t5 1 RRRR
dy RRRR
R =
R = 1.
R = 4
dx RRRR
dt dt RRRR
5t 1 RRRR
Rt=0
Rt=0
Rt=0
So the tangent line at the point t = 0 or (0, 0) has slope 1. Thus, the normal line at this
point has slope 1. Its equation is thus y 0 = 1(x 0) or more simply y = x.
The points where this normal line intersects the curve is thus given by the system of
equations y = x, x = t5 + t, and y = t6 t. Putting these together, we have t5 + t = t6 t
t (t5 t4 2) = 0. So t = 0 or t 1.45 (calculator). (We know by the Fundamental Theorem
of Algebra that there must be six roots altogether in this case, only two are real, while
the other four are complex.)
So the normal line intersects the curve C again at the point where t 1.45 or where
(x, y) (7.88, 7.88).
Exercise 174. A curve C is described by the pair of parametric equations x = t5 + t and
y = t4 t. Find the tangent lines to the curve at the points where t = 0 and t = 1. Find the
intersection point of these two tangent lines.(Answer on p. 1128.)

Page 444, Table of Contents

www.EconsPhDTutor.com

45.4

Connected Rates of Change Problems

Example 430. Sand is being unloaded onto a flat surface at a steady rate of 0.01 m3 s-1 .
Assume that the unloaded sand always forms a perfect cone with equal height and base
diameter. Find the rate at which the area of the base of the cone increasing at the instant
t = 20 s.
1
First, recall that a cone has volume V = r2 h, where r is the radius of the base and h is
3
the height. Since the base diameter equals the height (or h = 2r), we can rewrite this as
2
d
dV
dr
V = r3 . Applying , we have
= 2r2 .
3
dt
dt
dt
The base area is A = r2 . So the rate at which the base area is increasing is
dA
dr dV
= 2r =
r.
dt
dt dt
dV
= 0.01 at all times.
dt
3V 1/3
0.3 1/3
V t=20 = 20 0.01 = 0.2, so that rt=20 = ( )
= ( ) . Altogether then,
t=20
2

The volume of the sand is increasing at a rate 0.01, i.e.

Page 445, Table of Contents

dA
0.3 1/3

= 0.01 ( ) = 0.0219 m2 s1 .
dt t=20

www.EconsPhDTutor.com

Exercise 175. (Answer on p. 1129.) Illustrated below is a cone with lateral l, base radius
r, and height h. You are given that such a cone has total external surface area (excluding
1
the base) rl and volume r2 h.
3

A manufacturer wishes to manufacture a cone whose volume is fixed at 1 m3 and whose


total external surface area (excluding the base) is minimised. Find out what its height
should be. (You can follow the steps below.)
(a) Express r in terms of h.
(b) Use the Pythagorean Theorem to express l in terms of r and h. Hence express l solely
in terms of h.
(c) Now express the total external surface area A (excludes the base) solely in terms of h.
dA 3 h63
6 1/3
(d) Show that
=
. Hence conclude that the only stationary point is h = ( ) .
dh 2 A

(e) Use the quotient rule to show that


12

d2 A 9 h4 A2 ( h3 )
=
.
dh2 4
A3
d2 A
. Replace A2 with the expression for A that you found
dh2
in (c). Now fully expand this numerator. Observe that it is a quadratic and prove that it
is always positive.

(f) Consider the numerator of

(g) Hence conclude that the stationary point we found is indeed the global minimum.

Page 446, Table of Contents

www.EconsPhDTutor.com

45.5

Finding Max/Min Points on the TI84

Example 431. Define f [0, 2] R by x xsin (0.5x). We can easily find the minimum
point of f analytically:

df
= 1 cos ( x) = 0
dx
2
2

2
cos ( x) =
2

x=

2
2
cos1 0.560664181.

But as an exercise, lets find it using our TI84.


After Step 1.

After Step 2.

After Step 3.

After Step 4.

After Step 5.

After Step 6.

1. Press ON to turn on your calculator.


2. Press Y= to bring up the Y= editor.
3. Press X,T,,n SIN 0 . 5 . To enter , press the blue 2ND button and then
(which corresponds to the button). Now press X,T,,n ) and altogether you will
have entered x sin(0.5x).
4. Now press GRAPH and the calculator will graph y = x sin(0.5x).
Note that in the question given, the domain is actually [0, 2], but we didnt bother telling
the calculator this. So the calculator just went ahead and graphed the equation y = x
sin(0.5x) for all possible real values of x and y.
No big deal, all we need to do is to zoom in to the region where 0 x 2.
5. Press the (ZOOM) button to bring up a menu of ZOOM options.
6. Press 2 to select the Zoom In option. Using the < and > arrow keys, move the cursor
to where X = 1.0638298, Y = 0. Now press ENTER and the TI will zoom in a little,
centred on the point X = 1.0638298, Y = 0.
(... Example continued on the next page ...)

Page 447, Table of Contents

www.EconsPhDTutor.com

(... Example continued from the previous page ...)


It looks like starting at x = 0, the function is decreasing, then hits a minimum point, then
keeps increasing. Our goal now is to find out what that minimum point is.
After Step 7.

After Step 8.

After Step 9.

After Step 11.

After Step 12.

After Step 13.

After Step 10.

4. Press the blue 2ND button and then CALC (which corresponds to the TRACE
button). This brings up the CALCULATE menu.
5. Press 3 to select the minimum option. This brings you back to the graph, with a
cursor flashing. Also, the TI84 prompts you with the question: Left Bound?
TI84s MINIMUM function works by you first choosing a Left Bound and a Right
Bound for x. TI84 will then look for the minimum point within your chosen bounds.
6. Using the < and > arrow keys, move the blinking cursor until it is where you want your
first Left Bound to be. For me, I have placed it a little to the left of where I believe
the minimum point to be.
7. Press ENTER and you will have just entered your first Left Bound.
TI84 now prompts you with the question: Right Bound?.
8. So now just repeat. Using the < and > arrow keys, move the blinking cursor until it is
where you want your first Right Bound to be. For me, I have placed it a little to the
right of where I believe the minimum point to be.
9. Again press ENTER and you will have just entered your first Right Bound.
TI84 now asks you: Guess? This is just asking if you want to proceed and get TI84 to
work out where the minimum point is. So go ahead and:
10. Press ENTER . TI84 now informs you that there is a Zero at X = .56066485,
Y = .2105137 and places the cursor at precisely that point. This is our desired
minimum point.
(Notice theres a slight error, because the TI84 uses slightly-imprecise numerical methods.
Analytically, we found that the minimum point was x 0.560664181, while the TI84 claims
it is X = .56066485.)
Page 448, Table of Contents

www.EconsPhDTutor.com

45.6

Finding the Derivative at a Point on the TI84

This example will also illustrate how to graph parametric equations on the TI84.
Example 432. The curve C has parametric equations x = t5 + t and y = t6 t, t R. Well
find dy/dx using our TI84, even though this is easily found analytically:
t=1

dy
6t5 1
5
= 4
= .
dx t=1 5t + 1 t=1 6
After Step 1.

After Step 2.

After Step 3.

After Step 4.

After Step 5.

After Step 6.

After Step 7.

After Step 8.

1. Press ON to turn on your calculator.


2. Press MODE to bring up a menu of settings that you can play with. In this example,
all we want is to plot a curve based on parametric equations. So:
3. Using the arrow keys, move the blinking cursor to the word PAR (short for parametric)
and press ENTER .
4. Now as usual, well input the equations of our curve. To do so, press Y= to bring up
the Y= editor. Notice that this screen looks a little different from usual, because we are
now under the parametric setting.
5. Press X,T,,n 5 + X,T,,n and altogether you will have entered T 5 + T in the
first line.
6. Now press ENTER to go to the second line.
7. Press X,T,,n 6 - X,T,,n and altogether you will have entered T 6 T in the
second line.
8. Now press GRAPH and the calculator will graph the given pair of parametric equations.
Notice that strangely enough, the graph seems to be empty for the region where x < 0. But
clearly there are values for which x < 0 for example, t = 1.1 (x, y) (2.71, 2.87).
So why isnt the TI84 graphing this?
(... Example continued on the next page ...)

Page 449, Table of Contents

www.EconsPhDTutor.com

(... Example continued from the previous page ...)


The reason is that by default, the TI84 graphs only the region for where 0 t 2 (at least
this is so for my particular calculator). We can easily adjust this:
4. Press the WINDOW button to bring up a menu of WINDOW options.
5. Using the arrow keys, the number pad, and the ENTER key as is appropriate, change
Tmin and Tmax to your desired values. In my case, I decided somewhat randomly to
enter Tmin = 10 and Tmax= 10.
6. Then press GRAPH again and the calculator will graph the given pair of parametric
equations, now for the region Tmin t Tmax, where Tmin and Tmax are whatever
you chose.
After Step 9.

After Step 10.

After Step 11.

After Step 13.

After Step 14.

After Step 15.

After Step 12.

dy
,
Actually, the last few steps were really not necessary, if all we wanted was to find
dx t=1
as we do now:
7. Press the blue 2ND button and then CALC (which corresponds to the TRACE
button). This brings up the CALCULATE menu, which once again looks a little different
under the current parametric setting.
8. Press 2 to select the dy/dx option. This brings you back to the graph.
Nothing seems to be happening. But now, simply ...
9. Press 1 and now the bottom left of the screen changes to display T = 1.
10. Hit ENTER . What youve just done is to ask the calculator to calculate
point where t = 1. The calculator tells you that dy/dx = .83333528.

dy
at the
dx

dy 5
Again, theres a slight error the exact correct answer is
= = 0.8333..., so again the
dx 6
TI84 is a tiny bit off.

Page 450, Table of Contents

www.EconsPhDTutor.com

46

The Maclaurin Series


46.1

Power Series

Recall what polynomials are:


Example 433. 4 + x + 3x2 is a 2nd-degree polynomial. 18 + 5x x2 + x4 is a 4th-degree
polynomial.
You can easily imagine what a -degree polynomial is. Only we dont call it a -degree
polynomial. Instead, we call it a power series.
Definition 99. A power series is simply any infinite series

ai xi = a0 + a1 x + a2 x2 + . . . ,
i=0

where each ai is a real constant and x is the variable.


Example 434. 1 + 2x + 3x2 + 4x3 + 5x4 + 6x5 + . . . is a power series, with a0 = 1, a1 = 2,
a2 = 3, . . . , ak = k + 1, . . .
So too is 1 x + x2 x3 + x4 x5 + . . . , with a0 = 1, a1 = 1, a2 = 1, . . . , ak = (1)k+1 , . . .
As we learnt before, a series can either be convergent or divergent.
Example 435. 1 + x + x2 + x3 + x4 + x5 + . . . is a power series, with a0 = a1 = = ak = = 1.
It is, moreover, a convergent power series, provided x < 1. Indeed, provided x < 1, we
1
know that this is an infinite geometric series that converges to
and we may write
1x
1 + x + x2 + x3 + x4 + x5 + =

1
.
1x

In contrast, if x 1, then 1 + x + x2 + x3 + x4 + x5 + . . . is a divergent power series.

For H2 Maths, the only power series well be interested in is called the Maclaurin series.

Page 451, Table of Contents

www.EconsPhDTutor.com

46.2

Maclaurin Series

Definition 100. Let f be an infinitely-differentiable function. The Maclaurin series of f


at x is denoted M (x) and is defined to be the power series
M (x) = a0 + a1 x + a2 x2 + + an xn + . . . ,
f (3) (0)
f (n) (0)
f (0)
, a3 =
, ..., an =
, ...
where a0 = f (0), a1 = f (0), a2 =
2!
3!
n!

Written out explicitly or in summation notation, we have:


f (0) 2 f (3) (0) 3 f (4) (0) 4
f (n) (0) n
M (x) = f (0) + f (0)x +
x +
x +
x + +
x + ....
2!
3!
4!
n!
(i)
f (0) i
=
x.
i!
i=0

We are often interested in finite-order Maclaurin series:


Definition 101. Let f be a n-times differentiable function. The nth-order Maclaurin series
of f at x is denoted Mn (x) and is defined as the nth-degree polynomial (or finite series)
Mn (x) = a0 + a1 x + a2 x2 + + an xn ,
f (0)
f (3) (0)
f (n) (0)
, a3 =
, ..., an =
.
where a0 = f (0), a1 = f (0), a2 =
2!
3!
n!

Example 436. Lets compute the Maclaurin series for f R R defined by x ex . We


have f (0) = e0 = 1. We also have f (x) = ex , so that f (0) = e0 = 1. Similarly, f (x) = ex ,
so that f (0) = e0 = 1. Indeed, for any k Z+ , f (k) (x) = ex , so that f (k) (0) = e0 = 1. Hence,
the Maclaurin series for f is
M (x) = 1 + 1x +

1 2 1 3
x2 x2
x + x + = 1 + x +
+
+ ...
2!
3!
2! 3!

Here are the first four finite-order Maclaurin series for f :


M0 (x) = 1,

M1 (x) = 1 + x,

Page 452, Table of Contents

M2 (x) = 1 + x +

x2
,
2!

M3 (x) = 1 + x +

x2 x3
+ .
2! 3!

www.EconsPhDTutor.com

Exercise 176. Write down the third-order Maclaurin series for each of the following functions: (Answer on p. 1130.)
(a) f R R defined by x (1 + x)n ,
(b) g R R defined by x sin x,
(c) h R R defined by x cos x,
(d) i R R defined by x ln(1 + x).

Remark 8. The A-level syllabuses make no mention of the Taylor series and so we wont
talk about it. But just so you know, the Maclaurin series is simply a special case of the
Taylor series specifically, it is the Taylor series about 0.

Page 453, Table of Contents

www.EconsPhDTutor.com

46.3

The Amazing Maclaurin Series

The Maclaurin series is simply an (infinite) series. And as we saw in Part II, an infinite
series may or may not be convergent.
The following is very powerful theorem:
Informal Theorem. If f satisfies a nice property at a, then M (a) converges to
f (a). That is, M (a) = f (a).
(See section 971 in the Appendices for a more thorough and formal discussion of this
theorem.)
This table is in the List of Formulae you get, so no need to memorise.

f (x)

f (0)

xf (0)

x2
f (0)
2!

...

xn (n)
f (0)
n!

...

(1 + x)n

nx

n(n 1) 2
x
2!

...

n(n 1) . . . (n r + 1) r
x
r!

...

(x < 1)

ex

x2
2!

...

xr
r!

...

(all x)

sin x

x3
3!

x5
5!

...

(1)r x2r+1
(2r + 1)!

...

(all x)

cos x

x2
2!

x4
4!

...

(1)r x2r
(2r)!

...

(all x)

ln(1 + x)

x2
2

x3
3

...

(1)r+1 xr
r

...

(1 < x 1)

1. The first row of the above table says that if x is a value at which the function f satisfies
the nice property, then f (x) is equal to the Maclaurin series of f at x.
2. The second row says that g R R by x (1 + x)n satisfies the nice property for all
n(n 1) 2
x (1, 1). Thus, for all x (1, 1), we have (1 + x)n = 1 + nx +
x + . . . We
2!
say that (1, 1) is the range of values for which g has a convergent Maclaurin
series.47
3. The third row says that h R R by x ex satisfies the nice property for all x R.
x2
Thus, for all x R, we have ex = 1 + x +
+ . . . We say that R is the range of values
2!
for which h has a convergent Maclaurin series.
47

We should be careful to state that if n < 0, then the domain should be restricted to exclude 0.

Page 454, Table of Contents

www.EconsPhDTutor.com

4. The fourth row says that i R R by x sin x satisfies the nice property for all x R.
x3 x5
Thus, for all x R, we have sin x = x + . . . We say that R is the range of values
3! 5!
for which i has a convergent Maclaurin series.
5. The fifth row says that j R R by x cos x satisfies the nice property for all x R.
x2 x4
Thus, for all x R, we have cos x = 1 + . . . We say that R is the range of values
2! 4!
for which j has a convergent Maclaurin series.
6. The sixth row says that k (1, ) R by x ln(1 + x) satisfies the nice property for
x2 x3
all x (1, 1]. Thus, for all x (1, 1], we have ln(1 + x) = x + . . . We say that
2
3
(1, 1] is the range of values for which k has a convergent Maclaurin series.
In the syllabus, these five particular Maclaurin series are called standard series.
x2 x3
+ + . . . for all x R.
2! 3!
Instead, we will merely verify that this equation is plausible, for x = 0, 1, 5. (Try these
out yourself using the sheet Maclaurin series at the usual link.)
Example 437. Here we will not rigorously prove that ex = 1 + x +

For x = 0, we have ex = e0 = 1. And M0 (0) = 1, M1 (0) = 1, M2 (0) = 1, and indeed Mn (0) = 1


for all n. So it does appear plausible that e0 = M (0).
1
For x = 1, we have ex = e1 2.718. And M0 (1) = 1, M1 (1) = 1 + 1 = 2, M2 (1) = 1 + 1 + = 2.5,
2

1 1
M3 (1) = 1 + 1 + + = 2.67, ..., M7 (1) 2.718. It appears that Mn (1) 2.718 for all n 7.
2 6
So it does appear plausible that e1 = M (1).
25
For x = 5, we have ex = e5 148.413. And M0 (5) = 1, M1 (5) = 1 + 5 = 6, M2 (5) = 1 + 5 +
=
2
1
25 125
= 39 , ..., M18 (5) 148.413. It appears that Mn (5) 148.413
18.5, M3 (1) = 1 + 5 + +
2
6
3
for all n 18. So it does appear plausible that e5 = M (5).

Exercise 177. (Tedious, use the sheet named Maclaurin series at the usual link.) Verify

x3 x5
that for x = 0, , 2, it is similarly plausible that sin x = x
+ . . . (Answer on p.
2
3! 5!
1131.)

Page 455, Table of Contents

www.EconsPhDTutor.com

46.4

Finite-Order Maclaurin Series as Approximations

One important practical use of Maclaurin series is that finite-order Maclaurin series can be
used as approximations.
Example 438. Consider h R R defined by x ex . We have h(1) = e 2.718.
The 0th-order Maclaurin series is a pretty terrible approximation: M0 (1) = 1. The 1st-order
Maclaurin series is slightly better: M1 (1) = 1 + x = 1 + 1 = 2. The 2nd-order Maclaurin series
is even better: M2 (1) = 1 + x + 0.5x2 = 1 + 1 + 0.5 = 2.5. The 3rd-order Maclaurin series is

very good: M3 (1) = 1 + x + 0.5x2 + x3 /3! = 1 + 1 + 0.5 + 0.16 = 2.66.


We see that it tends to be that the higher the order of the Maclaurin series, the better the
approximation.

I emphasise the phrase tends to be, because the approximation can sometimes get worse
before it gets better, especially if were looking at a value that is far from 0. The next
example illustrates.

Page 456, Table of Contents

www.EconsPhDTutor.com

Example 439. Consider i R R defined by x sin x. We have h(2) = sin(2) = 0. If


we do the tedious computations, we find that
M0 (2) = 0,

M1 (2) = M2 (2) = 2 6.283,

M3 (2) = M4 (2) 35.059,

M5 (2) = M6 (2) 46.547.

The 0th-order Maclaurin series gets it exactly right. But each subsequent finite-order
Maclaurin series then drifts ever further from 0! Having computed the 5th- and 6th-order
Maclaurin series, it certainly does not look like the approximations will get any better. Yet
if we perservere, we find that
M7 (2) = M8 (2) 30.159,

M9 (2) =M10 (2) 11.900,

M11 (2) =M12 (2) 3.195,

M13 (2) =M14 (2) 0.625,

M15 (2) =M16 (2) 0.093, M17 (2) =M18 (2) 0.011,

M19 (2) =M20 (2) 0.001,

M21 (2) =M22 (2) 0.000,

...

Indeed, Mn (2) 0.000 for all n 21. So it does indeed look like the Maclaurin series for
sin x converges. Graphed below are sin x and M21 (x). We see that M21 (x) almost perfectly
approximates sin x for x [7, 7]. But for larger values, M21 (x) veers far away from sin x.

x
-12

-7

-2

12

(... Example continued on the next page ...)

Page 457, Table of Contents

www.EconsPhDTutor.com

(... Example continued from the previous page ...)


Graphed below are y = sin x, M1 (x), . . . , M10 (x). We see that the 1st-order Maclaurin series
M1 (x) = x is indeed a good approximation for values of x that are close to 0, but terrible
for larger values.
Low-order Maclaurin series work well as approximations, provided we are looking at small
values of x (i.e. values that are close to 0).
But for large x, even if the Maclaurin series eventually converges, low-order Maclaurin
series may fare very poorly as approximations. Indeed, as we saw on the previous page, for
sufficiently large values of x, even a relatively-high-order Maclaurin series like M21 (x) will
fare poorly as an approximation!

Page 458, Table of Contents

www.EconsPhDTutor.com

If a is not within the range of values for which the Maclaurin series for the function f
converges, then M (a) f (a). That is, the (infinite) Maclaurin series does not converge.
Hence, there is no reason to expect that Mi (a) f (a) (i.e. that any finite-order Maclaurin
series will serve as a good approximation). Example to illustrate.
Example 440. Consider k R R defined by x ln(1 + x). The range of values for which
the Maclaurin series converges is (1, 1]. Suppose we pick x = 2, which is certainly outside
this range. Then we have k(2) = ln 3 1.099. Lets see what the finite-order Maclaurin
series look like:
M0 (2) = 0,

M1 (2) = 2,

1
M4 (2) = 1 ,
3

M5 (2) = 5

M8 (2) = 19.314,

M9 (2) = 37.575,

1
,
15

M2 (2) = 0,

2
M3 (2) = 2 ,
3

M6 (2) = 5.6,

M7 (2) 12.686,

M10 (2) = 64.825,

M11 (2) 121.356.

Unlike before, further perserverance will not pay off here. Indeed, the Maclaurin series will
grow without bound. For example, M50 (2) 14.9 trillion! The Maclaurin series simply
does not converge for x = 2. So there is no reason to expect any finite-order Maclaurin
series to be a good approximation.

Page 459, Table of Contents

www.EconsPhDTutor.com

46.5

Product of Two Power Series

Informally, if two power series converge, then so too does their product; and to get this
product, simply multiply the two series together as if they were finite polynomials.48
1
1 3
x + . . . and cos x = 1 + 0 x2 + 0 + . . . .
3!
2!
Thus, for all x R, we have sin x cos x = c0 + c1 x + c2 x2 + c3 x3 + . . . , where
Example 441. For all x R, sin x = 0 + 1x + 0

Constant Term

c0 = 0 1 = 0,

Coefficient on x

c1 = 0 0 + 1 1 = 1,

Coefficient on x2

1
c2 = 0 ( ) + 1 0 + 0 1 = 0,
2!

Coefficient on x3

1
2
1
c3 = 0 0 + 1 ( ) + 0 0 + ( ) 1 =
2!
3!
3

Take a moment to convince yourself that c0 , c1 , c2 , c3 are as stated. So for all x R,


2
2
sin x cos x = 0 + 1x + 0x2 + ( ) x3 + = x x3 + . . .
3
3
The expression on the RHS is, of course, simply also the Maclaurin series for sin x cos x.
You are asked to show this in Exercise 178.
Exercise 178. Let f R R be defined by x sin x cos x. Evaluate f (0), f (0), f (0),
and f (3) (0). Hence, write down the 3rd-order Maclaurin series for f and verify that this is
consistent with what we found in Example 441. (Answer on p. 1132.)

The next example illustrates that one must be careful about when the Maclaurin series is
convergent:

48

This assertion is formally stated and proven at Fact 97 in the Appendices (optional).

Page 460, Table of Contents

www.EconsPhDTutor.com

1
Example 442. For all x R, we have sin x = 0 + 1x + 0 x3 + . . . For all x (1, 1],
3!
1 2 1 3
we have ln(1 + x) = 1x x + x + . . . And so for x (1, 1], we have sin x ln(1 + x) =
2
3
c0 + c1 x + c2 x2 + c3 x3 + . . . , where
Constant Term

c0 = 0 0 = 0,

Coefficient on x

c1 = 0 1 + 1 0 = 0,

Coefficient on x2

1
c2 = 0 ( ) + 1 1 + 0 0 = 1,
2

Coefficient on x3

c3 = 0

1
1
1
1
+ 1 ( ) + 0 1 + ( ) 0 =
3
2
3!
2

1
1
And so sin x ln(1 + x) = 0 + 0x + 1x2 + ( ) x3 + = x2 x3 + . . . , for x (1, 1] this set
2
2
is simply the intersection of R and (1, 1], which are respectively the ranges of values on
which the Maclaurin series for sin x and ln x converge.
The expression on the RHS is, of course, simply also the Maclaurin series for sin x ln(1 + x).
You are asked to show this in Exercise 178.
Exercise 179. Let f R R be defined by x sin x ln(1+x). Evaluate f (0), f (0), f (0),
and f (3) (0). Hence, write down the 3rd-order Maclaurin series for f and verify that this is
consistent with what we found in Example 442. (Answer on p. 1132.)

Page 461, Table of Contents

www.EconsPhDTutor.com

46.6

Composition of Two Functions

Informally, if f (x) = a0 + a1 x + a2 x2 + . . . for x S and g(c) S, then


2

f (g(c)) = a0 + a1 g(c) + a2 [g(c)] + . . .


That is, to get f (g(c)), simply plug in g(c) into the power series for f .49
Example 443. Define f R R+ R by f (x) = (1 + x)1 and g R R by g(x) = 2x. We
know that for all x (1, 1), we have f (x) = (1 + x)1 = 1 x + x2 x3 + . . . .
Thus, f (g(x)) = (1 + 2x)1 = 1 (2x) + (2x)2 (2x)3 + . . . for all g(x) = 2x (1, 1).
Equivalently,
f (g(x)) = (1 + 2x)1 = 1 2x + 4x2 8x3 + . . . for all x (0.5, 0.5).
Example 444. Define f R R by f (x) = ex and g R R by g(x) = x2 . We know that
x2 x3
+
+ ....
for all x R, we have f (x) = ex = 1 + x +
2! 3!
2

(x2 )
(x2 )
Thus, f (g(x)) = e = 1 + x +
+
+ . . . for all g(x) R. Equivalently,
2!
3!
2

x2

x4 x6
+
+ . . . for all x R.
f (g(x)) = e = 1 + x +
2! 3!
x2

In the case where g also has a convergent Maclaurin series, we can likewise also simply
plug in the Maclaurin series for g.50 Example:

49
50

For a more careful and formal version of this assertion, see Fact 98 in the Appendices (optional).
Again, for a more careful and formal version of this assertion, see Fact 99 in the Appendices (optional).

Page 462, Table of Contents

www.EconsPhDTutor.com

Example 445. Define f (, 1) (1, ) R by f (x) = 1/(1 + x) and g R R by


g(x) = sin x. Write down the Maclaurin series for f g, up to the 4th-order term.
Method #1 (composition method). We know that for all x (1, 1), we have f (x) =
x3
1x+x2 x3 +. . . . And for all x R, we have g(x) = x +. . . Hence, for all g(x) (1, 1),
3!
i.e. for all x k/2 (for k Z), we have
1
2
3
= 1 g(x) + [g(x)] [g(x)] + . . .
1 + sin x
2
3
x3
x3
x3
+ . . . ) + (x
+ . . . ) (x
+ ...) ...
= 1 (x
3!
3!
3!
1
5
= 1 x + x2 + x3 ( 1) + = 1 x + x2 x3 + . . .
3!
6

f (g(x)) =

Find the general term for a Maclaurin series is explicitly excluded from the A-level syllabuses. Usually youll just have to write down the first few terms.
Method #2 (direct method). Let h(x) = 1/(1 + sin x). We have h(0) = 1. We also have
R
R
cos x RRRR
dh RRRR
R =
= 1,
2 RRR
dx RRRR
R
(1
+
sin
x)
RRx=0
Rx=0
R
R
R
2
(1 + sin x) sin x + 2 cos2 x RRRR
d2 h RRRR
(1 + sin x) sin x + 2 cos2 x(1 + sin x) RRRR
RRR =
RRR
R =
4
3
dx2 RRRR
R
RRR
(1
+
sin
x)
(1
+
sin
x)
R
Rx=0
Rx=0
x=0
R
sin x + 2 sin2 x RRRR
=
RR = 2,
3
(1 + sin x) RRRRx=0
3
2
R
(1 + sin x) (cos x 2 sin x cos x) (sin x + 2 sin2 x) 3 (1 + sin x) cos x RRRR
d3 h RRRR
RRR
R =
6
RRR
dx3 RRRR
(1 + sin x)
Rx=0
Rx=0
1231
=
= 5.
1

Thus,

2
5
5
1
= 1 + (1)x + x2 + x3 + = 1 x + x2 x3 + . . .
1 + sin x
2!
3!
6

In the above example, I gave two methods. Use whichever seems to be easier or quicker.
Heres another example:

Page 463, Table of Contents

www.EconsPhDTutor.com

Example 446. Write down the Maclaurin series for sec x up to the 4th-order term.
Method #1 (composition method).
x2 x4
1
1
=
[1

(
sec x =
=

+ . . . )]
cos x 1 x2!2 + x4!4 . . .
2! 4!

x2 x4
x2 x4
=1+(
+ ...) + (
+ ...) + ...
2! 4!
2! 4!
1
1 2
x2 5x4
x2
4
+ x [ + ( ) ] + = 1 +
+
+ ...
=1+
2!
4!
2!
2
24
Method #2 (direct method). Let f (x) = sec x. Then f (0) = sec 0 = 1. And
f (x) = sec x tan x

f (0) = 0,

f (x) = sec x tan2 x + sec3 x


f (3) (x) = 6 sec2 xf (x) f (x)
2

f (0) = 1,
f (3) (0) = 0,

f (4) (x) = 12 sec x [f (x)] + 6 sec2 xf (x) f (x)


Thus,

f (4) (0) = 5.

1 2 0 3 5 4
x2 5x4
sec x = 1 + 0x + x + x + x + = 1 +
+
+ ...
2!
3!
4!
2! 24

Exercise 180. Write down the third-order Maclaurin series for sin [ln(1 + x)]. State also
the range of values for which the Maclaurin series converges. (Answer on p. 1132.)

Page 464, Table of Contents

www.EconsPhDTutor.com

46.7

How the Maclaurin Series Works (Optional)

Theorem 8. Let f [c, c] R be an infinitely-differentiable function. Suppose that for


all x (c, c), we have
f (x) = a0 + a1 x + a2 x2 + a3 x3 + a4 x4 . . .
Then the coefficients in the above power series are as given by the Maclaurin series. That
is, for each i = 0, 1, 2, . . . , we have
ai =

f (i) (0)
.
i!

Proof. Observe that


f (x) = a1 + 2a2 x + 3a3 x2 + 4a4 x3 . . . ,

f (x) = 2a2 + (3 2)a3 x + (4 3)a4 x2 . . . ,

f (3) (x) = (3 2)a3 + (4 3 2)a4 x . . . , f (4) (x) = (4 3 2)a4 + . . .


Thus, f (0) = a0 and
f (0) = a1 ,

f (0) = 2!a2 ,

f (3) (x) = 3!a3 , f (4) (x) = 4!a4 .


Rearranging, we have a0 = f (0), a1 = f (0), a2 = f (0)/2!, a3 = f (3) (0)/3!, a4 = f (4) (0)/4!,
..., ai = f (i) (0)/i!, ..., as desired.
The above theorem is merely a tantalising hint of why the Maclaurin series works. This is
because the theorem merely says this: If we make the very big assumption that the infinitelydifferentiable function f can be written down as a power series, then the coefficients of the
power series are as given by the Maclaurin series.
But this is not very useful, because how do we know that the function can be written
down as a power series? For a continuation of this discussion, see section 88.14 in the
Appendices.

Page 465, Table of Contents

www.EconsPhDTutor.com

47

The Indefinite Integral

Definition 102. Given functions f and F , we call F an indefinite integral (or antiderivative
or primitive) of f if for all x in the domain of f ,
F (x) = f (x),
In Leibnizs notation, we may write
F = f (x) dx or more simply F = f dx.
The statement F = f dx is thus completely equivalent to the statement F = f .
Example 447. Consider the functions f, F R R defined by f (x) = 2x and F (x) = x2 .
We see that F is an indefinite integral of f , because F (x) = 2x = f (x) for all x. We can
equivalently say that f is the derivative of F . We can also write
F = f dx or

dF
= f.
dx

The statement the value of F at 5 is 25 can be written as F (5) = 25. It can also be
written as
f dxx=5 = 25 or

f (x) dxx=5 = 25.

The symbol is called the integration sign it is an elongated S. The symbol dx is


called the differential of the variable x it informs us that the variable of integration
is x. The function f to be integrated is called the integrand.
Just like with summation, x is a dummy variable. We can replace x with any other letter
and the function F will still remain exactly the same function.

Page 466, Table of Contents

www.EconsPhDTutor.com

Example 448. The following two expressions are equal because i on the LHS and r on the
RHS are simply dummy variables.
n

i=1

r=1

i = r.
Similarly, the statement F = f (x) dx is equivalent any of the following three statements,
because the letters x, a, b, c, etc. are merely dummy variables:
F = f (a) da,

or F = f (b) db,

or F = f (c) dc.

So the statement the value of F at 5 is 25 can also be written F (5) = 25 or any of the
following four statements:
f (x) dxx=5 = 25, f (a) daa=5 = 25,
f (b) dbb=5 = 25,

Page 467, Table of Contents

f (c) dcc=5 = 25.

www.EconsPhDTutor.com

47.1

The Constant of Integration

Example 449. Consider f R R defined by f (x) = sin x.


F R R defined by F (x) = cos x is an indefinite integral of f , because F (x) = sin x =
f (x) for all x R.
Are there any other indefinite integrals of f ? Yes, certainly.
For example, G R R defined by G(x) = cos x + 200 is also an indefinite integral of f ,
because G (x) = sin x = f (x) for all x R.
Indeed, any H R R defined by H(x) = cos x + C where C R is also an indefinite
integral of f , because H (x) = sin x = f (x) for all x R.

In general:
Fact 59. If F is an indefinite integral of f , then so too is G defined by G(x) = F (x) + C,
for any C R.
We call C the constant of integration.
Proof. Since F (x) = f (x) for all x, we also have G (x) = F (x) + C = F (x) + 0 = f (x) for
all x. And so by definition, G is also an indefinite integral of f .

Page 468, Table of Contents

www.EconsPhDTutor.com

47.2

The Indefinite Integral is Unique Up to the C.O.I.

The indefinite integral is unique up to the constant of integration. That is, if F


and G are both indefinite integrals of f , then it must be that F and G differ only by a
constant.
Example 450. Say f has indefinite integral F defined by F (x) = sin (ex 3x+5 ). Suppose
G is another indefinite integral of f . Then it must be that F (x) = G(x) + C, for some
C R.
2

Formally:
Fact 60. If F and G are both indefinite integrals of f , then there exists some C R such
that F (x) = G(x) + C for all x.
Proof. Since F and G are both indefinite integrals of f , by definition, F (x) = G (x) for all
x. And thus (F G) (x) = 0 for all x. But the only functions whose derivative is always 0
are constant functions.51 Thus, F (x) G(x) = C, for all x, for some C R.

Exercise 181. (Answer on p. 1133.) Let f R R, F R R, G R R be defined by


f (x) = 4 sin 4x, F (x) = cos 4x, and G(x) = 8 sin2 x cos2 x .
(a) Show that F and G are both indefinite integrals of f .
(b) F and G seem to be very different functions. Yet both are indefinite integrals of f .
Why does this not contradict our assertion that the indefinite integral is unique up to a
constant?

51

The alert reader will note that this assertion has not actually been proven in this textbook. Well simply take it for granted
that the only functions whose derivative is 0 are constant functions.

Page 469, Table of Contents

www.EconsPhDTutor.com

48

Integration Techniques

As before with our notation for differentiation, lets be clear (pedantic). To take an example,
the notation
sin x dx = cos x + C
is simply shorthand for the following long-winded statement:
Consider a function with mapping rule x sin x.
Its indefinite integrals are functions, all of which
have the mapping rule x cos x + C.52

52

This shorthand statement fails to to mention the domain and codomains of the function and its indefinite integral. However,
the careful writer will of course have specified these nearby.

Page 470, Table of Contents

www.EconsPhDTutor.com

48.1

Basic Rules of Integration

Proposition 10. Let k, n R be constants with n 1. Let f and g be functions with


indefinite integrals F and G. Then
k dx
n

x dx

kx + C,

xn+1
=
+ C,
n+1

1
x dx = ln x + C,
x
e dx

(x 0 if n < 0)
(x 0)

sin x dx

= cos x + C,

cos x dx

= sin x + C,

f (x) g(x) dx = F + G + C,

ex + C,

kf (x) dx

kF + C,

where in each case, C is the constant of integration.


Proof. In general, to prove that f (x) dx = F , it suffices to prove that F (x) = f (x) for
all x.
d
And so to prove that x1 dx = ln x + C, it suffices to prove that
(ln x + C) = x1 for
dx
all x 0. This we now do. First note that

ln x + C,
ln x + C =

ln (x) + C,

Thus,
(ln x + C) =

dx

x
And so indeed

for x 0,
for x < 0.
for x 0,
1
,
x

for x < 0.

d
(ln x + C) = x1 for all x 0.
dx

You are asked to prove the remaining rules of integration in Exercise 182.

Exercise 182. Prove the remaining rules of integration listed in Proposition 10. (Answer
on p. 1134.)

Page 471, Table of Contents

www.EconsPhDTutor.com

48.2

More Basic Rules of Integration

No need to memorise the following rules of integration, because the List of Formulae contains a (slightly less general) version.
Proposition 11. Let a 0. Then
(a)

(b)

1
x2 + a2 dx

1
dx =
2
a x2

1
x
tan1 ( ) + C,
a
a
x
sin1 ( ) + C,
a

for x < a,

(c)

1
x2 a2 dx

1
xa
ln
+ C,
2a
x+a

for x a,

(d)

1
a2 x2 dx

1
a+x
ln
+ C,
2a
ax

for x a,

(e)

tan x dx

ln sec x + C,

(f)

cot x dx

ln sin x + C,

(g)

csc x dx

= ln csc x + cot x + C,

(h)

sec x dx

ln sec x + tan x + C,

for x not an odd multiple of

,
2

for x not an multiple of ,


for x not an multiple of ,
for x not an odd multiple of

,
2

where in each case, C is the constant of integration.


Proof. We prove only (a), (c), and (e). (You are asked to prove the remaining rules of
integration in Exercise 183.)
d
1
d 1
x
1
1
1
tan1 x = 2
. Hence,
[ tan1 ( ) + C] =

=
dx
x +1
dx a
a
a ( x )2 + 1 a
a
a
1
1
x
. So indeed 2
dx = tan1 ( ) + C.
2
2
2
x +a
x +a
a
a

(a) By Corollary 2,

(... Proof continued on the next page ...)

Page 472, Table of Contents

www.EconsPhDTutor.com

(... Proof continued from the previous page ...)

(c) Let x a. Case #1:

xa
0.
x+a

d 1
xa
d 1
xa
1 d
( ln
+ C) =
( ln
+ C) =
[ln(x a) ln(x + a)]
dx 2a
x+a
dx 2a x + a
2a dx
1
1
1
1 x + a (x a) 1 2a
1
=
(

)=
=
=
,
2a x a x + a
2a (x a) (x + a) 2a x2 a2 x2 a2
1
1
xa
so that indeed 2
dx
=
ln

+ C.
x a2
2a
x+a
Case #2:

xa
< 0.
x+a

xa
d 1
ax
1 d
d 1
( ln
+ C) =
( ln
+ C) =
[ln(a x) ln(x + a)]
dx 2a
x+a
dx 2a x + a
2a dx
1
1
1
1
1
1
1
=
(

)=
(

)= 2
,
2a a x x + a
2a x a x + a
x a2
1
xa
1
so that again 2
dx =
ln
+ C.
2
x a
2a
x+a
(e) Let x not be an odd integer multiple of /2, so that sec x 0.
Case #1: sec x 0.
d
d
sec x tan x
(ln sec x + C) =
(ln sec x + C) =
= tan x,
dx
dx
sec x
so that indeed tan x dx = ln sec x + C.
Case #2: sec x < 0.
d
sec x tan x
d
(ln sec x + C) =
[ln ( sec x) + C] =
= tan x,
dx
dx
sec x
so that again tan x dx = ln sec x + C.

Exercise 183. Prove the remaining rules of integration listed in Proposition 11. (Answers
on pp. 1135, 1136, 1137, and 1138.)

Page 473, Table of Contents

www.EconsPhDTutor.com

48.3

Trigonometric Functions

The following indefinite integrals are NOT on the List of Formulae and you are definitely
required to know how to derive them on your own!
Fact 61. Let m, n R. Then
(a)

2
sin x dx

1
sin 2x
x
+ C,
2
4

(b)

2
cos x dx

sin 2x
1
x+
+ C,
2
4

(c)

2
tan x dx

tan x + x + C,

(d)

1 cos [(m n)x] cos [(m + n)x]


+
} + C,
sin(mx) cos(nx) dx = 2 {
mn
m+n

(e)

sin(mx) sin(nx) dx =

1 sin [(m n)x] sin [(m + n)x]


{

} + C,
2
mn
m+n

(f)

cos(mx) cos(nx) dx =

1 sin [(m n)x] sin [(m + n)x]


{
+
} + C,
2
mn
m+n

where C is the constant of integration.


Proof. (a) The trick is to recall the trigonometric identity cos 2x = 1 2 sin2 x (this is in the
List of Formulae, as are several other trig identities). And so:
2
sin x dx =

1 cos 2x
1
sin 2x
dx = x
+ C.
2
2
4

You are asked to prove the remaining rules of integration in Exercise 184.

Exercise 184. Prove the remaining rules of integration listed in Fact 61. (Answer on p.
1139.)

Page 474, Table of Contents

www.EconsPhDTutor.com

48.4

Integration by Substitution (IBS)

The method of integration by substitution (IBS) is the Chain Rule in reverse. Before
we explain why it works, here are two examples of how it works.53
cos x
. Next, observe that
Example 451. Lets find cot x dx. First, observe that cot x =
sin x
d
du
sin x = cos x. Let u = sin x (this is our substitution), so that we also have
= cos x.
dx
dx
So:
cos x
1 du
cot
x
dx
=
dx
=

sin x
u dx dx.
So far, nothing unusual has happened. Now were going to do something strange, which is
to take that last expression and merrily cancel out the dxs:
1 du
1
u dx dx = u du + C1 .
du
is NOT a fraction? So why are we
Didnt we repeatedly insist earlier that the derivative
dx
allowed to merrily cancel out the dxs!? Shortly well explain why this move is legitimate.
For now, let us blindly perservere:
1
u du + C1 = ln u + C = ln sin x + C.

Another example, before we explain why exactly we can merrily cancel out the dxs:
du
Example 452. Lets find 2x cos x2 dx. Let u = x2 , so that we also have
= 2x. Now,
dx
du
2
2x cos x dx = dx cos u dx.
Again, we merrily cancel out the dxs and write:
du
2
dx cos u dx = cos u du + C1 = sin u + C = sin x + C.

53

Actually we secretly already used this method a few times above, though not very explicitly.

Page 475, Table of Contents

www.EconsPhDTutor.com

We now explain why it is OK to merrily cancel out the dxs. In fact, saying that we
merrily cancel out the dxs is merely a mnemonic (memory device). We are not actually
cancelling out the dxs. Instead, we are appealing to the following result:
Theorem 9. Let f D R be any continuous function. Let u be a real-valued differentiable
function. Assume Range(u) D, so that the composite function f u exists. Then
du
f dx dx = f du + C.
dP 1
du
dQ 2
du
dx and Q = f du. In other words,
=f
and
= f.
Proof. Let P = f
dx
dx
dx
du
2

Using first the Chain Rule and then =, we have

dQ dQ du 3
du
=

=f .
dx du dx
dx

du
. And so
dx
by Fact 60 (uniqueness of the indefinite integral up to a constant), P and Q must be equal
(or differ by at most a constant). That is, P = Q + C or
1

Examining = and =, we see that P and Q are both indefinite integrals for f

du
f dx dx = f du + C.

The above result says that when doing integration, we are allowed to merrily do two
things:
du
du
dx with du (cancel out the dxs from
dx to get du);
dx
dx
du
dx
du
2. Replace du with
dx (multiply du by
= 1 to get
dx).
dx
dx
dx

1. Replace

Of course, we are not actually doing any such things as cancelling out the dxs or muldx
tiplying by
= 1 these are merely mnemonics. Instead, all we are doing is appealing
dx
to the above theorem.54
Lets try more examples, now that we have a better understanding of how this works:

54

dy
dx
dy
= 1/ . The IFT is true NOT because
and
dx
dy
dx
dx
dy
dx
are fractions. Nonetheless, as a convenient mnemonic, we can pretend that the IFT holds because
and
are
dy
dx
dy
fractions even though strictly speaking, such thinking is wrong.

This is analogous to the Inverse Function Theorem, which states that

Page 476, Table of Contents

www.EconsPhDTutor.com

du
Example 453. Lets find esin x cos x dx. Let u = sin x, so that we also have
= cos x.
dx
Now we can write
du
1
sin x
cos x dx = eu dx = eu du + C1 = eu + C = esin x + C,
e
dx
1

where = uses Theorem 9. Purely as a mnemonic, we may think of this step = as cancelling
out the dxs, even though strictly speaking, we are doing no such thing; instead, we are
appealing to Theorem 9.

50

Example 454. Lets find (x3 + 5x2 3x + 2) (3x2 + 10x 3) dx. One method would
be to fully expand the integrand to get a 152nd-degree polynomial, then integrate this
polynomial term-by-term. This is doable, but absurdly tedious.
A better method is to observe that 3x2 + 10x 3 =
x3 + 5x2 3x + 2. Then we can write

d
(x3 + 5x2 3x + 2). Thus, let u =
dx

50
3
2
2
50 du
(x + 5x 3x + 2) (3x + 10x 3) dx = u dx dx
51

u51
(x3 + 5x2 3x + 2)
= u du + C1 =
+C =
51
51
1

50

+ C,

where once again = uses Theorem 9.

In the next three examples, we go in the opposite direction. That is, instead of cancelling
dx
out the dxs as was done in the previous few examples, we instead multiply by
= 1.
dx

Page 477, Table of Contents

www.EconsPhDTutor.com


Example 455. Lets find
1 u2 du. Well use the substitution u = sin x. Note that

du
2
1 u = 1 sin2 x = cos x. Moreover,
= cos x. So
dx

dx
du
1
1 u2 du = cos x du = cos x du = cos x dx + C1

dx
dx
sin 2x
2 1
= cos x cos x dx + C1 = cos2 x dx + C1 = x +
+ C,
2
4

1 1
2 sin x cos x sin1 u + u 1 u2
= sin u +
=
,
2
4
2
1

where = uses Theorem 9 and = uses Fact 61.


x2
Example 456. Lets find
dx. Well use the substitution u3 = 1 + 2x. Note that
3
1 + 2x
1 3
dx 3 2
x = (u 1) and
= u . So
2
du 2
2

2
2
[ 12 (u3 1)]
x2
(u3 1)
(u3 1) du

dx =
dx =
dx =
dx

3
3
4u
4u du
1 + 2x
u3
2

(u3 1) dx
(u3 1) 3 2
3u (1 2u3 + u6 )
=
du + C1 =
( u ) du + C1 =
du + C1
4u du
4u
2
8
1

3
3 u2 2u5 u8
3 2 1 2u3 u6
4
7
u

2u
+
u
du
+
C
=
(

+
)
+
C
=
u (
+ )+C
1
8
8 2
5
8
8
2
5
8

3
2(1 + 2x) (1 + 2x)2
2/3 1
= (1 + 2x) [
+
]+C
8
2
5
8
3
20 16 32x + 5 + 20x + 20x2
3
20x2 12x + 9
+ C = (1 + 2x)2/3
+ C,
= (1 + 2x)2/3
8
40
8
40
1

where = uses Theorem 9. (The last line is just further simplification, which is nice but not
necessary.)

Page 478, Table of Contents

www.EconsPhDTutor.com

1
Example 457. Lets find
dx. Well use the substitution u = tan x or x =
1 + 3 cos2 x
tan1 u. Note that
cos2 x =
So

1
1+

3
1+u2

dx =

1
1
1
=
=
2
sec2 x 1 + tan x 1 + u2
1
1+

3
1+u2

and

dx
1
=
.
du 1 + u2

du
1
1 dx
1
1
dx =
du + C1 =
du + C1
3
3
du
1 + 1+u2 du
1 + 1+u2 1 + u2

1
1
1
2 1
1 u
1 tan x
du
+
C
=
du
+
C
=
tan
(
)
+
C
=
tan
(
) + C,
1
1

1 + u2 + 3
22 + u2
2
2
2
2

=
1

where = uses Theorem 9 (multiply by

du
2
= 1) and = uses Proposition 11.
du

Usually, the hard part is to figure out the appropriate substitution to make. Fortunately,
in the A-level exams, youll always be told what substitution to make.

Exercise 185. (Answers on pp. 1140 and 1141.) (a) (i) Use the substitution x = 3 sec u
9

to find
dx.
x2 x2 9

9
9

(ii) Now use instead the substitution x =


to find
dx.
2 x2 9
1u
x

1
(iii) Show that sin (sec1 y) = 1 2 . Then explain why your answers in (i) and (ii) are
y
consistent.
x3
3
tan u to find
dx.
3/2
2
(4x2 + 9)
x3
2
(ii) Now use instead the substitution u = 4x + 9 to find
dx.
3/2
(4x2 + 9)
1
(iii) Show that cos (tan1 y) =
. Then explain why your answers in (i) and (ii) are
1 + y2
consistent.
(b) (i) Use the substitution x =

Page 479, Table of Contents

www.EconsPhDTutor.com

48.5

Integration by Parts (IBP)

The method of integration by parts (IBP) is the Product Rule in reverse.


Theorem 10. (Integration by Parts.) Let u and v be differentiable functions, which
have continuous derivatives u and v . Then uv dx = uv u v dx.

Proof. Differentiate the RHS to get u v + uv u v = uv . This shows that uv dx =


uv u v dx, as desired.
dv
, Exponential, Trig,
dx
Algebraic, Inverse trig, Log. (This is because exponential functions are easiest to integrate,
followed by trigonometric functions, etc.)

To choose v , use the rule of thumb DETAIL D stands for

Example 458. Find xex dx.


By the DETAIL rule of thumb, we should choose v = ex . Now,

u v
x

x
x
x
x
x
x e dx = uv u v dx = xe e dx = xe e = e (x 1).

Sometimes we need to apply IBP more than once:

Example 459. Find x2 ex dx.


By the DETAIL rule of thumb, we should choose v = ex . Now,
u v

2 x

2 x
x
x e dx = uv u v dx = x e 2xe dx
= x2 ex 2ex (x 1) = ex (x2 2x + 2). (Use the previous example.)

Exercise 186. Find x sin x dx and x2 sin x dx. (Answer on p. 1142.)

Page 480, Table of Contents

www.EconsPhDTutor.com

49

The Fundamental Theorems of Calculus (FTCs)

The problem of finding the definite integral is the problem of finding the area under a curve.
The problem of finding the derivative is the problem of finding the slope of the tangent.
The two Fundamental Theorems of Calculus (FTCs) show that, surprisingly enough, these
two problems are intimately (indeed inversely) related.
This chapter is a largely-informal discussion of the intuition behind the FTCs.

49.1

The Area Function

Given a continuous real-valued function f , its area function is denoted A and is, informally, defined by the mapping A(c) = Area bounded by the graph of f , the horizontal
axis, and the vertical lines x = 0 and x = c.
Example 460. Graphed below is the continuous function f R+0 R defined by f (x) =

x + 1.
The area A(6) is highlighted in red. It is the area bounded by the graph of f , the horizontal
axis, and the vertical lines x = 0 and x = 6.
Using a graphing calculator, A(6) = 15.79795897... Is there a way I can figure this out
without a graphing calculator? Heres one possible approach lets approximate the area
by using three rectangles.

(... Example continued on the next page ... )

Page 481, Table of Contents

www.EconsPhDTutor.com

(... Example continued from the previous page ...)


Well use three rectangles of equal width so each rectangle has width 2. The leftmost
rectangle will occupy the interval [0, 2], the middle rectangle will occupy [2, 4], and the
rightmost rectangle will occupy [4, 6].
For each rectangle, we choose its height to be the lowest value attained by the function in
that interval. In the interval [0, 2], the lowest value attained by f is f (0). So the leftmost
blue rectangle has height f (0) and thus area Base Height = 2f (0).
Similarly, the middle green rectangle has height f (2), because in the interval [2, 4], the
lowest value attained by f is f (2). Hence, it has area Base Height = 2f (2).
The rightmost grey rectangle has height f (4), because in the interval [4, 6], the lowest value
attained by f is f (4). Hence, it has area Base Height = 2f (4).

x
-1

Altogether, the total area of these three rectangles is

SL3 = 2f (0) + 2f (2) + 2f (4) = 2 [( 0 + 1) + ( 2 + 1) + ( 4 + 1)] 12.828,


where SL3 stands for Lower Sum in the case of 3 rectangles with equal width. This is our
very first approximation of the area A(6). We see that this is a fairly poor approximation,
because the true area is A(6) = 15.79795897... Nonetheless, it is useful we know that SL3
is a lower bound for A(6). That is, we know that SL3 A(6).
Well next try a different approximation SU 3 . Can you guess what this involves?
(... Example continued on the next page ... )

Page 482, Table of Contents

www.EconsPhDTutor.com

(... Example continued from the previous page ... )


Well again use three rectangles of equal width (width 2), occupying intervals [0, 2], [2, 4],
and [4, 6]. The difference now is that for each rectangle, we choose its height to be the
highest value attained by the function in that interval. In the interval [0, 2], the highest
value attained by f is f (2). So the leftmost blue rectangle has height f (2) and thus area
Base Height = 2f (2).
Similarly, the middle green rectangle has height f (4), because in the interval [2, 4], the
highest value attained by f is f (4). Hence, it has area Base Height = 2f (4).
The rightmost grey rectangle has height f (4), because in the interval [4, 6], the highest
value attained by f is f (6). Hence, it has area Base Height = 2f (6).

x
-1

Altogether, the total area of these three rectangles is

SU 3 = 2f (2) + 2f (4) + 2f (6) = 2 [( 2 + 1) + ( 4 + 1) + ( 6 + 1)] 17.727,


where SU 3 stands for Upper Sum in the case of 3 rectangles with equal width. This
is our second approximation of the area A(6). We see that again, this is a fairly poor
approximation, because the true area is A(6) = 15.79795897.... Nonetheless, it is again
useful we know that SU 3 is an upper bound for A(6). That is, we know that A(6) SU 3 .
Altogether, we know that 12.828 SL3 A(6) SU 3 17.727.
Can we do better than this? Yes, certainly. An obvious follow-up would be to increase the
number of rectangles we use. Lets next use 6 rectangles instead.
(... Example continued on the next page ... )
Page 483, Table of Contents

www.EconsPhDTutor.com

(... Example continued from the previous page ... )


Well now use six rectangles of equal width (width 1), occupying intervals [0, 1], [1, 2],
[2, 3], [3, 4], [4, 5], and [5, 6]. To calculuate the Lower Sum SL6 , we give the first rectangle
height of f (0), the second f (1), ..., the sixth f (5). So each rectangle has, respectively, area
1f (0), 1f (1), ..., and 1f (5). Hence, SL6 = f (0)+f (1)+f (2)+f (3)+f (4)+f (5) 14.382.

-1 0 1 2 3 4 5 6 7 8 9 -1 0 1 2 3 4 5 6 7 8 9
Analogously, to calculuate the Upper Sum SU 6 , we give the first rectangle height of f (1),
the second f (2), ..., the sixth f (6). So each rectangle has, respectively, area 1 f (1),
1 f (2), ..., and 1 f (6). Hence, SU 6 = f (1) + f (2) + f (3) + f (4) + f (5) + f (6) 16.832.
Once again, A(6) has lower and upper bounds SL6 and SU 6 . That is, 14.382 SL6 A(6)
SU 6 16.832.
You can see where this is going. We can get ever better lower and upper bounds, by
increasing the number of rectangles we use.
(... Example continued on the next page ... )
Page 484, Table of Contents

www.EconsPhDTutor.com

Exercise 187. Continuing with the above example, find SL12 and SU 12 . Hence give lower
and upper bounds for A(6). (Answer on p. 1143.)

(... Example continued from the previous page ... )


Let n be the number of rectangles we use. We will always have SLn A(6) SU n .
As n increases, we have increasingly-many, increasingly-slim rectangles. As n , we have
infinitely-many, infinitely-slim rectangles, whose total area should approach A(6).
Indeed, this slim rectangles approach is exactly how the area function is formally and
rigorously defined see Section 88.17 in the Appendices for the details (optional).

x
-1

It appears then that we need to do more maths to figure out how to add up all these
infinitely-many, infinitely-slim rectangles. ... But it turns out though that there is an
absolutely-fantastic shortcut we can use.

Page 485, Table of Contents

www.EconsPhDTutor.com

49.2

The First Fundamental Theorem of Calculus (FTC1)

Given a function f , we sketched an idea of how to find its area function A approximate
the area under the curve using infinitely-many, infinitely-slim rectangles and add up the
total area of these rectangles. This though was merely a sketch of an idea. How do we go
about adding up the area of these infinitely-many, infinitely-slim rectangles? Easier said
than done!
It turns out though that well take an entirely different approach. Strangely enough, instead
of finding the area function A, we shall try to find the the area functions derivative
A . This seems utterly bizarre. If we dont know what A is in the first place, how could
we possibly figure out what A is? This is analogous to asking someone, who has no idea
where Singapore is, to find the Singapore Flyer!
But surprisingly, it turns out to be much easier to find A than it is to find A! Well recycle
the example from the last section:

Page 486, Table of Contents

www.EconsPhDTutor.com

Example 460 (continued from the previous section). Pick some x thats just a little
larger than 6. A(x) is the area bounded by the graph of f , between the vertical lines x = 0
and x = 6. And so A(x) is just slightly larger than A(6).

x
-1

Consider the thin green vertical strip. This green strip is roughly rectangular in shape
its left, right, and bottom edges are all straight. Only its upper edge is not straight.
This green strips area is exactly A(x) A(6). Moreover, we know that its base is x 6, its
left side is f (6), and its right side is f (x). Hence,
(... Example continued on the next page ...)

Page 487, Table of Contents

www.EconsPhDTutor.com

(... Example continued from the previous page ...)


Area of rectangle with
Area of thin green
Area of rectangle with
base x 6 and height f (6)
vertical strip
base x 6 and height f (x)



(x 6) f (6)
<
A(x) A(6)
<
(x 6) f (x) .
Rearranging, we have
f (6) <

A(x) A(6)
< f (x).
x6

Now consider what happens if we pick another x that is slightly smaller but still larger
than 6. Then the above pair of inequalities will still hold. Indeed, for all x > 6, the above
pair of inequalities hold. If we let x approach 6, the above pair of inequalities becomes
A(x) A(6)
lim f (x).
x6
x6
x6

lim f (6) lim


x6

(For why the strict inequalities < became weak inequalities , either you simply trust me
or see Fact 7 in the Appendices.)
Of course, lim f (6) is simply f (6). And by the continuity of f , lim f (x) = f (6). Hence,
x6

x6

A(x) A(6)
f (6),
x6
x6

f (6) lim

A(x) A(6)
= f (6). But wait a second ...
x6
x6
A(x) A(6)
lim
? It is simply the value of the derivative of A at 6!!
x6
x6
which means of course that lim

what is

A(x) A(6)
Def
A (6) = lim
.
x6
x6
We thus conclude that astonishingly enough, A (6) = f (6). And this is more generally true
given a continuous function f , the derivative of its area function is simply the original
function itself! This is the First Fundamental Theorem of Calculus.

Page 488, Table of Contents

www.EconsPhDTutor.com

Note that earlier we defined the area function so that we started counting the area from
x = 0 (vertical axis). But this was just to keep the above arguments and diagrams simple.
It makes no difference if we start counting the area from any other x = a instead.
Theorem 11. (First Fundamental Theorem of Calculus [FTC1], informal statement.) Let f be a real-valued continuous function with area function A. Then A = f .

In words, the FTC1 says that the area function of a continuous function is simply
the function itself ! Equivalently, an indefinite integral (or antiderivative) of a
continuous function is the area function.55

Exercise 188. Why did I use the indefinite article an, rather than the definite article the,
in the last sentence above? (Answer on p. 1143.)

Next is a familiar example from physics to illustrate the FTC1.


Example 461. Graphed below is the velocity v (ms-1 ) of a car as a function of time t (s).
Recall that the area under the graph is the distance travelled by the car. For example, the
shaded red area A(5) is the total distance travelled by the car after 5 s.
But the derivative of the distance travelled with respect to time is precisely the velocity!
Hence, this example illustrates the FTC1: the derivative of the area under the graph of a
function is precisely the function itself!

Velocity (ms-1)

Time (s)
0

55

For a formal, rigorous statement of FTC1 and its proof, see section 88.17 in the Appendices.

Page 489, Table of Contents

www.EconsPhDTutor.com

49.3

The Definite (or Riemann) Integral

p f (x) dx is the area under under the graph of f , between p and q. (Compare this to the
q

area function: A(k) is the area under the graph of f , between 0 and k.) We call
the definite (or Riemann) integral of f between p and q.

56

f (x) dx

x + 1. The definite integral f dx (simply


p
the area under f , between 1 and 3) is highlighted in blue. Similarly, the definite integral

Example 462. Define f R+0 R by f (x) =


q

p f dx (simply the area under f , between 5 and 8) is highlighted in red.

y
x
0

IMPORTANT REMARK
q

The indefinite integral f dx and the definite integral f dx have very similar
p
names and notation. But do not make the mistake of believing that weve simply defined
them so that theyre similar we have not.
The indefinite integral f (x) dx is an antiderivative of f . (It is also a function.)
b

The definite integral f (x) dx is the area under the graph of f , between a and b. (It
a
is also a number.)
A priori, there is no reason whatsoever to believe that some antiderivative of f and
some area under the graph of f have anything in common.
It is the two FTCs that establish the connection between the two. This is what makes the
FTCs remarkable and surprising.
And it is because of this connection that we give these two distinctly-defined mathematical
objects such similar names and notation.
56

For the formal definition of the definite integral, see section 88.17 in the Appendices.

Page 490, Table of Contents

www.EconsPhDTutor.com

49.4

The Second Fundamental Theorem of Calculus (FTC2)

Our (informal) definitions tell us that:


A(q) is the area under the curve between 0 and q;
Similarly, A(p) is the area under the curve between 0 and p;
q

And f dx is the area under the curve between p and q.


p
q

Thus, f dx = A(q) A(p). From this and also with the aid of the FTC1, we can easily
p
prove the FTC2.
Theorem 12. (The Second Fundamental Theorem of Calculus [FTC2].) Let f
[a, b] R be a continuous function and p, q [a, b]. Then
q

p f dx = f dxx=q f dxx=p .
Proof. By Theorem 11, the area function A is an indefinite integral of f . And so by
Fact 60, A and f dx differ by at most a constant. That is, for all r [a, b], we have
A(r) = f dx

x=r

+ C. Hence,
q

p f dx = A(q) A(p)
= [ f dx

x=q

= f dx

x=q

Page 491, Table of Contents

+ C] [ f dx

f dx

x=p

+ C]

x=p

www.EconsPhDTutor.com

50

Definite Integrals

To repeat:
The indefinite integral f dx is an antiderivative of f .
b

The definite integral f dx is an area under the graph of f .


a
A priori, there is no reason to believe that the two are in any way related. It is the two
FTCs that establishes their remarkable relationship:
b

a f dx = f dxx=b f dxx=a ,
b

To compute f dx, one method would have been to painfully add up the area of the
a
infinitely-many infinitely-slim rectangles. Thanks to the FTCs, we have a wonderful
alternative method that is much easier:
1. Find any indefinite integral of f .
2. The difference of the values of this indefinite integral at b and a is our desired area.
We can simply apply all the rules of integration we learnt earlier.
b

Notation: Well let [f (x)]a be shorthand for f (b) f (a).

Page 492, Table of Contents

www.EconsPhDTutor.com

50.1

Area between a Curve and Lines Parallel to Axes

Example 463. Find the exact area bounded by the curve y = x2 and the horizontal lines
y = 1 and y = 2.
Its always helpful to make a quick sketch (given below). Our desired area is labelled A
below. To find a desired area, there are usually multiple methods, some quicker than others.

Method #1. The entire rectangle A + B + C + D has area 2 2 2 = 4 2. B has area

3 1
x
1
2
2
2
21
2
.
2 x dx = [ 3 ] = 3 ( 3 ) =
3
2
1

By symmetry, D has the same area as B. C has area 1 2. Hence, A has area

2 21
4
2 21
+2+
) = (2 2 1) .
A + B + C + D (B + C + D) = 4 2 (
3
3
3

Method #2. The right branch of the parabola y = x2 has equation x = y. The right half
y=2
y=2
2
2
2
4
x dy =
of the area A is
y dy = [y 3/2 ]1 = (2 2 1). Hence, A = (2 2 1).
3
3
3
y=1
y=1

y
y=2
A
y=1
B

x
-2

-1

Exercise 189. Find the exact area bounded by the curve y = x3 , the horizontal lines y = 1
and y = 2, and the vertical axis. (Answer on p. 1144.)

Page 493, Table of Contents

www.EconsPhDTutor.com

50.2

Area between a Curve and a Line

Example 464. Find the area A bounded by the curve y = x2 and the line y = x + 1.

1 5
.
By the quadratic formula, the curve and line intersect at the points x =
2

(1+ 5)/2

(1+ 5)/2
3
x

x2

2
(15)/2 x + 1 x dx = [ 2 + x 3 ]
(1 5)/2

3
3
2

(1 + 5)2
(1 + 5) (1 5)
(1 5)

1
+
5
1

=
23
2
3 23
23
2
3 23

6 + 2 5 1 + 5 16 + 8 5
6 2 5 1 5 16 8 5
=[
+

][
+

]
8
2
24
8
2
24

3 5 1 5 2 5
7+5 5 75 5 5 5
3+ 5 1+ 5 2+ 5
+

][
+

]=

=
.
=[
4
2
3
4
2
3
12
12
6

A
x
Exercise 190. Find the exact area bounded by the curve y = sin x and the line y = 0.5, for
x (0, /2).(Answer on p. 1145.)

Page 494, Table of Contents

www.EconsPhDTutor.com

50.3

Area between Two Curves

Example 465. Find the area A bounded by the curves y = x2 2x 1 and y = 1 x2 .

1 5
. So
By the quadratic formula, the curves intersect at x =
2
A=

0.5(1+ 5)

0.5(1 5)

1 x2 (x2 2x 1) dx = 2

0.5(1+ 5)

0.5(1 5)

1 x2 + x dx

5 5
+ ]
=
,
= 2 [x
3
2 0.5(15)
3
x3

0.5(1+ 5)
2
x

where weve simply recycled our tedious calculations from the previous example.

x
A

Exercise 191. Find exact area bounded by the curves y = 2 x2 and y = x2 + 1. (Answer
on p. 1145.)

Page 495, Table of Contents

www.EconsPhDTutor.com

50.4

Area below the x-Axis

Example 466. Find the area A bounded by x2 4 and the x-axis.


The definite integral calculates the signed area under the curve and above the x-axis. So
if the curve is under the x-axis, the computed area will be negative, as we now see:
2

8
8
32
x3
.
x 4 dx = [ 4x] = ( 8) ( + 8) =
3
3
3
3
2
2

But of course, an area is simply a magnitude, so well take the absolute value and conclude
32
that the desired area is .
3

x
A

Exercise 192. Find the exact area bounded by x4 16 and the x-axis. (Answer on p.
1146.)

Page 496, Table of Contents

www.EconsPhDTutor.com

50.5

Area under a Parametrically-Defined Curve

Example 467. Consider the curve described by the equations x = t3 2 and y = 4t5 . Find
the exact area bounded by the curve, the lines x = 2 and x = 1, and the horizontal axis.
It helps to graph this curve on your graphing calculator:

Note that x = 1 t = 1, x = 2 t = 0, and dx/dt = 3t2 . So the area can be


computed as:
x=1

x=1

x=1

t=1

x=2 y dx = x=2 4 t dx = x=2 (4 t ) 3t dt = t=0

3t8
(4 t ) 3t dt = [4t
] = 4.
8 0
5

Exercise 193. Consider the curve described by the equations x = t2 + 2t and y = t3 1.


Find the exact area bounded by the curve, the lines y = 1 and y = 2, and the vertical axis.
(Answer on p. 1146.)

Page 497, Table of Contents

www.EconsPhDTutor.com

50.6

Volume of Rotation about the y- or x-Axis

Example 468. Consider the line y = 1. Rotate it about the x-axis to form an (infinite)
3D cylinder. Now consider the finite portion of the cylinder between x = 1 and x = 2. By a
primary school formula, its volume is Base Area Height = 12 (2 1) = .

Height

Radius
Volume
We can also compute this same volume using integration. The intuition is that were adding
up infinitely-many infinitely-thin circle-shaped slices, laid on their sides, from x = 1 to x = 2
(left to right). The face of each of these circles has area y 2 . In this particular example, y
is constant (simply 1). Thus, the total volume is
1

Page 498, Table of Contents

y 2 dx =

2
1

dx = [x]1 = .

www.EconsPhDTutor.com

Example 469. Rotate the line y = 3x about the x-axis to form an infinite double cone.
Consider the finite portion of the cone between x = 0 and x = 2. By the formula for the
1
1
volume of a cone, we know its volume is r2 h = 62 2 = 24.
3
3
We can also compute this same volume using integration. Again, the intuition is that were
adding up infinitely-many infinitely-thin circle shaped slices, from x = 0 to x = 2. Again,
the face of each of these circles has area y 2 . In this particular example, y = 3x. Thus, the
total volume is
2

0 y dx = 0

x3
(3x) dx = 9 [ ] = 24.
3 0
2

Height
Radius

Volume

Now consider instead the finite portion of the cone between x = 3 and x = 5. This looks
like a pedestal tilted sideways (not illustrated). We can easily compute its volume using
integration:
5

3 y dx = 3

x3
(3x) dx = 9 [ ] = 294.
3 3
2

Computing its volume using geometric formulae is possible, if slightly more tedious. The
1
1
finite portion of the cone between x = 0 and x = 3 is V1 = r2 h = 92 3 = 81. The finite
3
3
1 2
1
portion of the cone between x = 0 and x = 5 is V2 = r h = 152 5 = 375. Hence, the
3
3
desired volume is V = V2 V1 = 375 81 = 294.
Page 499, Table of Contents

www.EconsPhDTutor.com

We can just as easily find the volume of rotation about the y-axis.
Example 470. Consider the curve y = x2 . Find its volume of rotation about the y-axis,
from y = 0 and y = 5.
In this case, there are no familiar geometric formulae we can apply. So we really just have
to compute this same volume using integration. Again, the intuition is that were adding up
infinitely-many infinitely-thin circle-shaped slices, but this time these circle-shaped slices
are stacked from bottom to top, from y = 0 to y = 5. The face of each of these circles has
area x2 , where in this particular example, x2 = y. Thus, the total volume is
5

0 x dy = 0

y2
y dy = [ ] = 12.5.
2 0

Volume

Exercise 194. Compute the volume of rotation of y = sin x about the x-axis from x = 0 to
x = . (Answer on p. 1146.)

Page 500, Table of Contents

www.EconsPhDTutor.com

Vous aimerez peut-être aussi